Download as pdf or txt
Download as pdf or txt
You are on page 1of 254

Delhi University CBCS Syllabus

MANAGEMENT ACCOUNTING —
B.COM (HONS) SEMESTER-V
Duration : 3 hours Max. Marks: 75

ae
i te
Objective: To impart the students, knowledge about the use of financial, cost and ot her data
for the purpose of ma nagerial planning, control and decision making.

UNIT 1: INTRODUCTION -
Meaning, Objectives, Nature and Scope of management accounting, Difference between cost
accounting and management accounting, Cost control and Cost reduction, Cost manacerient.

UNIT I: Budgetary Control


Concept of budget, budgeting and hudgetary control, objectives, merits, ard limitations.
Budget administration. Functional budgets. Fixed and flexib: ‘e budgets. Zero base budceting,
. Programme and perforniance budgeting.

UNIT ITI; STANDARD COSTING


Meaning of standard cost and standard costing, advantages, limitations aid applications.
Variance Analysis - material, labour, overheads and sales va-iances. Disposition of Variances,
“ Gontrot Ratios.

~ UNIT IV: MARGINAL COSTING —


- Absorption versus Variable Costing, Distinctive features and income determination. Cost-Volume-
Profit Analysis, Profit Volume ratio, Break-even analysis-algebraic and graphic methods. Ancle
of incidence, margin of safety, Key factor, determination of cost indifference oint.

‘UNIT V: DECISION MAKING


steps in Decision Making Process, Concept: of Relevant Costs and Benefits, Various short-term
decision making situations - profitable product mix, Acceptance or Rejection cf special/expart
offers, -Make-or buy, Addition or Elimination of a product line, sell or process Further, operate
‘Shut downs Pricing Decisions: Major factors influencing pricing decisions, various methods of

sey
tT VI: CONTEMPORARY ISSUES
peestilty Ac¢oiinting: Concept, Significance, Different Responsibility Centres, O-visional
formance Measurement: Financial and Non-Financial measures. Transfer Pricing.
Contents

g i. Nature and Scope of Management Accounting


a Introduction
Meaning and definition of management accounting
Nature of management accounting
Scope of management accounting
Ojectives or functions
Tools and techniques used in management accounting
Comparison with financiat accounting
Comparison with cast accounting
Limitations
Cost control and Cost reduction
Cost management
111
Key terms
1.12
Objective type questions
1.43
Essay type questions
1.43

an hetgting and Budgetary Control 2,1~2.68


Introduction
2.1
Meaning and definition of budget and budgetary control
2.2
Objectives
2.3
Advantages
2.4
Limitations
2.4
Essentials of effective budgeting
2.5
Budget admmistration
2.6
Functional budgets
2.8
Master budget
2.16
Fixed and flexible budgets
2.16
Revision of budgets 2,20
+. Budget reports
2.21
.. , Zero base budget
2.22
Performance budgeting
2.24
Problems and solutions
2.25
Key terms
2,58
Objective type questions
2.58
Essay type questions
2.59
Practical questions
2.69
Vftandard Cesting and Variance Analysis 3.1-3.103.
, Introduction . 3.1
Meaning and definition of standard cost and standard costing
3.2
Applicability 3.2
se

(ig (xiii)

Standard cost and estimated cost "* Key'terins


Standard costing and budgetary control Objective type questions
Advantages , Essay type questions
Limitations Practical questions
Prelimineries
Variance analysis §. Decision Making
Material variances =: introduction
Labour variances Short-term and Long-term decisions
Overhead variances Relevant costs and relevant revenues
Sales variances Cost and non-cost factors in decision-making
Deposition of variances - Decision-making and variable costing
Control ratios Fixation of selling orices
Summary of formulae Exploring new markets
Problems and solutions Export sales
Key terms Make or buy decisions
Objective type questions Product mix decisions
Essay type questians Plant shut-down decisions
Practical questions Differential cost analysis vs. variable costing
Decision making and differential cost analysis
4, Absorption Costing versus Variable (Marginal) Costing 4.41-4.29 _ Adding or dropping a product line -
Introduction 41 Evaluating make or buy alternatives
Product costs and peried costs 41 Decision regarding further processing
Absorption costing — meaning, characteristics and income determination 42 Problems and solutions
Marginal costing — meaning, characteristics and income determination 4a Key terms
Distinction between absorption and marginal costing 47 Objective type questions
Difference in profit under absorption and marginal casting 49 - Essay type questions
Advantages and disadvantages of marginal costing 4.40 Practical questions
Problems and solutions
Key terms dh Responsibility Accounting 7.1-7.14
Objective type questions Introduction TA
Essay type questiens Meaning and definition 72
Practical questions Pre-requisites 7.2
fi ar? Responsibility centre and its types fe
5. st-Volume-Profit Analysis 5.1-5.88: Features 7.4
Introduction Advantages 74
Meaning of CVP analysis Measurements of divisionaliperformance a4
Break-even analysis Retum-on investment (ROI) 7.6
Assumptions Residual income (21) 7.8
Contribution and marginal cost equation Transfer Pricing 7.8
P/V ratio ; Problems and solutions 7.10
Caiculations in break-even analysis Key terms 7,13
Margin of safety Objective type quéstions 4.13
Key factor Essay type questions 7446
sid Angle of incidence
Break-even chart Appendix — A +.Problems and Solutions for Revision A.1-A,53 |
Profit volume chart “
Uses and limitations of Break-even analysis
Summary of formulae _ Appendix — B : Latest Question Papers B.1-8.50
Problems and solutions i
jaca

ul
ee
a:
4 | NATURE AND SCOPE OF
t
x MANAGEMENT ACCOUNTING |

Introduction; Meaning and definition of management


accounting; Nature of
Management accounting; Scope of management accountin
g; Ojectives or functions;
Tools and techniques used in management accounting;
Comparison with cost
accounting; Limitations; Cost control and Cost reduction;
Cost management; Key
“terms; Examination questions.

“The most exciting and innovative work in management today is


found in accounting”
Peter F. Drucker
~ Tntroduction
Accounting is a part of information system of an enterprise. Such informati
on is provided to peaple
who have an interest in the organisation, such as shareholders, managérs,
creditors, debentureholders,
bankers, tax authorities and others, Broadly speaking, on the basis of
type of accounting information
and the purpose for which such information is used, accounting may
be divided into three categories:
\ Accounting

Finantial or Generat |" Cost ’ Management


Accounting . Accounting Accounting
FINANCIAL ACCOUNTING
Financial accounting is mainly concerned with recording business
transactions in the books of
‘account ‘for the purpose of presenting final accounts. It is defined as
“the art of recording, classifying
and summarising in a significant manner and in terms of money, transactions
and events, which are in
“part at least, of a financial character and interpreting the results thereof."*
The information supplied by
Financial account
ingis summarised in the following two statements at the end of the accounting
period, generally orie year.
(2) Bofit and Loss Account showing the net profit or loss during the

period.
‘American: Institute of Certified Public Accountants (AICPA}
i : 1d
i

ij
1.2 Management Accounting 13
Rature and Scope of Management Accounting
fl
(by Balance Sheet showing the financial position of the firm ata point of time, .
“\-"- decision-making. Management accounting should serve as a decision support system to all levels of
The objective of. financial accounting is to provide information to external parties such .as - o
" ghareholders, creditors, employees, potential investors, government agencies, etc,
management, ~
|
Relationship of Management Accounting to Cost Accounting and Financial Accounting
COST ACCOUNTING The three types of accounting, i.e, financial accounting, cost accounting and management
accounting are closely linked. The management accounting uses the priniples and practices not only
Cost accounting is a branch of accounting which specialises in providing information about the
detaited cost of products or services being supplied by the undertaking. Compared with financial of cost accounting but also of financial accounting. Information provided by nancial accounting
proves extermely useful for management accounting, For example, Profit and Loss account and
accounting, cost accounting is relatively a recent development. It has primarily developed to meet the;
needs of management. Profit and Loss Account and Balance Sheet are presented to management by the Balance Sheet become the basis of ratio analysis and comparative financial statements, etc, which are
used by the management accounting as important tools of planning and control. Financial accounting
financial accountant. But modern management needs much more detailed information than supplied by
levels of records also become basis of preparing detailed cost computation and reports. Cost accounting is a
these financial statements. Cost accounting provides detaited cost information to various
by cost accounting more detailed application of financial accounting and provides detailed cost information about
management for efficient performance of their functions. The information supplied
optimum use of scarce resources and ultimately add to the products, services, departments, etc. This information is used by management accounting for planning,
acts as a tool of management for making
controlling and decision making purposes. Fig. 1.1 shows the evolution of management accounting
profitability of business.
and its relationship to cost accounting and financial accounting.
MANAGEMENT ACCOUNTING
Meaning and Definition PREPARING
It - PROFIT & [
The term ‘management accounting’ is the modern concept of accounts as a tool of management. LOSS ACCOUNT _ FINANCIAL
In
is a broad term and is.concerned with all such accounting information that is useful to management. AND ACCOUNTING
simple words, the term management accounting is applied to the provision of accounting
information BALANCE SHEET ; |
for management activities such as planning, controlling and decision-making, etc.
which
According to the Institute of Chartered Accountants of England, “any form of accounting
enables a business to be conducted more efficiently” may be regarded as management accounting.
and evaluate
Management accounting information can help managers identify problems, solve problems
= / ” en ay
performance. ANALYSING COSTS,
In the words of Robert Anthony, “Management accounting is concerned with accounting information FOR CONTROL |
that is useful to management.” eo Do AND MAXIMISING cost
ACCOUNTING |
The Institute of Cost and Works Accountants of India {ICWAT) has defiled management EFFICIENCY
to an
accounting as “a system of collection and presentation of relevant economic information relating ne |

planning, controlling and decision making.” . :


enterprise for
authoritative
The Chartered Institute of Management Accountants (CIMA) of UK has given a very
and comprehensive definition as follows: Ce
presenting
“Management accounting is an integral part of management concerned with identifying,
ni ae erg

and interpreting information used for — Tey


MANAGEMENT
(} formulating strategy; ACCOUNTING
(ii) planning and controlling activities;
{iif) decision-making:
(iv) optimising the use of resources;
(v) disclosuze to sharéholders and others axtemal to the entity:
Fig., 1.4. Relationship of Financial, Cost and Management Accounting.
fvi} disclosuzé to ertiploypes; and ~*~
(vii) safeguarding assets.” Management Accountant is The Controlier
According to National Association of Accountants (USA}, management accounting is, “the
:
«process. of identification, ‘measurement, accumulation, analysis; preparation nimunication. of
and comitt ae or ‘Financial Controller’ ar-d is a part of the
In USA, the management accountant is called ‘Controller’
top marieae aiei nent team. .
"_ finencial information used by mafiagément to plan, evaluate and control within the organisation and to Try |
Management accountant plays a very important role in an organisation. He analyses and interprets
- assure dppropriate use and accountability for its resources.
accounting information and meets the informational needs of management at different levels. In’an
~ Th“These definitions*mak at that, management accounting plays avital role in providing.the afyaiiidation,‘a'ntanagenient accountant géneially performs a staff function i.¢. advisory role. But if he
necessary information to-managers in performing their functions of planning, controtling, organising and is:pemnitted-to participate in planning and decission making, he is a’part of the inaagement team and

Management Accounting
_, Mature and Scope of Management Accou
nting .
ise
thus becomes a part of the line function. It is very important that
status of the management
accountant in-the organisation is clearly defined so that the scope of his work submission to top Management, middle
and Tesponsibilities are order management and operating level
accordingly determined. on their requirements, management depending -
6. Cost conirot procedures,
Any system of management
CHARACTERISTICS OR NATURE.OF effective cost control procedures accounting is incempiete witho
MANAGEMENT ACCOUNTING like inventory control, labour ut
control, etc. contral, overhead cantral, budge
It is clear from the definitions of management accounting tary
that it is concerned with accounting
data that is useful in decision making. The main character 7. Statistical tools. Various tools of
istics of management -accounting are as analysing and presenting statistica
follows : charts, etc., are used in preparing report { data like graphs, tables,
; s for use by the management.
1. Useful in decision making, The essential aim of Managem 8. Internal control and internal
ent accounting is to assist management audit. Management accountant
ia decision making and control. It is concerned with all financiai controls like internal audit heavily depends on internal
such information which can prove useful to and internal check to piug loop heles
management in decision making. the concern. in the financial system of
2. Financial and cost accounting information, Basic accounti 9. Financial analysis and interpreta
ng information useful for Management tion, Management accountant emplo
accounting s derived from financial and cost accounting records, analyse and interpret financiat data to make it understand ys various “echaiques to
3, Internal use. information provided by management analysis helps management to achie abie and useable to the Management.
accounting is exclusively for use by ve objectives of Management in a Such
management for internal use. Such information is 10. Office services. Management accou more efficient matiner,
net to be given to parties external ta thebusiness ntant is expected to maintain and
like shareholders, creditors, banks, etc. procedures like filing, copying, conmm controt office routines and
unicating, electronic data processing
4 Purely optional, Management accounting is a purely and other allied services.
voluntary technique and there is no
statutory obligation. Its adoption by any firm dezends upon FUNCTIONS (OR OBJECTIVES)
its utility and desireability. OF MANAGEMENT ACCOUNTING
5. Concerned with future. As Management accounting is concernad with decisionmaking, it is Main functions of management accou
telated with future because decisions are taken for future course of nting ate as follows:
action and not the past. 1. Planning. Information and data
6. Flexibility in presentation of information, Unlike provided by management accounting
financial accounting, in Management forecast and prepare short-term and heips Managelent to
accounting thece are no prescribed formats for presentation long-term plans for the future activi
of information to management, The form of formulate corporate strategy. For ties of the business and
this purpose Management accounting
presentation of information is left to tre wisdom of the standard costing, marginal casting. proba techniques like budgeting,
Management accountant-who decides which is bility, correlation and regression, etc.,
the most useful format of providing the relevent informat 2. Coordinating. Management accou are used,
ion, depending upon the utility of each type nting techniques of planning also
of form and information. business activities. For example, while help in coordinating various
preparing budgets for various depar
purchases, etc., there should be full tments tike production, sales,
coordination so that there is no contr
SCOPE Teporting, management accounting adiction. By proper Financial
GF MANAGEMENT ACCOUNTING helps in achieving coordination in
accomplishing the set goals. various business activities and
Management accounting has a very wide scape. It includes ,
not only financial accounting and cost 3. Controlling, Controlling js a very impor
accounting but also all types of internal Financial controls, tant function of management and Mana
internal audit, tax accounting, office helps in’ controlling. performance by gement accounting
services, cost control and other methods and control procedur control techniques such as standard
inter alia includes the following:
es. Thus scope of management accounting, control ratios, internal audit, etc. cos' ting, budgetary centrol,
/ 4. Co munication. Management accou
1, Financial accounting. Financial accounting provides basic nting system prepares teports for
historical data which helps management . levels of management which show the presentation to various
to forecast and p'an its financial activities for the futtte period. performance of various sections
Thus for an effective and communication in the form of repor of the business, Such
‘Management accounting, there should be a proper and successful ts to various levels of manageme
well designed financial accounting system, control on vartous business activities nt helps to exercise effective
2, Cost accounting. Many of the cechniques of cost and successfully running the business.
contro} like standard costing and budgetary 5, Financial analysis ‘and interpreta
control and techniques of profit planning and decision tion. In arder to make accounting
-making like Matginal costing, CVP analysis and the management accounting offers data easily understandable,
"differential cost analysis are used by the Management accounti various techniques of analysing,
ng. data in rion-accounting language so that interpreting and piesenting this
"3. Budgeting and forecasting. In order to plan business every one
in the organisation understands j
activities for the future, forecasting and cash flow and funds flow statements, trend
budgeting play a very analysis, etc., are some of the ma hage
significant role. Forecasting helps in the preparation techniques which maybe used for finan ment accounting
of budgets and budgeting cial analy
. helps Management accountant in exercising budgetary control.
| 6. Qualitative information. Apart from monesis and interpretation,
.
. Povides. qualitative information which tary and quantitative data ; Manage
helps in taking better decisions, Qu ality ment
ge Tax“planzing. In order to take advantage accounting
of various provisions of tax laws, management
~, accounta
nt bas to depend pon tax accounting and planning to minimise its cand employees, tegal judgements, of goods, custoiners
tix abilities and save opinion Polls, togic, etc.;i-@2€ some
Of the examples of qualitative
mage, fun th sins. .
orting to management. For effective vo gt Tax policies, Management accounting
and timely: decisions, thefe should be a system of
“intelligent Jeporting to management. Both-routine and supervises ani coordinates the reports prepa system is responsiblé for ta
special ‘reports’ are prepared for red by various authoritie
, Lo. os 1mak ing! Correct decision-making is crucial to the s, ~
. ... &,8] Decision-mak
accouriting has¢ertain special-techniques Success of a busin
ee which help management in short- term and ess. Management
long-term decisions.
‘ '
'
6 Management Accounting | Nature and Scope af Management Accounting

For example, techniques like marginal costing, differential costing, discounted cash flow, et¢.,. help in- Basis Cost Accounting
decisions such as pricing of products, make or buy, discontinuance of a product
Management Accounting
line, capital: wa Pa
expenditure, etc. , 3. Techniques Various techniques used by cost Management accounting also uses
Tools and Techniques used in Management Accounting employed accounting include standard costing and all these techniques used in cost
variance analysis, marginal costing and accounting but in addition it also
Management accounting uses-a number of tools and techniques to help management in achieving
cost volume profit analysis, budgetary uses techniques like ratio anzlysis,
business goals. Some of the important tools and techniques are as follows:
control, uniform costing and inter-Sirm funds flow statement, statistical
1. Budgeting . comparison, etc, analysis, operations research and
2 . Standard costing and variance analysis.
certain techniques rom various
3 . Marginal costing and cost volume profit analys:s.
4 . Ratio analysis, Dranches of knowledge like
mathematics, scanomics, etc.,
5. Comparative financial statements.
whichsoever can help management
& . Differential cost analysis.
in its tasks,
. Funds flow statement.
3 . Cash flow statement. 4. Evolution Evolution of cost accounting is mainly Evolution of management
§. Responsibility accounting. due to the limitations of financial accounting is due to the limitations
1. Accounting for price level changes. accounting. of cast accounting, In fact,
11. Statistical and graphical techniques. Management accounting is an
12. Discounted cash flow. extension of the managerial aspects
13. Risk analysis. of cost accounting.
14. Learning curve, 5, Statutory
15. Value analysis. Maintenance of cost records nas been Management accounting is purely
requirements made compulsory in selected industries voluntary and its use depends upon
16. Work study, etc.
as notified by the Gavt. from time to its utility to management.
time.
COST ACCOUNTING AND MANAGEMENT ACCOUNTING — COMPARISON 5. Data b
—_ . i. ase it is based on data cerivéd from It is based on data derived from
An examination of the meaning and definitions of cost accounting and Management accounting -
indicates that the distinction between the two is quite vague. Many eminent writersteven consider 4 financial accounts. cost accounting, financial accoun-
ting and other sources.
these two areas as synonymous while others distinguish between the two. Horngren, a renowned authér 7. Status in
un the subject, has gone to the extent of saying, “Modern cost accounting is often called management 4 In the organisational set up, cost Management accountan: is generally
omanisation account-ant is placed at a lower level
accounting. Why ? Because cost accountants look at their organisation through managers eyes.” Thus .3 placed at a higher levei of hieraschy
Managerial aspects of cost accounting are inseparable from management accounting. One point on in hierarchy than the management than the cost aecountant.
- which all agree is that these two types of accounting do not have clear cut teiritorial:boundaries. accountant,
" However, distinction between cost accounting and management accounting niay ‘be made.on the Cost accounting system can be installed Management accounting cannot he
following points: withdut management accounting. installed without a proper system
of cost accounting.
Basis Cost Accounting Management Accounting
LIMITATIONS OF MANAGEMENT ACCOUNTING
“1. Scope Scope vasof cast accounting
:
is limited
:
to | Scope.
.
of “inanagement” accounting, 5
providing cost information for] is broader than thata \of- cost: vem oe ement ac belowtilting is a very useful took sf management. However,
as stated it suffers from certain
managerial uses, ‘accounting as it provides all ‘types Emita ow ;
#./ . 1, Based on historical data. Management accounting helps management in making decisions for
of information, f.e.. cost accounting
sthe future butit is mainly based on the historical data supplied by financial accounting and cost
as well as financial, accounting
counting. This implies that: historical data is used for making future decisions. The accuracy and
‘information for managenat Uses. dability of such data wil ll ledwe their mark on the -quility “of managerial decisions. In other:
2, Emphasis Main emphasis is on cost ascertainment | Main emphasis is on, pl ning s if ‘past data is not accurate, man agement decisions may not be correct:
. and ¢ost control to ensure maximum | controlling and ecisipy-m at 1) 2 Lack of wide nowledge. The management accountant should have knowledge of not only
profit. \ | maximise profit. .- ". al drid. cost bacé
acéotinting:
but. also many-allied subjects like economics, management, taxation,
fight and mathematical techniques etc. Lack. of knowledge of these subjects on the part of
agement accountant limits the quality of management accounting.
x
1.8
Management Accounting Nature and Scape of Management Accounting
oo rg
3. Complicated approach, Management accounting provides mass of data
using various acounting
and non-accounting subjects for decision making purpose. But COST REDUCTION
sometimes management avoids this
complicated and lengthy course of decision making and makes decisions based on Cost reduction is often confused with cost control. Cost
intuition. This leads reduction is much wider in scope and
to unscientific approach ta decision making. consists of effecting savings in cost by continuous research
for impyovement in products, methods,
4, Not a substitute of management. Management accounting only provides procedures and organisational practices. Cost reduction is defined
information to by C.LM.A. London as “the achievement
management for decision making but it is not a substitute of managemen of real and permanent reduction in the unit cost of goods
t and administration, manufactured or services rendered without
5. Costly system. The installation of management accounting system in an impairing their suitability for use intended.” This definition
organisation is a costly reveals the following characteristics of cost
affair as it requires a wide net-work of management information system, reduction:
rules and requlations. All this , ~
requires heavy investment and smatl concems may not he able to afford it.
_(i} Cost reduction must be real - sav. through increase
6. Developing stage. Management accounting is a relatively recent developme in Productivity, change in product
nt and it has not design, improvement in technology ete.
fully developed as yet. This limits the utility of this system to managemen
t in making perfect and {iI) Cost reduction must be permanent — temporary reductio
correct decisions, ns in cost due to windfalls, change
in tax rates, changes in market prices, etc., do not come in the
7, Lack of objectivity. The interpretation of information provided purview of cost reduction.
by management accounting (iit) Cost reduction must not impair the suitability of products
may be influenced by personal bias of the interpreter of data. or services for the intended use. In
This tells upon the quality of other words, cost reduction should not be at the cost of
Inanagerial decisions. assential characteristics of the
8. Resistance from staff. The existing accounting and manageme products or services,
nt staff may not welcome the
introduction of management accounting system. This may be because they The cast reduction is, therefore, the term used for planned and
positive approach to the improvement
look at the system with
suspicion thas it will add to their work and responsibilities. of efficiency. it can be viewed in many ways, such as increasing
productivity, elimination of waste,
improvement in product design, better technology and techniques,
incentive schemes, new layouts and
COST REDUCTION AND COST CONTROL better methods, etc. if the cost reductions are not based on sound
reasons, like improved methods,
then very quickly the costs will grow back to their oziginal size,
New-a-days businesses are operating in a globaity competitive
environment, where in order to
increase profits, increase in selling prices of products/services may not be possible Cost Control and Cost Reduction — Comparison
because of
fears of
shrinkage in volumes. Thus in order to Improve profit performance,
businesses have to be extremely Cost control and cost reduction are two effective tocls of cost Manageme
cost conscious and improve their performance in cost. In other words, in nt to improve efficiency.
this-age of competition, in Cost control and cost reduction are two separate phases of cost
order to survive and improve profitability, managers have to make improvement. Cost reduction begins
continuous efforts to find out ways where cost contro! ends. The main points of distinction between the
and Means to control and reduce costs. two are as follows:
:
Cost control { Cost reduction
COST CONTROL
1. Cost control is the achievement of pre-" 1. Cost reduction is the achievement
Cost con-rol is an essential component of any system of cost accounting. determined targets of costs.
of the
It is exercised ‘through teal and permanent reduction in costs,
comparing actual cests with pre determined standard costs so that the difference
between the two can 2. Cost control tends to assume a static state of | 2. Cost reduction
be measured end then analyzed according to reasons for taking assumes the existence of
corrective action. CIMA, Loridon’ has - affairs and that standards: once set are not
defined cost control as, “the regulation by executive action of the cost concealed potential savings in the standards ;
of operating an undertaking challenged. oF pre-determined costs set for cost contral
particularly where ‘action is guided by cost accounting”. Cost control
is thus simply thevutilisation of the and that these standards are always subject
available resocrces ecohomicaily and prevention of the wastage within the
existing environment. it is
the functioofn keeping costs within the prescribed limits, to challenge.
; : J. Cost con
is tro
concernedl
with predetermining| 3. Cost reduction is not concérned with
Steps in Cost Control : In designing a system of cost control, the following steps
are taken : costs, comparing it with actual costs, maintenance of performance according to
1, Establishing norms: The first step in cost contro} is to set analyzing the variances and taking corrective
norms or standards which may predetermined targets. It is rather concemed
serve as yardsticks for measuring periarmtance. These standards action,
or-norms are set on the basis of past : with finding out new product designs,
performance adjusted ‘for. changes in futuye and-on. the basis of stiidies conducted :
, a methods, ete,
2, Comperison’ with actual : The actual costs incured are compared with 4. Cost control is a preventive function as it}
established standard 4. Cost reduction’ is a corrective function
costs to. know.-he level-of achievement. The variations-are analysed aims to prevent the costs from exceeding the
so as to arrive at the causes which because it challenges the predetermined
are controllable:and,uncontiollable. predetermined targets.
"300 ‘ targets and seeks to improve performance
‘tion. :: Remedial. or corrective “measures are ‘taken to avoid. the
variations { iecunence of by correcting the targets.
revision OF staitdards; whelever necess ary, 15. Cost control is a part of cost accounting | 5. Cost reducti
. . Amongst® ised for cost ‘control, ‘the-two mest popular are budget on may be achieved even ‘when
ary control and function. , Ro cost accounting system is in operation.
standard ‘ths bose L . \ ft 6. Cost control lacks dynamic approach to cost 6. Cost
reduction is-a more dynamic approach
improvement. , : to cost improvement and elimination of
x
| waste.
"Management Accounting Nature and Scope of Management Accounting
Ld
Scope and Areas of Cost Reduction (a) Cost reduction by economising expenditure and increasing productivity, and
The scope of cost reduction is so wide that it is not practicable to develop fully the areas in _ {b) Cost reduction by improving the use value and esteem value of products.
‘Use value refers to
~~" “which cost reduction may be applied. Wherever costs are incurred, there is scope for their reduction 4 those qualities and characteristics which make a product useful
and esteem valua refers to
and the management should not feel that there is no room for cost reduction in any particular area. those properties which make it attractive and create in it an aesthetic value.
Effort should, therefore, be made te reduce costs right from the top level to the shop floor level. Value analysis attempts to reduce cost by operating af the latter method. Value analysis
may thus
However, in the following areas, scope of cost reduction is the jargest : be described as a systematic analysis ‘and evaluation of the techniques and functions in the various
1. Product design. The design of the product provides the greatest scope for cost reduction. spheres of an organisation with a view to exploring channels of performance improvement, sa that
the
Product design being the first step in production, if cost reduction can be made at design stage, then value in the preduct or service may be bettered. It is a technique which is used to analyse
all aspects
it is likely that the benefits can be availed to the maximum. of an existing product or service to determine the minimum cost necessary for specific
functional
There are tvs basic points that should be kept in mind while effecting cost reduction in product requirement. Basically, the idea is to weigh cost against value.
design :
Tools and Techniques of Cost Reduction and Cost Control
(e) The product should perform all the functions for witch it is intended, and
Various tools and technique used for cost reduction include the following :
(b) The product should retain its ‘esteem’ or ‘aesthetic’ value. This is in the case of many products
which have the shape or other characteristic which pleases the eye. 1. Standard costing
Improvement in product design may result in cost reduction as illustrated below: 2, Budgetary control
3. Inventory control
(!) Material cost—-Change in design of the product may result in saving in material cost. 4, Preduction planning and control
Economical substitution for existing material may also be considered. For example, in 5. Standardisation and simplification -
manufacturing kitchen utensils, brass may be substituted by cheaper alloys. In curtain rings, 6 . Operational research and statistical techniques
metal may be substituted by plastic. 7 . Value analysis -
(if) Labour cost— Improvement in design may result in reduced operating time. 8 . Automation
(iit) Factory overhead—Reduced operating time not only helps in saving in labour cost but also in 9, Design improvement
factory overhead. _ , 10. Market research
(iv) Packing and transportation—Compact design of a product results in reduced cost of packing 11. Job evaluation and merit rating
and transportation. , 12. Work study’
(v) Cost of tools, jigs and fixtures can be reduced through design improvement. 13. Organisation and methods study
2, Organisation. Cost reduction may also be achieved by improving factory organisation in the 14. Quality control.
form of clear-cut lines of authority and responsibility, well-defined channels of commumications, co-
ordination and co-operation between various executives, ete.
' COST MANAGEMENT
"3, Production. A cost reduction programme should make a study of sequence of operations to find
‘out the best one, to use the most suitable machines for the work, to use jigs and fixtures to reduce The present day business environment is significantly different from that of the yesteryears. Tha
‘operating time, to reduce idle time, to reduce scrap by the use of better quality tools, to provide better Tapid pace‘ of change is because of factors such as ~~ globatisation of markets, spectacular advances in
«working conditions conductive to efficiency, etc. vO | ‘information and manufacturing technology, deregulation, increasing emphasis on total quality
“. 4, Administration. Items under this head include savings effected by modifying the fange of cash management, etc. The emphasis on quality of products and productivity Tequires new -aeasures
of
discounts to customers, introducing mechanical and electronic aids to office routine, modifying internal control Cost accounting information. plays a very important role in determining the appropriate
jand external communication system,- ete. Strategic direction for the organisation and effective management. Of course, cost accounting methods
“- &, Marketing, In this function, costs can be reduced by revising the méthods of remuneration of and techniques are important but equally important is to understand how to use theta for
effective
lesmen, re-arrange territorial responsibilities of sales representatives, modifying current methods of decision jaking. Cost- management addresses these’ changing requirements. In the changed
business
vertising, improving product-design and production quality so as: to reduce after sales- service, = *Scenario, the managers have tumed towards cost managerient to find ways and means to continuousl
y
ecénomising channels of distribution, improving packing, etc. . weduce costs to improve product/service quality and at. the same time maintain Profitabitity,
6. Finance. A cast reduction programme should aim at securing capital at economical cost, Definition
@mploying capital to give maximum return and eliminating over and under capitalisation and wasteful ‘
es - Cost Tanagement is a new entrant in the terminology’éF management-andis widely used
mse ‘of capital, etc. ~ these
e days in the business world. Somehow, no uniform definition of this‘term Has aleveloped so -far,
Reduction and Value Analysis ; : _ In the words of Horngrer, cost management
is used “to describe the-ajprdaches and activities’ of
li analysis is a scientific approach to cost reduction. {t aims at cost reduction by increasing ‘managers‘in the short term and-long term planning and control decisions ‘that jnetedse
value for
value in a product. In fact, cost reduction may be effected in two ways : -Customeys.and:lewer costs-of products and services." vos
Management Accounting oe Nature and Scope of Management Accounting

According to Hansen and Moven, “Cost management identifies, collects, measures,. classifies
and 3 EXAMINATION QUESTIONS
reports information that is useful te managers in costing (determining what something costs),
planning,
controlling and decision making.”
It may thus be said that cost management is the practical application and use of cost accounting
Objective Type Questions
methods and techniques by the management to improve business performance, True and False ?
Information provided
hy cost accounting system helps managers make decisions regarding the amount and kind of materiats » Management accounting is a branch of accounting that produces
1 information for managers within an
used, changes of product designs, changes in plant processes, etc, ? organisation.
. Management accounting is based on double entry system.
Advantages : - Management accountant is also called controller.
The following are the advantages of cost management: 5 . Cost accounting is a part of management accounting.
. Management accountant must be a qualific ALO a Chartered
heey enn
Oud ntant or Management Acccuutant
1. Cost management helps in cost based strategie planning. {t provides critical information from the
that 3 Institute of Chartered Accountants of India.
manager needs to develop and implement successful strategies, In taking a strategic emphasis, 6+ Small firms may find it difficutt to afford
a system of Management accounting which is quite costly.
cost management looks to long term competitive success of the firm. 7. Management accounting system cannot be installed without a proper
system of cost accounting.
2, Cost management helps in improving factoral productivity ard profit Margin, ;. Management accounting is a branch of financial accounting.
3. Cost management ensures that planning and control of costs are directly linked with revenues
10 » Ratio analysis and funds flow statements are part of Management accounting.
and profit planning. For example, the effect of additional cost incurred on sates promotion » Main emphasis of management accounting is an cost ascertainm
ent and cost contyol ioy maximisation of
and product modification on the increase in revenue and profit is studied, profit,
. Cost management methods and practices are used to-help the firm in gaining success. [Ans. 1. True; 2. False; 3. True; 4. True; 5, False; 6. True: 7. True: 8
False; 9, True: 10. false}
. It improves understanding of processes and activities. The same cost may be analysed in
different ways to study its relationship with the activity. This helps in more effective planning. Essay Type Questions
Cost management is an integral part of the general management, planning and monitoring + Define management accounting and explain its “objectives.
mechanism and if is not practised in isolation, It can be effectively used by both production (B.Com Hons Delhi)
and . What are the functions of management accounting ?
service arganisations. Use of cost management can significantly benefit many organisations by increasing - What is managment accounting? In what respects it is different from financial
accounting?
resource productivity and margins and facilitating effective management of casts,
(8.Com Hons Deihi)
+ In what tespects cost accounting is different fram Management
accounting ?
Cost Management, Cost Accounting and Management Accounting . Discuss the role of management accountant in a modern organisatio
n,
Cost management has a much broader focus and encompasses both cost accounting and management: Explain the nature and scope of management accounting.
accounting. Cost accounting is concerned with ascertainment of costs of products/services, » Discuss management accounting as an effective tool of financial
But cost control.
management is not only concemed with how much something costs but also with the factors th » “There is an intimate relationship between management
accounting and finance functions.” Elucidate,
» “Management accounting is concemed with accounting informat
dtive costs such as quality of the product, productivity of factors, cycle time, etc. Thus cost ion which is useful to management.”
Explain. i
management needs a deeper understanding ofa firm's cost structure. The cost information provided by
> What is cost reduction? Give examples,
cost accounting plays an important support role for management in cost planting, controlling an 1, Distinguish between cost coytrol and cost reduction. (B.Com Hons., Delhi)
decision making. , fot - . What'is meant by cost foxtrot and cost reduction?
(B.Com. Hons Deihi; CA, P.E.-IT)
Management accounting is concerned with not only cost information but also with non-cost a . “Product design provides for‘ greatest scope for cost reduction”. (B.Com., Bangalore)
Discuss. (5.Com Hons Dethi)
other financial information that can be used for planning, controlling and decision: making. ‘14, Briefly describe the importarice of value analysis in cost reduction.
overall objective of mariagement accounting to make sure that organisation makes effective use’ 452 Wie a-note on cost management. (CA., Inter}
resources so that business objectives may be achieved. 16. What is: meant by cost -mahagement ? Discuss its advantages. (B.Com Hons Dethi)
x
‘W. Mention a few areas in which cost reduction is possible in a large manufacturing concer.
~~.. KEY TERMS 18, In a competition market,. profit can be sustained or increased
.
mainly by cost controt and cost reduction.
- 7 . " : : . . y Discuss and state the areas of cost reduction you would
Management. Accounting: It is concerned with accounting information that is useful like to explore. (LC WA, Inter)
to;
management. oo
Management Accountant: He is.an official who analyses and interprets accounting information’
t. . 4 - . wet

col valso_cajled controller of Gnancial controller.


redetermined targets of cost. be
d:penmanent reduction in-cost; 2 or. a
practical application and use of cost accounting hy the managem
ent-t
performance
oo, a Management Accounting
2.

Characteristics ~ Budgets have the following characteristics -


{a} A budget is ‘primarily a planning device but it also serves as a basis for
performance evaluation and control.
(b) A budget is prepared either in money terms or in quantitative terms oF in bath.

BUDGETING AND (c) A budget is prepared for a definite future period, ~


Purpose of a budget is to impletnent the policies formulated by managemen t for attaining
(d)

BUDGETARY CONTROL
the given objectives.

Budgeting
In the words of J. Batty, “the entire process
The act of preparing hudgets is called budgeting.
"of preparing the budgets is known as budgeting. “
Meaning and Definition of Budgetary Control
preparation of budgets. Budgeting
Budgetary control is a system of controlling costs through
ary control
is thus only a part of the budgetary control. According to C.I.M.A., London, “Budget and the
introduction; Meaning and definition of budget and budgetary control; Objectives; of executives of # policy
is the establishment of budgets relating to the responsibilities
Advantages; Limitations; Essentials of effective budgeting; Budget admmistration; either to secure by individual
continuous comparison of the actual with the budgeted results,
Functional budgets; Master budget; Fixed and flexible budgets; Revision of budgets; or to provide a basis for its revision .”
action the objective of the policy
ing costs
Budget reports; Zero base budget, Performance budgeting; Problems and solutions; In the words of Brown and Howard, “Budgetary control system Is a system of controll
establisting
the departments and
Examination questions. which includes the preparation of budgets, co-ordinating
responsi bilities , compari ng actual perform ance with the budgete d and acting upor results to
achieve maximum profitability.”
Introduction _Characteristics—The main characteristics of budgetary control are :
Business budgeting is the most widely used and highest rated management. tool of planning (a) Establishment of budgets fot each function/department of the organisatio
n.
and control. Planning is the key to gaod management as it involves looking systematically at: th {b) Compariso n of actual performan ce with the budgets on a continuou s basis.
performance to
future. Business budgets help managers in developing financial plan to guide their in allocatin (c} Analysis ‘of variations of actual performance from that the budgeted
their resources aver a specific future period. Controi is the process of measuring and corzectin know the reasons thereof:
actual performance to ensure that plans for implementing the chosen course of action are catrig {d} Taking ‘suitable remedial action, where necessary.
out, . u _{e) Revision of budgets in wiew of changes in conditions.
to those followed by the captain of
Re fe. principles involved in budgeting have been likened
Meaning and Definition of Budget account such factors as shipping
etary hip. Before the voyage, he will plan his route, taking into
Budget refers to a plan relating to a definite future period of time expressed.in, ritds, tides and possible advérse weather forecasts. During the journey, he
will record details
or quantitative terms. In relation to business, a budget is a formal expression of the axpect that planned. Though trying to keep to
Sarogress and frequently check actual progress with
_, imcomes and expenditures for a definite future period. The Chartered Institute.o plan; he! may have to. deviate from the plan if prevailing circumstan ces require it. On
i ‘Accountants (C.1.M.A.) London, has defined a budget as “a financial and/or quantita f he encounter ed with these he expected.;
iletion of the journey, he will compare the conditions
for
prepared prior to a defined period of time. of the policy to be pursued ‘during that period rience so gained will be used by ‘him in planning similar voyages in the future.
This
enptcyment
“Purpose of attaining a given objective.” It may include income, expenditure and used in budgeting and budgetary control,
pe ae le analogy serves to illustrate the basic practice businesses
capital. ‘la fé technique of budgetary control is now widely used in the business world. Many
In this words of Gorden Shitlinglaw, a Husiness budget is “a pre-determined detailed have reveated that the business should not
and as ‘a paltia basis Hecause of lack of efficient planning which could
action, developed and distributed as a guide to current operations ahead.
4 heen started or that one should have been prepared to face serious dangers
_ subsequent evaluation of performance.”
"According to Brawn:
and Howard, “A budget is a pre-determined statement of managem
policy duringa given period which provides a standard for comparison withthe results d¢
7 ‘important to ‘note carefully the distinction netween forecast and a budget.
It
2 8% Horecast sa. prediction of what may happen as a result of a given set of circumstances.
of probable futiire=eevents.
ssment
ot
. Bs ;
23 | 24
A budget, on the other hand, is a planned exerci Management Accounting
se to achiev
-and” cons of a forecast. Forecasting thus precedes the preparatione a target. It is based on the Pras _ 6. Performance evaluation. A budge
t provides a useful means of infor
of budget. they are performing in meeting ming managers how well
Thus the main points of distinction between the two are thus targets they have previ ously helped to set. In many
- there is a practice of tewarding companies
-
. 4 * promotion of a manager may be
employees on the basis of their
achieving the budget targets or
Budgets relate to ‘planned events’, i.e, policies and programmes to be pursued. linked to his budget achievement
Forecast is record.
concerned with probable events, Z., events expected to happen under anticipated
conditions. Advantages of Budgetary Control -
2. Budget, being a formal business plan, can be prepared
hut forecast ean be ma de by anybody. only by the anthorised management <q _ Budgetary contrat provides the following advantages
:
a! 1 Budgeting compels Managers to
3. Budget is a tool of contro! while the forecast issimply st think ahead—to anticipate and prepare
an anticipation of events. - "
= for changing conditions.
4, Forecasting is a pre-requisite for budgeting e it increases pedecto n ae tes 1
2 vrs i
while budgeting is not a pre- requisite for Teste cena function
] af ete
s ofthe boas,
business.
forecasting. 4. tt pinpoints efficiency or tack of it
5. Budgets relate to economic activities : 5. Budgetary control aims at Maximisation
of business, enterprises, government or of profits through careful planning
Forecast may reiate to economic as well as non others. 6. It provides a yardstick against which and control
economic activities, eg., weather actual results can be compared,
forecast, stock market forecast, etc. 7. It shows managem ent where action is needed to remedy a situation
8. It ensures that working capital is available
.
Objectives of Budgetary Control 9. It directs capital expenditure in for the afficient operation of the business.
the most profitable direction.
The-following are the main objectives of a budgetar : 10. It instills inte all levels of Managem
y control system. ;
ent a timely, careful and adequate
consideration of
1. Planning.A budget provides a detailed plan of action all factors before: reaching important decisions.
for a business over a definite period 11. A-budget motivates executives to attain the given
a
of time. Tatailed plans are drawn up relating
te production, sales, raw material tequirements, 12. Budgetary control syste m
goais.
labour needs, advertising and sales promotion perform creates necessary conditions for
ance, research and development activities, costing technique. the introduction of standard
capital additions, ete: Planning helps in anticipating 13. Budgeting also aids in abtaining bank’
many problems long before they may arise
and solutions car. be sought tsrough careful credit.
study. Thus most business emergencies can be 14. A budgetary controt system
avoided by planning, In brief, budgeting forces manage assists in deiegation of authority
ments to think ahead, to anticipate-and responsibility. and assignment of
prepare for the situation, 15. Budgeting creates cost consciousn
ess and introduces an attitude of mind
2. Co-ordination. Budgeting aids managers in co-ordi and efficiency, cannot thrive. in which waste
nating their efforts so that objectives of
the organisation as a whole harmonise with the objecti Limitations of Budgetary Contzol
ves of its divisions. Effective planning and
organising contrikute a lot in achieving co-ordination,
There should be co-ordination in the The list of advantages Given above is
budgets of various departments. For example, the impressive, but a budget is not a cure
budget of sales should be in co-ordination with ills. Budgetary control system suffers all for organisation
the budget of production. Similarly, the production from certain limitations and these using
budget should be prepared in co-ordination be fully aware of them. the system should
with ‘
the purchase ‘budget, and so on. The main limitations-are ;
me Te :
3. ‘Comm nication. A budget is a communication 1, The budget plan is based: on estim :
device. The approved budget : copies aq ates. Budgets are based on forecasts
distributed: to all management netsoniel which provide cannot be ah exact ‘science. Absolute and forecasting
s not only adequate understanding and:3 accuracy, therefore, is Rot possible
knowledge of the programmes and policies to be followed budgeting. The strength or weakness in forecasting and
but also alerts about the restrictions- of the budgetary control system depen
to be adhered to, on the accuracy with which’ estim ds to a large extent,
ates are
It ig Rot. thie’ budget itself that facilitates commu
. . “budget is 'based on estimates thust be kent made, Thus, while using the system, the fact that
nication, but the vital information “is in view.
comminicated’in'thte act ‘of preparing budgets and partici ' &. Danger of rigidity. A‘ budget
pation of al responsible individuals in programme must he dynamic and conti
nuously deal with the
thi “ae s achanging business conditions. Budge
ts will iose much af their usefulness
mo . . and are not: revised with the changing if they acquire rigidity
4. Motivation:'A budget is a useful device for motivat oo circumstances.
ing inanagers to perform in line with " 3, Budgeting is only-a tool of mana ,
the company. objectives. If individuals have actively gement, Budgeting cannot take the
participated in the prepaiation of -budgets, : but is onlya todl of management. place of management
if actsas a strong motivating force to achieve “The budget should be regarded not
the targets. ot / Servant.” Sometimes itis believed as a Master, but as a
“5, ‘Cont a, that intreduction of a budget progr
iControl is “necessary to ensure that 0 ehsure Its success, Execution ‘of a amme is alone sufficient
plans and objectives as laid down ‘in’ budget
chieved. Contrdt-as applied to budgeting, the
is a systematised effort to keep th tire organisation must participate enthu will not occur automatically. It is necessary that the
|*of whether: the- planried Performance is being achieve siastically in-the programme for the
d or not.. Forth Indgetaty goals, cf realisation of the
n is:made
between plans and actual
two-is ‘Yepottgd'to ‘the management for taking correct performancé. The difference between
the “Opposition from staff, Employees
may not like to be evaluated and thus
ive action, , f udgetary contrat system. ‘As"sitth,
inefficient managers may try to create
oppose introduction
difficulties in the
2.6 Management Accounting
25
Budgeting and Budgetary Control

of a budgetary control system isa BUDGET ADMINISTRATION


5. Expensive technique. The installation and operation e en
costly affair as it requires the employment of specialised Pre-requisites for the successful implementation of a budgetary control system are as follows:
Essentials of Budgeting
“staff and involves other expenditure which small concerns 1. Creation of budget centres. A budget centre is a section of the organisation of an
may find difficult to incur. However, it is essential that the 1. Support of top management, undertaking for which a separate budget is prepared. A budget centre may be a depariment or
cost of introducing and operating a budgetary control system 2. Participation by responsible a part thereof. Budget centre must be clearly defined because a separate budget has
to be set
"executives. in the
should not exceed the benefits derived therefrom.
. Reasonable goals. for each such centre with the help of the head of the department concerned. For example.
preparation of purchase budget, the purchase managet has to be consulted. Simitarly, while
. Clearly ‘efined organisation.

ew
ESSENTIALS OF EFFECTIVE BUDGETING preparing tabour cost budget, the personnel manager will be of great hetp.
_ Continuous budget education. be so designed
A budgetary control system can prove successful only 2. Introduction of adequate accounting records. The accounting system should

Ow
. Adequate accounting system. must also
the information required. The budget procedures
_when certain conditions and attitudes exist, absence of | . Constant vigilance. as to be able to record and analyse
Compansons
which will negate to a large extent the value of a budget empioy the same classification of revenues and expenses as the acecunting department.

mn
. Maximum profits. corresponding with the
system is any business. Such conditions and attitudes which . Cost of the system. cannot be made if the classifications do not coincide. A chart of accounts
ate essential for effective budgeting are as follows: . Integration with standard costs. budget centres should be maintained.

ow
for a successful
3. Preparation of an organisation chart. Proper organisation is essential

a
1. Support of top manageme nt. If the budget system ~
member of management and the impetus chart should be prepared which clearly shows the plan of the
is ta be successful, it must be fully supported by every budget system. An organisation
exact scope of his authority and
from the very top managemen t. No control system can be effective organisation. Each member of management should know the
and direction must come
copies of the organisation
unless the organisation is convinced that the top manageme nt considers the system to be responsibility and his relationship to other members. For this purpose,
budget idea as well as to the chart and written supplements should be distributed to alt concerned.
important. Thus the top management must be committed to the
system. . A specimen of
principles, policies and philosophy underlying the The organisation chart will depend upon the nature and size of the company.
ion by responsibl e executives . Those entrusted with the performance of the ‘the organisation chart is given in Fig 2.1
2. Participat
budgets should participate in the process of setting the budget figures. This will ensure proper
implementation of budget programmes. Managing Director
reasonably attainable
3. Reasonable goals. The buu,v: figures should be realistic and represent
goals, The responsible executives should agree that the budget goals are reasonable and attainable.
Budget Committee

4. Clearly defined organisation, In order to dezive maximum benefits from the budget
Tite
system, well defined responsibility centres should be buitt up within the organisation. . Budget Director
controllable casts for each responsibility centres should be separately shown. = ¥
5. Continuous budget education. The best way to ensure the active interest of the responsible ” y 4 4 t 4
+
af
supervisors is continuous budget education in respect of objectives,, potentials and techniques Sales Purchase 4 |Production Pezsonnel Chief Research &
preparation Manager coin Development
budgeting. This may be accomplished through written manuals, meetings, etc. whereby Manager Manager Ԥ | Manager
: Manager
of budgets, actual results achieved etc., may be discussed. a
6. Adequate accounting system. There is close relationship between budgeting and accounting. TT TT 4
For the preparation of budgets, one hag to depend on accounting department for jeliable historical ;
¥

Purchases fl « Production labour e Cash Research &


« Sales
data whick primarity forms the basis for many estimates. The accounting system. should be so 4] Budget Cost Budget Development
ps
designed so as to set up accounts in terms of areas of managerial responsibility. In other words,
Budget
« Selling e Plant Budget « Capital Budget
responsibility accounting is essential for successful budgetary control.
Utilization > Exp.
7. Constant vigilance. Reports comparing budget and actual results should be promptly Cost
Budget Budget
prepared and special attention focussed on significant exceptions—figures that are significantly Budget
«_ Distribution « Master
different from those expected. mo
Budget
8. Maximum profits. The ultimate object of realising the maximum profit should alivays be’; . Cost Budget
kept uppermost. e Advertising .f
» 9, Cost of the system. The budget system should not cost: more than it is worth. Since, it; Budget
is not practicable to calculate exactly what a budget system is worth, it only implies a caution : Fig 2.1 Organisation Chart for Budgetary Control.
against adding expensive refinemelts uniess their value clearly justifies them.
and execution of the
. 10. Integration with standard costing system. Where standard costing system js also used, 1 ; A Establishment of budget committee, In large concerns, the direction
management. The
“it should be completely integrated with the budget programme, in respect ofboth ‘budget Drdget is delegated to a budgef committee which reports directly to the top
preparation and variance analysis.
_ Budgeting and Budgetcry Contral
"7
oY
us
a7 2.8
Management Accounting
financial controller is usually appointed to serve
as. the budget director. He is incharge of
re rut rt ar err te

preparing budget. manual of instructions and accumu electricity companies which incur very heavy
lates the budget and. actual figures for capital expend
iture, the need for new power stations
reporting. Other members of the budget committee usually comprise various is forecast possibly five to ten years
heads of functional in advance. Such long term budgets are
departments, like sales manager,
purchase manager, production Manage short term ones. supplemented by
r, chief accountant, ete.
as shown in the above organisation chart. Each 7, Determination of the principal
member Prepares his own departmentat budget budget factor er key factor, Also
which are then considered by the committee for coordi (s} known as limiting
nation. factor ot governing factor, the key factor
Functions: The main functions of a budget means the factor which limits the size
is define d as “the of output. [&
committee are as follows - factor the extent of whose influence
must fust be assessed in order to
that functional budgets are capable of fulfil ensure
(a} %o provide historical data =o all departmental heads to help ment” Such a factor is of vital importance
them in estimating. all budgets to and affects
(5) Fo issue inscructions to departments regarding
a large extent.
requirements, dates
of submission of
estimates, etc, . The kev factor serves as the starting noint fo, th: reparation
of budgets. For instance, when
(c) 30 define the general policies of the Management in relatio sates potentiat is limited, sales is the key facts.
n to the budget system, Therefore, sales budget should be prepared first.
(d) To receive budget estimates from various departments for consid Production and other budgets will follow the sales
eration and teview, budget. Thus a key factor determines priorities
() To discuss d:ficutties with departmental heads and sugges in functional budgets. Among the various key
t possible revisions, Factors which affect budgeting are te following
(f) To evaluate aad revise the estimates before preparing :
the final budget. (2) Sales
{g) To make recommendations on budget matter
s where there is conflict between departments.
(h} To prepare budget summaries. ; (i) Low market demand
{?) To prepare a master budget after functional budget
s have been approved. (if) Shortage of experienced salesman.
(7) To inform departmental heads of any revisions (b) Materials
made in their budgets by the committee,
{k) ‘Tp coordinate all budget work.
4 {i} General shortage and seasonal shortage
(} To analyse variances and recommend corrective action,
where necessary. (i) Restrictions imposed by licences, quota, ete.
5. Preparation of budget manual
, A budget manual has been define (c) Labour
as “a document which sets out the responsibili d by C.LM.A., London 4
ties for the persons engaged in the routin {i} General shortage
the forms and records required for budget e of and 4
ary control.” A budget manual is thus (i} Shortage of specialised labour in a Particu
budget policies. It tays down the details of a statement of 3 lar process.
the organisational set up with’ duties and (d) Plant
of executives including the budget commi Tesponsibilities
ttee and budget director and the proced
Programmes to be followed for developing budget ures and (?) Limited! plant capacity
s for various activities. ‘‘ (if) Bottlenecks in certain key processes,
Contents: The contents of a budget manual are
summarised as follows : (e) Management
(a) Description-of the budget systere and its objectives.
(]) Shortage of experienced executives
(6) Procedure ard forms to be used in budget preparat
ion. {ii} Paucity of know-how.
{c) Responsibilities of operational executives,
budget committee and budget director. in this age.of competition; most often, sales is the key factor
(d) Budget calendar, specifying definite dates for the in industry.
completion of each part of the-budget and
" submission
af the teports. It is possible that more than one key factor
is Operating at the same time. Under such
(e) Method of accounti te conditions, the relative impact of such factors
ng and account codes in use, is considered in Sudget preparation. Moreover,
factor is not necessarily a permanent factor. The key
(f} Procedure to be adopted in operating the system. Management may be provided with opportunities
{g} Follow-up procedures. to avercome the limitations imposed by key factors.
For example, plant capacity can be incteased
. by the.instaliation of new and improved plant
&. Budget period. Budget period is a length and machinery which may be financed by the
of time for which a budget is prepared of new shares, issue
operated: Budget” periods: vary between and , :
short term
laid dewn-for all: budgets, ‘It. varies‘among concerns and tong term and no specific period can be
and industries for several factors. * FUNCTIONAL BUDGETS
A budgetis usually prepared for one year which corresp
onds to the accounting year, ft is then A functional budget is one which relates to a particu
sub-divi into. ded
quasters and.in turn each quarter is broken lar function of the business, e.g., Sales
When-a business éxpeziences ‘seasonal fluctuat down into three separate months. Budget, Production, Budget, Purchase Budget,
ions, the budget period, may be fixed etc. These are components of master budget. Specifi
seasonal cycle. Ifthe to‘caver one functional budgets to be prepared in a busines c
yn “cycle covers say two or three yea a rs,
long térm budget should s vary from organisation to organisation.
The
common types of functional budgets for a manufa
/Ehat-period:. The dong period may then be cturing ‘concern are discussed here in brief.
broken down into smaller periods
\e L + .o
Sales Budget
penditire.ate: usually prepared~on 1 long term
.

basis. Por exaraple, in In most companies, the sales budget is not


only the most impartant but also the most
ee budget! to prepare. The importance of this difficult
sa!
FL
3 + budget arises from the fact that if sales figure
:
5, is
'
2.9 Management Accounting
Budgeting and Budgetary Control 2.10

Types of Functional Budgets


“* “incorrect, then practically all other budgets witl be .
The principal considerations involved in budgeting production are :
- gffected. The difficulties in the pieparation of this (a) Sales budget. When sales is the principal budget factor, the production budget will be
1, Sales Budget.
“ Budget arise because it is not easy to estimate 2. Production Budget. based on the volume of sales forecast by the sales budget.
consumer demand, particularly when a new product is 3. Production Cost Budget. {b) Inventory policy. The management decision regarding quantities needed in stock at atl
introduced. times to meet customer requirements is an important factor. Ii deciding on the inventory
The sales budget is a statement of planned sales in 4. Raw Materials Budget.
policy, factors like storage facilities, length of the production period, perishabitity of product,
| 5. Purchases Budget.
terms of quantity and value. It forecasts what the tisk of price changes, etc. have to be given due consideration.
company can reasonably expect to sell to its customers 6. Labour Budget. :
7. Production Overhead Budget. fc) Production capacity. The production capacity of each department should be worked out
during the budget period. The sales budget can be and budget figures should be within these fimits.
a. Selling and Distribution Cost Budget.
prepared to show sales classified according to products, However, when production capacity falls short of sales requirements, the following alternatives
9. en Cost Budget _
salesmen, customers, tertitories and periods, ett. may be considered :
Y satel Eve diture Budget.
Factors: The factors to be considered in forecasting
(i) Purchase of additional plant and machinery.
sales are the following:
(ii) Introduction of additional shift.
i. Past sales. Anatysis of the past sales shows the (iff) Introduction of overtime working.
not difficult to suggest
trends to date and any seasonal or cyclical fluctuations. it is, therefore, (iv) Hiring machinery.
future trends fram the analysis of the past sales Sub-contracting production of components.
the market and thus, they may {v)
2. Reports by salesmen. The salesman are in close touch with role in
be required to prepare detailed estimates of sales that they are likely to make in their respective (d) Management policy. Production policy of the management plays an important
component part
areas during the budget pesiod. The report of each salesman should be studied in the light of budgeting production. For example, management may decide to buy a particular
his past assessment and actual sales. from outside instead of manufacturing it. This will influence production budget.
company and their effects
3. Company conditions. Any change in policies and methods of the Production Cost Budget
additional spending on advertising, introduction of
on sales should be considered. For example, (explained above)
new channels of distributions, introduction of new products, etc. should all have some effect of This budget shows the estimated cost of production. Tre production budget
s,of production are expiessed in terms of cost
a sales budget. shows the quantities of production. These quantitie
cost budget. The cost of production ig shown in detail in respect of material cost,
related business in production
4. Business conditions. Any changes in economic conditions and that in Production Budget,
obtained labour cost and factory overhead. Thus Production Cost Budgzt is based upon
activities and their effect on company sales should be considered. Information should\be Material Cost Budget, Labour Cost Budget and Factory Overhead Budget. |
and about the customer
about competing industries to assess the strength of competition
requirement to determine their demand. itlustration 2.1
external developments of Precision Tools Ltd. for
5, Special conditions. In the preparation of sales forecast, any hew The following information fas been made available from the records
ures products for
taking place should also be considered. For example, when an industry manufact stry. A tyre the six months of 2017 {and the sales of January 2018) in respect of product X;
it will be necessary to analyse the trend of sales in that-indu
another industry, fi) The units to be sold in different months are : .
on which tyres are used. _
manufacturer would estimate the sales of cars or scooters
s depend upon market analysis and reseatch to measure
July 2047 . 1,100 November 2017 2,500
6. Market analysis. Some companie August 2017 1,100 December 2017 2,300
demand for their products. Such an analysis reports on the state of the market, 2,000
‘the potential
and other ‘September 2017 1,700 January 2018
- i fashion trends, the type of products design required, activities of the competitors October 2017 1,900 .
‘Mfactors which may have a bearing on the sales of the company. (i) There will be no work-in-progress at the end of any month. .
(iii}Finished units equal to half the sales of the next month will bein stock at the end of
“Production Budget
drawn up in * every month (including June 2017).
{The production budget is a plan of production for the budget period. It is first 2017 are thus:
_ quantities of each product and when the remaining budgets have been compiled and cost of Kes, {iv) Budgeted production and production cost for the year ending 31st Dec.,
are translated into money terms, . ad Production (units) 22,000
-preduction calculated, then the quantities of production cast
Direct materials per unit z 10
Hat in effect: becomes a production cost budget. weet = 4
Direct wages per unit
“The production budget is the initial step in budgeting manufacturing operations. In addition | Total factory overhead apportioned to production < 88,000
ng activities of a
P roduction budget,. there are three other budgets telating to manufacturi You are required to prepare : _-
gmpany. ‘These are raw materials budget, labour budget and production overhead ‘budget. (a) Production Budget for the six months of 2017, and
{b) Surimarised Production Cost Budget for the same period.
(B.Com. Hons., Bethi)
.
Budgeting and Budgetary Control
Sos 24¥
Management Accounting
Solution
Production Budget (e) Purchase ofders placed ‘before the budget pericd against which ‘supplies will be received
for the six months ending Dec. 2037 during the peried under consideration,
(f} Policy of the management regarding materials or components
July Aug. Sep. Oct.
to be manufactured within the
Nov. "Dee. Total business as distinct from those purchased from outside.
finits Units Units Units Units Units Purposes: The main purposes of @ purchase budget are as follows
Estimated sales 1,100 1,100 1,700 3,900 —
:
2,500 2,300 (a) To enable the purchasing department to plan its purcha
Add; Closing stock 550 ses and enter into long term contracts,
856 95a 1,250 1,150 - 1,000 where advantageous.
rr te
~ 1,650 1,950 2,650 (b) Te record the material prices on whirh the pla represented by
dess: Opening stock
3,150 3,650 3,300 the budget is based,
550 S50 850 gu (c) To facilitate the management of finance of the business by
1,250 1,150 defining the cash requirements in
Production “4100 1,400 1.800
Tespect of the budget period and for shorter runs.
2,300 2.400 2.150 11,050
The purchase budget differs from the raw material budget in that
purchase budget specifies
Production Cost Budget hoth quantities and rupee costs, whereas raw material budget is usually limited to quantities
for the six months ending Dec. 2017 only. Secondly, purchase budget iacludes direct and indirect materials
_. .
, finished goods for resale,
Sore nee (Production:: 11,050
ORUCHION 11,050 units)
units) services like electricity and gas, etc. while caw material budget includes only direct material
requirements,
Direct materials e
@ © 10 for 11,050 units
Brrect wages 1,10,500
@ @ 4 for 11,050 units
44,200
Hlustration 2.2
"Factory overhead

sieht
@ € 4 for 11,050 units
44,200 The sales manager of Mahindra & Co, Ltd -eporis that next year he
Total Production Cost expects to sell 50,000 units
1,98,900 of a certain product. -
“Factory overhead per unit = % 88.000 = 22,000 The production manager consults the storekeeper and casts his Agures
units = 2 4 as follows: Two kinds
Raw Material Budget of raw materials A and B are required for manufacturing the praduct.
Each unit of the preduct
requires 2 kg of A and 3 kq of B, The estimated opening balances
This budget at the commencement of the
shows the estimated quantities af all the raw next year are—Finished Product, 10,000 units; 4, 12,000 kg: B 15,000
production demanded by the production budget. materiats and components needed for kg. The desirable closing

oe
Raw material budget: serves the following purposes: balances at the end of the next year are: Finished product, 14,000 units;
4, 13,000 kg; 8, 16,000 kg.
(a) It assists purchasing department in planning the purchase Draw up a Materials Purchases Budget for the next year.
s. : (B.Com. Hons. Delhi Adapted)
(5) It helps in the preparation of purchase budget.
Selution
(c) It provides data for raw materiat control.
Estimated
t production quantity. ‘ during the year is nat given,
it should be noted. that raw material budget it is calevlated as under.
generally deals with only the direct materia
Indirect materials and supplies are includ is, ... Sales during the year °
ed in the overhead cast budget. 50,000 units
Purchase Budget Add: Desired stock at the end of next year
14,000 units
Total ~ , 64,000 units
Careful planning of purchases offers one
of the Most significant areas of cost saving Less; Expected: stock at the beginning of the next year
companies, The purchase manager should be assigned the direct Tesponsi-bility in many 10,000 units
for preparing a , Estimated production
detailed plan of purchases for the budget 54,000 units
period and for submitting the plan in the
purchase ‘budget. form of a
, 5 . ” “Purchase Budget. for the period -..........
The purchase budget provides details of the
purchases which are planned to be madé during
the period to meet the needs of the busine ss. It indicates: "Item
(a) The quantities of each type of raw materi
: Material A Materiql B
al and other items to he purchased; kg kg
(8) The timing of purchases;
Consumption during the year :
_ (c} The estimated cost of material purchases. coe A—54,000 units @ 2 kq per unit 1,68,000 _
Factots: In preparing a purchase B—54,000 units:@ 3 kg per unit
budget, a number of factors must —
the following: ™ be considered, including
_ Add: — Desized stack at the end of next year
1,862,000
, : 13,000 16,000
(a) Opening,and closing stocks to be maintained as it will affect materi he wa!
(9). Maximum antl-minimum, stock quaati al Tequifements, 1,21,000 , 1,78,000
ties. \ Te a Less; Expected: stock at the commencement
(6) Econoinic order quantities. j of next,'yeat © © 12,000
(d)- Financial resources, available. 15,000
\. Quantities
of materials: to be purchased 1,09,000 1,63,000

ws
Budgeting and Budgetary Control 243 _ 2.14 Management Accounting
ag

Labour Budget Selling and Distribution Lost Budget


‘Labour cost is classified into direct and indirect. Some companies prepare a labour budget that a This is closely related to sales budget and represents the forecast of all costs incurred in selling
includes both direct and indirect labour, while others include only direct labour cost and include and distributing the company’s products during the budget period. As a general sule, the sales
inditect labour in-the overhead cost budget. budget and the selling and distribution cost budgets are prepared simultaneously, since each has
‘the labour budget represents the forecast of labour requirements to meet the demands of the a definite impact on the other.
company during the budget period. This budget must be linked with production budget and The sales manager is responsible for selling and distribution cost budget. He prepares this
production cost budget. The method of preparing labour budget is like this. The standard direct budget with the help of heads of sub-divisions of the sales depaytiment. Some companies prepare
labour hours of each grade of labour required for each unit of output and standard wage rate a separate advertising budget, particularly when spendings on advertising are quite heavy.
for each grade of labou: are ascertained. Multiplication of units of finished goods to be produced
Administration Cost Budget
by the labour cost pér unit gives the direct lahowr cost. The indirect labour is normally a fixed
This budget represents forecast of all administration expenses like directors’ fees, managing
amount, so should be easy to calculate in total for the period.
director's salary, office lighting, heating and air conditioning, etc. Most of these expenses are
Purposes: The labour budget serves the following purposes :
fixed, so should not be too difficult to forecast.
{a To estimate the labour cost of production.
{b) To determine the direct labour required in terms of labour hours and hence the number and Capital Expenditure Budget
grade of workers required to meet the production requirements. This budget represents the expenditure on all fixed assets during the budget period. It includes
{c) To provide the personnel department with personnel requirements so that it may plan such items as new buildings, machinery, land and intangible items like patents, etc.
recruitment activities. - Special Features: The capital expenditure budget has certain characteristic features which
(d} To provide data for determination of cash requirements for payment of wages. distinguish it from other functional budgets. These are :
{e) To provide data for managesial control of labour cost. (a) Capital expenditure budget deals with items not directly related to profit and loss account.
Expenses related to capital expenditure such as depreciation, repairs and maintenance, etc. are,
Production Overhead Budget however, corelated to this budget and they are included in overhead budgets.
After budgeting of material and labour cust, next logical step is to prepare a budget for {5} Capital expenditure is frequently planned a number of year in advance, perhaps five to ten
production overheads. The production overhead budget represents the forecast of all the production years, in which case it is broken down into convenient, periods like years or months. As compared
overhead (fixed, variable and semi-variable) to be incurred during the budget period. The fact to this: other functidnal budgets are normally prepared’ for a shorter period, say, one year.
that overheads include many dissimilar types of expenses creates considerable problems im {e) This budget involves large amount of expenditure which needs top management approval.
(a) The allocation of production overheads to products manufactured, and The capital expenditure budget is, therefore, subject to a strict management control.
(b} Contre! of production overheads. Cash Budget
The production overhead budget involves the preparation of overhead budgets for each department The cash budget is one of the most important and one of the last to be prepared. It is a
of the factory as it is desirable to have estimates of manufacturing overheads prepared by those ‘detailed estimate of cash receipts from all sources and cash payments for all purposes and the
individuals who have the responsibility for incurring them. The budget expenses for each sub- resultant cash balances during the budget period. it makes certain that the business has sufficient
period during the budget period should be indicated and the classification of expenses should cash available to.meet its needs as and when these. arise. It is a device for coordinating and
‘be the same as used by the accounting department. The budgeted overhead costs of service controlling: the financial side of the business to ensure solvency and provide a basis for planning
departments are totalled and apportioned to production departments according to the services anid financing Tequired to cover up any deficiency in cash. Cash budget thus plays an important
received by each such production department. The budgeted overhead costs of service departments Tole in the financial management of a business undertaking.
_are.totalled and apportioned to production departments according to the services received by
_ Purposes: The; ‘main purposes of cash budget are outlined below:
each such production department.
Factors: The factors to be considered in preparing this budget are as follows: (a) It ensures that sufficient cash is available when required.
~ {b) % indicates cash excesses and shortages so that action may be taken in time to invest any
{a) The classification of all overhead costs into fixed and variable elements. In ithe case of semi- excess cash orto borrow funds to sweet any shortages.
vatiable items, the degree of variability should be ascertained, . Phe level of output at which (e} Tt establishes a Soiintd basis for credit.
fixed costs change also be determined. (d) It'‘shows whether capital expenditure may be financed’ internally.
(2) The level of activity likely’te.be achieved during the budget petiod tke units of output, (ey Te establishes 2a sound basis for controt of cash position.
~ labour hours, etc. eparation of: fash!Badget. There are three methods of preparing cash budget:
(¢). Policy of management - regarding matters like overtime work, number of shifts tobe woiked {a) Receipts’ and|
depreciation, replacement of hand labour by machines, ete. “(b} Jadjusted Profit and Loss Method.
. (a): Individual items ofcost incurred in the past. {e)’ ‘Balance Sheet Method.
Budgeting and Budgetary Control ° 2.15
a
Management Accountin
g
. “aR a) Receip
| ipts and : Payments Method. This mathod is usually used for short term cash forecast (8) Adjusted Profit and Loss Meth
and is much more detailed than the other two methods. od
This method is suitable for long
i i with
begins i i
the opening balance o f cash in in Ihand and at bank. To this will term cash forecast. It is based
that is the source of cash
in the business. The profit on the view that it is the profi
a oth reach teccpte ‘from various sources and from this will be deducted a payments of as per profi t
into cash figure by preparin t and loss accounts is conv
_ hahwhe ‘her on capital or revenue account. The resultant figure is closing cash
cash, g an Adjusted Profit and erted
balance. expenditure (like depreciation Loss Account. All those item
, provisions etc.) which s of income and
ipts 1 itua
t situations i
arise les, collections from.debtors, interest on
from cash sales, are adjusted in the forecast do not involve an inflew or
ed Profit figure to arrive outflow of cash
i fain nts oad ‘ans, sale of capital assets and miscelianeous sources, In the “se ot credit profit. at the figure of cash made
available by
sales, adi
sales, adivetipent should be made for the time lag between the point of sale and realisation
(c) Balance Sheet Method.
This method is aise used for
on i purchases, direc’
far raw material i é labour,
a out ; of pocket expenses '
ket exp forecastin g cash requirements
similar to adjusted Profit and for long periods and
capita enciture projets : dividends. etc. The period of credit appropriate to the payment
tal ee ncitate loss account m ethod discusse is rather
balance sheet is prepared with d above. Under this method,
concerned should be taken into account. all items of assets and liabi a budgeted
The two sides of the balance lities excepting cash or bank
sheet ‘are the n totalled and balance.
tion 2.3 . If the liabilities are more than the balancing figure is take
assets, this reveals 2 bala n as cash,
exceed liabilities, this revea nce of cash and/or bank,
ame ny is ex ecting to have € 25,000 cash in hand on 1st April 2017 and it eaultes you ls a bank overd tafe. and if assets
t oO :prepar
spare cash hudget for the three months April to June 2017. The following information is
supplied to you: MASTER BUDGET
Sales Purchase Wages xpenses When all the functional budg
ets have been p teparec, thes
gz = z as a Master budget. Thus e are summarised into what
a master budget ‘3 @ cons is known
0 40,000 8,0 00 6,000 budgets. According to C.I.M olidated summary of all
.A., London, “Masier audu the functional
wach 0.000 56,000 component functional budgets et is a Summary budg
8,000 7,060 and which is finally azaroved et incorpareting its
heal 92,000 52,006 9,00¢ oon A master bud get has two parts , adopted and employed,”
(i) operating budget, t.22,
ie 1,00;600 60,000 10,000 8,0 (i) financial budget, i.e. budg budgeted profit and loss acco
eted balance sheet. Thus, unt, and
Han 1,20,600 55,000 12,000 9,000 a balance sheet together cons 2 projected profit and loss
\ e OM,
titute a master budget, account and
Other information : The master budget is’ prepared
(a) Period of credit allowed by supptiers is two months; by the budger director (or budg
the budget committee far appr et officer) and is presente
oval. If approved. it is subm d to
(h}25%
woo OFof Sale
sale is for cash and the period of credit allowed to customers for credit sale is one : approval. The Board may make
certai N amendments/aiteratio
itted to the Board of Dire
ctors for Anal -
anth; ns before it is finaliy appr
oved,
" {c} Delay in payment of wages and expenses one month; .
vas
(4) Income tax & 25,000 is to be paid in June 2017. . .. Madurai
een
FEXED AND FLEXIBLE BUD
GETS
Solution . -
) ‘;, Based-on level of ctiv
atid Flexible budget.
a ity of capacity utili
sation, budgets are classified
into fixed budget
Cash Budget for three months ending June 2017.
April May _ dune
fixed budget.is one whic
z zo iz budget “which is designed
h is Prepared keeping in mind
one level of output. It is
Opening balance 25,000 53,000 81,000 to. remain unchanged irre defined as
Receipts .:.
7 ft actual output differs from spective of the level of activ
budgeted level of output, ity attained ”
. ° 30,000 prepared variances will arise, Fixed
..23,000 25,00 0 i
uracy. This means that‘in thos can be estimated with
budget is
Debton | 60,000 69,000 75,000 e situations a fair degree of
where
Total
Ota: 83,000 94,000 - 1,05,000 ible Budget
aa .
| oe edit 40,000 50,000’ ~~ $2,000 contrast to. a fixed budg
et, a flexible budget is
“Siages, “8900 9,900 |. ~49,080 atigined,”* The underlying
one “wh toh is designed
to change in relation
can 9600" 7,600 > + 8,080 f uflittle use unless cost antli principle of flexible bud
rey emue are related to the ac get is that a budget
ann \- geting has been developed tual volume of Productio
25,000 _ with the chjective of chan n. Flexible .
i gin g the budget figures to cor
55,000 ._ 66,000 95,000 mo,
ey -CIMA, Lokdon terminol
respond
ogy.
53,000 81,000 91000
Management Accounting
_ Budgeting and Budgetary Control

-Regarding the behaviour of variable costs, it is important to note that total variable cost
with the actual output achieved. Thus a budget might be prepared for various levels of activit
increases in proportion to increase in the tevel of activity and vice versa. However, varichle cost
say, 70%, 80%, 90% and 100% capacity utilisation. Then whatever the level of output actuall
reached, it can be compazed with an appropriate level. per unit does not change with the change in level of activity.
Semi-variabie cost should be separated into fixed and variable components. Fixed cemponent
Flexible budgets are prepared in those companies where it is extremely difficult to forecast#
of the semi-variable cost will not change between levels but variable part of the semi-variable
output and sales with accuracy. Such a situation may arise in the following cases.
cost will change in the proportion of level of activity. This is explained in the following Mlustrations.
1. Where nature of business is such that sales are difficult to predict, eg., demand for luxy
goods is quite unpredictable. Illustration 2.4 . .
2. Where sales are affected by weather conditions, eg., soft drink industry, woollen garments, | Draw up a flexible budget for overhead expenses on the basis of the following data and
ete. : . . determine the overhead rates at 70%, 80% and 90% plant capacity. At 80% canacit
3. Where sales are affected by changes in fashion, eg., readymade garments. ‘s pacity
4, Wharze company frequently intwuuces ew products.
.
6. Where large part of output is intended for export.
; . . . . Variable overheads:
Uses of Flexible Budgets The figures in flexible budgets are adaptable to any given set of indirect labour 12,000
Stores including spares ‘000
operating conditions. It is, therefore, more zealistic than a fixed budget which is true only in ones
Semi-variable overheads : 20,000
Set Flexible budgetsconditions
of operating are also useful from control point of view. Actual performance of an executives Power (30% fixed, 70% variable)
variable} 300
Repairs and int nance (60% fixed, 40%.
should be compared with what he should have achieved in the actual circumstances and not with g e ,
Fixed read mainte fo , 40%
what he should have achieved under quite different circumstances.
SENG Demeciation 11.000
In brief, flexible budgets are more realistic, practical and useful. Fixed budgets, on the othe :
eprect -
hand, have a limited application and are suited only for items like fixed costs, insurance 3,000
Salaries - . 10.000
Distinction between Fixed and Flexible Budgets
The main points of distinction between the two are as follows : Total overheads } 62,000

1. Fixed budget assumes static business conditions whereas flexible budget is based on Estimated direct labour hours” / : _1,24,000 hrs,
assumption of changing business conditions. ; (B.Com., Maduras)
2, Fixed budget is prepared for only one level of activity but flexible budgets may be prep Solution
for different capacity levels ct for any level of activity. Flexible Budget for the period .......
3. Hxed budget figures are not changed when actual level of activity changes, But in
At 70% At 80% At 90%
budgets, the figures are adjusted according to the actual level of activity attained.
. - 7 capacity capacity capacity
4. When actual level of activity differs from budgeted level of activity, then in fixed bud
meaningful comparison between actual and budget figures is not possible. But in fl Variable overheads: z z z
budgets, such comparison are quite realistic. *" Gndirect labour 10,500 12,000 13.500
5. Under changing business environments, fixed budgets have very limited use for control. Stores including spates ' 3,500 4,000 4.500
flexible budgets are very useful for cost control and performance evaluation under chaz Semi-Variable : overheads:
business environments. “Power: Fixed . 6,000 6,000 6.000

: Preparation of Flexible Budgets . Repairs and M sintennee . 12,250 14,000 1.750


The preparation of flexible budgets necessitaces the analysis of all costs into fixed and vari Fixed - 1,200 1,200 1,200
~ components. This analysis, of course, not peculiar to flexiblé budgeting, is more importami , Varjabie 700 800 S00
> flexible budgeting than in fixed budgeting. This is so because in flexible budgeting, varying le Fixed overheads:
of output are considered and each class of overhead will be different for each level. In flex oe + Depreciation . 11,000 11,000 11,000
budgeting, a series of budgets are prepared for every major level of activity so that whatever! Insurance 7,000 3,000 3,000
“ aétial level of output, it can be compared with appropriate budget or can be interpd Salaries . 40,000. 10,000 __18,000
between budgets of the activity tevels on either side. For example, budgets may be prepa A) Total overhead 58,150 " 62,000 65,350
ay, 50%, 70%, 80%, 90% and 100% levels of activity. If the actual level of activity is 85%, Estimated direct labour ‘hours Tios500 Ya 1i9 30077
budget allowance for 85% activity should be computed. it lou hour Fate: “(h :B) z 0.536 0.500 0492
Hie computing fixed cost at various levels, it is to be noted that fixed cost in total aj
iréiiains unchanged at various levels of activity. However: fixed cost per unit decreases whe
utput i increases and vice versa, i.e., fixed cost per unit increases when level of activity de
2
SESS oer
marge Stila
Budgeting and Budgetary Control
220°
Management Accounting

is
Working Notes :
_oin flexible budgets, the following important points should be noted -
70 1. Total fixed casts for each level remains unchanged.
1. Indirect labour cest at 70% =_ 12,000.x 7 = 10,500
2. Per unit fixed cost decreases when level of output
increases and vice versa,
90 3. Total variable cost increases in Proportion
at 90% = 12,000 x a Z 13,500 to increase in the ievel of output and.
4, Per unit variable cost remains unchanged at each level.
Similar calculation for other variable item, i.¢., stores.
2. Power - Fixed = 6,000, Variable = % 14,900.
REVISION OF BUDGETS
7U
Variable powar at 70% = 14,000» s> = % 12,250 Someti
30 " mes the original budget prepared may have to be revised due te one or more of
following factors. the

at 90% = 14,000 x #0 = 2 15,750
1, Changes in management policies and other internat factors
Similar caleutation utilisation or addition.to the like change in the capacity
for repairs and maintenance production capacity, ete.
70 2. Unforeseen changes in uncontrollable or
3. Direct labour hours at 70% = 1,24,000
external factors like change in market prices
x 80° 1,08,500 of materials and other inputs, changes in fashions ang
consumer tastes, etc.
90 3. Errors committed in the preparation of Original budge:
al 90% 1,24,000 » ~~80 = 1,39,500 While preparing a revised budget, changes in all the
Hv

factors requiring consideration should be


illustrations 2.5 taken into account. The method of preparing a revised
budget may be similar to preparing a
flexible budget so tong as it pertains to changes in level
Ke a flexible budget for production at 80 per cent and 100 per cent of output or capacity utilisation,
, activity on the basis of the
oll owing informatio: n : : Mustration 2.6
,
oducts and budgets ar 60% ivi
‘ Raw materials ~ % 80 per unit
Direct labour 50 per unit
Direct Experses & 1s ger unit Product A Product B
; :
Factory Expenses € 50,000 (50% fixed) Raw material cost per unit
Administration expenses to v7.50 - 3.50 -
* 60,000 (60% variable}. (B.Com, Bangalo Direct wages per unit
Solution — = 4.00 3.00
‘ . - Variable overhizad per unit
Flexible Budget for the periad........... serreeees
z 2.00 1.50
_Fixed overhead per unit, . = 6.00
80% Capacity 4.50
100% Capacity Selling price per unit -
8,000 units 10,000 units % 20.00 _
15.00
Cost Production and sales (units): 4,000
Per unit Total Per unit ’ 6,000
Total _ The managing director is not satisfied with the budgeted
z z = results as stated above and wants
Raw materials
z improve thé performance. The managing director proposed that
86.00 6,40,060 80,00 8,00,000
the sales quantities of products
Direct labour and B could be increased-by 50% provided the selling
50.00 4,00,000 50.00 price was reduced by 5% in the case of
Direct expenses uct A and 10% in the,case of product B. The price reduction should be made applicab
15.00 1,20,000 “15.00 le to
Prime cost entire quantity of sates of each of the two products,
145.00~--} 11,60,000 ~ 145,00 are required to present the overall profitability under
Factory expenses- the original budget and revised
Variable ‘ budget after taking the increased sales into consideration,
5.00 - 40000 5.06 2 (C.A., Inter)
Fixed . os
3.125 25,000 |: "2.50
Works cost “153.125 12,25,000 | 152.50
Administration expenses: ae ,
‘Variable 2, /
7.200 ° 57,600 | 7.20
Fixed . _ 3.00 24,000 "2.40 ,
Total cost ts :
163 .325 13,06,600° |” “162,10 * x “
; _ Be Be Management Accounting
_ Budgeting and Budgetary Control
. ‘Solution Form: of Budget Report
Original and Revised Budget “The budget reports may be presented in the form of financial Statements or diagram as
for the year 201 illustrated below:
Original Budget Revised Budget
Budget Report
:
A B Total A B Total c , Period | oe.
Budget Centre ..... nan
Sales funits) 4,000 6,000 6,000 9,000
z z code item of Budget Actual Variance
t g z z
g No. Expenses ¢ zg Adverse | Favourable Reason 7
1,709,000 | 114,000 1,21,5002,38,500_ 2 z
(A} Sales {value} 80,000 90,000
Costs &
30,000 21,000 51,000 45,000 . 31, 76,500
Raw material
16,000 18,000 34,000 24,000 27,000 51,000
Labour
Variable overhead 8,000 9,000 17,000 12,000 13,500 25,500
Fixed overhead 24,000 27,000 * 51,000 24,000 27,000 51,000
A) Total cost 78,000 75,000 1.53,000 , 105,000 99,000 2,04,000 dG

Profit (i + B) 2,000 15,000 17,000 $400 22,500 41,500 Fig 2.3 Budget Report in Statement Form

Note : Revised sales figures are computed as follows :


Working
A B
g4
Selling price per unit 20 15

Less: 5% and 10% ti 1.50. 3 (= Budget


. 2% Actual
ee Add
3
Sales value A = 6,000 units x © 19 = 2 1,14,000
5
B = 9,000 units x % 13.50 = % 121,500

BUDGET REPORTS
flow of budget reports; Jan. -. Feb. March April May June
Establishing budgets in itself is of no use unless there is a continuous
n of actual and budget figures. Budget reports should be prepared at regular fi ‘ Month
showing compariso
and budget
intervals (say, every month) showing the reasons for the differences between actual Fig 2.4 Budget Report in Bar Diagram Form
‘the responsibility for
figures. The reports should be prepared in such a way that they establish
le ‘aant,
‘the variances. Reports should also reveal whether a variance is favourable or unfavaurab
dlso whether a variance is controllable or uncontroll able. ZERO BASE BUDGETING (ZBB)
leve
‘ the contents of the budget report differ according to the need of. managerial ZEB; is a,‘recent development in the area of management control system and is steadily gaining
provided with detailed reports of such act
‘example, lower level of management is generally yaportance in the business world. Before preparing a budget, a base is determined fram which
with which the manager is directly concerned. Thus a forettan will:be concerned ‘with*repa dget process begins. Quite often current year's budget is taken as the base or the starting
of detail
concerning his own section. As the level of management grows higher, the amount Bintint “for ‘preparing the next year's budget. The figures in the base aré-changed as per the plan
_ becomes less although the coverage of the report will widen.
the next _year, This, approach of preparing a budget is called incremental budgeting since the
° i ” Essentials of a Budget *epers*The following essentials should be kept in mind while prepari dget. process is concemed ihainly with the increases orc] changes i in operations that are likely
i pudget reports : écur during the budget period. For example, sales-of the current’ year’s budget may be taken
us
~~ (a) The budget reports: should be simpie and suitable for the level of undentanding for the : 1e base and next year's budget for sales will be curtent year's’sales plus.an allowance for price

>. (b)-Reports should be presented promotty. dases; fand expected changes in sales volumes, The main drawback of this approach | is that it
“) Reports should be accurate. but the extreme accuracy should not be at the cost of romp ie i. the pasti inefficiencies.
~ (d) The principte1 of exception should be utilised, where possi.
(e) The teports should contain only essential information according 40 the needs of the us

Budgeting and Budgetary Contra! 2.23 2.24 .
Te
Management Accounting

/ budgetingi (ZBB) isi an alterna’ tive nent budgeting.


i to incremental
i i ZBB was inintroduced
C at 2. ZBB discards the attitude
Phyr, who is known as the father of ZBB, it is not of accepting the current posit
ns Inari ants in SA in 1969 by Peter questioning and challenging each item ion in favour of an attitude
of budget. of
beced on inarementat approach and previous year's figures are not taken as the base ior preparing 3, In the course of ZBB process, ineff
ic mean icient and lo ss making operations
neat yes a et, Instead, the hudget figures are developed with zero as the vase removed. are identified and may be
thatat >a budget wil be > prepared as if it is being prepared for a new company for the firs . 4. It adds psycholegical push to emplo
yees to avoid wasteful expenditure,
i
has defined ZBB as “ “a planningi an d budgeting i process whichi requizes
i eac A manager 5. HE is an educational process:
and can promote a management
eter nes tye budget request in detail from scratch {hence zero base). Fach manager States People who tend to Promptly respo team of talented and skillfut
nd to changes in the business envi
een wld s end any money at all. This approach requires that all activities be identifie as 6. Cost behaviour patterns are more ronments.
closely examined.
Wea ie ° ackages which will be evaluated by systematic analysis ranked in order of importance. ~ 7, Deliberatety inflated budget reque
sts get automatically weeded
cut in the 2BB process.
ee to C.I.M.A., London, ZBB is defined as “a method of budgeting whereby allarenes Disadvantages: Despite being a usef
ul technique, 2BB suffers
are evaluated
rey each ‘time a budget is : set. Discrete
‘ levels of each activity are valued and a disadvantages: from the following
inatt
combination chosen to match fun ds available. . 7 1. 2BB leads to an enormous incre
ase in paper work ang results in
budgets every year. high cost of preparing
imple words, ZBB is a system whereby each budget item, regardless of whether i is neve
wxisting ust be justified in its entirety each time a new budget is prepared. it isa orm ; 2. Managers may resist new ideas
and changes. They may feel threa
expenditures are questioned and need tened by ZBB because all
st vem ofo budgeting for the a activities
system ar of an enterprise as if each activity were being performe to be justified,
i i {
Le, zero
fram zero base. | 3. In ZBB, there is danger of empha
he first time, sising short-term gains at the expen
4. It has a tendency to regard any se of long-term benefits.
he rel part of the ZBB is the requirement that the budgeting process starts at “rw activity not foreseen and Sanctioned
as illegitimate. 'in the mast recent 2BB
ion cnditores to be completely justified. This contrasts with the usual approach, in whic 3
5. For introducing ZBB, Managers need
ne tain lev u el of expenditure is allowed as a starting point
cer and the budgeting process focuses to be given pro per training and educ
hew concept, its pros and cons and imple ation regarding this
requests for incremental expenditures. . mentation
6. It may not always be easy to proper
l y rank decision packages and
’ B, budget requests for appropriation are accepted on the basis of cost/benefit approach conflicts, this May give rise te
sche res value for money. it questions lang standing assumptions and Sustematicaly exacines
ana ree“hapsPineabandons
find no any
placeunproductive projects. This means that those of the activites wich
in the ‘forthcoming PROGRAM AND PERFORMANCE
budget even though these might ave ween BUDGETING
n
Program and performance budgeting
integoral part of the past budget prepared under the traditional approach.
int ' 2BB in | a way systems are contrary to conventionai
budgeting was first developed by Lyndo budgeting. Pregram
n Johnson when he was the president
locate those activities which are not essential. Program budget describes and gives detail of USA during 2963 to 1969,
ed cost of every activityt or program
given budget. The sum of all activities or that is carried out with a
Main Features of Zero Base Budgeting (ZBB) programs constitu te performance budg
et. Performance budgat is
a budget’ that reflects the input of resources
The main features of 2BB are as under : and the outpu t of services for each unit
of the organization.
Tn case of traditional budgeting, both
input and output are mostly measured
1, All budget items, both old and newly proposed, are considered totally afresh. performance budgeting lays emphasis in monetary unit while
on achievement of physicat tar
2. Amount to be spent on each budget item isto be totally justified. ‘ established in such a mannex that each item of expenditure related to a
: e
3. A detailed cost benefit analysis of each budget programme is undertaken and each programm
‘haste compete for scarce Tesources. .
te ,
De objectives are linked to corporate goal : Se
e the ate sede ‘h Tot in’ Tow much" a department will spend but on why it needs to spend . This type of budgeting is commonly
used by governments to show the
;, Provided to activities and the outco link between the funds
& Managers
‘6: Manaoers at all levels particip
ticipate in 2BB process and they have corresponding | account
accountabilities me thereof. The Government of India
has also decided to introduce the
performance budgeting in all its depar
ZEB is now-a-days widely used, In Fact, when Jimmy Carter became a President ‘ im ‘ tments in a phased manner, An exam}
Ls ‘government. agencies-adopt ZBB. On a Teview 6 er
tha all nme ncies adopt iew of literature on the ’ !
7BD. Fiend that i taky’ organisations, ZBB has led to- a, considerable improvement in the

¢ budget are justified on cost benefit considerations whict


ore effective allocation of resources.
2.25 2.26
. : Budgeting and Budgetary Control Hanagement Accounting

or Planning, .
Program and performance budgeting is sometimes called Prograrime Budgeting —- Variable 500 600 700
Prograinme and Budget System (PP8S}. Fixed: Gverhead :
Depreciation 16,500 16,508 16,500
Steps in Performance Budgeting Insurance 4,500 4,500 4,500
centre. First
irst of ofall, respo
all, responsi re are established. A
bility centres ’ Salayies 15,000
‘ i
blishme nt of responsibility
ibili 15,000 15,000
responsi centre is a segment of ar organis ation where an individu al manager 7s held responsible for Total Overhead (A} | 85.900 ~~ 93,000 i
- .
the performance of the segment. Estimated Disect Labour Hours (B) 1,55,000° 1,86,000 2,147,000"
each responsi bility centre, targets are set in terms of
. Establishment of performance targets. For Direct Labour Hour Rate (A + B} 0.554 0.500 0.461
be achieved . For example , for sales departm ent, which is a responsibility centre,
physical performance to
: budget period. For production
targets may be set in terms o F number of units to be sold-during the a *Working Note
d.
department the target would then be the number af units te be produce
3, Estimating financial requirements. In this step, the financial support needed to achieve the 50%
1. Direct labour hours at-50%-= 1,86,000 x
physical targets is estimated. In othe: words, the amount of expenditure involved under various heads to 60%
‘Oo
7 1,55,000 hrs.
7
hysicat performance is forecasted.
ree cone vison of actual with budgeted performance. This is a usual step in budgetary control to 70%
at 70% = 1,866,000 x = 2,17,009 hrs.
evaluate the actual performance. 60%

6 Reporting and_action. Variances from budgeted performance are analysed and reported for
Problem 2.2 , . _
corrective action to be taken.
The following data are available in a manefacturing company for a yearly period:
PROBLEMS AND SOLUTIONS
Fixed expenses : 2 lakhs
lem 2.1
for the overheads of Damyanti itd. from the following data and _ Wages and salaries \ 9.5
are’ flexible budget
: Rent, rates and taxes ~~ 6.6
értaifé the overhead rates based on direct labour hrs, at 50%, 60% and 70% capacity . Depreciation . . .. 74
3
At 60% capacity
Sundry administration expenses . : 6.5
: Variable overheads :
: oon Semi-variabte’ expenses (At 50% of capacity):
Indirect material Maintenance and repairs 3.5
; :
Indirect labour Indirect labour =~ 79
.
Semi-variable overheads : Sales department salaries, etc. 3.8
: OO
Electricity (40% fixed, 60% variable) Sundry administration salaries 2.8
0
Repair (80% fixed, 20% variable)
= 16.500 Variable expenses (At 50% of capacity):
Fixed overheads :
: ee Materials 217
Depreciation
wees Labour : 20.4
Tnsurance
, Other expenses 79
Salaries
Estimated direct labour Rours 1,86,000 hrs. (B.Com. Hons. Dethi Total cost "$8.0,
ee
Assuine that the fixed expenses remain constant for all levels of production; semi-variable
: ti . ; expenses remain constant between 45 per cent and 65 per cent of capacity, inereasing by 10 per
Sotution Flexible Budget for the period........... ecent between 65 per cent and 80 per cent capacity and by 20 per cent between 80 per cent and
50% 60% 00 per cent capacity.’
z F : Sales at various levels are :
Variable Overhead + 000
: 6
«000 700
S0%:capacily Z 100 Lakhs 75% capacity @ 150 Lakhs
‘ i terials m 60% capacity % 120 Lakhs
15,000 18,000 21,00 90% ‘capacity % 180 Lakhs
: indiect tabour 52.08 100% capacity % 200 Lakhs
- Semi Variable Overhead
12,000 12,000. mn ! Srépate a flexible budget.for the year and forecast the profit at 60 per-cent, 75°par cent, 90
“Electricity— Fixed
VWVariable 15,000 48,000 wa
240 cent and 106 per cent of capacity,
ON ‘(B° Com. Hons. Dethi)
Repairs -~ Fixed 2.406 2,400 .


, Budgeting and Budgetary , Contraf oe . 2.27 2.28
Management Accounting
Solution ” Solution
Flexible Budget for the periad ... Flexible Budget for the periad.....
(Figures in & Lakhs)
6,000 units 8.000 units
Capacity Levels — 50% 60% 75% 90% 100%
10,000 units
Per unit Total Ber unit
- (A) 5 ales
les ” 100 120 150 180 200 Total Per unit Total
z # z | z z x
Fixed expenses: . 45 9.5 Materials 70 | 420,000 |
Wages and salaries 23 i ee ° ee 7 | 5.60,000 70 | 7.00900
Labour 25 1,30.000 |
Rent, fates and taxes 5. . : 5.4 . 25 2.00,000 25 | 2,56,000
Depreciation 74 ie i i ie Direct exp, (variable) 5 30,000
TA. 74 5 | 40,000 5 50,000
Sundry adm. expenses 6.5 : : . Variable overhead 20 1,20,000 | 20 1,60,000 20 | 2,0¢,000
Fixed overhead 16.67 4,00.000 |
{B}) Total fixed cost 30.0 30.0 30.0: 36.0 30.0 12.50 | 1,00,000 10 | 1,900,000
Selling exp : Fixed 2.17 13.000 | 1.63 | 13,000 1,30 13,000
Semi-variable expenses: Variable 11.70 70,200 |
Maintenance and repairsi 3.5 ’ ; :; i
4.2 Ae
4.2
1.70 93.600 11.70 1,17,000
Distribution expenses:
Indirect labour 7.9 oe te 2 ; -
Fixed 2.33 14,000 175
Sales dept. salaries 3.8 a v s3 - * 14.900 1.40 14,000
Variable 5.60 33,600 |
Sundry adm. expenses 2.8 - . . 5.60 | 44,800 5.60 56.000
Adm, exp, : Fixed 8.33 50,000 | 6.25 |
{C} Total semi-variable cost 18.0 18.6 19.8 21.6
$0,000 5.00 50,000
21.6 Tatal Cost 166.80 | 10,00,800) 159.42 | 42,75,400 195.00 | 15,50,000
Variable expanses:
Working Notes :
Material 217 26.0 32.5 39.0 43.4
vabow
Lavo 20.4 . 24.5 30.6 36.7
é 40.8 . Material, labour, direct expenses and variable

me
overhead are variable costs and do not change
Other expenses 7.9 9.5 11.9 14.3 15.8 unit. Total amount changes in Proportion to number pez
of units produced.
. Fixed overhead total amount remains at

™,
= 1,0¢,000 at all levels of output, Per unit
(D} Tota? variable cost . 50.0 60.0 75.0 90. 0 100.0 overhead is F 1,00,000 divided by the number fixed
of units produced,
3, Adm. expenses are also fixed. It is calcula
(FE) Total cost [{B) + {C} + (D)] 98.0 108.0 124.8 141-6: 151.6 ted in the same manner as fixed overhead.
4. Selling expense s are 10% fixed when output is 10,000 units, ie,
Profit (A - E) 2.0 12.0 * 25 25.2 38.4 48.4 ',
% 23,000 (% 1.30 ~ 10,000
uplits). Variable selling expenses per unit are
90% of 13, ie, % 11.70. Total fixed cost of F
: 7 13,000 remains the same at each level
and per unit is calculated by dividing 13,000 by the
Problem 2.3 . _ . aa . " number of units at eaci level. Vartable selling expense
per unit is ¥ 11.70 which remains the
same at each level.-Total variable selling expense
To expenses budgeted for production of 10,000 units in a factory are furnished ‘below : s are calculated by, multiplying % 11.79 by the
- number of
units at each activity level:
\Ww & per unit. ‘
5. Distribution expenses are calculated in the same
Materials
i 70
way as selling expenses.
In the! above flexible budgets, the following importa
nt points should be noted :
Labour L A / aa
1. Total fixed costs for each level remains unchanged.
Variable overheads 2. Pet unit fixed cost decreases when level of output
increases,
Fixed overheads @ 1,00,000) , 0 3. Total variable cost increases in proportion to increase
in the level of output.
Varlable expenses (direct) 2 , . 4 Per unit variab ‘cost remains unchanged at each
level,
Selling expenses {10% fixed) 13 Problem se
Distribution expenses (20% fixed) te 7 A department of -Alstom India Company attains
Adniinistration expenses (50,000) oo capacity.. Its“expenses. are given below
sales of © 6,00,000 at 80% of its normal
:
coe ea - Total. . 455.2: .
.
Prepiate a'bidget: for the; production. of (a) 8,000 units, and (6) $6,000 units. eo e
Office salaries. .
-Assimie'that adiniristration expenses are rigid fér all levels of production. General expensés
90,000
{ 4 fo me i . - .
(BBM Bangalo‘e; ¢.A. inter)! Depreciation
2% of sales
! 3 an
7,500
_ Budgeting and Budgetary Controt 2290 2.30 Management Accounting

8,750 . Problem 2.5 '


‘Rent and rates
the accounting
Selling Cost: ’ The budget manager of Jupiter Electricals Limited is preparing flexible budget for
Salaries 8% of sales year starting from 1 July, 2017.
Travelling expenses 2% of sales The company produces one product—-DETX H. Direct material costs 7 7 per unit. Direct labour
re
Sales office 1% of sales averages 2.50 per hour and requires 1.6 hours to produce one unit of DETX II. Salesmen_a
a commission of Re 1 per unit sold. Fixed selling and administrat ive expenses amount to
General expenses 1% of sales paid
% 85,000 per year.
Distribution Cast: oa
volumes:
"45,000 Manufacturing overhead is estimated in the following amounts under specified
Wages .
1% of sales . . 1,50, 000
Rent “ ee :
4% of sales
Volume of production (in units)
Other expenses
Expenses :
Draw up Flexible Administration, Selling and Distribution Costs Budget, operating at 90 per 3,30,000
Indirect material
cent. 100 per ceat and 110 per cent of normal capacity. 1,87,500
(B.Com. Hons., Delhi; C.A. Inter) Indirect labour
1.12,500
Inspection
Solution 84,009 1,02,000
Maintenance
Flexible Budget . 1,98,000 2.34,000
Supervision
for the pened .... $0,000 $0,000
Depreciation of plant and equipment
$4,000 94,000
80% 99% 100% 110% Engineering services
* z = z - 9,790,000 11,50,000
Total manufacturing overhead
6,00,000 6,75,000 7,560,000 8,25,000
Sales
1,40,000 units of production. [B.Com. Hons., Delhit)
Prepare a Total Cost Budget_forcee
Administration Costs:
Office salaries (fixed) $0,000 90,000 90,000 _ 90,000 § Solution
General expenses (2% of sates) 12,000 13,500 15,000 16,500 }
- ' Budget for the year ending “30th June, 2018
Depreciation {fixed} 7,500 7,500 7,500 . 7,500 i Output 1,40,000 units
Rent and rates (Fixed) 8,750 8,750 8,750 + 8,750 z
Variable costs :
9,80,000
(A) Total Adm. Costs 1,18,250 1.19.750 1,271,250 * 1,22,750 ‘4, Direct material @ % 7 per umt
’ 2. Direct labour @ % 4 per unit 5,50,000
Selling Costs: 1,40,000
66,000 "3. Salesmen’s commission @ Re 1 per unit
Salaries (8% of sales} 48,000 54,000 60,000, 3,08, 000
4. Indirect materials @ & 2.20 per unit
Travelling expenses (2% of sales) 12,000 13,500 15,000 16,500 1.75,000
‘6, Indirect labour @ 1.25 per unit
Sales office (1% of sales) 6,000 6,750 7,500 8,250 .- 1,05,000
6. inspection @ *% 0.75 per unit
4 General expenses {1% of sales} 6,000 6,750 7,500 8,250
Semi-variable costs :
72,000 81,000 90,000" 99,000. 4) Supervision — Fixed 54,000
' ? {B) Total Selling Costs
~~ Variable @ @ 1.20 per unit 1,68,000
-
Distribution Costs: , !
~- Fixed 12,000
15,000 15,000 2. Maintenance
Wages (fixed) 15,000 15,000 84,000
. —- Variable @ % 0.60 per unit
Rent (1% of sales) 6,000 6,750 7,500 8,250
Fixed Costs +
Other expenses (4% of sales) 24,000 27,000 30,006 33,000 90,000
fet 44. Depreciation
48,750 52,500" 56/250 . 94,000
| @) Total Dist. Costs 45,000 2. Engineering services
expenses . Coe O 85,000
~ 2,355,250 2,49,500 2,63,750 2,78,000 3, “Selling and distribution
- q, Total Cost (A + 2 + C) Totat 28,55,000
Note: All fixed, costs have been assumed to remain unchanged even at 110% capacity. However, i /
ce, fixed costs may change when capacity utilisation exceeds 00%. ve “Working Notes:
, ‘Fixeand variable components of each item of cost are determined as follows «
4
\ t
: ‘Bildgeting and Budgetary Contrat
: 2.32
2.31
Management Accounting
L. Indirect material

(6) Budget allowance for 8,500 repair hours


Change in cast t 3,30,000-2,64,000
Variable cost per unit
= = Fixed cost + Variable cost for 8,500 repair hours.
” Change in output ;
1,50,000-2,20,000 units
= 36,000 + (8,500 hrs, x % 7.99)*
"| = © 1,03,150.
£6,000 5 on
30,000 “Working Notes : Liea
Variable indixect material 120,000 1. ies
Employee salaries isi a fixed
i cost an d thus is the same at al levels, 7
x = 2,20 = © 2,64,000 for
1,20,000 units. 2, Indirect repair material is a variable cost @ % 6.70 (ie.,
Hence there is no fixed cast eleme
nt and this item is purely 40.200 + 6,000) per repair hour.
labour and inspection. variable. Similar calculat 3, Misc. cost is a semi-variable item. It is separated into fixed and variable components as follows
ion for indirect
2. Supervision
Difference incost ‘16,800 ~ 13,260 _ 3,606
Variable cost per unit — Changeee inin cost
cost” 2.2,33,
74000,0
~— 1,98
1,98
00 000
,000
Change in outpur — 1,50,000—1,20,000 anie Variable * Difference in hours 9,000 - 6.000 3,000
it

= % 1.20 per repair hour.


a Fixed = [€ 13,200 ~ (6,000 « % 1.20}] = % 6,000.
= 36,000 Z 1,20 per unit.
36,000 Total fixed cost Employee salary + Misc. cost (fixed)
Fixed supervision cost =_ 1,98,000 - {1,20,000 x 1.20)
= % 54 000 30,000 + 6,000 = % 36,000.

a
Similar calculation for maintenanc Totat variable cost per hour = Indirect material + Misc. cast
e cost,
3. Depreciation and en ginee =, 6.70 + 1.20 = % 7.90 per hour. -
ring services costs ate the same
fixed costs. at two levels of production, Thus
these are
Problem 2.6 t Manufacturers can produce 4,000 units of a certain product at 100% capacity. The
The manager of Repairs and Maintenance owing information is obtained from the books of account :
Department in response to
following budget estimates for a request, submitted the
his department that are to b @ Aug. 2017 aept. 2017
to be used during the coming used to construct a flex
budget year. : ible budget Units produced 2,800 3,660o
Details of cast
Planned at 6,000 z z
Planned at 9,000
direct repair hours i i ance 500 560
Employee salaries direct repair hours
30,000 meee ne ue 1,800 2,000
Indirect repair materials 30,000
40,200 Shop labour no hoo
900
Miscellaneous cost, ete. 60,300
13,200 Consumable stores : ,
(a) Prepare a flexible budget 16,800
for the department up te activ Salaries , 1,000 1,000
{use increments of 1:000 hours ity tevel of 10,0 00 repair hours
). - i co 240
(h) What wouid be the budget
allowance at 8,500 direct repai Depreciation
Deuccinth 1 ‘ ton :
1,400
r hours?
Rate of production per hour is 10 units. Direct material cost per unit is % 1 and direct wages
Solution {CS Inter;) per hour is F 4,
Flexible Budget; You are required to : ; .
for the peried, {i) Compute the cost of production at 109%, 80% and 60% capacity showing the
Direct repair hours variable, fixed and semi-variable items under the flexible budget.
(fi) Find out the overhead absorption rate per unit at 80% capacity.
Employee salaries
Indirect materia‘s
30,000 (CS. Inter)
. 40,200
Misc..casts : Fixed
6,000
Oo ee” Variable”
ee
eee 7,200
.
~ Budgeting and Budgetary Control!
2.33 2.34 Management Accounting
Solution
Semi-variable power cost at 60% = 41,100 + 600 {i.2, 2,400 units @ 0.25)
Flexible Budget
= @ 1,700
100% 80% Similar calculations for inspection and repairs and maintenance.
60%
£000 nits 3,200 units 2,400 units Problem 2.8
z
Variable costs ; For the production department of HYL Ltd you are required to :

Direct materials [@ Re 1 per unit.]
4,000 3,200 {a} Prepare a fixed budget of overheads;
Direct wages [@ % 4 per kour for 16
units} 1,600 1.280 (5) Prepare a flexible budget of overheads at 70% and 110% of budgeted volume:
Shop labour ‘ he
1,090 ‘80 +00
(c) Calculate a departmental hourly rates of overhead absorption as per (a) and (b) ebove. The
soetg
Consum:hle storas _ 2.000
cane 1 : +00 budgeted level of activity of the department is 5,000 hours per period and a study of the various items of
tal ‘A 1,200
00
8,a0u 6,886 expenditure reveals the following :
Semi-variabte costs : 5,160
= = per Ar.
Power —-
2,100 1 Indirect wages 9.40
inspection
260 0 ep Repairs Upto 2,000 hours 100
Repaitss and maintenance
590
ed 530
250 4700
For each additional 500 hours
; Total ‘B
2 950 2,650 up to a total of 4,000 hrs. 35
Fixed costs ; 2,350
Additional from 4,001 to 5,000 hrs. 60
~ Salaries . Additional above 5,000 hrs. _ 70
1,000
Depreciation Rent and rates 350
. a 1,400 —_ 1700 100
Total ‘C 400
Power Up te 3,600 houss G.25
2 FOG
Total Cost (A + B + ¢) 13,950
2,400 2,400 For hours above 3,600 0.20
Cost per unit (Total cost + Units)
11,930 3, Consumable supplies 0.24
—_ 3.49
49 3 ’ 73 =
4.13 Supervision Up to 2,500 hours 400
i
(8) Cateulation .
of overhead absorption ; Additional for each extra 690 hours
rate per unit at Sn
80% capacity
Total ces cost at 30% | . above 2,500 and up to 4,900 hours 100
z 11,930 Additional above 4,900 hours 150
Less : Direct material and direct wages
(ie. % 3,200 + 1,280) 4,480 Depreciation Up to 5,000 hours 650
Overhead cost
7,450 Above 5,000 hours and
Overhead rate per unit = % 7,450 + 3,200 units an up to 6,500 hours 820
= F 2.33
Cleaning Up te 4,000 hours 60
Working Notes:
, ‘Above 4,000 hours - 80
Calculation of semi-variable costs, From 2,100 hrs. to 3,500 his. 120
Heat and lighting
_ From above 3,500 hrs. to 5,000 hrs. 150
.
Variable cost per unit = Di ~tference iaes Above 5,000 hours 175
Difference ia units (LC WA, Inter)

Power = : 2000 > 3.800 = ® 200


3,600 ~ 2,800 units aoounits O85
At 70%, fixed element in power
cost= 1,800 - 700 (/.c.. 2.800 units
@ 0.25 per unit}
= € 1,100
Semi-variable power cost at 100%
= 1,100 + 1,000 (ie, 4.000 units @
0.25)
. = ¥ 2,100
Semi-variable power cost at 80% = 1,100 + 800 (/.2., 3,200 units @ 0.25)
= F 1,900
a,
Budgéting and Budgetary Contral
ae, AF,
pe Fo . 8 2.36 Management Accounting
Solution ,
Solution
Fixed and Flexible Budget for the Period .... na Before preparing material procurement budgel, it is necessary to estimate the production during the
items of Overhead Nature of Overhead Fixed Budget Flexible Budget { next year. This is done as given below: ‘nit
pm nits
100% } Estimated sales ~ 40,000
; 0% 110% Add: Closing stock
I nen oe meAD Bs. | 3,500 bs. 5,500 hrsSS
7,000
[re 47,000
Indisect wages Variable g = z Less: Opening stock
:

- 2,
6,000
Repairs Semi-variable . arr
0 1-400 2,200 4 Estimated Production during the y<ar 42,000
Roni and taxes Fixed 350 ne 378 |
Power Semi-variable 1,180 875 350 4 Materials Procurement Budget (Quantitative)
Consumable supplies Variable 1.200 Bao 1.260
Supervision A 7
Semi-variable 950 600 eee
Depreciation —do~ 0 Units Units
on ~ Po ES
Cleaning do 650 650 820 Units of material required
Heat and lighting do 80 69 80 . (@ 3 units of A and 5 units of 8 for
wets 2 150 120 175 42,000 units of production) 126,000 2,160,000
otal overhead 6,880 5400 | oT Add: Closing stock required 15,000 25,000
. P7545 Material on order 8,000 23,000 10,000
Hourly rate of overhead (Total overhead + ours) — 23,008
Rs, 6,860 Rs. 5,100 Rs. 7,545 - 549,000
5,006 hrs. 3.500 brs.
2,45,000
5,500 bre, Less: Gpening stack 42,000 20,000
ener =F 4.37 z Materiat on order 7,000 __19,000 11,009 37,000
~ + =< Lib = 2 137 Units ta be procured 1,30,000 2,146,000
Problem, 2, 9
Draw a Material procurement Problem 2.10
budget (quantitative) from the
slimated sales af a product following information:
is 40,000 units. Each unit of
material A and 5 units of material B. the product requires 3 units The following are the estimated sales of 2 company for eight months ending 30.13.2017:
of
Months. Estimated sales (units)
nee opening balances at the commencement of the next year:
Apriio17
insied brocut 5,000 units May %2017
12,000
13,000
Material B 12,000 units Jung 2017
Materials . 20,000 units July 2017
9,000
vMstaia 8,006
August 2017 10,000
Material 8 7,000 units September 2017 12,000
The destrabi
Tata . closing balances at the end of the rext year: 11,000 .units Octdber 2017 14,000
November 2017 . 12.000
e TOs
Material A uct me _ 7,000 units As a’matter of policy, the company maintains the closing balance of finished goods and raw
Matérial B 15,090 units materials as follows:
Materials on arder: 25,000 units Stock item Closing balance of ve
Matezial A: rat . Finished goods ' 50% of the estimated sales fo next month
MaterialB . sens ts " Raw miateriats Estimated consumption for the next month.
oT * ay
Rye ~ . (B.Com. Hons,
‘ units
Dethi Adapted):3 = at : : aa
Every unit of producti on. requires 2 kg of raw material costing’ 5 per kg.
we , ‘ "Prepare. Production Budget {in units) and Raw Material Purchase Budget (in units and cost} of:
the company for the half year ending 30 September 2017.
\ (B Com Hons Dethi; 1.¢.WA., Inter)
A
Budgetii
udgeting and Budgetary Controf
2.37 2.38
Management Accounting

Solution
Budgeted prices for the purchase of materials are ;
| Production Budget for the half year ending 30-9-2017
Apri May
Material A — @ 3 per kg, Material B — @ 2 per kg.
une sa . Prepare a Materials Budget for the first quarter of 2017 ina logical form showing the quantities
units unite nite baie Aug. Sep, of each type of material to be purchased every month. Also prepare a Puschase Budget.
: 5 units units
Estimated Sales 12,000 13,000 9.000 8,000 10,000 12,000
- B.Com. . Belh " 2
Solution
Add: Closing stock 6,500 4,500 ome Hons. Del feng)
: 4,000 5,000 6,000 7,000 Before preparing material budget, production has to be estimated by preparing a production
hese Open 18,500 17,500 budget.
13,000 13,000 = 18,000 19,000
éss: Opening stoc
pening 6,099
: Production Budget
6,500 4,500 4,000 5,000 6,000
Past ~ For the Quarter Ist Jan. to 31st March, 2017
:
Estimated Production nae
12,500 11,050 8,500 9,000 11,000 ~ 13,000 f
Particulars Budget Period
Raw : Jan Feb.
Material Purchase Budget for the year ending 30-9-2017 March April
units units waits °
April May June July Aug. Sep, Estimated sales 10,800 15,600 12.200 — 10,400
go kg kg kg kg kg Add : Closing stock 3,900 3,050 2,600 2,450
Material @ 2 kg pu. . 44,700
of production 18.450 14,500 12,850
25,000 22,000 17,000 18,000 22,000 26,000 Less : Opening stock 2,700 3.900 3.350 2,500 —
Add:
dd: Closi stock
Closing 22.009 17,000 18,000 22,000 26,000 26,000 ; =
: Production 12,000 14,750 11.750 10,250
Openi ;
47,000 38,000 35,000 40,000 «48,000 52.9000
Less:
Purch pening stock
' , . king
Working Note:
Nate: . .
25,000 22,000 17,000 18,000 22,000 26,000
chases 22,000 17.000 18,000 22,000 26,000 26.000
Catculation of closing stock and opening stock
Cost of purchases _ . . Closing Stock Opening stock 7
@ 25 per kg z= 1,140,000 85,000 90,000 . Jan.25% of 15,600 units = 3,900 . 25% of 10,800 units for Jan. = 2,700
110,000 1.30,000___1,30,000 - ‘Feb. 25% of 12,200 units = 3.050 Closing stock of Jan. is opening
Problem 2,11 ' Ay March 25% of 10,400 units = 2,600 stock of Feb and closing stock of
. . 3 Feb. is opening stock of March. -
Flex Food Products Limited has prepared the
following Sales Budget for the first five months
of 2017 : —
. Material Budget
,

Sales Budget? For the Quarter Jan. I to March 31, 2017

j (in units) « Particulars Jan. Feb. | March


Febr tan 10,800 A 8 A a
Mach 15,600 . oo kg. kg. kg. kg. ka. kg.
April 12,200, : Material for production oe
m _ 10,406 @ 4 kg. per unit for A 48,000 60,000 | 59,000 73,750 47,000 48,750
ay 9,800 @ 5 Kg per unit for 8
or inventory of finished products atthe end of every month is to be equal te 25 percent
Add ; Closing stock 29,500 36,875 | 23,500 29,375 | 20,50 75,625
“8 we sales estimate for the next month. On 1 January 2017, there were 2,700 units of product
77,500 96,875 | 82,500 1,03,125 | 67,500 84,375
on hand. There is no work-in-process at the end of any month, Less ; Opening stock 24,000 30,000 | 29,500 36,875 | 23,50C 79,375
_ Every unit of product Tequires two types of materials in the following
E + ae . . ;.
quantities:
: : piace et
- Quantity to be purchased 53,500 66,875 | 53,000 66,250 | 44,000 55,000
Material A. 4 ka. : = 7
Material B . 5 kg.
PARE faterials
Fer equal : 40 oné-half
: " of the next month’s producti
ath’s productign. are to bena in ha hen
in gne =
ymonth. This requirement was.met on 1 January, 2017, nd_at' the end of
t
Budgeting: and Budgetary Control _ ; ae 2.39
. .
Working Notes:
Management Accounting
Solution
Closing stock for each month is calculated as under :
. . Master Budget for the year anding...
For A
:
Jan. -~ 50%. of 59,000 kg. = 29,500 kg. Sales:
Feb, -. z
50% of 47,000 kc. = 23,500 kg.
Mar. - Toughened Glass
50% of (10,250 x 4 kg.) = 20,500 kg. §,00,000
Bent Glass
For B - - 2,00.000
Jan. -- 50% of 73,750 kc. Total Sales 8,00,000
« 36,875 kg.
Feb. — 50% of 58,750 kg. Less: Direct materials (60% of % 8,900,000)
= 29,375 kg. 4,860,000
Direct wages (20 workers x F 150 «x 12 months)
Mat, —~ 40% of (10,250 «3 kg.j = 25,625 kg. __ 36,000
Prime Cost 516,000
Purchases Budget
Fixed Factary Overhead :
For the Quarter Jan 1 af March 31, 2017 ‘
qo Works manager's salary (% 500 x 12 months} 5.000
M 7
— Jan. Feb. Fareman’s salary (% 400 x 12 months)
ar ‘otal Depreciation 4.800
Material Qty. 2.600
Price Amt. ty. Price Amt. Oty. Price Amt. Amt, Light and power
| ky e ky ® e ge 3.000 26,400
z z Variable Factory Overhead :
4 (53.500 3 1,60,500) 53,000. 3 1,59,000 | 44000. 3 13,000 "
451,500 9 ~ Stores and spares 20.000
B (66.875 2 1,33,750 | 66,250 2 132,500 | 55,000 2 1,160,000 | 3,76,250 . Repairs and maintenance
Total - 2,94,259 8,000
291,500 2,42,000 8,27,750
oor" Sundry expenses - 3,800 31,600
Problem 2.12 Works Cost — — 574,006
~~ Asahee Glass Company requires you té present the budget for the next Gross Profit 2,26,000
year from the following Less: Adm., selling and dist. expenses:
informat ion : 30,000
. : ! Net Profit
Sales : 1,96,000
Toughened Glass -
\ §- Problem 2.13 a ,
Bett Class % 6,00,000 i The follow:
= 2,00,000 hi
e following data relate to Bookhive /
Direct material cost Ltd:
. 60% of sales The financial manager has made the following sales forecasts
Direct wages 20 workers @ & 150 per month for the first five months of the
coming year, commencing from ist April, 2017:
Factory overheads:
Indirect labour — o Month =
| Sales (%)
Works manager April 40,000
500 per month May
Foreman 45,000
¥ 400 per month June:
Stores and spares 55,000
_— 2:5% on sales vo July
Depreciation dn machinery 60,000
% 12,600 : August. -
Light and power :
50,000
Repairs and maintenance z 3,000 Other data: . .
z 8,000 {!) Debtors’ and creditors’ balance at the beginning
Other sundries me of the year are % 30,000 and z 14,000
10% on direct wages respectively,
Administration, selting and distribution expenses The balance of other relevant assets and liabilities are:
= 30,000 per year Gash Balance
. : - 500
oc
. — Prehene pacter Buotpat. ° (B.Com. Bharathidasan} Accrued Sales Commission
(ii) 40% sates are on cash basis. Credit sales
1,000
= 3,500
are collected in the month following the sale,
(if} Cost of satés in 60 per cent on sales,
’ 1 (iv) The only other variable cost is a 5% commi
ssion to sales agents. The Sales Commission
\ ifs paid in a month after it is earned. * .
(v}i-Enventoty (stock) is kept equal to sales
requirements for the next two month budgeted
‘sales.

Fadaetary Cot
2.41 2.42
Management Accounting

fv) “Trade creditors are paid in the following month after puzchases. (b) Credit terms are :
(vii) Fixed costs are% 5,000 per momth including % 2,000 depreciation, Sales and debtors —~ 10% of sales are on cash, 50% of the credit
“You are required to prepare a cash Budget for the month of April, May and June, sales are enllected next month
2017 and the balance in the following month :
respectively.
" Creditors — Materials 2 months
(B.Com, Hons., Delhi)
Solution Wages 1/4 month
Cash Budget Overheads 1/2 month
For the month of April, May and June, 2017 {ce} Cash and bank balance on Ist Aprit, 2017 is expected to be 7 6,000.
April May June {d} Other relevant informations are :
= z = (}) Plant and machinery will be installed in February 2017 at a cost of ¥ 96,000.
Opening Balance 7,508 33,600 37,000 The monthly instalment of € 2,000 is payable from April onwards,
Receipts : (ii) Dividend @ 5% on Preference Share Capital of % 2,00,000 will be paid on
Cash Sales (40% of sales) 16,000 18,000 22,000 Ist June.
Receipt for debtors 30,000 24,000* 27,000* {iit) Advance to be received for sale of vehicles % 9,000 in June.
Total cash available (A) 53,500 75,000 86,000 (iv) Dividends from investments amounting to % 1,000 are expected to be received
Payments ; ia June.
Creditors 14,000 33,000" _ 36,000* {vy} Income tax (advance) to be paid in June is % 2,000, (LCWA., Inter)
Fixed cost {5000 - 2000) 3,000 3,000 3,000
Commission to sales agents 3,500 2,000 2,250 Solution
Tetal Payment (B} 20,500 38,000 41,250 Cash Budget
Closing balance (A - B) 33,000 37,000 44,750 For three months ending 30th June, 2017

“Working Notes : _ April May June Total


‘ zg ‘ € < z
1. Calculation of rieceipts from debtors, purchases and payment to creditors (assuming that ail purchases
are ar on credit basis} : Balance b/f 6,000 3,950 3,000 6,000
Receipts :
. April May June July. August
Sales* 14,650 15,650 16,650
e z z z ft 46,950
Dividend . - ~ 1,000 1,000
Credit sales 60% of total sales 24,000 27,000 33,000 36,000 30,000
Advance against vehicle - _ 9,000 9,000
Cost of sates 60% of total sales 24,000 27,000 33,000 36,000 30,000
‘” Total 20,650 19,600 29,650 62,950
Desired closing inventory at cost 60,000 69,000 66,000
Payments : . ,
84,000 96,000 99,000 _
Creditors (materials) 9,600 9,000 9,200 27,800
, Less: Opening Inventory 51,000 60,000 69,000
Wages 3,150 3,500 3,900 10,550
-’ Purchases 33,000 36,000 30,000 °
Overheads 1,950 2,100 2,250 6,300
Payment to creditors 14,000 33,000 36,000
Instalment for plant 2,000 2,000 2,000 6,000
Pref. dividend _ _ 10,000 16,000
Problem 2.14
Income-tax advance — _ 2,000 2,000
Prepare a Cash Budget for the three months ending 30th June, 2017 from the informati
given below : Total © 16,700 16,600 29,350 5?,650
“4, pgs Closing balance 3,950 3.000
. (a) Month Sales Materials Wages Overheads 300 300
, z z ME oo. . ¥
_ February 14,000 5,600 3,000 ---— 1,700
March coe 15,000 9,000 3,060 “71,900
Apil * 46,000 9,200 \ 3,700 °° =. 2,000
May 8 17,000 10,000 3,600 "2,200: i ;
dune ~- : _18,090 10,400 ‘4,000 2,300
era! weramamaaeedees , smn n pele separ
Sudgating and Budgetary Control
2.43 2.44
Management Accounting
"Working Notes :
Solution
1. Caiculation of collection from debtars
Cash Budget for the period ......
feb. f = March < April * May = June ¢
Sales Ist Qtr, 2nd Qtr. 3rd Qtr. 6th Qtr,
14,000 15,000 16,000 17,000 18,000 g z z z
Cash zales (70%) 1,400 1,500 1,600 1,700 1.800 Opening Cash Balance 10,000 15,000 15,000 15,325
Cradit sales 12,600 13,500 14,400 15,306 16,200 Add: Collections from customefs 1,25,0680 1.56,000 1.60,080 2,21.000
50% callection <a next month 6.750 7,200 7,650 Total Cash Available (A) 1,35,000 1.645.060 175,000 2,36,325
50% collection in the foltowing month 6,300 6,750
Peal
7,200 Paymsits :
Tetal collection 13,050 13,950 24,850 Purchase of materials 20,000 35,006 34,000 54,200
Cash sales 1,600 1,700 7,800 Other expenses 25,000 20,000 20,000 17,000
rrr a a
Cash ceceipts from sales 14,650 15,650 Salary and wages 90,000 95,000 95,000 1,869,200
16,650
2. Payment to creditors for materials, wages and overhead have been computed Income tax 5,000 _ _— ~
on a similar pattern,
(7} For exampe, payment for creditors for materials will be ; February Purchases Purchase of machinery ~ =— — 20,000
will be paid in April,
March puschases will be paid in May, and April purchases will be paid in June.
Total Cash Payments {8} 1,40,000 1,50,000 1,50,000 2,00,400
(xf) Payment for wages in April = (3.000x }/)+ (3,200
x 34) = 2 3,150 Minimum cash balance required 15,080 15,000 15,009 15,000

May = (3,000 x W)+ (3,600 x M}= % 3.500 Total cash required (£) 1,55,006 1,65,060 1,65,000 2,15,400
Excess (Deficit) (A) - (C) (26,000) - _ 10,000 20,925
June = (3000 Yj)+ (4,000x ¥)e & 3,900
Financing :
(iil) Payment for overhead in April = (1,900 « 4} + (2,000x 4} = 2 1,550, Borrowing 20,000 _ ~ _
Similar calcutations are made for May and June, (Repayment) . _ _ {4.000} (11,000)
Problem 2 Interest payment | — - (675) (1,100)
sw 0 he following information prepare Cash budget of a company : Total effect of financing (D) 20,000 _ (9,675) (12,100)
Closing Cash Balance (A+ D- B) 15,000 15,000 15,325 23,825
Ist Qtz., 2nd Qtr. 3rd Atr. 4th Qtr.
|. Probiem 2. Wwe -

mace
e < z z

aaerieruees
Opening cash balance 10,000 _ _ a. Thecepét“accountant of a manufacturing company provides you the following details for the

“ Collection’ from. customers 1,25,000 1,50,000 1,60,000 2,21,000 year 2017:
; Payments : ‘ , z , z
Purchase of mazeriats 20,000 35,000 35,000 54,200 Direet. materials 1,75,000 Other variable costs 80,000
Other expenses 25,000 20,000 20,600 17,000 Direct wages 1,00,000 Profit 1,15,000
Salary and wages 90,000 95,000 95,000 1,09,200 ‘ Fixed Factory overheads 1,00,000 Other fixed costs 30,000
Income tax 5,000
Purchase of mechinery
_ — _ ". “Variable factory overheads 1,00,600 Sales 7,50,000
= _ _ 26,000 During the year, the company manufactured two products A and B and the output and costs
The company desired to maintain a cash balance of € 15,006 at the end of each quarter. were;
Cash
can Le borrowed oz repaid in multiples of = 500 at an interest of 10% per annum. Management
does not want“ta botrow‘cash more than what A B
is necessary and wants to repay as early as
possibie. In any-event; loans cannot~be extended Output (units) 2,090,000 1,00,000
beyond four quarters. Interest is computed and
paid-when the 'pri cipat i
jat“is repaid. Assume that borrowings take place at the beginning and ;
.
Selling price per unit
., Direct materials per unit
-%-2.00 z 3.50
repayments-are wade ‘atthe end of the quarters. \
= 0,50 % 0.75
\ Direct wages per ‘unit. X 6.25 X 0.50
(LC.4.4., Inter)
Variable factoty overhead are-aheorbed as a percentage of direct wages and other variable costs
have béen computed as: 7"
Management Accounting
Budgeting and Budgetary Control 2.45 | 2.46

Product A % 0.25 per unit, and BZ 0.30 per unit. ~~ The budgeted capacity of the company: is % 20,00,000 but the key factor is sales demand. it
g capacity, the selling price of this only product
, Buring 2018, ‘ it is expected tha the demand for T ct il f all b y 25% and d f fo T B b yo 50
B19 Mo, is proposed that in order to utilise the existin
_ P oduct A will
y should be reduced by 5%.
manufactured by the compan
: +
It 1S fecided to manufacture a additior 4
the costs etc. for which are est] ated a
show the effect of the proposed reduction
You are required to prepare a budget which should
ed during the coming year. The following
Product € - in selling price and include any changes in costs expect
|
Output (units) 2,00,006 additional information is given:
on).
Selling price per unit F175 1. Sales forecast ¥ 19,00,000 {after reducti
to increase by 2%.
Direct materials per unit ~ 0.40 2. Direct material prices are expected
x
to increas e by 5% per unit.
Direct wages per unit 2 0.28 3. Direct wage rates are expected
expect ed to increas e by 5% per unit.
It is anticipated that the other variable cosis per unit will be the same as for Product A 4. Variable overheads are
will increase by 7 26,000.
Prepare dq budget to present ta the a lacement, showing
i the current position
iti and for 2018 5 . Fixed overhead (B.Com. Hons., Detht)
Comment on the comparative results. (B. . Com . Hons. a Dethi:)
vu

Solution
Solution
Budget for the period....... rt tg
a
Budget Showing Current Position and Position for 2018
19,00,000
(Particulars) Currant Pasition Position for 2018 Sales
-
fess: Variable costs :
A Bot Total | A a Cc Total 2% 6,12,000
Direct materials (4.50,000 x 4/3)" +
(A+B4C} x 4/3) + 5% 4,20,000
Direct wages (3,00,000
Sales (units) 2,00,006 1,00.006 | _— /1,50,000 | 50,000 | 2,400,000 _ 1,68,000 12,00,000
5%
Variable overhead (1,20,000 * 4f3)* +
z z! zi z z z z —— 7 00,000
Contribution
(A) Sales {% ) 4,00,000 | 3,50,0G6 - 7,50,000 | 3,00,000 | 4,75,000 | 3.50,000 | 825,000 _ 4,60,000
Less: Fixed cost (4,40,000 + 20,000} 2,40,000
Direct matertats 1,060,000; 75,00¢°14,75,000 75,000] 37,500] 80,000] 1,92,500
: . :
Profit
Direct wages 50,0001 50,000 /1,00,000} 37,500] 25,000| 50,000 112.500
Factory
Grariable)overhead 50,000,
' 5€,000
: | 1,00,0
00,000: 37, 500 25,000 ‘o
50,000 %4 4 1,172,500
1.12. *Working Hote :
20,00,000

koe
5 in relation to present sales =

f
Other
ee varizble 50,00
s000) 36,000) 80,000, | 37,500) 35,000) 50,000} | 1,02,500, 15,00,000

{8) Marginal cost


(€) Contribution {A - B)
2,50,000 | 2,05,000 | 4.55,000 | 187,500!
150,060 | 1,45,000 |2,95,000 14,12,500}
1,02,5001 2,30,000)
72,500 1,20,000
5,260,000
305,000
aan Pred Sudoo’
single product company estimated its
sales for the next yeal, quart i
er-wise as under :
Fixed cast:Factory 1,60,000 1,00,000 .
Quarter Sales units
Others 80,000 “40,000 I 30,000
(B) Total fixed cost 1,80,000 180,000 t 37,500
Profit (C - D) 1,15,000 | 1,25,000
F f 1, lil . 41,250
Vv 45,000
. Comments. S. Introduction of product € is likely y ttoo i increase profiti by % 10,000 (ie, :
units and the company expects to maintain the
% 1,25,000) in 2018 as compared to 2017. Therefore, introduction of product os ‘ecommended
The opening stock of finished goods is 10,000 in
at the end of the year. The production pattern
Problem 2.17 closing stock of finished goods at 16,250 units of the next quarter .
of the current quarter and 20%
Tké profitability statement of Gourmet Co. Ltd. has been summarised as follows : each quarter is based on 30% of the sales closing
ls in the beginning of the year is 10,000 kg. and the
ne z z The opening stock of raw materia unit of finishe d output
be maintained at 5,000 kg. Each
Sales . stock at the end-of the year is required to
Direct riaterials 4,50,000 0,000 requires 2 kg. of raw materials.
Direct Wages 3,600,000 4
Variabie overheads 1,20,000 :
Fixed overheads 4,40,000 13,10,000
Profi
rofit 1,960,000
- Budgeting
ae
and Budgetary Control: ‘gag
4 2,48 Management Accounting

The company proposes to purchase the entire annual requirement of + aw Materia ee i Budget i
:t kg.
three quarters
in the proportion and at the prices given below: isin the first Eon areal reqised fo vadsttion ” 3,20 000 i
Quarter Purchase of raw materials % of total Price per k Add : Closing stock 3.000
ee annual requirement in quantity 7 325,090
3 Less : Opening stock ___ 10,000 _ -
I 30%
3 Materials to be purchased 315,000
q 50%
M1 20% 4 Raw Material Purchase Budget (in 7 }
- ane ee Quarter Oty. to be purchased Rate per kg. Amount
Te value of the opening stock of raw materials in the beginning of the year is F 20,000 ze +
ou are required to present the following for the next year, quarter-wise: ee
I 3,15,000 x 30% = 94,500 kg. 2 Le3.0re
) Production budget in units. Hl 3,15,000 x 50% =1,57,500 kg. 3 4,72,500
(i}} Raw material consumption budget in quantity. I 3,15,000 « 20% = 63,000 kg. 4 2,52,000
(it) Raw material purchase budget in quantity and value. Total 3,15, 006 kg. 9,13.500
nner
————-
ty) .
Pri Priced stores tedger card of the raw material on oy
(1) using first-in-first-out method. (9) Stores Ledger Card (FIFO Methed)
: ( B.Com, Ha ns., Dethi:is C.A. Inter) =
Quarter Receipts Issues Batance
Schition

Date Oty. Rate = Amount Qty. Rate = Amount Oty Raté Amount
Cateulation of Estimated Production Units
Estimated sales per year (30,000 + 37,500 + 41,250 + 45,000) 153.750 kg. z z kg. é e kg. e z
Add : Closing stack _ -
‘ __ 16,250 1 _ — — — = —{ 40,000 2 20,000
94,500 2 =1,89,000 _ _ —j 1,04,500 2 2,039,000
: 1,70,006 —
lass
ne :: Opening stock '
10,000 -— — — | 63,090 2 1,26,000 41,500 2 83,000
Estimated production —— : ‘ 83,000
il 1,57,500 3 4,72,500 = _ { 44500 2
Oo te
Production Budget
1,60, 000.
. | _ - — (741,500 2 83,000
L753 472500
ee Quarters Total 35,000 3 1,05,000| 1,22,500 3 367,500
Sal . I I I Iv ¥ I] 63,000 14‘4 2,52,00
2,52,000 a —| f71,22,500
be 3 -3,67,500
bf, if j
£5 (units) 30,000 37,500 41,280 45,000 | 1,53,750 63.000 ee
Production (89% of sales 24,000
, 30,000 = Te
33,000» RE
(36,000 | Stet
. _ _ — | 84000 2 2,52,000] f 38,500 3 1,15,500 is4
of current quarter) oo . 63,000 4 2,52,000 “|
rotection (20% of sales 7,500 8,250 9,000 12.250 y _ - — 1f38,500 3 1,125,500 :
. of nest quazter) ‘ . ' 58,000 4 2,32,000 5,000 4 20,000
roduction ——_— +
en _ 31,500 38,250 42,000 48,250 | 1,60,000 Problem 2.19
(in Raw Materiat Consumption Budget in Quantit
y The dperating resultsof GQ£. Ltd. for the year ending 31st December, 2017 are summarised
Raw material consumption in kg, = Units of Production x Consumption per unit below:
Quarter I 31,500*2kg. = 63,000 kg. . © in lakhs
. HM 38,250-« 2kg. = 76,500 kg. 1 Sales - 40,000 units 48,00
84,000 kg. a Less: Trade discount 2.40 i
HE 42.000 x 2kg. =
WW 48250 2kg. = 96,500 kg. Net sales 45.0d) ites ‘he i
: Total: °3,20,000 kg. a Cost of sales:
TT . . Direct materials 14.40
\ Direct’ labour 12.60
Budgeting and Budgetary Contre!
2.49 2.50 Management Accounting

Factory overheads a
Administration overheads : - Problem 2.20
Selling and distribution overheads 45 0 oe GESCO Ltd. is currently operating at 75% of its capacity. In the past two years’ the levei of
company
. operations were 55% and 65% respectively, Presently, the production is 75,000 units. The
The folowing changes are to be inco rporated in the bud level during 2011-12. The cost details are as follow :
udget for the year 2018 - is planning for 85% capacity _
(a} Sal : .
es (units) to be increased by 25%; 55% — 85% 75%
(@) Material price to increase by 15%: Particulars :
# z z
A Direct wages to go up by 12%;
(4) Overheads-factory overheads will increase by 15%. In addition, a new fa iti Direct materials 11,00,000 13,00,000 45,00,008
Direct labour 5,560,000 6,50,000 7,50,000
. é added to the factory laboratory at a Tecurring cost of = 12,500 per year ty
Administrative andmG SONG dicteihety,
and Glo
calling aia . ;
‘ .
. Y
~ 3,106,000 3,30,000 3,509,000
TlOULIGi Ove ;
“i Mledus are estimated to increase Factory overheads 3,20,000 3,60,000 4,00,000
by 10% and 14% respectively; Selling overheads
cea gs 160,000 160,000
{e} Inventory—no change; Administrative overheads 1,690,000
() Profit target f ;
24,40,000 28,00,000 31,60,000
(3) No change th the ae has been fixed at @ 6 lakhs; and during the year:
© in ratepri of trade discount.
the
unit selling
Profit is estimated @ 20% on sales. The following increases in costs are expected
Calculate the g price and present the budget for 2018 in a meaningful format In percentage

(B.Com. Hons Delhi; L.C.W.A., Inter) Direct materials 1B:


Solution + 14,

Tmiole
ark factory overheads ! 5
Sitges for the yeat ending 31st Dee, 2018 8o
(Fig in & takhs) Variable selling overheads
Year 2017 Fixed factory overheads 104
Year 2018
Fixed selling overheads 15
(0,000 units} {40,000 units)
Sates ! Administrative overheads HW,
Less: Trade discount 7” £4.00 48.00
ascertain the
320 2.40 ' ¥ Prepare a fléxible budget for the period 2011-12 at 85% lavel of capacity and
{A) Net Sales (B.Com. Hens; Detht)
60.80 45.60 1 profit on sales.
Cast of sales:
: Solution
Direct materials , | Flexible Budget for the period 2021-12
Direct labour 78 44,40
sO For 85,000 units
is 12.66 -
Factory overhead -
overhead . 6.30
' Adm. 3.96 18,36,000
. 5.13 3.60 Direct materials {€ 20 + 8%} x 85,000 units
Selling and dist. overhead
Direct labour (% 10 + St) x 85,000 units 8,92,500
(B) Total Cost 35 4.50 2,20,000
500 A1i@ Factory overhead - Fixed (2,00,000 + 10%)
Profit (A - B) 4.20 , Variable(Z 2 + 5%) « 85,000 units 4, 78,500
: os : 1,76 ,000
Unit Selling Price = © 64,00,000 Administration overhead (1,60,000 + 10%)
"50000 unite 128 1,415,000
Selling overhead -Fixed (1,080,000 + 15%)
Working Notes : Variable (7 4 + 8%) = 85,000 units : __ 3,67,200
For the :
Total cost 37,85,200
i Sales unitecost Figures are caleulated.as follows : Profit (25% on cost or 20% on sales} __ 9,46,300
9 Materiat 125%
= 40,000 x d 50,000 units.

== 20.30 Sales as —sA193 1,500


& 0 3'Labour" cost = 1440 « 1.15 * 125%4 : by the following
\ : = 12.60 x 1,12 17.6), Note : Factory overhead and selling overhead are semi-variable-items and are segregated
4. Factory overhead , = (6.30 « 1.15) . toe . =z
ON , 78737 methods :
Adin. “overhead = 3.60 x 110% 3.96 . . :
> Selling and di
g and dist overhead ‘
= 4.50 x 114% ° . : =% i 4
Eee
in overhead at two levels
Difference
Lat a Variable overhead
“ : % Difference in output
Budgetingvand Budgetary Control
Ae nee 2 2,52 Management Accounting
51

Variable factory overhead _ Rs. 3,30,000 - 3,50,000 Solution


=% 2 per unit
10,000 units (i} Principal budget factor is determined as under :
Fixed
ie factery overhead = 3,30,000 ~ (65,000 units « = 2)= & 2,060,000 Forecast I Praduct Total

u
Similar calculation may be made for selling overhead.
- A 8 Cc e
Problem 2.21 Sales quantity (units) 22,000 40,000 6,000
its budget for the period ahead, : ’X’ Limited is consider ~ Labour cost per unit (2 ) 2 1.50 1
i
ering two pos. i
its three products as follows : Total labour cost (< } 44,000 60,000 6,000 1,108,000
; possible sales forecasts
i Labour available (= } 1,30,uuu
Products Principat budget factor for Forecast I is sales because tavuur 1s available more than needed to produce the
Forecast
agA B c given sales quantities.
¢ = z
I Sales (units}
22,000 40,600 6,000 Forecast IE Product Total
Selling price per unit
= 10 a) z ? 50 A B Cc .
I Sales (units) Sales quantity {units} 30,000 50,000 7,000
30,000 30,000 7 000
Selling price per unit Labour cost per unit ( ) 2 156 1
zg % 5.50 = 7 50
Variable costs per unit are expected to be the same Total labour cost (% } 60.006 75.006 7,000 142,000
a t the differen
i t level i
variable costs per unit are as follows : Labour available (= ) 1.30,000
Hes OF Possible sales. The
Principal budget factor for Forecast EI is labour because iabour is in shert supply.
roducts
A B c Sales Budget
. Org Forecast I (When sales is the principal budget jactar)
Direct materials
3.00 2.00 4.00 Product , Units SP. Amount
Direct labour
2,00 1.50 1.00 : to be sold = ¢
Variable overheads
1.00 0.50 1.00 A : 22,000 10 2,206,000
' Fixed overheads are expected to total € 1,00,00
0. These are expected to be unaffected by
the
Bo 40,000 6 2,40,900
c anges in activity which are being considered.
Due to recent high labour turnover problems c 6,000 7.40 45,000
abour will be restricted to a maximum of Z 1,360,0 direct
00 in the period. It can be assumed that all labour 5,095,000
iS of the same gradedi @ nd is fr eeiy t Tansfér a ble between products 4 Other Tesources ar are €xp ec ted to b e
Computation of Expected Profit
You are required te:
Taking Proditct Totai
each of the possible sales forecasts in turn
{i}. Say what the principal budget factor is for , A B Cc ?
each of the forecasts. Selting price per unit (2 ) 10 6 7.50
{if) orofite
For each forecast cal
calculate the sales budget that you would Tecommend to maximise Less : variable cost per unit (7 ) 6 4 6
{A) Contribution per unit & } . 4 2 1.60
{uit) What profit would you expect from each
sales budget? (B) Sales {units} : . 22,000 40,000 6,000
Assume that the products will be sald accardin
g to the sellingi i est i
price Total contribution (A x B) (% ) 88,000 80,000 9,600 1,77,000
and no interchange of the forecast is allowed.
“P mated 38 Ber the forecast Less : Fixed cost - . 1,900,000
Profit 77,000
(C.A. Inter)
esi
Budgeting and Budgetary Controt 2.53 2.54 Management Accounting

Forecast I] (When lagour Is the principal budget factor) 2018 Prospects :

Product Units SP. Amount


Sales _ 1,50,000 (60,006)
to be sold* € z Raw material 5% price increase
A 30,000 9 170,000 Wages 10% increase in wage rate
B 42,000 4.50 2,343,000 5% increase in productivily
c ~ 7,000 7.50 52,500 Additional plant One lathe % 28,000
5,53,500 One drill % 9,000 {[Ar“f- >*
Rate of depreciation is 10%. — -
Computation of Expected Profit —3
( Adapted)
Product Total Solution
4 B C qv Budget for 2018
Selling price per unit (2) 9 5.50 7.50
Actuals for 2017 Budget for 2018
Less : Variable cost per unit (%) A 4 6 Gutput (units) 40,090 60,000
(A) Contribution per unit (%} 3 1.50 1.50
Sales % 100,000 2% 1,50,000
(B) Sales (units)* 30.000 42,000 7,000
Variable costs :
Totak Contribution (AxB) (%) 50,000 63,000 10,500 1,63,500
less : Fixed cost 1,00,000 Raw materials 53,000 83,475
Profit 63,500 Wages - 11,000 17,286
Variable overheads 16,000 24,000
* Working Note :
80,000 124/61
When labour is the principal budget ‘ac tor, ranking of products is determined as follows : Fixed overheads: 10,000 13,7000
Per unit of product
Total cost 90,000 1,38, 464
Profit (Sales - Total cost) 10,000 42,539 A
ay

(4) Selling price Working Notes: .


[we

Direct material . 60,000 units


(a) Material cost in 2018 = 53,000 x 40,000 units
unis 105% = % 83,475
(8) Direct labour
more

Variable overheads 60,000 units


(C}) Total variable cost (6) Wages in 2018 = 11,000;x 40,000
O00 units
units = % 16,150
|» |o

(D} Contribution {A - C}
ray

oS

Add: 10% increase due-to wage rate __ 1,615


Contribution per rupee of direct . . Total 18,150
labour (D = BY 1.50
Less : Saving due to increase in productivity
Ranking 1

As products 4 and { are more profitable, 30,000 units of A and 7,000 units of C should be produced. This 4 [iss x *.| 864
will utilise labour cost of 7 67,000. (ie. 30,000 x 2+ 7,000 « % 1). The remaining T 63,000 (i.e., 1,30,000 -
105 °
67,000) of labour cost is te be utilised for the manufacture of 42,000 units of product 2, i.e., ‘ 17,286
¥ 63,000 + F 1.59,
(c) Fixed overhead in 2018 = F 10,000 + 10% dep. (on 28,006 + 9,000)= = 17700
Problem 2.22
B oblem, 2.23
From the following information retating to 2017 and conditions expected to prevail in 2018,
Viveka Elementary: School, has a total of 150 students consisting of 5 sections with 30 students
prepare a budget for 2018.
per section. The sthool’ plans for a picnic around: the city during the weekend to places such as
201? Actuals : =
the zoo,“the amusement park, the planetarium, etc A’ private transport operator has come
Sales 1,00,000 (40,000 units)
forward to lease att the ‘buses for taking the students. Each bus will Have a maximum capacity of
Raw materials 53,000
50 (excluding 2/seats reserved for the teachers accompanying the students}: The school ,will
Wages . 11,000 employ two teachers for each bus, paying them an allowance of % 50 per teacher. It will aiso lease
Variable overheads 16,000 Out the required number of buses. The following are the other cost estimates:
Fixed overheads 16,000 \

I
Budgeting and Budgetary Control - | Lo. a ; 2.55 2.56
Management Accounting
Cost per student (c) Break-even number of students
. t5
Breakfast
10 zg
Lunch Colléction per student
3 45
Tea fess: Variable cost per student (5+ 10434
2) 26
3
Entrance fee at zoo Contribution per student
25
Rent € 650 per bus Semi-fixed cost changes for every 50 students.
Thus, fixed and semi-fixed cost for the follo
levels of students are taken for consideration, wing three
Special permit fee ¥ 50 per bus.
Block entrance fee at the planetarium 7 250. No. of students
Up to 30 _ 59-100 IGL.E50 _
Prizes to students for games © 250, TO z yo
No costs are incurred in respect of the accompanying teachers (except the allowance of Fixed + Semi-fixed costs (a)
1,300 2,100 2,900
@ 50 per teacher} Contribution per student (b)
25 25 5
Break-even point (No, of students to fully
You are required to prepare :
cover fixed and semi-fixed cost) (a+)
(a) A flexible budget estimating the total cost for the levels of 30, 60, 90, 120 and 15¢ students. 52 84 116
Each item of cost is te be indicated separately, Thus upto 5¢ No, of students, break even Point
is $2 student which is outside the No. of 50. Bet
. {wo B.E. Points., Le, 84 and 116 are within the other
(b) Compare the average cast per student at these levels. their presctibed ievets. Therefore, 84 and
116 aie ts0 break-even
points for two different levels.
ic) What will be your conclusions regarding the break-even level of students if the school
proposes to collect = 45 per student? Problem 2.24
{€.A. Inter) ‘ABE Ltd. produces and sells a single product. Sales
budget for calendar year 2018 b Y quarters
Solution is an under :

Flexible Budget for the period... Quarter No. of units to be sald


[ 12,860
+ | No. of , students (A) ) 30 60? a0: 120 | 450:
uo, 75,006
I} 16,500
eat * 150 300 450 600750 Iv 18,000
1 in h " 300 600- 900 1,200 1,500 The year is expected to open-with an inventory of 4,000
units of inished product and close with
ea
fea 90 ¢ 180 270° 360 450 an inventory of 6,500 units.
Entrance fee 60 128 180 240 300 Production is customarily scheduled to provide for two-thir
ds of the current quarter's sales demand
600 1,200 1,800 2,400 3,000 plus ‘one-third of the following quarter's demand. Thus,
production anticipates sales volume by
Semi-fixed Casts : about.one month. The standard cost details for one unit
of the product are as follows:
Rent of bus 650 1,300 1,300 1,950 1,950 Direct materials 10 ths. @ 50 paise per Lb.
Permit fee 50 100 1h 150 150 Direct labour 2 hour 30 minutes @ Z 4 per hour,
Allowance to teachers 100 206 200 300
0} 300
Variable overheads 1 hour’30 minutes @ % 1 per hour.
800 1,600; 1,600 “2,400 2,400
. Fixed overheads 1 hour 30 minutes @ % 2 per hour
Fixed Costs : a based on 2 budgeted production volume of
90,000 direct labour hours for the year.
Block entrance fee 250a” 250 nn 250
oes 250
a
{i} Prepare a Production Budget for 2012, by quarters
Prizes to students 250 250 , showing the number of units to be
produced and the total costs of direct material,
. Total Cost (F) ) 1,900
50 . 3,300 3,90 0 . §,300- 5,900 direct labour, variable overhead and fixed overhea
(i7}. If the budgeted selling price per unit is = 17, ds,
whai would be the budgeted profit for the
(b) Average cost Per'student (8 + Al 63.33 55 43.33 44a 39,33 year as a whole?
{iit} In which quarter of the year is the company
expected to break-even,
(6.4. Inter)
Budgeting and Budgetary Contral 2.57

2.58
oe Production Budget Management Accounting
tent ate ne
(For the year 2018 by quarters) ‘Total sales by the end of Il quart
er will be 43,50 O units (ie, 12,000 + 15,000 + 16,5007.
Quarters
compa ny will break-even in the later
part of IT quarter, Thus the
fi I itl iv Total
_
te
KEY TERMS ee
Units Grits Units Units Units
Budget is a plan relating to a defin |
ite future period of time, expressed
2/3 of current quarter's sales 8000 19,000 11,000
’ 12,008 41,000 quantitative terms or in both. either in monetary or
1/3 of the following quarter's : -
Budgeting The process of preparing the
budgets is known as budgeting.
sales in [, Tf and [II quarter and » «one «500 sao Badgetary-control is a system of controllin |
5,5 ' . ‘ g through preparation of budgets.
closing stock in IV quarter 5,000 Functional Budgets For each function }
of the business, a separate budgets is prepa
budget, production budget, purchase budge red, @g., sales
Units
i to be produc
duced :
13,000 ';
15,500 :
17,000 ;
18,500 64,000 t, labour cost budget, cash budget, ete.
Master Budget It is a summary of atl the funct |
ional budgets,
Fixed Budget It is a budget which is prepa |
"Direct material
i (@ 2 5 per unit)
i :
65,000 77,500 85,000 92,500 3,20, 20,000 red keeping in mind one level of output,
Flexible Budget It is a budget “which
*Direct labour (@ @ 6 per unit) 78,000 93.000 =1,02,000 1,11,000 3,84,080 is designed to change in relation to the
attained.” Level of activity
*Variabi head
Zero base budgeting (ZBB) is a method
nee 150 - unit 19,500 23,250 25,500 27,750 96,000 of budgeting in which previous year's figure
the base for preparing next year's budget. Instea s are not taken as
fi
*Fixed ove whead 45,000 45,000 45,000 45,000 1,80,000 d, the budget Figures are developed with 28:0
base, i.e. the budget is Prepared as if it as the
is being prepared for a new company for i
reductia Cost
Production :
2,07,500 2,38,750 2,57,500 2,76,250 9,80,000 Ferformance budgeting is a relatively the first time,
new concept which focuses on functions,
“es activities as it lays emphasis on achievemen programmes and
*Working Notes : t of physical targets,
1. Birect material cost per unit = 10 lbs. @ F 0.50 . 5
2, Direct labour cost perunit = 1 hour 30 mts. @ f 4 per hour EXAMINATION QUESTIONS:
3, Variable overhead per unit = 1 hour 30 mts. @ € 1 per hour = F 1.50
4, Fixed overhead = 90,000 hours @ < 2 per hour = & 1,80,000 per year
Objective Type Questions
Fixed overhead per quarter =% 1,80,000 = 4 = 7 45,000 True or False ?
1. A master budget is a summary of all .
Statement of Budgeted Profit functional budgets.
for the year2018 - Fixed budgets are most Suited for fixed
expenses.

ww
All functional budgets should be coordinated .
= with the sales budget which is always prepar
- The budget that is prepared. first and all other ed first.
10,45,500° budgets are subordinate to it is Cash Budget.

&
Sales * 61,500 units @ & 17 per unit i .
A system of budgetary contiol can he used
} Less : Variable cost (61,500 units @ % 12.50* per unit) i . even when standard costing is in use in a concer

Ao
. A budget is a forecast of future expenditure. n.
Contribution eta
: Principal budget factor is the same as
Less : Fixed cost 1,80,0aa | key factor.
Won
VA budget coordinates the activities
of various departments,
Profit 36. .
. A budget centre is a part af the organi
sation for which a Seperate budget is
-: prepared.
Mee.

“Working Notes : . , Ina fixed budget, figures are adjusted accord


et.

ing to actual level of activity.


oa

1. Total number of units sold for the whole year is computed below : [Ans, 1. True; 2. True; 3. False: 4. false: 5. True;
6. False; 7. True; 8 True; 9. True:
12,000 + 15,000 + 26,500 + 18,000 = 2 61,500 units 16. False|
Multiple Choice Questions. Choase the
Variable cost per unit is material cost + labour-cost + variable overhead, a: correct answer in each of the following:
na

1. Of little or no relevance in evaluating


fie, ©5426
+E 4.50) =F 12.50 - * (a) Fledibie budget
the pezformance of an activity would
be
(5) Fixed-budget--"""’.
3. Contribution per unit = = 17 - 12.502 4.50 --*" {c) -difference between planned and actual
resutts
ml .* (> planning and control of future activities
Fixed cost _ 180,000 = 40,00G,units.”
{iii} Break-even point= Contribution per unil 4.50 \ ae A-budget that gives a summary of all the functional budgets and
nay as: f
projected Profit and Less A/c is
Lt
(a) Yapital budget (5) Flexible budget
{ce} Master budget (a) Discretionary budget
PNG a amy cen
Sudgeting and Budgetary Control . : 2.59. 3 2.60
3. The budget for a specific activity level Management Accounting
during a year was & §0,000, while actual
cost for the same period
was 45,000. Considering these facts, it can be said that the plant manager has done a better than
21. Mr. Managing’ Director is
surprised that his proéit
expected jab in controlling the costs if Or expects to achieve, Some every year is quite diff
/ one advised him to install erent from what he want
a the cost is variable and actual production was 80% of budgeted production, fresh cost accountant to a form, al system af budg s
do this. For twa years, the eting. He employs
(b) the cost is variable and actuat productian was 90% of budgeted production. based on previous years acco unta nt faithfully make a
accounts. The problem rema s all the assumption
the accountant on what steps ins unsal ved. Advise

7 Leste
(c) the cost is variable and actual production equalled budgeted production Mr, Managing Director and
. they should take. Make assu
*. The basic difference between a fixed budget and a fexible paaget mptions about what
is thet a fixed budget f
i
is lacking.
ly fixed costs, and a flexible budget on y varia € costs ; ; (CA. Inter; LC. Wa. Inter
: )
tS ies tee tor a single level of some measures of activity, while
a flexible budget consists of several Practical Questions
, don different activity levels. -
(¢} ease with feture acquisition of fixed assets. 1. ABC Ltd. has prepared
while a flexible budget is concerned with the budget fer the prod
expenses that vary with sales. Wwe manufactured by it uction Gf a takh waits
; ; for a costing period as unde of the only commodity
(d} cannot be changed after a fiscat period begins. r -
Raw material
5. In flexible budgeting. Direct labour z B52 per unit
; .
(a) statements included in the budget report vary from period to period. 0.75 per unit
Direct expenses
4) budget standards may be adjusted at will. ;
Werk overheads (60% fixed 0.10 per unit
4 seporting dates vary according to the activity level reported Upon, )
Administration overheads 2.50 per unit
(a) planned activity level is adjusted to the actual activity level (80% fixed)
before the budget comparison report is 8.40 per unit
prepared. Selling overheads {30%
fixed}
The actual production durin 0.20 per unit
{Ams. 1. (b), 2. (c), 3. (c), 4, (}. 5. fay) g the p period was only $0,00
7 Calculate the revis 0 units. 7 o>
ed budgeted cost per unit.
Essay Type Questions
{5.Com.)
EL Befine budget and budgetary control. State the advanta dget ry contr [Ans. Revised cost per
ges of budgeta ntrol in(Bcon
an organis
eeation.
Det 2. The cost at a capacity
unit % 7.75]
tevel of 5,000 units is given
or below this level, the below, For a variation of
2% Distinguish between budget and forecast. individual expenses vary 20 if capacity above
(B.Com. Hons., Delhi} as indicated,
Ue Write explanatory note on functions of budgeting. ; {8.Com. Hons., Dethi) Material cost z
4. List the Important functional budgets prepared by a business. . Tee
{B.Com. Hons,, Dethi: C.A. Inter) vip
5.. Discus
oT: :
s briefl jecti "Labour cbst 25,000(100% varying)
y the objec imitations of budgetary control.
tives and limita we = 15,000{100% varying)
4
(B.Com, Hons. Delhi. Andhra: . ‘ Power. -
6. List down any five objectives of budgetary control, Nagarjuna}. Repairs and maintenance 1,250 (80% varying}
(CA. . me Inter £000 (75% varying)
Ve-What are the main objectives of a system of budgetory control. Stores
Do you think B.Com Fons bet:
subject to certain limitations? ' Inspection 1,000(100% varying)
: ttl sytem in an sonata iS., ;
4 Discuss the necessary steps to the success of a budgetary control system in an Depreciation « 500 (20% varying)
o: Cin.
isation. Hens, Deth
Adm, overheads 16,000 (00% fixed)
:
<9, What do you think are the essentials of an effective budgetary control Selling overheads :
5,000 (25% varying)
system. (B.Com. fone. Deli
dp. Discuss the nature and functions of a budget committee. . 3,000 (50% varying)
UE Cor ‘ona ; Total
WAC What is sales budget? How is it prepared? “Cost per unit 62,759
/ , . Cont Wa inten /
i2. What factors would you consider when preparing-a-purehase 12,55
budget? (LC. WAL: Hs
istinguiish between fixed
« Bistingu i budget and flexible
i bu dget. Briefty state
5 the circumstances ne in which flextht
ae
a budgets are used. 3. Production
{8.Com. Hons.. costs of Oriental Enterprise
We What is a cash budget ? What are its uses? ~~~ s Limited are as follows ;
oT (B.Com. ho nel
JS, Explain (a) Zero base budgeting (b) Master budget.
(B.Com. Hons., Beth _ Level of Activity
. Distinguish between zero base budget and conventional
budget. 60% 70%
Output (in units) 80%
— ie * :

l¥. Write a short mote an principal budget factor. ~

7 2
1,200 1,400
\-T8.Write a short note on zero base budgeting. Costs {in %): : 1,609
"49. What is zero hase budgeting? What are {8.Com. Hons., Delhi Direct materials .
the advantages of zero base approach over saan 24,000
anode Direct Jabour . 28,000
/ ; 32,000
20 rite a note on performance ‘
Factory overheads 7,200 8,400
budgeting. . \ (B.Com, Aons:; B n 12,800 9,500
13,600 14,400
Works cost
44,000 56,000 56,000
a.

ee
ore
Budgeting and Budgetary Contra!
2.61 ! 2.62
: Fr
A proposal to increase production te 90% level of Management Account
activity is under the consideration of Management. Wen centeine it
The proposal is not expected to involve any increase in fixed | The sales during: the perio
factory overheads. Prepare a statement d were estimated as follows:
showing the prime cost, total marginal cost and ,
total factory cost at 90% level of activity, Counter saies
(B.Com. Hons., Delhi} Travelling salesmen'’s
=
fAns. Factory cost at 90% 2 52,000 é
4. For production of 10,000 Electrical irons, the following are budgeted ] (7) 8G.000
expenses: d 10.000
(4) ¥,20,000
se ste Per unit % 15,000
Ww Birect materials
(ii) 1,40,008
” 60 20,003
Direct labour (B.fom. Hons. Delhi: LO Wa.
30 Ttte
Variable overhead (Ans. Totai saies overhead at
25 sales of 2 90,000 » ¥ 17,3
Total sales overhead at Sales
Fisca overhead (% 1,50,000) of 2 1,35,000 = % 18,4
15 Total sales overhead at sales of

cree
Variable expenses (direct) % 160,000 = % 19,40
5 7, From the following details,
Prepare a Factory Overhead Budge
Selling expenses (10% fixed) t for 8,000 direct labour hours
15 Direct labour hours
Administration exBenses (% $0,000 rigid of alt tevels of production} _— 6,000
5 10,000
Distribution expenses (20% fixed Q zg
5 Factory Overheads:
Total cost of sales per unit Indirect materials
160
12,000
Prepare a budget for production of 6,000, 7,000 and 8,000
Trans, showing distinctly marginal cost Indirect labour 20,000
and totat cost. .
~
Depreciation 9,000 15,060
(C4. inter)
fAns. Total cost at 6,000 units, Z 16,50,000,
per unit & 275;
— . Lighting 25,0068 25,000
Total cost at 7.000 units % 11,87,500, per unit. Ra Supervision
3,000 3,000
169.64;
Total cost at 8,000 units, % 13,25,000, per unit | 13,200 48,000
7165.63} Power”
9. The monthly budgets for factory overhead of a company for = a Maintenance 3,800 6,000
two levels of activity were as follows - 9,800 15,000
60% 100%. :
Budgeted production (units) 600 (B.Com. Adapted
* 1,000 [Ans. Total factory ovezhead
for 8,000 direct labour hours ~
2 v 8. Gemini Steel Ltd. manufactur = 88,900]
Wages 1,200 2,000 1 es a single product for which
marke: demand exists for additi
ye ‘
quantity. Present sales of = 60,00
0 per month utilises only 60% capacity of the plant. Marke
onal
Consumable stores 900 Manager assures that with the ting
1,560 reduction of 10% in the price,
Maintenance the sale by ahout 25% to 30%, he would be in a position to

1,100 * 1,509
. inciease

Pewer and fuel 11,600 2,000
The follawing data are available :
Depreciation 4,000 (i) Selling price
nm
Insurance 2,000 (ii) Variable cost @ 10 per unit
000
Total 9,800 (it7} Semi-variable cost € 3 per unit
12,000
You are required to : = 6,000 fixed + 50 paise per
o (iv) Fixed cost % 20,000 at unit
, Present level estimated to be
(f) indicate which of the-items are fixed, _Nariable and_semi-variable, You are required ta Prepare
Z 24,000 at 80% output.
{t} prepare a budget the following statements showing
for 80% capacity, So . (i} The operating profits at :
So 60%, 70% and, 80% levels at
(ii) find the total cost, both fixed and variable, her unit of output current selling .ptice, and
at 80% capacity. (i) The operating profits at
Proposed setling price at the
: ——TE-tom. Hons., Delhi) above levels. (B.Com, Hons., Dathi)
(Ans. Profit at current sellin
fAns. Cost per unit at 80%—Z% 13.625] ! g prices = 13,000; 19,500
Profit at proposed selling price and 22,000;
6. You are required to prepare 2 sales overhead budget from % 7,000: % 12,500 and 14,090)
the following estimates: oe 9. Goldman Company Limited
operates on a system. of Flexi
‘ t information, you are reqilred to ble Budgets. With the aid of
prepare Flexible Budget at 80%, 90% and the following
Advertisement showing the profits that would 100% level of activity
2,300 result. at.-these levels :
Salaries of sales department ~ 3 (i) The present sale of §,00,000 .
_ 5,000 units.at = 10 each is.at-the norma
Expenses of sales department : increased to 90%, the selling l level pf 80%. Lf the output is
Le 41,500 price will be reduced by. 2.5%
Counter-salesmen's sataries and dearness allowance the original selling rica will ‘be and ifthe output reached
100%,
\ 6,000 " reduceby d 5% in order ‘to “teach a wider market,
(Commission to counter salesmen at 1% of their sales, i fii) The prime cost per unit is
Travelling salesmen’s commission at 10% on 5, made up of direct materials
‘. “2 expenses ® 0.25, If output Z 3°50, direct labour % 1.25 and
their sales and expenses at 5% on their sales}, ; \ reaches 90%. level of activity dizect
“effected in the purchase and above, a Saving of 5%
price of raw materials. can be
t
i
Budgeting and Budgetary Control

(111) Variabke overhead—salesmen’s commission Management Accounting


will be 5% of the sales value,
(iy) Semi-variable overhead at normal tevel of activity are: At 60% working, material cost per vait
increases by 2% and selling price Per unit
80% working, material cost per unit increases by 5% and falls by 2%. At
z selling price per unit falls by 5%,
Supervision Estimate profits of the factory at 60% and 80% worki
80,000 ag and offer your comments,.
Power .
Heat and light 70,600 {CWA Tater)
40,000 fAns. Profit at 50%; = 50,000; At 60% % 53,000:
Maintenance At 80% % 53,000]
(Comments By increasing capacity from 50% to 60% c. profit increases by € 3,000 to % 53,000
Salesmen expenses 5,000 .
However, by increasing capacity from 60% to 80%, profit remains at 2 53,000 because it
. 60,006 gets
Endirec- labour neutralised by increase in cost and reduction in sellir
1,00,000 hd price}
Transpert costs
2,00,060 12. The following overhead expenses relate ta a cost
These are expected ta increase by 5% centre Operating at 30% of normal capacity. Draw
if output reaches 90% level and by a further up a flexible budget for the cost centre for Operating
the 100% level. 10% if it Teaches at 75%, 300% and 125% of normal capacity.
Indicate the basis upon which you have estimated
{v) Fixed overheads are ; each item of expenses for the different operating
levels.
z
Rent and rates z
1,00,000 Foremen
Depreciation 60
4,00,000 Assistant Foremen
Advertisement 4g
5,00,000 Inspectors
~ Acministration 65
7,568,006 Shap labourers
Sales department 40
2,00,000 Machinery repairs
General = 100
50,000 Defective work
: 25
_ (G.€om, Hons, Bethi} Consumable stores .
{Ans. Profit
20
at 80% % 10,00,000; at 90% F Overtime bonus
13,63,750, at 190%. 15,907,000]
10. From the following data, pre pare Nil
a flexible budget for production of 40,000 Machine depreciation
75,900 units of product X, distinctly units, 60,000 units and __ HG
showing variable and fixed Cost as _ Total
WW ‘fid:cate element wise cost per unit. Budget well as total cost. Also 460
ed output and costs per unit are :
Budgeted output
1,00,000 units
4 {L¢.W.A. Inter}
(Ans, 755, % 610;
= 790; 125% © 1,170] 100%,
Per unit cost ¥ (Hint. Answer will vary according to the assumptions
Direct materials made regarding nature of variability of different
90 items of cast).
Direct laboua
45 13. Draw a Material Procurement Budget (quantitative)
Direct variable expenses from the following information :
io Estimated sales*of product 40,000 units.
Manufacturing variable overhead
40 : Each of the product requires3 units-of material 4 and 5 units
Fixed production overhead of material B.
5 Imated openiag balance at the conimencement of the next year
Administration overhead {fixed) ,
;
- 5 Finished product , 5,000 units 3 Materiais on order :
Selling overhead Be
10 (10% fixed) . oa Material A 12,000 units * Material 4 7,000 units
Distribution overhead
15 (20% fixed) Material B 20,000 units Material B 14,000 units
(B.Com: Hansie.a The desirable closing batantes at the end of the next year :
-[Ans. Total cost € 96,40,000 at 40,000
. thits:”
Finished product 7,000 units Materials on order :
© 1,37,60,000 at 60,000 units" 1,68,50,000-at 75,000
Gs factory is currently Tanning at 50% capacit
units) _sfaterial A 15,000 units Material A 8,000 units
y and produces 5,000 units at a cost Material B 25,000 units Material £
as per details below: of % 90 per unit 10,000 units
' Materials 1
w50 AR! . a (B.Com., Madurai)
15 / [Ans. Production 42,000 finished units,
ot o ;
15 .
(R°6/- fixed) Ao Purchases: A - 1,30,000 units, 3 ~
2,14,000 units]
. aed 14. ‘Due Age producing
on alloy. Production of one ton of alloy requires
1.5 tons of Iran and 0.5 ton of
10 (® 5/- fixed) ie TE Producer plans to sell 50,000 tons
of alloy during the year 2017. Prepare Materials
Procurement Budget for-the yéar"2017 from the following:
’ Balance as on Jan. 1, 2037:
:
Stock of Alloy
6,000 tons

OO
2.66 Dec., 2031
i . yeat ending yist
! udgeting and Budgetary Control Budget for the (8.Com. Adapted}
get ard Mal terials Purchase
2.65
Prepare the Mat
e: vial Usa ge Bud
. .
ges, units: X-1 ,00 ,000, ¥-35,000;
Stock of izon ore 12,000 tons
fans, Budgeted wa : ¥ 1,02,500, ¥-36,0
00;
ck. of Zin‘ c
Stoock. Units to be purchased 00,° ¥-¥ 4,0 8,0 00]
o4,00oO0 tons - Purchase cost at X-% 2,05,0
ne or an order 16
it requires you
Iren on

tder
5.0
han d on ist April, 2017 and pp ed t you :
3000 ten h in is su
B
‘alance as desired on Bec. 31, 20 ue ons ex pecting to
have &, 35,000 cas 2017. The following jnformation
. 31, 2017 : co mp an y ig Apr il to Jun e, Exp ens es
UW. A for three months, Wages
sot of Alloy prepare a budget Sales Purchase
:
[5.000 tons zg
z 6,000
Irone ore 8,000 tons & 000
ock ofofZin
;an 79,000 -
40,000 7,000
3-000 tons -
50,000
8,000
7,000
he ore on order 7,000 tons Ka 80,000 4.000
Ihe on order (< March
52,006 > Brvd
2,000 tons 92,006
(0,000 —s> 18000
j,60,000 —->
? 9,000
> April
pens : Dethi} _—_ 12,000
fAns. EstiEsti mated production of
teBL
2> May1,00,000 4,290,000
55,000
° terials
Mater, to
— 49,000 tons:
re procured Iron or 71,506 tons; 4 Zinc 22,500
! to.ns] "June ,
1 Engineerin g Co, Ltd. . ma on: One
nufactures tw ‘o Products X and ¥. An estimate of Tumber other informati d by suppliers
: two months
;
to custome rs
for credit sales
ected t to b e § sold o TL
units
cre d’ it all owe dit all owe d
in in the firsti 1S s even months Ss af
{a} Period of dof cre
cash and perio
201
Ol? is given h 2 Law,
sa les are for
Product ¥ Product ¥ (b} 25% of the “
Janua 50
one month; ala}
: y 1,400 month.~ and ex penses: (3. Com, Ker
on ~ t of wages
‘abruary in pa ym en e 201 7. 1,0 6,0 00]
° 1,400 (c} “ielay be paid in Jun on 30th sure
=
March me tax of 2 26,000 is to {Ans. Ealag ye
800 {d) In co bud get
Aptil
1,200
prepare a cash
1,000 it ur e. you até required to 10, 000 .
K 1,000 and expend =
sts of income ptember was
: 1,20 ance on yet Se
m ay
j 800 following foreca mb er . The bank bal y Opies
200 Sea h No ve Fac tar
une hs en ding 30t Wages enses
‘of three mont
1,200 860 s exp
Pu rc ha se exp ens es
July 1,006 Sales can
_ It is anticipated
904 Month z
that : z 10,000
a zy 00
8 3,9
te)
(a) fakes
there wi illane
d work-in-progréss at the end of 5 600
40,000 . 12,000
end of tay equal $0,000 4,100
onth n anwi
to half the anticipated to the pes July 5,800 14,000
d instock at the 42,000 4,200
) The budgeted bivdvetio
for the san month (including December 201 5) The bohetrs 76,500 5,800
August , 16,006
nding 31st December 2017 nea s foll
follaws : production costs 78,000
34,500
“5,900
5,100
Septembat 37,500 13,000
Product X 99,000 5,900
6,c00
the gales due
s
October 43,000 moth in which
95,009 h followin the wil be pu rchased
November mont 65,0 00
hue (units) on of 44, on sales,
due in the assets worth © pst witl
; ee
A sales commissi ble in-addition to office exper ses. Fixed 1
es
Materiats per unit nth credit
a
are coll ec te d; is pa ya lowing month. hs and one mo
Me wages per unit id for in the fol cu d 16 of each
in September t
mi all owe d to
iod of credit an ave paid in
sortase charges ober. The per es are paid om
° 7 | be paid in Oct pre ces tin g such days. (M .Com.)
portionable ble to each type of product — - ; m sup p the dat e
33 000 igs obtained fro
Yo Ware required
ire to prepare :
48,900 -
pec t of due s Sep t. 2 24,000,
month in res
ic h the y até due. [an s, Cas h balance 30 )
wh . z (-) 40,610
x
(a) A producti
uction bud get showing
i the number of units t ‘o ‘bbe manufactur ed the month in 3,150; 30 Rov
each GHonth . 31 Oct. = (-) an
(b } A summarise d produ
od ction cost bud g @t~for : the 6-month
= peral od. — January to. June
2017. requires you 6 prepare
Ast April, and information
(B.Com., Adapted) © 37, 500 . cas hin hand of and Jun e. T ne tolcwing
expects to pave hs -A pr il . May
49. A company ee mont
Ans. Production
guring the thr
for six months : X—5550 units,
T‘otal cost F 1,17,000; ¥ Selling
US, Y—6350
cash position
units
Yoel of Office
companmy y A man ufac ts A and 8 by mak.i 0}
ofematser € 190X,50and :
to you Factory es _aypxpens
(& Y, Product
Manuregijfarbectur19°esytwtwount produc A of ¥
AOeunits of Hand 3 units ¥ ing use of tip typ ig supplied Sales Purchases *,
Wages
expenses expenses

pee seTegites ' 5 units of erials
viz.
u
The
ms
3 Produc t.8 re quires X-and 2 units ve a ns
of &
F per ym and that
of
imated the sales of product Ate op
a. fi *

went
°
z zz 7,500
that “OF 9,000

ended
heay
sand
igétaid ‘period is’ 2/500 eee a 75,000 45,000
at of
Fis 3,000-1 fo
February
SHAN is 5:000-un
ae

ue a
Management Accounting
_ 8.67 2.68
TT — - c
BO
7 : cae " 3 Machine Shop and the
847000 48.000 °° «9:750
tM. 8.250 6,000 carefully planned and monitored in the any month the indirect
Indirect manufacturing costs are 10%
an)’ | PERG 9,000 oon 4,500 of the
is paid a savings as bonus when in
29:00 52,500 10,500 foreman of the shop
0 5,250 the budgeted provision.
manufacturing cost incurred is less than
60,000 13,500 11,250
1,20,000
ae 6,000 6,570 for the month of January.
14,250 14,000 he should be paid a bonus of # 38.50
x ad : 335,000 60,000 The foreman has put in a claim that bonus when the compan y has lost a sizable
7,000 one can claim a
. . , 7,000 The works manager wonders how any
Other informations : . are as under : _
1. Period of credit allowed by supplier s 2 months contract. The relevant figures : Actuais In
of credit fl, : . Expenses for Pianned for
7. 20% of sales are for cash and period a owed to customers for credit is one month
Indirect Manufacturing January
Jonuary
- expens es—1 monttt. cost i normal month
3. Delay in payment of all . z z z
is d +d 0, , 1,000
4. Income tax y of is % t 57,500
. , ue to be paid on June 15th, . 1,000 1,000
5. ‘the compan : Salary of foreman 900 600
ona 15,000 and & 22 720
.
respectively in the oath ase shareholders and bonus to workers of Z Indirect labour 1,600 700
‘ ‘500 Indirect material gad -
6. Plant h as been ordered to be received “ and paid in May, It will cost @ 1.20 0 ' : 600 650 600
‘ 20,000 . Repairs and maintenance 800 875 740
- : e .
- . . 8.Com., -- M Madurai)i
P
owe! 320 400 300
: ( :
i Tools consumed
an dist May © 91.050.
[Ans, Cash batanc
e 30th April = 11,700; Cash shortage 150 150 180
fr er ° '\ Rates and taxes 300
¥ (050; Cash shortage:a n 30th June = 2,15,370} 800 800
20? G: Depreciation 4
i100
Tepare a Cash Flow Budget of Madern Septemb | 100
we
arments Ltd. for the six month ending 100
the basis of the-following information nt ™
er 2011 on t Insurance
5,875 4,990
5,290 ra .
A. Sales forecast;
a
for the performance in
ee
Is the foreman entitled to any bonus
; _

Bo you agree with the works manager?


June ~
J anuary z 2,40,000 (C.A, Inter)
and figures.
= 1,830,000 ' January? Substantiate your answer with facts = z 4,705
February 2 2,00,000 = 2,109,000 {Ans. Budgeted expenditure for January
July
Z2 50,000 not entitled to any bonus]
March
August % 2,00,006 against actual of 7 4.990, Foreman is
f Apri z 140.0
z 2,00,000
: variable . Renairs and mainten ance, power até semi-
tools are
‘May? 1,50,000 40.008 September (Hint. Indirect labour, material and
® 1.60,000 ‘ ‘ jable i -
B, Cash sales represent sO a ofof tatotal sales, 60% of i : collected in 30 days, 30% i variable items.)
ee are g budget for 2011-12
60 days and balance in 90 days. Presunte ROO DaE co ‘ 22, SK Ltd, has prepared the followin
Percent
is maintai ned at 20% o f sellin s
€. Grogs profit margin + of selling price.
‘ : . “o25
8. Anticipated sales of @ ; . , Raw materia ls ;
3s purchas ed and paid in cash in“the precedin | Direct wages 10
and wali month d as fol 9 month.
E. Administration
April ing and distribution expenses are budgete r Factory overheads (variable) 5
July ows. Factory overheads (fixed} 6
- = 44,000 + ds (variable)
August % 14,500 “4 5 Administration and selling overhea 12
= 12,000 ds (fixed)
*Y
73 3 Z 14,500 - Administration and selling overhea
dune September F500 7 | 100-
F Payment: for installati f 1500
is d ue an ist May"oma 204. ° & Profit iue.,
G1- installadeposit 2 tack costing & 80,000
tion ofof a % machine - yo Total Saies value. would not
Interest on fixed
12% p.a is payable quarterly in April and
July. it was felt that the budget volume of sales
H. Advance tax of2 24.000 j KS @ After considering the half yeariy performance, ent to a sales value of % 160 lacs. At
. nly. d to achieve 80% equival
Interest on invest UN is payable on 14th June, 2014. , , be obtained but the company expecte for its usual line of products. The prime cost of
the
le i oe
quarter is receivab‘able in June and September, | | ‘this stage the company receives an export order
stment amounting to % 10,000 per export expenses are est imated at % 40,990.
J the ten,259 lb
Balance the value of an asset
sold for % 1 lac is ‘due in : expoit order is <<€'13 lacs and speciai l
ne pany has a cash balance of€ 75,500 on 3ist Marck August,: 2611. : : :
! You are, required-to : showing the
fF 10 aon pou If at any time cash balance revised budget based on 80% achievement
reduces below % 50,000 level, short-tertito
i (i} Present the original budget and the
interest payable on monthly basis. | ‘ a time is arranged @ 18% p.a;” quantum of profit or‘ toss. ry sates.
=
(EC.W.A. Inter) ishing percentage of overhead recove
ted cost for establ
[Ans. Closin g balance % ~Apri (if) Prepare a statement af budge your comments.
April 1,32,500; May 74,100; June
55,800; ate the lowes t quotation for the export order and offer (C.A. Inter}
, (ii?) Calcul
;
St B0% shows joss € 3.60 lakhs
2%. Action Plan Manufacturers norma
oduct [Ans, (i) Revised budget at
Shop. For the ‘month ‘af: in a month in their Machi - Fixed 20% of wages, Variab le 40% of wages.
Jarua pecductionof 10,000 units ‘win (ii) Qverhead rates:-Fac tory cost
Suddéh cancatlation:of. e te cost, Variable - 7.5% of works
they ©could produce only Selling and.adm. ohds.: Fixed - 15% of works above € 16.525]
January. co ohly 6,00
6,0099 units
waits in / - iit) Any price
we
i
9 nt Accounting
Manageme
3.2
.
Meaning and Definition of Standard Cost st
or a crite nt on. Stanntdard cost is thus a criterion cost whic
iteri
: dst standard means ‘a{ norm’ . has been incn tred
the efficiency with which actual cost
aa ed as a yardstick to measure that shou ld be incur re
pre-determined costs ot target costs
in noother words, . standard costs are: veccdur

STANDARD COSTING AND nder efficidlient operatini g con ditions.


. According to Chartered Institute
vy the pre-determined cost based
of Management Accountants (C.LM.A.
on techn ical esti mate s for mater ials,
), London
igbour and @
ee

ermine
.

VARIANCE ANALYSIS |
”.
a prescribed set of working conditions
for a selected period of time for [ d cost Oe 7 eesr ve
mine
rd, “fhe standard cost 1s a predeter
if
‘ In the words of Brown and Howa I i umstances.” ‘Thus stand
circ ard costs a
ice should cost under given main ahiec
cr
teach apereatin
set t ofoF oper g cond itio ns,
tition s. The
7
houldd bebe attaatained under a given is
what each promt shoul the cost shou ld be’ as disti nct from wn
forward and assess what
of stanoddacore st is to S took
ote
has been in the past.
the cost
Standard Costing
introduction; Meaning and definition of standard cost and standard costing;
Meaning and Definition of

ee
5 are compRS uted
Applicability; Standard cost and estimated cost; Standard costing and
$i y the name given i toa techniqu i e whereby standard costs
ing isis simpl betw ee he ae
-budgetary control; Advantages; Limitations; Prelimineries; Variance analysis; costs to find owt the differences
a certly compared with the actual ee on
Material variances; Labour variances; Overhead variances; Sales variances;
varia nces) are then analysed to know the causes ql
Disposition-of variances; Control ratios; Summary of formulae: Problems these fer nces (known as stan dard costi ng as “tne ’ Se ysing
.A. London has defin ed
nov : asis of control. The ¢.1.M actual costs an a in
them to measure the variations from
and solutions; Key terms; Examination questions.
Pan de tf costs and apply ing t dard eof
mum effic iency in Pa
view to maintain maxi

ee
h vourses of variations with a cost
saaxd ined it, it, ° “as a techniqu ji e of cost : accoun ting which ve compares ations se
Introduction rmin e the effic iency of the oper
to ! dete
u
_each eeprodduct sevice with the actuai l costs,
Standard costing is a specialised technique of cost accounting to canteol the cost. From cost actio n may be taken immediately”.
any reme dial - ou
control point of view, ‘what a product should have costed’ is more important than ‘What it rd costi i ng syst em invol
invo ves the following steps :
, anda ig, material, lavour
did actually cost’. The actual cost is the past cost or historical cost and historical costing for different elements of cost,
te he setting of standard costs
is a system in which actual costs incurred in the past are ascertained. overheads. «costs “awn
ining actual costs. een the two
costs to determine the differences betw
Limitations of Historical Costing
3 aeparing standard with actual
Ascertainment of actual costs does not serve any useful purpose and has certain limitations, as ‘variances. . : . vetive
f.
Firstly, suck costs are obtained too late and cannot be used for price quotations. ~: i Hiances for ascertaining reasons thereo for appro priat e corre
analysis thereof tc management
Secondly, historical casts do not serve the purpose of cost control because the ‘cost: has ‘ fencing of these variances and
already heen incurred before cost figures are available for. managerial control. ara , action, where, necessary.
seg tue ae
‘Thirdly, historical costs do aot provide any yardstick against which efficiency. can be Measured. ing
‘Applicability of Standard Cost
‘These limitations encouraged the development of a more satisfactory standatd costing approach ng requires certain conditions
to be fulfilled. The
based on predetermined costs. Standard costing is not a ‘method of costing like’ job otdér on The @ application of standard costi d be produced.
-products or comp onen ts shoui
Process costing. It is a special technique to control costs and can be used in conjunction with (a) A sufficient volume of standard ed.
any other system like job costing, process costing or marginal’ tosting) ete. should be capable of being standardis
HS ee (b) Methods operations and processes
Standard costing is one of the most important tools to control costs. Ini this techitique, all be capable of being controlled.
{cy A sufficient number of costs should i.e ,
nat e, .e.,
itive inin natur indus
1m a tries us1ug
the
costs are pre-determined, i.e., casts are determined in advance of production. Such, pre-determined W Hh
il standardisi ed produc ts which are
cing repetitive
us ries produ
ndust p
the system can be us d
costs are. then compared with the actual costs. The difference between the actual costs and all the above conditions and thus
pre-determined costs, known as. variances, are thes, analysed and investigatedto know their P 5 @ vosting method, fulfil t, steel, sugar, etc.
. Examples aye fertilisers, cemen
reasons. Variances ate teported to management for taking remedial steps so that actual costs bestes savarttage’ i I such industries a full Sys tem
e of
f st
stan
di d
he
worthw h ile to d deveiiop.p.. a nd employ
-,- adhere to pre-determingd or standard costs. Poles yen
ye tee 1
Wet .. in J obbin ig i sust
di ti B85, yl tis not
h b n may he diff eren t
from anol her
such industri es @ jo undertake
\ we : rT ii as is b a cause in i ‘ ac
costing. ‘ ries, z therefore,
3.1 si Vi e. in, such indust
for each jab may prove diffic WLlia and expen
ang : stand
fon ards1
% aq d setti
Standard Costing and Variance Analysis 3.4 Management Accounting
- wan 3.3

a partial systen may be ado pted ini appropriate


. OP! i i
circumstances. . For exam ple, certaini

oe sak
costing Budgetary contret
operations perforined may be of a repetitive nature and thus the Basis Standard
principles of standard costing
y be applied by setting standard for each such process or operation. for different functions
“ed 1. Scope. Standard costs are developed mainly for; Budgets aie compiled
as sales, purchases, pro-
the manufacturing function and sometimes | of the business such
Standard Costs and Estimated Costs - Comparison capital expenditure, researcn
also for marketing and administration func-| duction. cash,
tions. “ Dand deveiopment, etc.
Bo th standard costs and estimate
i d costs are predetermined
i costs computed in advance of production in nature and
But their ob ective Sa re norm alt ¥ differen t . The differences 2, Intensity. Standard costing is intensive in applica-' Budgetary control is extensive
between the two are summaris e d as
tion as it calls for detailed analysis of! the in - of analysis tends te be much
. T variances, ibyess than that in standard costing.
Basis Standard ve normally
cost Estimated cost 3. Relation to: in standard costing, variances are usually "in Buinetary control, variances
: nol revealed through accounts and control is
1. Aum, Standard cost aims at Estimated cost is an assess accounts. | re-vealed through accounts
what the cost ent 9 exercised by statistically puttin, dudgets and
SHOULD be. cost WILL be. * what the Factuals side by side.
¢. Basis. Standard casts are planned costs which are
Estimated costs are based on average xsually represent an upper limit on
determined on @ scientific basis after of the 4. Usefulness. Standard costs represent realistic yardsticks : Budgets
past” figures, taking into consideratio
n and, are therefore, more useful for con-' spending without considering the effective
caking inte account certain level of anticipated changes in future.
officiency. trolling and reducing costs. ness 2 expenditure in terms of output.
3, Relation to! 'n standard costing system, standard costs Standard costs are usually established! Budgets may be based on previous year's costs
Estimated costs are used as Statistical data 5. Basis.
being paid to efficiency.
accounts. a usually incorpora
/ are ¢ ted ::nto th e accounts, | for comparing with actual fi
ures. S' after considering such vital matters as/ without any attention
tam which variances of actual from! are not entered ia the books production capacity. methods employed and: -
f wcouge
on standard are asce-tained, other factors which require attention when
maces
4 a. Use. Standard costs are meant to be used fora | estimated cost: determining an acceptable levei of
i . se ee
cancer
system, aperating on a standard costing | gperati efficiency.
berating on aany he eecostin system.
historical any concern of cost’ Budge: is a projection of financial accounts.
6. Projection. | Standard cost is a projection
5. Purpose. ee
Standard-costs serve the purpase of cost | accounts,
Estimated costs do not serve the purpose
i control. of
cost control. Such costs serve other purpose
like quoting selling price of new product
s
ADVANTAGES OF STANDARD COSTING
woe eee bo s,
decision to buy or manufacture, etc,- of standard costing will vary from one business
advantages to be derived from a system

eR
The
achieved and the acceptance by the
Standard Costing and Budgetary Control — Comparison to another. Much depends upon the degree of sophistication
ges are as follows :
management of utility of the system. Possible advanta

MERA
ental costing and budgetary control have the common objective ge of standard costing is that it facilitates
of cost control by establishi 1, Effective cost control. The most important advanta
BP rmined targets. The actual performances are measured and compared with and
actual performance with standards
the predeteminede the control of casts. Contzol is exercised by comparing

IN
targets
.
taking action on the basis of variances so revealed
q 4 Or contrel
tr p pu es. a Bot h
Poses. tl @ te chnique: 3 are of importa
p nce in their respecti
P veé fields and

planni ng. Establi shing standar ds is a very useful exercise in business planning
2. Helps in
Points of Similarity of thinkin g in advance.
which instils in management a habit
ves and motivation to work with greater
1.: Provides incentives. Standards provide incenti
ig ete
T here
L T
are certain basic princy7p 1 BS wh ICH
i h are common te both standard costin
i g and
ted to reward those who achieve or surpass the standard. This
effort. Schemes may be formula
budgetary

increases efficiency and productivity. ;


1. The establishment of predetermined targets of performa
nce.
4, Fixing prices and formul ating policie s. Standard costs are a valuable aid to management
2, The measurement of actwal performance,~ ~~.
ion policies. For example, prices may be fixed by
tA

in determining prices and formulating product


3. The comparison of actual performance with the predetermined cost. Similarly, standard costing furnishes cost
targets adding a standard margin of profit to standar d
4. The analysis of variances between the actual and the standard
performanc estimates while planning production of new products.
5. To take corifirective measures, where necessary. off-standard performance
a 5. Facilitates delegation of authority. In order that responsibility for
ation chart is prepared which
Points of Difference
a
may be identified directly with the persons concerned, an organis
” ty and establi shes respons ibility of each executive.
shows delegated authori
In spite of so much-simiil it
__MSpite ol uch‘similarity between standard costin nation . While establi shing standar ds, the performance of different
: 6. Facilitates coordi
important differences between the two, which are as talons taken into account . Thus through the
? Ngetary contre, there are some departments such as production, sales, purchases etc. is
coordin ation of various functio ns is achieved.
working of standard cost system,
Standard Casting and Variance Analysis 3.5
Management Accounting
3.6
7, Eliminates wastes. . By fixing standard, , certain wast e such as material
i w i i
In other words, any variance that has a favourable effect on profit is favourable variance and
lost machine hours, ete. are reduced, *stage, dele time,
any variance whick has an advetse or unfavourable effect on profit is unfavourable variance.
; 8. Valuation of stocks. Standard costing simplifies the valuation of stock because the stock
Many students experience difficulty in ascertaining whether a variance is favourable or adverse.
is valued at standard cost. The difference between standard and actual cost is transferred to a
In the formutae given in this book, positive (+) variance will indicate favourable variance and
variance account, This ensures uniform pricing of stocks in the form of raw materials, work-i
designated by
progress and finished goods. “ negative (-) variance will indicate adverse variance. Favourable variances will be
{F} and Adverse by (A).
3. Management by exception. Reporting of variances is based on the principle of management
by exception. Only variances beyond a predetermined limit may be considered by the management CONTROLLABEE AND UNCONTROLLABLE VARIANCES
for corrective action. This also reduces the cost of preparing reports. person, with tne result that
H a variance can be regarded as the responsibility of a particular
awn ; ; Tae + . .
18, Economical and simple. Standard costing is an economical and simple means of cost
]
of efficiency can be reflected in its size, then it is said to be a contraliable variance.
his deqree
accounting and generauty results in savings in tie cost of costing system. it results in reduction responsibility of the foreman concerned.
For example, excess usage of material is usually the
defective, the responsi:lity may rest
However, if the excessive usage is due to material being
in paper work in accounting and needs fewer number of forms and records. This leads to
considerable saving in clerical labour. non-dete ction of the defects.
with the Inspection Department for
beyond the control of management, it is known as
LIMITATIONS OF STANDARD COSTING If a variance arises due to certain factors
For example, change in the market prices of materials, general increase
uncontrotlable variance.
canta costing system may suffer from certain disadvantages. This may be because of lack insurance premium, ete. are rat within the
of education
| and communication aand
nd 1resultant misunderstandin g on the part of m i in the labour rates, increase in the rates of power or
the company. Responsib ility for uncontrol lable variances cannot be
Possible disadvantages are: ~ ° anagenat Saft control of the management of .
assigned to any person or department.
1. The system may not be appropriate to the business. is extremely important. The
The division of variances into controllable and uncontrollable which
2. The staff may not be capable of operating the system. on controlla ble variance as it is these variances
Management should place more emphasis
3A business May not he able to keep standards up-to-date. In other wards, a business may not tion and possibly correctiv e action. The uncontrol lable variances, on the othe:
require investiga
of exception” whereby those matters
revise standards to keep pace with the frequent changes in manufacturing conditions Firms hand, may be ignored. This follows the well known “principle
performance are investigated.
may avoid revising standards as it is a costly affair. , which are going right are ignored and any deviations from efficient
4. Inaccurate and unreliable standards cause misleadin g tesults and thus may not
confidence of the users of the system. ,
enj
Paley the
METHODS VARIANCE
ion are kept in view. If, for some reason
5, Operation of the standard costing system is a costly affair and smal! firms cannot afford it While setting standards, specific tnethods of product
of product ion is adopted, it will give rise to a different amount
or the other, a different method
6. Standard costing is expensive and unsuitable in j job order industries
i i which
i are mam3 i
of cost, thereby resulting in a variance. Such a variance is known as methods variance. Thus a
non-standardised products. . Macturing than those specitied. According to
methods variance arises‘ due to the use of methods other
“the difference between the standard cost of
“C.LM.A., London Terminology, methods variance is
VARIANCE ANALYSIS
-a product or operation produced or perform ed by the normal method and the standard cost of a
employed.”
Cast Variance—Difference between standard and actual is known as variance. Cost variance is Pp roduct or operatio
PB n produced or performed by the alternative method actual'y
the “difference between a standard cost and the comparabl
eee i
parable actual cost incurred i a period.
during i *
REVISION VARIANCE
be revised on account of unavoidable
Variance analysis is the process of analysing variances by sub-dividing the total variance i After setting standards, sometimes standard cost has to
wages, material s etc. The standard costs once set are not
such a way that management can assign responsibility for any off standard performance, Acc di “4 changes in prices of various factors like revision
factors. Rather a
to C.LM.A., London, Terminology, variance analysis is “the process of computing the amour to disturbed every now and then to account for these uncentrollable
to stand. This revision variance is the
variance and isolating the causes of variance between actual and standard.” An im ortant oo a variance is created and the basic standard cost is allowed
the standard cost originall y set and the revised standard cost.
of variance analysis is the need to separate controllable from uncontrollable vatianees A detailed difference between
analysis of controllable variances will help the management to identify the per vesponsibl tem eee te
:
for its occurrence so that corrective action can be taken. ‘ persons responsible
Thus
Revised standard
]
Revision .. ‘Original standard
| variance cost of actual outpal east of actual output
FAVOURABLE AND UNFAVORABLE-VARIANCES
nt which allows the standard costing
‘ were the actual cost is less than standard cost, it is known as ‘favourable’ or ‘credit’ variance Creation of revision variance is only an interim adjustme
changes in standard costs.
nt e other hand, where the actual cost is more than standard cost, the difference is referred system to operate usefully even when there are
to as ‘unfavourable’, ‘adverse’ or ‘debit’ variance.

3.8 . Management Accounting
Standard Costing and Variance Analysis 34

Total Cost Variance


Example
A furniture company uses sunmica tops for tables. It provides the foliowing data -
Standard quantity of sunmica per table 4 sq. ft.
Material Cast
Overhead Cost Standard price per sq. ft. of sunmica €5
Variance 1,000
Vanance Actual production of tables
Sunmica actually used 4,300 sq. ft.
Actual purchase price of sunmica per sq. ft. % 5.50

f Material Material cost variance wiil-be ealculaied as uncer :


Price MCV = (SQ x SP} - (Aa x AP)
Variance
MCV = (1,000 x 4 x % 5) - (4,300 % 5.50)
20,000 - 23,650

u
v = 3,650 (A)
Labour § usage variance.
Material Material Labour idle The material cost variance may be further divided ivto price variance and
Mix Yield Mix f Time § Yield f
Variance Variance Variance § Variance § Variance | Material Price Variance
between
This is “that portion of the material cost vartance which is due to the difference
price specified and the actual price pard".* It is calculated by the following
the standard
formula : .

Material Price Variance = (Standard price - Actual price) x Actual quantity


MPV = (SP - AP) x AG
by actual quantity.
Thus, this is the difference between standard price and actual price multiplied
Expenditure or Efficiency Expenditure or 6 Volume
Budget Variance Variance Budget Variance RI Vatiance Example:
be calculated as follows:
‘ f
With the figures in Example given above, the material price variance will
MPY-= (SP - AP) x AQ
aed

MPV= (5 - 5.50) * 4,300


Efficiency Capacity Calender’ = = 2,150 (A)
Variance Variance Variance
price Variance. This variance usually arises due to the following
Reasons for Material
Fig. 3.4 Cost Variance Analysis reasons +

1, Change in the market prices of materials.


MATERIAL VARIANCES paid.
2. Failure to purchase the specified quality, thereby resulting in a different price being
Material Cost Variance . 3. Change in the quantity of materials purchased. thereby leading to lower/inigh er quantity
discount.
This is the difference between the standard cost of direct materials specified for the output
achieved and the actual cost of direct materials used. I: is calculated as : : . Not availing cash discounts, when standards set took into account such discounts.

OF Oe
. Inefficient purchasing.

ae
Material Cost Variance = Standard cost of actual output - Actual cost
MEV = 5C - AC . Change in the delivery costs.
One: . /
A . Rush purchases.
7 “ Material Cost Variance = Standard quantity ‘y Standard . Actual x one |
ny

_ Purchase of a substitute material on account of non-availability of the material specified.


nono

poe for actual output + — price quantity price


MCV = (SQ x SP) - {AQ » AP) , * C.LM.A., London Terminolegy
3.9 3.10
Management Accounting
Standard Costing and Variance Analysis

oe
se tax, etc.
g, Change in the rates of excise duty, purcha
products like jute, cotton, etc.
10. Off-season purchasing for certain seasonal

Material Usage {or Quantity) Variance

ee
jance which is due to the difference between the

in 2
This is “that portion of the material cos:
quantity used”,e Its formula is :
standard quantity: specified and the actuwl i guantty

Price
Standard quantity Actual. Standard
. . - x .

Variance *| for aztuc: outrut quantity | pre


Maienai Usage
| MUV = {SQ - AQ } x SP Quantity in sq. ft. 4,000 4,300
: . a the
the differe nce betwee n standa rd quanti ty and actual quantity multiplied by Fig. 3.2 Analysis of Material Cost Variance.
Thus, this is
standard price.
MPV = (75 -% 5.50) x 4,300 = % 2,159 (4)

ae
calculated as under:
il! be be calcu
i e will
usage variane
MUY = (4,000 - 4,300} x %5 = % 1,500 (A)
Continuing example given above, material

el
The difference between the areas of standard test rectangle and the actual cost rectangle i.e.
MUV = (SQ - AQ) « SP shaded area represents material cost variance, which has been analysed into material price
variance and material usage variance. The variances are adverse because the actual cost rectangle
= (4,000 - 4.300} x 4
is larger in size than the standard cost rectangle.
= © 1,500 (A)
Illustration 3.1
l usage variance may be caused by some
Reasons for Material Usage Variance. The materia
or all.of the following reasons : The standard cost card shows the following details relating to material needed to produce 1
kg. of groundnut ‘oil :-—
1, Use of defective or sub-standard matesials.
Quantity of groundnut required - 3 kg.
_ Carelessness in the use of materials.
Price of groundnut ,
NH

% 2.50 per kg.


. Pilferage.
Actual production data :
Fw

. Poor workmanship. Production during the month 1,000 kg.


. Defect in plant and machinery. Quantity of material used 3,500 kg.
product.
aw

_ Change in the design oF specification of the Price of qroundnut & 3 per kg.
ls, \
. Change in the quality of materia Calculate : (a) Material Cost Variance {b) Material Price Variance (c) Material Usage Variance.
want

. Use of substitute materials. (B.Com. Calicut)


_ Use of non-standard material mixture. Solution
d yield.
Yield from materials in excess of or less than standar
be

Basic Data
Q

. * Standard Quantity (S0}


Check = 1,000 kg. of praduction x 3 kg. = 3,000 kg.
material usage variance should be equal to Standard Price (SP) = % 2.50 per kg.
The algebraic sum of material price variance and
material cost variance, Thus : Actual quantity (AQ) = 3,500 kg.
“TMCV = MPV + MUV |
Actual price (AP) sti perky 20+!
3,650 {A} =€ 2,150 (A) #-& 1,500 (A) Calculation of Variances :
variances which has been prepared with the (a) Material Cost Variance = SC - AC
' Figure 3.2 shows the graphic analysis of material
-Rqures of the abave example. In this chart, the solid rectangle indicates the standard cost (4,000 - = (50° SP) - (AQ x AP)
‘ ne 4) and the dotted rectangle shows the actual cost (4,300 x © 5.50). = (3,000 x 2,50) - (3,500 x 3) = & 3,000 (A)
+
a

i
4
Standard Costing and Variance Analys
is
3.12 Management Accounting
(4) Material Price Variance = (SP ~ AP) « AQ
- 3} x « SP3,500
(2.50- AQ) = @ 1,750 (A) Solution
. Usage Variance
(c) Material == (SG
Calculation of Revised Standard Quantity (RSQ).
Cheek - = (3,000 - 3,500) x 250 = 4,250(A)
RSG of X a « 110 = 44 units
Material Cost Variance = Price Variance
+ Uszge Variance
3,000 (A) = 1,750 fA) + 1,250(A) . . 60
RSQ of ¥ “Too * 140 = 66 units

= =
Classification of Material Usages Varia
nce

i
Material Mix Variance = (RSQ - AQ} » SP
Material usage variance is further sub-d

REE REITER REESE


ivided inte ; Material X = (44 ~ 50) « 50 = $300 (A}
{a} Material mix variance
Material ¥ = (66 - 60) x 40 = %240 (F)
(b) Material yield variance. (Or Material
sub-usage variance) MMV “=(A)
%60
Material Mix Variance
Material Price Variance = (SP - AP) « AQ
This is sub-variance of material usage Material Y = (50 - 50) = 30 = Nil
variance. It arises on} y where more

ce
Material is used for producing the finish than one type of
ed product. A comp any may be Material ¥ = (40 - 45} » 60 = %300 (A)
materials
ials wwhich does not ¢ omply with using a mixture of
i the pred i ndard mixture. This gives MPV = %300 (A)

Se
material mix variance. rise to
presets’ sa
‘ue The materi al mix vari Material Usage Variance = (SQ - AQ} « SP
anlance isi defin
1 ed as that portic
porti n af the material7 usa
industriee fe no weeween standard and actual compositio i ich i Material X = (40 - 50) + 30 = %500 (A)
n of material it may wich n
final nerd ke chemicals, ntbber, , etc. et. where a Material Y = (60 - 60) « 40 » Nil
a numbeb r of raw materialsi are mixNed toon
componente Gf ee, from the stand MU = 500 (A}
ard mix may be due to non-availabilit
oportion , . @ mix or due to non-p y °f on rece 8
urchase of materials at Proper tinte, Inc

eee IP
nae or cheaper mateciats resuits in favourable se in the
m:x variance and vice versa th ise of Material Sub-usage (or Material Revised Usage} Variance
p ® materials in larger proportion results in mae
adverse variance jaNG use of move variance and represents that porticn of the
This is a sub-variance of the material usage
material usage variance which is attributed to reasons other than those which qive rise to
ae

oy rene
material mix variance. Thus the algebraic sum of this revised usage variance and material mix
Material mix variance = { Revised standard — Actual formula is :
quantity ~ want x Standard variance is equal to material usage variance. Its
price
___ MeV = (RSG - AQ) « SP Material revised (Standard _ Revised .
standard ia Standard price,
usage variance = (Geant quantity
The | revised3 standard quantity
ity isi nothing
i but the star i
quantities of all the materials. This is calc MRUV = (SQ - RSQ} « SP
ulated as under mare Pesponion OF total of actual
In Illustration 3.2 materiat revised usage variance is calculated as follows :
_ Standard quantity of one material Total of actual quantities
Total of standard quantities of | all materials MRUV = (SQ - RSQ) x SP
of all materiats
X = (40 - 44} = 50 = = 200 (A)
Ilustration 3.2 Y¥ = (60 - 66} x 40 = & 240 (A)
F rom ‘allowwing
the follo i MRUV = % 440 (A)
variances.
data, calculate material nix variance.
mix Also calculate price and and usage Check
Raw material MUV = MMV + MRUY
Standard
’ Actual
x: 500(A) = 60 {A} + 440 (A)
40 units
i @-3_50 per unit |
50 units @ # 50 ‘per unit
60_units @ % 40 per unit Material Yield Variance
60 unit:i 5 i
Totai "400 vnits
110 white Ober unit This is also a sub-variance of material usage variance. It arises in process industries, like
chemicals, where loss of materials in production is inevitable. While setting standards, the normal -
Ox standard joss is taken into account. But actual \oss may differ from norma or standard loss.
This results in actual yield or output being different from standard yield.
Standard Costing and Variance Analy Management Accounting
sis 3.14
33
Thus materiat yield variance is that
portion of the material usag illustration 3.4
difference between standard e variance which is due
yi eld specified and actual yiel to the follows:
d obtained. The standard. yiel product isis asas follow
output expected te be obtaine d
from the actual usage of d is the The standard mix to produce one uniti of
yield variance as used in stan raw materiats. it should ee
dard casting is the same thin be noted that Material A 60 units @ = 15 pet uniti = 900
in the other, costing syste g as abnormal loss or abno 80 units @ € 20 per unit - et
ms. rmal gain Material B
100 units @ © 25 per unit = —
One important feature of yield Material ¢
variance which differentiates .
(price, usage and mix variance it from other material 240 units
s) is that yield variance is an variances
out put variance while othe tia
co ASUMptLAN was as Ll
ouows
, yleld yarian ce represents a gain rs are Burin q the mom th ¥,
of f J July, 10 O 4 units er
were actually duced
pr oduce acu
or loss on output in term a
s of Material A 640 units @ % 17.50 per tntit = x 11.200
é
950 units @ 7 18.00 per uni * aes
Material B
870 units @ ¥ 27.50 per umit = 23,92)
Material C
Material Yield | Actual Standard Standard 52.225
= -
2460 units ns
x
| Variance yieid yield (BBM, Bangalore)
output price
Calculate ail material variances.
I

| MY = (AY — Si) x SOP |


!

Standard output price (SOP) is


the standard material cost
per unit of output. _ Materia
‘al Standard for 10 units : or ans
Actual
for 10 units an .
Tlus
a tration 3,3
During the month
aty. Rate Amit. :
of May, the following data applies: units z ee ze ee we tte
Raw material
Standard mix ee g.000 17.50 11.200
Actual mix
Sers Price Amount Units
eon e 6 ago 18,00 17,100
ip. Price Amount
oe = t Ka. z
ene “° »5 00 27,50 23,925
z 1,000 @5 , om
Xx
50 52,225
¥ Total
: 2,400
, ;, $0,000 | 4. ee?
4G
Total 7 100 | i
1, Material i
Cost Variance -= ae oe
Standard cost - Actua (Al
-2.225 { cost
Less: Loss if
. 30 | . at on
Yield MCY = © 2,225(A)
a ‘0 - Actual Price} » Oe v.
, ial Price Variance = (St. Price
The standard loss is 30%. Calc Material A = (15 ~ 17.50) x 640 -= 1.600 a
ulate: — =" <1
(a) Material yietd variance jal B = (20 - 18) * 950
(0) Material mix variance, = (25 - 27,50) » B70 = %2,175 (A)
Material C
Solution MPV = 1,875. A

St. 00Pricei)
(a) Material Yield Variance
= (A¥ - 5Y) x Sop | l Usage Variance = (St.Oty. - Actual Oty.) *
3. Materia
MYY = (74 - 70) x $0" = % 200 (F) . Material A = (600 - 640) x 15 = ss 000 (A)
,
“Standard material cost per unit of (800 - 950} x oe. - Post :
output is calculated as follows : . Material B=
= (1,000 - 870) * oe 3s i ;
Material €
SOP Standard material cost % 3.500
Standard output ~ i ce = { {Revised
i oh St. Price
St. Oty.a- Actua: L Qty).
(6) Material Mix Variance 7” @ 50 terial Mixi Varian
= (RSQ - AQ\ « SP
tot “mee Material A = (615* ~ 649) x 15 merit
“Material X = (60 - 55) x 25 = 2 100 (F) Material B = (820° - 959) « 20 a se are 0)
Material ¥ = (40 - 446).x 50 Material C= (1,025 - 870).x 25 =228 (
4
=F 200 (A) MMY = 7 900 (F)
_*, Note. In this case;
MMY = 2100 (A)
standard quantity and revised s tena,
is the same becaot
“actual quantity of all tandard quantity (RSQ)
the materials and total standard use total
quan tity is-the sanié, i.e, 10¢ unit
s.
Standard Casting and Variance Analysis
we
Management Accounting
3 15
3.16
"Revised Stz ndard (Quantiti y (RSQ) is calculat Oa
ed as follows:

Material A
2460
=-——— , Example
jag0 * 800) = 615 units :
The following information is given
i5
Mater ateri
al B = no "x Standard hours per anit
860 = 820 units 2 4 per nour
Standard rate
5 -
Materi Actual data:
- | aterial Co = a , * 1000 =1,025 4,000 uncts
units Actual production
» Material Yield Variance 35,300 burs
Actual hours
73.90 per hour
fi yteld va af ance
C #, certaini basici ¢ alcu
Actual rate
Icula { to r 5 a ay e h P macead q as fi follows:3
Calculate labaur cost variance,
Made

Actuale usageeof materials eee


Standard yi
yield. +2460
Stal ndard usage per unit: ofoupur * — “49 = 10.25 units Solution
= (SH for actual output « SR) - (AH » AR}
,SOPiP ‘St, Material
i cost per unit Labour Cost Variance
alerial Yield Variance
of output) ~ © 50,000 = 18 units = % 5 099 = (1,000 « 15 « 4) - (15.300 « 3.90)
= (AY -~ SY} «« SOP
vatetiat nes Loy = = 330 (F)
ce.
MYV = (10 - 10.25) « 3,000 = % 1.2 d into rate variance and efficiency varian
Labour cost vatiance is further divide
3
" ‘Sania Usage (or Sub-usage) Varianee
(urn
: r Quantity
i ~ Revised
i Standard Quantity) « Standard
Labour Rate Variance
Materia PriTice nce between the
atenia. A= (600 - 615) « 15 cost variance which is due to the differe
=& This is that portion of the labour formu la is:
en actual rate paid. Its
8 = (800 - 820} x 20 standard rate specified and the
ace.a! Rous
Tabour Rate Variance ~ (Standard rate - Fctuai ote)»
- ‘oon,
aterial C= (1,000 ~ 1,025) « 25
2% z _625(A)
wos,
ipv = (SR - AR} x AH
ee7 . lied by actual
standard an d actual races of wages, multip
Thus this is the differenc e between
Note.
mn ee Either M YV or MRUV
isi caleudated. Th esein
two areae
always equal
hours.
{i} i MOV = MMV + MYV (Or MRUV) Exampte
= 350 (A) |
= % 900 (F} + F 1,260 fA) le:
(i Using the data given in above examp ;
ERV = {SR - AR) « AH
or
ny MCV = MPV 4 MMV + MYP (Or
MRUV) = (4 - 3.90) * 15,300 = € 4,530 (Ff)
T 2,225 (A) == 7 1,875 (A) + € 990 (F1 « = Usual reasons are -
1,259 (A) Reasons for labour rate variance.
the ana
LABOUR VARIANCES 1. Change in the basic wage rates.
payment.
ysis and computation 2. Use of a different method of wage
of labour variances is from the standaré grades specified.
: ce
uite simi 3, Employing workers of grades different
Lai bour Cost Varian quite similar to materiat variances
4. Unscheduled overtime.
This is the diffs
achieved ops &. New workers aot being paid at full rates.
variance as labour rates are usually
and the ac :
tual ¢ di tect labour cost incurr
actual ed. It is calculated specified for the activity uncontrollable
S$ under : Often, labour rate variance will be an market. hacked by negotiable strength
ions in the labour
Variance = ofSt actual
i Labour Cost f Vari cost.: Actual labour
labouroutput determined by demand and supply condit
to the use of a grade of labour other than that
union. Where this varian ce is due
of the trade
as non-availability of the labour grade
LOY = SC - AC
AY
accept a ble explan ations
specified, there may well be such
I ssly employ s a wrong grade of labour on a job, he may be
or, specified, But when a forem an carete
held responsible.
j
_ Labour Cost Varian ,
ce | Sthous
for actual omtput Strate Actual
clual = Act Labour Time (or Efficiency) Variance
per ‘or {to ® . eri" hour | | This is that portion of the labour cost varian
ce which is due to the difference between
labour

EN (SH SR) ~ (Alt « AR) | expend ed. This varian ce is calcul ated
we ee,
hours specified for actual output and the actual
as follows :
labour hours
Managemen: mecuaccny

Standard Costing and Veriance Analysis


3.17
al
Example hours,
ming that idle time -5 200
for _ Actual the above example and furth er assu
{ Pabour Efficiency Variance =|“ Mows Using the data give in
x Standard rate | would be:
actual output ‘ows then the idle time variance
} !
v2 200 «4 = % 800 (A)
not of labour
of labour cost variance and
Thus this variance is the e differ a
difference Al) x SRand actual time valued z
between standatd idle time varia nce is treated as a sub-variance would be 15,300 - 200
When efficiency variance, the actual time
me valued at stand ard yate. the n-for labour 15,1 00 hours. Thus
Example efficiency variance, ulated on the basis of
hour s. jabo ur effi cien cy yatiance will be calc
Us} . = 15,100 SR
sing the data given in above example ance = (SH - AH} *
Labour Efficiency Vari 00) » 4 = = 490 (A)
2 (15 ,00 0 - 15,1 d Idle Time
Efficiency Variance an
7
LEV == (5H for actual output - AH} x
SR x

l ef Lab our Rate Va tiance, Labour


= (15,000 ~ 15,300) x 4 = % 1,200 (A) In this case, the tota
iance. Thus
al to Labour Cost. Var
The € otal of labour rate variance and labour effici en Cy Variance would be equ + [TV
vatlance is equal to labour cost LCV = LR¥ + LEV
variance. Thus : (A) + 800 (A)
330 (F) = 1,530 (F) + 408
. .

ng Composit on Variance)
= 330 (F) =F 15530 (F} + % 1,200 (A), Labour Mix Variance (Ga ont y when more than
one grade of
Reasons for labour efficienc y Variance. Thi . : ila r to mat eri al mix ¥ ariance. ft arises er from thos e specitied
This variance is sim diff
:
the f cllowing This variance is usually caused by o ne or more of com pos iti on of actu al grade of wor kers
reasons : workers are employed
and the
1, Poor workin g conditio
iti ns, 8, eg.,eg, inad equate i it is calculated with
the help of following
formula :
:
|J
j i « xcessive heating, etc ctes
2. Defective tools and plant and machine “eng and vertaton . Revised stan dard ~ Act ua!
hours x Standard rate
. +- hour s
3. Inefficient workers. ° Labour Mix Veriance
4. Incompetent supervisior. LMV = (RSH - AH) « SR
5. x of defectivi e or non-sta ; ndard materials
- neste
ime wastbyed v actars factors like
like waiti
waiting for materials, our cost 0 { which
tools j the standard direct lab
: Clent training of workers. Ltd. man ufa ctu res 4 patticulat product,
Coates India :
, se maine break-down, olves the following
se manufacture inv
et
: anes in the method of operation is £120 per unit who Rac e Amount
- Hours
- Non-standard grade of workers. Grade of z qo
ssification of Labour Efficiency Variance
Classificati
workers 30 2 60
A . Oo.
20. 3
Labour
( efficie ncy vari
arlance B 120
50
is furthe td 1ivi
¥. d e Md in
i to the following sub = varian ce ae

t of which was
fhe actual labour cos
(6) Labour mix variance
units of the pro duct were produced,
During a period, 100 ee
‘ies {c} Labourur Yi Yield Varia
i nce (or Labour revised-effici i as follows 2 e Amount
Hours Rat
le Time Variance
ee Grade of Fo
oo
This Variance Tepresents that tha portion
1 workers 1.50 4,800
of the tahour efficiency ’varia hee which 3,200
idle time: ‘ such
. i
as time
is du é to abnormal A 4,00 7,600

"12,400
lo s t due
ue , to machi
: ne
i break
;: down 7 POWer
OW failure e Strike 1,900

5,100
by valuing idle
et c. I t 18
i calculat ed B
hours at standara rare Thus

2Idle e Time
Ti Varianc
lanee = Idlele hours
hours x Standa
Standanrd rate {b) Labour Rate Variance
Variance
—_ _fIV = TH
x SR eqtculate (a) Labour Cost Labour Mix Variance.
Variance... . (dq),
As idle ho urs t€; p Tesen
{c). Labour Efficiency
es t a i Oss ie idle tim é vaitance is atwa vs u ayavo
i u F ‘abl e. <
Some accou:
Quntants do not treat Idle Time Vari :
treat it as apart of labour " alian¢e aS a part
vari
P
ost variance : Part of labour efficiency mcy Variance but
. ,
Standard Costing aad Variance Analysis Management Accounting
enna ee
3.20
Solution
=
Example
sts Brade of - Standard for 160 units 3.5.
Actual for 100 Using the data given in Iltustration
= (SH - RSH) * 38
; worker Labour Revised Efficiency Variance (A)
ae Hours Rate - 3,060} x 2 = 7120
:ft Amt.
i Hou Ts fate Amt. Grade A = (3,000
3 = %120 (A)
Grade B = (2,000 - 2,040) «
A; ne,
3,400 2
LREY = 2240 (A)
6,000 3,200 1.50 4,800
C _ : 3 6,000 1.900 4.40 7,600 Check
ja 000
LEV = LMV + LREV
12,000 5,100 240 (A}
Z 100(A) = % 140 (F) + %
12,4
reveals
(a) Labour is quite simil ar to Materia. 1 vicl¢ Variance. This variance
Cost Variance = $C - AC
— Labour Yield Variance. This yield being more 0: jess than the standard yield.
:
actual output or
LEV = 12,060 - 12,400 the effect on labo ur cost of
{6) Labour Rate Variance = {SR - AR) “AH
Tts formula +is:
rm “ St. labour cost
A = (2 - 1.50) x 3,200 (sua _ St. yield
=% 1,600 (F) = from actual tnput- per umt of output
B = (3 - 4.00) « 1,900 Labour Yield Variance yield
= 7 1,900 {A)
LRV = &
Illustration 3.6
(¢) Labour Efficiency Variance = (SH - AH) x SR 500 units.
° fo Standard output
A = (3,000 - 3,200) x 2 =2 400 (A 450 units.
Actual output
& = (2,000 ~ 1,900) x 3 =f 300 2 1000 hrs.
Standard time
LEV =F _100 (a) 2 20 per hour.
Check Standard rate
ECV = LRV + LEV
Calculate Labour Yield Variance
{d} Labour Mix Variance " . tsi" = a) cae Solution
A = (3,060 - 3,200) x2 hrs + 50G units = 2 his.
| =% 280 (a) St. time per unit = 1000
@ 20 = = 40,
B = (2,040 - 1,900) « 3 St. cost per unit = 2 his
a ®% 420 {F}
Actual st. yield St. cost per urit 1
; ; -ylela | x of output
| yield
Labour Yield Variance =
“Calculation of Revised Standard Hours (RSH)
ae 2,000 (A)
= (450 - 500) » Z 40 = %

RSH = St Aours of the grade x Total actual hours Titustration 3.7


ina week for a job are as under ;
Total st. hours
t and the actual labo ur engaged
The standard tabour employmen -skilled Unskilled Semi
Grade A x 3,000 2,000 OS Skilled warkers
5,000 5,100 = 3,060 his. Grade B = *5,100= 2,040 his, workers workers
5,000 6
32 12
gang
Standard no. of workers in the 18 4
28
Actual no. of workers employed 2 1
3
Standard wage rate per hour 3 2
4
Actual wage rate per hour work.
we ek, the gang produced 1,800 standard labour hours of
During the 40 hours working
Labour Revised Efficiency-¥ariance = ( St.hours for Revised Caicutate :
actual output” st. ines) x St rate (b) Labour-Rate Variance
{a) Labour Cost Variance
(d) Labour Mix Variance
EREV= (SH- RSH) x SR (c} Labour Efficiency Variance (B.Com Hans Delhi)
(e) Labour-Yield Variance
\y
Standard Costing and Variance Analysis g
3,21 Management Accountin
i a
ne
Solution
A
Standard Actual St.out, te per hour of work
-
ctual
= (set et ‘ Jer x Stra
iance
ok
Rate A Labour Yield Var
Category
ioe Amount Hirs.* mount
of workers
Hrs.
z 5,040
g 3.9997 % 504 (A)
z
Ly = (1,800 - 2,000) *
Skitled dar 1,120
3 3,840 4.480
1.280
Semi-skilled Q 960
240
72
‘so
;
3 2,160 Check
led
Unskilitle
“Be
240 1 2 LCV = LRY + LEV
(A) + % 424 (A)
320 {i}
2,000 z 2,424 (A) = = 2,000
-
2,000 5,040 5,960 Ley = LMY + LYV
: tii) & 504 (A)
‘ Hrs. = No. of workers « 40 hours. z 424 (A) = % 80 (F) +
S
OVERHEAD VARIANCE
expenses. Analysis
7 5,040
: his. =2 % 4,5536
labour and indirect
St. cost of actual output = « 1,800
2,000 hrs
ate of ind ire ct materials, indizect dir ect lab our variances and
Overhead is the aggreq fro m tha t of direct material and sons for this
Labour ‘ariance
Cost Vari = St. cost of actual output - Actual cost
of overhead variances
ig different are mainly two rea
ult par t of var ian ce analysis. There to est abl ish because
LCV =% 4,536
: - 6,969
' = 72,426 is considered to be
4 diffic
rate for fix ed overhead is difficult cha nge in the
standard overhead this rate ever tho
ugh there is 10
Labour Rate Variance = (SR - AR} x AH 2 difficulty. Firstly. put will dis tor t is det erm ine d on che
ume of out ad absorption rate
changes in the vol lly fix ed ove rhe fer ent ways
Skilled = (3 ~ ¢} » 1,120 = 71,120 (A} rhead cost. Genera terminology and dif
amount of fixed ove put . Sec ond ly, there is conflicting com put ed for fixed
Semi-skilled = (2 - 3) » 720 =% 720 (A} basis of normal vol
ume of out
rhe ad var ian ces may be separately variance
ad variances. Gve variance and fou:
unskilled = (1 - 2} x 160 == 160 (A) of computing overhe The n the re are two variance, three hou r oF pet unit
le overheads . per
overheads and variab . Mor eov es, ove rhead tate may be
overhead variances
- ERV = 72,000. (A
methods of analysing ad variance analysis.
the se lea d to confusion in overhe variable overhead
variances
Labour Efficiency Variance- (“SH , ‘cr actual
al output -- AH} x ~ & SR “ of output. All bee n cla ssi fied into fixed and
ad variances hav e
In this book, overhe ses.
Skilled = (1,152 - 1,120) x 3 =% 96 (F}
n furthe r ana lys ed according to cau variance is nothing
but under of
and the the outset tha t overtiead yariances are
- Semi-skilled = (432 - 720) x 2 =% 976 {A} und ers tan d at wit h ove rhe ad
it is important to n bas ic ter ms used in connecti on
overhead. Certai
over-dbsorption of
Unskilled = (216 - 260) x 1 =F 56 (F)
hour oF per
LEV =% 424 | explained firs t of all.
Rate: Thi s ove rhe ad absorption rate may
be computed per
js calculated as fol low s :
St. his. for actual output are calculated as follows: =. Standard Overhead of abso t ption. This
the method
unit, depending upon
~ “oy
I
ad
_ Budgeted overhe
. 18dg
Skilled =so00 * 1,280 9 = 1,152 hrs.
Standard overhead |
rate (per hour) Budgeted hours
1.800
Semi-skilledilled ==
you 480 = 432 his, Or ‘ !

nedgove
Budget
_. Bu ead
erhe
Standard overhead __
1,800
Unskilled
ilted = 77
7,000 * 240 = 216 hrs.
ed out put (in units)
Budget
rate (per unit) ads. separate
and vari able overhe
Labourur MixMi Variance
i = (Revised st. hrs. — AH} x SR
are sep ara tel y computed for fixed ad.
Where overhead var
iances and variable overhe
Skilled = (1,280 - 1,120} «3 =% 480 (F) ° for fixed overhead
overhead rates ate to be compu ted
Semi-skilled = (480 - 720) x 2 = % 480 (A)
per hour is used
Unskilied = (240 - 160} * 1 == 80 (F When overhead rate e hefore computing
variances.
bas ic cal cul ati ons should he mad ated when overhead
are
LM =%_80 (B) : The following
(SH AQ) . It is required to be calcul
for actual output er :
‘(a) Staridard hours tt is calculated as und
of ove rhead rate per hour.
absorbed on the basis
Testing and Variance Analysis
3.23 3.24 " Management Accounting
Budgeted hours
SHAO =
A0 Budgeted output * Actual output Solution
(d} Absorbed (or Recovered) overhead
. al Outp
actu : ut
Budgeted overhead Rs. 20,000
Tate pe T hour St, overhead absorption rate = Gudgeted hours “Toounhe. 7 7 2 per hour
(c} Standard overhead
hou Tat
te per hour 10,060 firs. ;
{d) Budgeted St. hours for actual output = 10,000 uniis * 12,000, units
overhead
hou rs Tate per hour
(e) Actnal overhead = 12,000 hours.
_ Actual x Actual overhead
hours OCV = (42,000 x 2) - % 22,000
fate per hour
When overhead rate per unit is = % 2,000 (F}
used
Overhead Cost Variance is divided into variable overhead and fixed overhead variances.
The following basic calculations
should be made :
(a) Standard output for actua VARIABLE OVERHEAD (VO) VARIANCES
l hours (S0AH) -~ Iti i
are absorbed on the basis of overhead rate per
unit.'S ItTequired to be calculated
is cateutated as follows: when overhead Variable Overhead Cost Variance : It may be defined as the difference between absorbed
SOAH = Budgeted output vatiable overhead and actuai variable overhead. its formula is o
(in units} :
x Actual hours : Variable Overhead St. hours far ; St.variadle . Actual overhead
Budgeted hours

en
Actual x St. overhead Cost Variance (actual output overhead rate *~ cost
(0) Absorbed overhead
output ~ rate per unit VOCV = (Absorbed V.0. ~ Actual 1.9.)

(¢)
as Standard overhead = _ St.St. output
» St overhead This variance is sub-divided into the following two variances :
for actual time (a) Variable Overhead Expenditure Variance. This is also known as Spending Variance or

ee
(d) Fudceted overhe rate per unit
ad < Budgeted ote Overhead Budget Variance. This variance arises due to the difference between standard variable overhead
output rate per unit allowed and actual variable overhead incurred. Its formula ts:
{2} Actual overhead

es
— Actual ,, Actual overhead . : St.variable dei lcnal overhead
output tate per unit VO. Expenditure Variance = * 47
1 “a EXP overhead rate | hours > cost
Overhead Cost Variance |
tant i the total overhead
= (Standard V0, - Actual ¥0.)
variance and can be
ndar overhead absorbed and
total actual overhea (b} Variable Overhead Efficiency Variance. This variance arises due to the difference between
standard hours allowed for actual output and actual hours. The reasons for this variance are the
actual overhead cost."
Thus same which give rise to labour efficiency variance. Its formula is as follows:
from the standard overhead, absothi i i
rbed and is simpl variable |
formula is - Y¥ under or over-absg i - . _ Stdours for . Actual) x St.
nd eh D | VO. Efficiency Variance = | a cial output = hours } overhead rate =
Overhead Cost Variance
e

= Absorbed overhead - Actu


al overhead | | = Absorbed V.0. - Standard 4.0.
ocr « ( 5t haurs for . SE overhead |
Check
actual output ~ absorption rate} ~ Actual overhead {
VO. Expenditure >_ 1.0. Efficiency
Example . VO. Cost Yartance = Variance Variance
Budgeted output :
Budgeted hours its, Hlustration 3.8
|
_, Budgeted “overhead 1000 = Calculate variable overhead variances from the following :
Actual -overhéad Z 20,006 Budgeted. Actual
‘Actual ontput t2 bo 20,000 19,000
Output (units)
Caleutate Dverhead Cos 12 20 000 units,
: its 5,000 4,500
t Variance (OC¥). 1 Hours
Overhead - Fixed . € 10,000 10,500
Variable = 5,000 4,800
Management Accounting
ll me
Standard Costing and Variance Analysis
a
coe i Ane
jane eeneeemn
3.26 nn ere

.

Solution Tilustration 3.9


ances in Illustration 3.8.
Basic calculations :
Calculate fixed ovethead vari
St. variable Budgeted overhead = 5,000
{0) qyerhead rate * Budgeted hours ~ 5,000 hours = 91 per hour Solution
~
Basic calculations :
Budgeted haurs _ 10,000 5
(6) St. hours for actual 5,01 overhead
dutput
x Actual output
P = 70,000 «19,000 St. fixed _ Budgeted fixed
* Budgeted output Budgeted hours 5,000 hrs.
(2) gyerhead rate”
= 4,750 hours, -
2 Budgeted hrs. x Actual output
Calculation of Variances St. hours for
actual output Budgeted output
(a) Variable Overhead ; - ( St. Rrs. for Actual variabl
ead Cost Variance = | actual output ™ rate | overhead . 200 x 19,000 = 4,750 his.
20,000
(4,780» 1) - 4,800 = % 50 (A).
(b) Expenditure Variance (Actual hours x St. rate) - Actual variable overhead Calculation of Variances
= (4,500 » 1) - 4,800 = & 300 (A). (a} Fixed Overhead Cost Variance

(c} Efficien : _ 1 Sh dows for _ Actual St. hours for _ Actual fixed
rte overhead _
) cy Variance == laetual ouput hows « St rate - = cca output *
= (4.759 - 4,500) « 1 =% 250 (F) = (4,750 x 2}- 10,500 =? 1,000 (A)
Check
Expenditure Variance
vo.
Cost
.
Variance =
7 .
Expenditure Variance
an
+ Efficiency Variance (b) Fixed Overhead
Budgeted Actual
50 (A) = 300 (A} + 259 (E).
* overhead overhead
= 10,000 - 1,500 = % 500 (A)
FIXED OVERHEAD (FO) VARIANCES
me Variance
Fixed Overhead Cost Variance. . ItIt isi the difference
i between standard fixe (c} Fixed Overhead Volu
actual output (or absorbed overhead) and actual fixed overhead. Its fovnls e
ie cost fr St. brs. for _ Budgeted x Strat
> lactual outpu t = hour s,
FO. Gost Variance« Si.fiowrs for | St. F 2) Actual
«F2 = % 500 (A)
actual output rate} 7 fixed overhead = (4,750 ~ 5,000)
|
= Absorbed overhead ~ Actual overhead. Check
Variance
ure Variance+ Volume
Fixed overhead cost variance is sub-divided into the following two variances : £0. Cost Variance= Expendit
(a) Fixed Overhead Expenditure Variance. This is also known as Spending Variance or Budget
1,000 (A}= 500 (A} + = 500 (A)
Varian ce. It t ar ES€,
StS due to th e di i erence between bud G eted Fixed overhead and actual fixed Mustration 3.10
:
The following data is given Actual
FO. Expenditure _ _ Budgeted Budget
Actual
Variance * fived overhead ~ fixed overhead 12,500 11,000
L Production im units 5,750
ta b)y Fyet : : .
sverhead Volume Variance. This variance arises due to the difference betw Man hours
6,250
put and actual output. It is defined as that portion of overhead variance
| ises
which atises Gverhead costs : 13,000
due to the difference betw een standard cost of ove T head abso. rbed
12,500
b y actuat produc ti Jon and t he
Fixed
50,000 45,000
Variable
| when :
F.0. Fehane .( St. hours for _ meee!) ‘ | Calculate overhead variances
Variance actual out kourg jf * St. rate per hour is used,
\ {A} Standard overhead rate
per unit is used.
(B} Standard overhead tate
[oo

. _ _= Absorbed Overhead - Budgeted Overhead.


___ Manageme conn?
Standard Costing and Variance Analysis
38
per unit is used the same. Thus
Solution (B) Wher overhead rate , the results will be
A . . hea d rate used is per ho ur or per unit e are two different
Whether standard over be identical. In fact, thes
h the methods w ould
) When overhead rate per hour is used
calcul ate d by bot
Basic calculations the variances
the same thing.
ways of looking at
Standard: fixed overhead % 12,500
rate (per hour) Basic calculations:
6.250 hrs. = 12,500
unitss
= 12,500 unit Z 4 per unit
rate
Standard
andard variuiable Vi
cverh e ad rate (per hat ) =
@ 50,000 (i} St. fixed overhead
=
6.250 hys.
z 50,006
per unit
ad rate = 72,500 units.” % 4
6,250 hes.
5 tandard hours for actual output # ——————
x 11,000 units = 5,500 hrs (ii} St. variable overhe
12,500 units.
Budget t x Actual hours
Calculation of Variances pet
for actual ours « busgetededouthrs.
A Variable Overhead Variances (ai) St. output Budget
') Variable Overhead Cost Variance .
12,500 units units.
a x 5,750 hieg.= 11.500
St. hours for St . Ove _ 6,250 his.
wn) overhead
. cca output * cost iances
Calculation of Var
;
ia = (5,500 x % 8- 45,000 = Variances
(it) Overhead Variable Qverhead
Expenditure or Budget Variance “ony “ee
the ad Cost Variance ~
(a) Variable Ove overhead
= ania overhead - Actual overhead ~ St. rate} ~ Actual
°= (Actual
(Act hrs. x St, rate) -~ Actual overhead = (Actual output = % 1,006 {A}
= (5,750 x % 8) - 45,000 on = (11,000 « 4} ~ 45,000 ce
Expenditure Varian
7 = 46,000 - 45,000 = ” (b) Variable Ov erhead rhe ad ) - (Actual overhe
ad)
(i) Overhead = (Stand ard ove
Efficiency Variance ony, eno
(on St, output \ entre
- St. hours for Actual rate
i

actual oufput ~ tur x St overhead rate output ~ for actual hrs


= = 1,000 (F)
check (5,590 - 5,75) x % @ = % 2,000 (a) e (11,500 x 4) - 45,000iance k
ad Efficienc y Var
(c} Variable Qverhe - Standard overhead
t
a VOCV = ORV + OEV « Absorbed overhead
" St. output Standard
,000 {A} = % 1,00 Actual rate
*
(b) Fixed Overhead Variances + 82.000 » | output” for actual hrs,
(i) Fixed Overhead Cost Variance | 4 = % 2,000 (A)
= (11,000 - 11,500) »
iances :
FOCY « St. hours for x St. overhead Fixed Overhead Var .
rate - Actual overhead rate ed Ove rhead Cost Variance
actual output (a} Fix
x%2)-1 = St, rate: Actual
. = {5,500 Actual
Fate) ~overheads
i) 0
(i) Overhead Expenditure or Budget Vatiante (081) mane a) = | output
= Budgeted overhead - Actual overhead = z 2,000 (A)
.
e (11,000 x 1) - 13,000
(if) Overheadad = 12,500 - 13,000 a F Expenditure Variance
Volume Variance (ov) 500 (a) {b) Fixed Overhead ~ Actual overhead
= Budgeted averhead 500 =.€ (A)
f
. l ta hrs. for _ Budgeted = 12,500 - 13,000
Variance
\ al preduction™ — Ars. r St, rate (c) Fixe d Overhead Volume
\

Check = (5,500 - 6,256) x % 2 = % 1,500 (A) J ' “Actual Budgeted


vient) St. rate
‘ = (aime
. FOCV © OBY + OW 1 = z 1,500 (A)
2 (11,000 - 12,500) x=
2,000 {A} - % 500 (A) + & 1,500 (A)
a
3.30
ee St. inNo. 0, a Strate per
day
ee Actual No. of 5 7 wo rk g a |
1
2 | working day:
Stand ard Costing
t and Variance Analysis Calendat Variance hour
ed _ pudgete| x. St. rate per
Revised budget rking
Sub-divisision of Overhead Volume Variance hours are ext ra wo
Or
=
ho li da ys , but if there
of extta
rse because favourable.
ance is adve
¥ olume
: . Vaa nance . ed into the following variances :
ance will be
i is further sub-divid
§ i
th is va ri this va ri
Generally, day s), th en
© § less holi
2. Capacity Variance. | days (because
3.44 245
3. Calender Variance. Tiustration ation is gi
ver =
hat porti (ee w whi ing inform 12,500
is defined as “t abe artion of volume variance
‘ ll ow hour )
Fixed Overh ead Effici iency Va
fo
Th e
erhead tate (per
to on Thi s which si. fixed ov
25
refle d ciency
eased or reduusce $hone t arising from effiantity mradte or isbelodiwffertheent stfranomdardstandardt Budgeted ho
urs 11.500
cts th e in cr
o riance thor iow ws eftha t thecy acoftualworkquers 0 o
ys
working da 22
pentitn nec - This uofvashiegher tower ficien in production. *
Its formula is: St. No. of
y b a Actual hour
s
days
of working
Effc en Variance = fj
A ibse bed fixed overn 2 ad - a da a
Standar fi ; x ed overhead Actual No.
riance.
lendat Va
St ri ate |
Caiculate Ca
actual output - Actual hour 5) x
(S , s. for

+ 500
Fixed Overh ead Capacit y Va a1riance
fance This
Thi is isis “ “that portion of theThuvolu me variance which ick isi. due
th Solution = 127,500
= 25 days . of days
af ‘ * Actuai No
s y
; : ard”. when hrs. pet da y
sannedis tovaribe nccee arises
the standa t da
to workin g at
.
highe rar lower ert
than
conu St. No. of s > St. ho ur s Pe
lan than th es . po pl ue to dgeted ho ur
= 11,000
or less mand stil , power failure, et Its formula i
ee e e@ ca
ee e Revised pu x 22
actually
r customer- de
= 00 * 25
eae ce
a
e, un d 0 T ove r ' » OTC. is: ~ 12,500)
e idle tim ' er = (11,000
Yariance
fixed overhead - Budgeted overhead} Calendat = goo (A)
i| Capacity Vari ance (Standard
=C (Standard
(St }
—- Budgeted
hours) x § per hr
hrs. work
vorked t. rate
Method * st, rate
. Or (Actual
UT = MEE Alternative . per day
pe r day= St. hrs 25 00
rs ad tate x HE
Example: St. overhe _ = 500 hrs x Sp. rate
; ance and c acity i on
atti f st No.of | per day
Calculate Fi efficiency vari variance from informma given i
n (i No.oys wo rk ing da ys
10d overhead
3.xe
ap
= \ working
da
ustration AQ,
:
Variance 2 7,509
{A}
Ca le nd at ) * 2, 50 0
= (227 25
Solution yanance as
ca lc ul at ing capacity
ce od of
St hrs far Actual x St. rate acity Yarian . the math
Effici :
iciency Variance = actual siupur ~ feral) Revised Cap yan e is to be calculated 18 4 follows.
ndar val that case
When cale formula in
«= (5,500 - 5,7, 50) x % 2? = % 500 {A
fi ed . the new St. No.6 \ Strate per da
y
Actual No. of — working “| x
'
to be mo di
canaci “
= Actual — Budgeted
ce = ( workining da
ys
pacity Variance hours | * St. rate city Varian catcul
| hours
nevis’ Ca pa ce will be
e, ca pa cit y varian 2,900 (F)
= (5,750 - 6,250) x €2 = 1,000 (A) given ab ov
* z= 9) * z
In the Tius
tration
= (1 1, 50 0 - 41,000)
Check an ce
pacity Vari
= - Vari
Variance + Capacity anance Revised ca of August
:
e month
Vari
Velume ance th
= Efficiency (A) 3.12 io n fo r Actual
Z 1,500 (A) + % 1,000 yustration snformat
= % 500 (A) e th e following Bu dget
2 a
at portion of th which is dt s furnishe
d yo u 32 ,500
| may b defined as “th we variance vumber of
Vari
Galendar
dar ence Vari. ancetw. ee. Ik
n na y be th e b o xyz (td. ha 39,000 33,000
feto anthee differ
he days in
er of working dget ts spelied" Calendarar
and the : 30,000 50,000
the period bu end vari ce
rian actually ts)
da ys in ea l the . Cal
of working a ys being
va Output (uni z 45,000
volume orkiwng t arising due to a cau se.
icular holiday singse. i
Le , a de d nu mb er of
63,306
‘Mifweat ho
ur s = 90,000
on the death
Ho
t of due to extra
26
osre veb ud ge y i claredptional itmusten a nation
al Fixed over
head
m. otthhe " as on ar is es onl ce
25
(C4. {nter)
le‘ ager oF any , Calend ar va ri an ce
e laying do y in ex ne ce s be ca us e
Variab le ov erhead
ai holidays
are taken in
] to account whil wit the st
andard.
Work in g da ys
ces.
When calend ar variance is c alculated, the calculation of capacit y variance has t o be modified ’ head varian
Calculate over
$0 as to iT variance into t the a nal 5)
1S. Calenda T Vafldl
lance 1$ calc UL!l , e
th: s addition a
- weouy urd variance Anal
Moe
ysis
A ett eens ee
o
3,32
Solution 3.31

.
Basic calculations Yatiable Overhead Variances
: = Recovered Overhead - Actual Overhead
V ‘arian e (A)
ti ‘ost
=
=@ : ,000
Budgeted hours 36,600 = 65,000 - 68,000
Standard hours ber unit (A)
ae Fo Overhea woo
Budgeted units hour St. Overhead - Actual
30000 Gi) ¥.0. Expenditure Variance = % 2, :
St. hrs. far actual out -= 6666,000 - 68,000
put = 32,300 units « 1 hr = 32,500 , rhe ad - St. ove rhe a
= Recovered Ove
Standard overhead rate Bucgeted overhead (vii) V0, Efficiency Variance =% 1,000 (A)
per hour = Budgeted hours = 65,000 - 66,000 -
:
Check Expenditure , Efficiency
a apacits Calendar
F or fixed overheads 45,030
= 30,000 5 Variance
aren ance
= 2 1.50 per hour = ce *
(i) FO. Cost Variance Varian (A) 2, } + 1,800 (F)
1,250 (A) = 5.000 (A) + 750
+ 2.700 (F
For variable overhe 60,000
ad
St. FO. rate per day
* 32,000 = 2.00 per hour
Efficiency | Variance»
Capacity Calendar
Wariance
= % 45,000 = 25 days = % 1,809 (ii) £.0, Volume Varii ance = Variance
Recovered overhead }
= St. hrs. for actual output ( } r { } ‘
For fixed cverhead « St. rate ' { )
cy
Expenditure , Efficien
= 32,590 his. x % 1.50 = © 48.750
For variable overhead * Variance
= 32,500 his. x Fe
= 65,000 (iii} V0. Cost Variance = Variance
Standard cvathead
0 14):
For fixed overhead
= Actual houzs « St. rate
3,000 (A) = 2,000 (A) + 1.00
= 33,000 « 1.50
For variah.¢ overhe = % 49,500
ad = 33,000
SALES VARIANCES
x 2
lab our and overheads. These
= = 66,000
Revised budgeted
ot ial,
ing to material,
ina system,
companieies calculate only cost variances eae advantage of standard costing sy
hours = Budgeted hours
« Actual days Some
> : butbul to o
Budgeted days £ course, invaluable, :
While cost variances are concerned wi ith cost and
variances are, 0 late sales variances. leg variances affect
= 33000

a6: 31,200 hours, companies also calcual due to favourable or adverse variances, the sales iation
Revised budgeted
25 Theit effect on budgeted profit du in sales revenue I.e.. change s caused by either 4 varia
overhead
the budgeted : p rofit2 due ges 7
(For fixed overhead) to chan
. |
or sales quan tities.
= 31,200 prices
s :
ulattiing sales variance
x 1.50 in selling
= % 46,800
Calculation of varian methods of calccula
ces There are two distinct
Fixed Overhead Var method
iances : (a) Turnover {or value)
(b) Margi;n (or profit) methoe various main
(7) FO. Cost Variance
sales.
= Recovered Overhead
- Actual Overhead e and sub-variances of
= 48,750 - 50,00G The folto wing chart shows Sales Variances
(T) 70.
= %1,259 {A}
Expenditure Variance
= Budgeted Overhead ~ Actu
al Overhead
= £3000 - 50,000 = & 5,000 (A)
(#) E.0. Volume Variance Margin Methed
= Recovered Overhpad Turnover Method
- Budgeted Overhead
fv)
~ 48,750 ~ 48,060 = & 3,750 (F) L
Bo, Efficiency Vari
ance = Recovered Overhead ;
~ Standard Overheac Price Variance
= 48,750 - 49,500 Price Variance Volume Variance (es)
{v} FO. Capecity Varian = 2 750 (A)
ce ~ Standard Overhead +

- = 43,500 - 46,809
— Revised Budgeted
= & 2,700 (F)
Gverhead
Quantity Variance Mix Variance Quantity Variance)
(vi) Calendar Variance
= (“aa - eee) x
Strate per day,
~ (26 - 25) « 1,809 = & 1,800 Fig. 3.3 Sales Variances
(Fy
Standard Costing and Variance Analysis
3,33 Management Accounting
TURNOVER METHOD OR SALES VALUE METHOD.
Under this method, the following variances
are calculated: = (Ad - BQ) * SP
Sates Volume Variance
(E}
= (13,000 - 10,000) « 3 = % 9,000
1. § ales Value Variai nce. Thisis is
j the difference between the bud
valu 2 of sales effected duringi a period. This variance is calculated oe falle
as followsn, - ad the actual Sales Price Variance = (AP ~ SP) x Aa
. Sales Value Variance 2 (Actual sales) = (3 - 3} « 5,000 = Nil
- (Budgeted sales) | {A)
(2.50 - 3) * 8,000 = 24,000
Sales vaiue Varia vari nce results due ta One or more of the follo
OWIN
wi g reasons : Tetal = % 4,000 (A)
(a) Actual sales volume being more or less
than the standard sales volume.
sales volume variance. This is expressed i in Check
(6) Actual sales price nrica receiv ed beingbe: high
-ecoived Sales Value Variance = Volume Variance + Price Variance
sales price. This is
expressed in sales price variance. ser OX lower than the standard 5,000 (F} «9,000 (F) + 4,000 (A)
one type of product, a oudyet
(c) A: mix of products h as been sold which a company is sel ling more than
i is di from the standard mix. This is expre 4, Sales Mix Variance. When If actual sales of different
products
in sales mix variance, ssed budaeted sales of each product. mix variance is
ns eirent will be prepared to show the variance will arise, Sales
ean as budgeted, a seles mix and
2. 8 2es alume Variance, i Volume refers to the number of physical is not in the same proportion difference between the stendare
‘ ce, therefore, represents that portion of the units. Sates the sales volume variance which is due to the which sales are
sales value variance which is due te ‘ “that portion of of product group of
ifference between the actuat vol ume _and the of the quantities of each product
standard velume of sales. The formula the actual inter-relationship
is : ° the composed”.
|; Sales Vollume Varian ariancce = (dctual quanti anti ty - Budgeted quantity) x Standard price ee
formula +
| It is calculated by the following
» Standa rd sales - Budgeted sales ctua quantity ~ Revi sed stan dard quantity) * Standard price’
Sales Mix. Variance = (Actual

dard sales.
= Stancard sales - Revised stan
Reasons, Thi @ usual reasons for this1 Variarice are: 1. Ineff
:2, 1Unex Sake
pected
Preecomp ee
ective advertisi ng an d
sales promotion. or
etitn
petitiion. 3. Lack of propep r su Pervision
isi be
and control of sales men
is calculated as under -
es price variance ; is that portion
i of the sales value vari
‘ar Revised standard quantity
is due to the difference betw een standard i products . Standard quantity
formuta for its calculation js :
price specified and the actual price charged.
. TheThe | Total of actual quantiles of all of one product
. ities of all products
Fotal of standard quant
|
3 -
ales Price Varianc ; 7
e = (Actual price ‘ce - Standard price) « Adnan
- Standard price} « Actual qu an tia},
i L =
|
sales and
ce hetween the budgeted
The variance is the differen
L

__= Actual Sales - Standard Sales


me 5, Sales Quantity Variance. .
| the If actual
actual price
price i is less than the formula is :
1
the s standard price, possible reason m y be unfo
rese en conipetition, revised standard sales. Its
Pp may also have to be reduced if a larger number Budgeted) x Standard
of unit ts have to be sold. Sales Quantity [Rese standard ! price
quantity
Illustration 3,13 Variance ~ quantity |
A com pany marketini g a product supp
ties the following information - = Revised stardard sales -
Budgeted sales.
a 4
a budget ed ratio of products. Where
quan tity in
Standard sales “means actual sales and
Actual sates Revised standard quantity tity this variance is adverse
,
‘qua ntit y is more than the revised standard quan rable .
. the budgeted eted quantit y, it is favou
Price Amt. Qty. quan tity is more than budg
Price Amt when the revised standard
ee : z Units z = z | Check ance.
Price Variance + Volume Vari
39,000 5,6 3 15,000 Sales Value Variance ance.
Mix Varlance + Quantity Vari
8,000 2.50 20,000 Sales Volume Variance e + Quantity Variance.
Caiculate sales value variances. Price Variar.ce + Mix Variance
- Sales Value Variance

1
Solution — ; (C.A. Inter)
Sale Ss Value Variance
i = Actual sales - Standard sales
~ [{5.000 » 3) + (8,000 x 2.50)] ~ (1@,000 x 3)
= 35,000 - 30,009 = 75,000 (F}
yy
Management Accounting
i:
Sales Margin Mi x Variance
: - Thi i
on © type of product. Its formula is is as varia

follows arises ont y when @ company iis selling
i
sa
more than
3.38

" — mo Revised St. profit


i Sales Marg:g-8 Revi
«Revised standard’
te
Revised St. Qty.
ia,
| Mix Varinnes = ( Actuala | Product z
quantity ard x Sandard profit (units)
| ix Variance quant ’ per unit
. / .
!
~ Revised standard profit !
Sales Marginin Quantity Standa
i , oei rd profit
To a 4 1,200 1.4814.00
It is the difference between revised « 3,150 = 1,181 1.00
audgeted profit. its fornala : ised standard profit Ess 3200.”
anne and
| Sales Margi — : 2,000 484
. [ Revsed Standard Budvetoct* Soe 5.30
agin * 3,150 = 1,969
i Quant,
quantity quantity Standard pi afi Kay 3200
| “aneety Variance Per unit 2,165.36
i
bo
» Revised standard prop ~ Budget ed profit
Check
Calculation of Variances
{t). Total Sales Margi argin Variance=
‘ Pri é : = Actual prefit - Budgeted Profit
+ Volume Variriance 1. Total Sales Margin Variance
(it) Sales Margin Volume Variances, MiMix “ariance
Variance + Quantity Variance = 2,115 ~ 2,200 =f B5 (A)

Therefore ; == Actual Profit - Standard Profit


2, Sales Margin Price Variance
(tii) Total Sales Margi
argin Variance= Price Vari : - = 2,115 - 2,225 2% t10 (A)
Nuustration 3.15 anance + Mix Variance + Quantity Variance nce = St, Profit - Budgeted Profit
3. Sales Margin Volume Varia (F)
=f 25
The foilowi ng data is3 suppl i = 2,225 - 2,200
ied to you for the month of January
202 1 = St. Profit - Revised St. Proi:t
4, Sates Margin Mix Variance
= 2,225 - 2,185.30 = 759.50 (Fi
Profit ~ Budgete< Profit
5. Sales Margin Quantity Yariance= Revised St.
= 2,165.50 - 2,200 = © 34.50 (A)
Check
nce
= Price Variance + Veiume Varia
(i) Total Sales Margin Variance
% a5 (A) = % 110 (A) + & 25 (F)
nce
10,000 Variance = Mix Variance + Quantity Varia
{Volume
budgeted costs are: Ess ~
= 4.00 per unit
@,090
% 2506) = 59.50 (F} + 34.50 (A)
aleulate sales Margin vari ances, , ~ Ray - & 1.50 per unit. Mix Variance
Technique of Computing Sales
Solution Quantity Technique and Value
or value technique.
by using either quantity technique
(Assuming that the products Ess and Sales mix variance may be calculated cts being sold are
ique is used when various produ
Kay are hom
Calculation of actual, standard ‘and budgeted Sgeneous,
profits quantity tecknique is used here) 4) Quantity Technique. This techn g different types of cars. In this technique.
example, Maruti Car Company is sellin
homogeneous. For Tnis technique of calculating
Product {| Actual Profit is similar to material mix vari ance,
.
Per unit
Actual
teu St. profit.7 | Stand, : computation of sales mix variance Illustration 3.14.
| variance was used in
oes.

Standard | Budgeted
1 ” profit per unit profi v4 quantity profit sales mix variance and sales quantity are not hamogeneous.
is used when products being sold
2. Value Technique. This technique
2 4
etc. In this technique, revisec
a

(2 « 3) ° TVs, computers, airconditioners


For example, a company is selling
g z gales quantity) calculated.
standard sales value is {instead of
enn ”
ce wil
Ess3
- |{ sales price variance and sales velume varian
Under value technique, sales value variance,
1.00
a5
bos o _ we
1,000
a L0G mix varian ce and sales
ie 1,060
Technique. However. saies
be the same as calculated under Quantity
1,200
300
quantity variance may be different.
Totat SO Tlustration 3.16
the date given in
method using value technique from
‘Compute ‘sales variances by turnover
Hlistration 3.14
st
Sta: ndard Costing
i and Variance Analysis Management Accounting
ee TI

3.39, 3.40
Solution
. Solution Sales Margin Variances assuming products
vari As st ated ahove, sales value, price and vol ume variance will not be Rlustration 3.15 computed
anances well be different. These are 4. Quantity technique.
com ted as follows: ® affected. Only mix and quantity variance will be the same
Basic calculations are homogenous.
der this technique; price variance and volume ce will be
. 2, Value technique . Un iance and quantity varian
(2) Standard sales = Actual ntity tec hnique sho wn above. However, mix var
quantities sold v. as in qua
th :
salesdard= Stan dard sat alued at standard prices. different as calculate
d below.
(5) its standard Stan
Revised TT sales
es value rearranggeded ini the budget s and revised standard
Pet Sales ($5) ed ratio,
Calcutatio n of standard sale
Product ;
(RSS Revised Standard Sales _
Actua? eet ete Standard sales _ Standard Sales Ratio Amount
_ | aly Ssdara Pnice ae atic Product Standards
= Price Amount <
:
¥
‘| Actual aty. ‘%
1,200 ; 25% aty. z z
, 6,000
1,400 ‘ 75% 5.000 * :
Total 14,000 _15,000 *
20,000
20,000
ses
eos
Revised standard sales = Bucgetee sales of individual products
Total budgeted sales — * Total standard sales Total
standard profit
Calculation of revised
x. 3.000, Revised st. profit
Revised st. sales
0 St, rate of profit
= 35000 * °0.000 = & 5.00 Product . 3
2x«3
Z 1,224
| 20.0 i 6,120
y — 23.000 1/5 = 20% 1,020
000 = @ 15,000 Ess 4,080
20.000 0.50/2 = 25%
Kay 2,244
Calculation of Vari . ne
a

~ vee
Sates Mix, Vari ance = 5S = RSs Total it
= St. Profit - Revised st. prof
: .
Fe
Sales Margin Mix Variance )
oy = 2,225 « 2,264 = © 19(A
= 1,000 (F)
"= 6. 000 -- 5,000
y
= 14,000 - 15,000 = 1,000 (A) = Revised st. prof it — Budgeted profit
L Sales Margin Quantity Variance 44{F }
. total= M
a . - 2,20' 0 =%
= 2,2' 24
Sales Quan ti = RSS ~ BS
; ity Variance
.
Variance
== 5,000 - 5.000 = Check Mix Variance + Quantity
000 = NIL - 0 Volume Variance = 4408)
¥ = 15.000 - 75,000 = z 25(F} =< 19(A) +
; Me
DISPOSITION OF VARIANCES
Total
, Check al= Ni vat
* : s of acco unt and are used only as a statisti
cal
Sales Volume Varitiance = Mi ‘ red in the book
matte * Quantity variance
cost s are not ente How eve r,
When standard the vari ance s.
=e
Ta end of the period for
ired at. the ledgers, there
Ml + Nil are men
adjust are requinto
tsrpor the accounting system t hrough journals and
Note : It should be noted that unde r in sian | whenrmat
info standar
ion, d nocosts inco ated
variances at the end of the acco unti ng period.
we
wit ways be zero because-it
jp tnover Method, white using value
t chnique, sales ndgat
mix eayarinee arises a question of adjustment and disposition of
is based on re-arréngement of standard sale . : no hard
~- . . of yaniances. Therefore,
he
total standard sales and total revi
standard sales represent the sa of opinion
:
as to the proper: disposition of d-sposition of variances
Use of Quantitity Techni que and Value T
me figure. ; There is no uniformity
down in this regard, The following methods
i
echniques under Margin Method
; and fast rules can be laid in certain firms.
7 nethod of sates variances : are based an pract i ce followed
Illustration 3.17 t ee
een 37 . ~
it
.
1. Transfer to Profit and
Loss Account
unt at the end of
the
gal
Compute e sales variances using quant : h | to Profit and Loss Acco s are
and valu
transferred cost of sale
shed stock and
¥ technique are
variances
given Illustrat
ion 3.15
Under this methed, alt ess and Aini standard costs
stocks of work-in-progr
4
accounting period. Thus, the
method is bas ed on the as sumption that inefficiency.
cost s. This dition s of
maintained at standard and alt forms of var
iances represen t con
teal costs
represent correct of
rd performance.
waste and below: standa
Management Accounting
Standard Costing and Variance Analysis ee
3.41 3.42
as ee of
The method h as the advant age of prompt and uaifo ™m valuati : This ratio indicates the extent
: : calender ratio is also calculated. the following
presentation of variand es as a separate group of items
. in the P an of inventories. Moreover Calender Ratio. So qmetimes pet ‘od. It is cakculated by using
on profit< or loss Tiafc led during ¢ he budget
‘ and Loss Statement reflects
attracts the attention of the manageme . t actual working days avai
TIL, ,
formula :
2A
. llocat Ion
int he budgdget od _ © 100
e peripeniod
Actual workinI g days i
oO f Variances to iy ¥ entor 1e: $ alt d Costs of Sales
| cale nder Rati o =
se the budget pertod
Under this. ime thod, vari . Budgeted working days in
and cost of sales. This will el are distributed over stocks of work-i of sales at finished stock ee
ae
showing inventories and costs actual costs. This (oe
method is based on the premiseise hethat standard costs are only tools of vontral at
th e true ard do not represe nt Iliustration 3.16 specifications
are re a
tr Me OY eas
nt of yyz_Co. Ltd. Their
i costs. A Ac cording to 0 th is rd ™m thod, th 45 actual u
manufactured in a departme
costs should be reflected 4
in the
Two arti cles A and Ba re hour , The budg eted production ¥
s of A an a8 unit s of 8 can be produced in one ng the mont h is auc
3. Combination Method show that 2 unit 8. Actual production duri

widbome siesta
0 f A and 1,600 units of There were 25
for a month is 400 units spen t in pro ductio n wer e 480.
Both the meth
ods explai
.
apply, but vari units of B. Actual hours were 26 during the month.
in. accordance with the pialned above are quite easy to units of A and 2,000 Howe ver, actuall y worked days
acrouating practice. The best and most lo lances are not disposed of working days specifie d in the budg et.
first two methods by anal to the vausee 4 a is to combine the ulate the following ratio
s +
according to the antesiying oo the variants according Your are required to calc
(Adapted)
existence and disposing them off Effi cten cy Ratio (d) Calende: Rano
sons for their . ity Rati o (b) Capa city Rati o {c)
{a) Activ
All
! those varie ances: which result f clors
executives, ~
which reat should ireb nasterred to Profiftit and
and Loss
Less Accen
Account. On the ‘other ha a those vata Solution
|i cost of goods wl
wien sone . nay inconteoltable causes
ample. vatianshould be atlen: Basic calculations i
l : roe. nveantories
variances month :
Budgeted hours for the = 200 hours
should be transfe and Lose Aceouee so A= 400 +2
in the market
tie 2 1600-8 =S200ehours *
(uncontrollable
: ae ould tb beerated
prorated toto ene Bo
inven toryieand cost DT goods
hangof node
na sold, Prices
ele 400 heurs
Total
CONTROL RATIOS Standard hours for actu
al production.
_ = 300 hours
| in ollin
contr addigtionope
to o variavari nce
nces, , certa
certain
i control ratioi s are common
A 600 +2
hours
+ 8 = 250
' 00% B 2,000 a
eae ae rans are generally expressed Wises 550 hours
indicates unforato vercentageff the ete Total
res a avourable position and versa, if th ratio x 1090
t. Bfficioney nant aneposition. Three impo : rtant contr
i less thon trove
; St. hours for actual production
¢ ol ratios ate given
fren bvelo : nom . 100% ‘ it =
hours
Budgeted
expressed aes oer (o} Activity Ratio
is defined as “the standard hours equivale) he
actual hours spent in production”. . Thus , this
atio
te show
tomss eh
whether
560 -
actual time taken inj ion j a —- «1005 :137.50%
200
of calculation is : production is more or less than the time allowed by th e standard. - Its meth od

Actual hours worked


| ers. Capacity Ratio See
1H

hours for actudl output xd


| Efficiency ratio = Standard
Actual hours worked
{b)
430
Budgeted hours

= ——
400 «1l00= 120%
2. Activity y Ratio
Rati It is defi
“the stand ard how c eeraent
defined asstandard i production * 106
expressed as percent, age ofis budgeted hours". This te ‘the work preduced
. . St.hours for actual
production faciliti 1e. ws the extent to whichch thethe Efficiency Ratio =
7
Actual hours
8 have been utilised as~eompared with that conteempla
ape
formula is - mpl ted in budgets! = 1 Its
{c) :
550 — 9)
430 x H0= 14.58%
a
Standard Bud
hours for actual output
Actual worked days x 100
Activity ratio =
>= 100°.
udgetary hours
Ra Tt cho
ti ou. —_
(d) Catender Ratio = — pudgeted working days
hour3.s. Capa citform
-Its acit Rati
y ula 2 it. shows the relationshi p between actual hours worked and the
budgeted = 26 x 100 = 104%
4
75
Check
capaci ratio = Atual hows worked Activity Ratio == Capacity Ratio = Efficiency
Ratio
: ted hours Budge x 100. = 120 % « 114.58 %
I 137,50 %
et et
g
Management Accountin
Stal ndard Costing
i and Variance Analysis

tho
3.43 3.44

Sales Variances
Material Variances
Gales Value Method = Actual sales
- Budgeted sales
iance
(1) Material Cost Varience (i) Sales Value Var - Budgeted sales
= St. = Standard sales
* (St. aty. for actual outpus ' cost of actual output ~ Actual cost (ii) Sales Volume
Variance
° = (AG - BQ) * SP
.
Pririce Variance Put« (St.« St. priceprice)= A- (Actualal : qty. x Actual price) Stan dard sales
(if) Material . e@- ctual price) » Actual aty. = Actual sales ~
iance
[
(te) Mater,ta ft
Maler sage
5 ig V arlan
r J Ace
= (S f‘OL ac tual outpe
u pet t - Act
(iti) Sales Price Var = (AP - SP) * AQ
Choa l ty , ® § t . price
Tl
ised st. sales
(tv } Mi q } ‘
= St. sales - Rev
Mate riial Mix} th arianc
a e = {Revis
1 ed $d - Actua l qty.fh }
yoywiaterial x St - price
iance
(iv} Sates Mix Var
rit
wy 5 ay « §°
Yield Vari
riance + fActual yield - St. . yield) x« $t St. output
price = (AQ - Revised s
1 { yt } Sal e 5 - Fudgeted sale
= Revised St.
M
Mate rr tal R evi
vised K bar ianin
Ur sage @=z (5 Of 0 tactual o EE Eput
put — VE.
R Revise $8.) Vartance ) * 5?
fv) Sales quantity
d x
Budgeted qty.
St. o ric 2
I
I
Labour Variances = (Revised sq -

(}} Labour Cost ‘ost ¥ Varia nce = St.


5 cost of actual output - Actual Sales Margin Me
thod - Gudgeted profit
= Actual profit
gin Vari ance
{i) Total Sales Mar
hrs
:
for
actual output « rs « Actual rat e j - Stancard profit
Put « Sl. rate) - (Actual hou
(St.
= Actual profit
(it) Labour Rate Variance Price Varinnce
per unit-St. pr
ot per unit} x
AQ
.* {$t,
wot, rate rate -- Actual rate) x Actual hours (ii) Sates Margin = {Actual profit
s
(i) Labour Effiecten i - h ° profit
= (St. hrs. for actual output - Actual Actual hrs.) » St. . ra Tate fit - Budgeted
Idle Time lariong Variance = Standard pro
tiv) rate ~ Volume Variance
€ = idle houzs « Standard (iii) Sales Margin = (AG - BQ) *
St. profit per anit
(v) Labour Mix/ Variance = (Revi - Budgeted prafit
.
sed st. hours ~ Actual hours) x St, . rat Tate, ce = Revised St. profit
(vt) Labour Field Variance
(iv) Sales Marg
in Quantity Varian BO} * St profit
unit
pet
; =
e {Revised $0 -
(Actual yield -
Revised St. pro fit
per unit of output
3t yie l d fo actual
= St. profit -
hour
U1 8) x Sk, cost
(v t } Mix Variance per unit,
Sd) * St. profit
oe
(v) Sales Margin
Revised Effici e cy Variance
(Aa - Re vise d
= (St. hours for actual Gutput - Re vised standard hours) x Standard
rat e
Overhead Variances
Vari Control Ratios |. 4a9
ariable Overhead (VO) Variances: al output
St. hours for actu
Actual hours wor ked
(1) Variable 0:
verhead Cost VariMande =
Absorbed overhead - Actual overhead (i) Efficiency Ratio
(ii)if VO Expenditure Variance ead actual outpat
Standard hrs. for hous
ing
= Standard overhead - Actual overh Budgeted
u VO Efficiency Variance =
» Absorbed overhead - Standard overh eada fii) Activity Ratio
ixed Overhead (FO) Variances
; worked | 499
(i) FO Cost Variance . Actual hours hou
rsrs
= Absorbed overhead - Actuat
overh io = —Fodgeted hou
cH FO Expenditure Variance
= Budgeted overhead = ‘Actual veeend (iii) Capacity Rat
at days e100
FO Yolume Variance
= Absorbed overhead - Budgeted ov: . Actual working
(iv) FO Efficiency Variance te g days
= Absorbed overhead - Standard wet (iv} Caterder Ratio Budgeted workin
7) FO Capacity Vartance s a
= Standard” overhead - Budgeted orat |
|L PROBLEMS AND SOLUTIONS
(vi) FO Revised Capacity Varia e
nce = St. overhead ~ Revised budgeted vee
ead
. StON
(vai) ¢
alendra Variance _ factual No. of
terial Variance s)
Problem 3.1 (Ma
) \ working days ~ working dey) x ne rte
compute: usage variance
er day
Or ng particulars, and (C) Material
(Revised b From the follow! 2 rice variance,
udegeted hows - B udegeted honrs) Mat eri al
a

hour, * St. . rate pe Y


variance, (B) 3,000 units
{a} Material cost ,
mat eri als pur cha sed = 9,000
4 Quantity of
als purcha sed
Value of materi
g
Standard Costin
g and Variance Management Accountin
Analysis ee
a
Standard quantity
of mater ~als requir 3.45
Standard price of ed Per ton of out
nateriat put 30 units
Opening stock of
Solution
waterials % 2.50 per unit * 70 kg = 3 go, oat kg
2 a 0 ,000 k g
Closing stock of Nil for actu at 1 o ow tpu
t=
materials Standard
Quantity (50)

Output during the _


period 500 units : i)
: WG0G mies c Be per eg.
80 tons ; ( ) : ( 252,000 28 as wa. Re 90 k
Solution Standard
c
=f
=
(B.Com., Hons. (AP)
Basic Dethi) Actual Price
calculations
:
a) Material Usage V - 2,80,000 kg.) a
Actital TUaatity
of materiel Piic = (3,00,000 - 2
hased
~% 28,000 (F)
= 3,900 units _ APY x AQ
Value of ma teri e Var jance = ‘ 9.90) x 280,000
als purchased = 2 9,000 (b) Material Pric
£)
Actual pric: e per .
unit =="
9,000 : 1 (sd SP) ~ (20 ~ » AP)(2,80.006
og 4 pez unit
. (c} Material Cost Variance = (3,00,00 ox -&
* G.90) == © 48,000 (
3,000 units B
Stancard price = 2.50 Trev % 48,000 (
per unit ee
Standard quantity
= 80 tons x 30 unit Check
Actual quantity s= 2,400 units tt
= Opening stock 1 ry ®© 20,20,000 (F)
muy
+ Purchase - Clo
Calculation of = Wil + 3,900 sing stack Meu 2 28,000
variances — 300 = 2 549 (F}
units
(2) Material Cos
t Variance = SC iances)
- AC TH 3.3>. (Material Var is i7 ( )
= (50 = SP) - (40 nd ) rd materiai
e Q Hrement
x ap) € k g- of gemca,
hi i nda
t e . sta
= {2,400 « 2.50) - (2, Fi or c
maki! q

MCV = % 1,500 (a) 500 3.00)


(5) Materiat Pri
ce Va Tiance = (5p js as Ut der
- AP) & AQ tia no fma
, ( als
teri )
con$sul
= (2.59 - 3.00) actu ait aM
MPY = & 1,250 (a) « 2.sog
produced. he
ood 0 kg 90 f G erm co we e
wet
|
(c} Material Usage During April.
Vari ance = (50 - ' Quantity (kg)
AQ) x sp -
5 oo
= (2,408 ~ 2,560) Vy
alt
tance.
x 2.59
Check MUV = % 250 (A) . b) M . teria
Pp i
rice Vai
c 4
Va ance, {c) Material an ht }

ost V arlance '


t
| late fetlia A
|a } M atef
{ alcu
MCV = MPY 4 AV
t 1,500 (A) = * 1,250
(A) + = 250
= 1.200 (A) = & 1599 (A} Solution
(A) . : Actual re 7000 kg.
Problem 3.2 (Material Var 2
Basic calculations | Standard for 100010 kg. — Amount
iances)
.
et

A manufact uring Rate Amaurt


information - concer which has adopted cy ‘
standard costing
furnishes the fol
Standard - towing
Material for 70
kg finished Produc
Price of Inaterial ts |
100 kg
Actual :
= 1 per kg
Output
iances
calculation of Var put - AC
= SC for ectual out
Material used
Cost of materials,
2,10,000 kg
2,86,000 kg (a) Materialial CosMCV
ian
t Variancce = ' 6,40 0 = 7,750 = © 1,350 (A)
Calculate me _. yarlance
; @ 2,52,000 = ~sp
(SP-2 7)AP)«75« AQ 0 == 750 A
{a) Material Usage variance,
(5) Materiat Price
variance, (c) Mat
(b) Material Price © B xf 3} = 500 = £500
erial cost Var
en
AY
iance, MPV = % 1,250 (A
)
(B.Com, Kerala)
Standard Costin g an. a Vari a nee
Anal :‘STS —_————
3.47 3.48
erial Usage Variance
{c) Materi
(50 ” BAB 2
= (50 - AQ) = SP = € 300 4 Material B=G
* 6 = 1.5 kg.

= (a00 = 750) x6
.
* 6 = 3 kg.
Material A=Q
B ~ 500) = 4 = % 400 th
=% 100 fA . 2
ae

* 6 = 1.5 kg.
Ss
Cheek MUV 5
; Material f=

_ MCV & 2,350 (A) ial Variances)


problen 3.5 pMater mixture is as
follows:
7
1.280 ( (A). t of a chemical
MPY ®€ 1,250 The standard cos at z 20 per ka
Problem 3.4 MUV F 100 (A) 40% material A kj iod
B at % 30 pet The cost records for a per
Variances) h0% material production.
TER 24 (Material aan is expectad in
From th s of 10% af
input
A standard los
:

nip ascmecrnetodiniee bf
information comnute
(a) Mix ots low ing usa ge: 48 per ka
(8) Price, and (c) Usage variance es: showed the fol al A at a cost of @
Ee
34 pet kg
|
ri al B at a cost of €
Standard
Act 110 kg mate uct.
: ual was i g2 kg of good prod (B.Com. Hens., Detht)
. , oduced
The quantity pr
Quantity Total I
ike ty Unit 1 Quantity
ae
price
all mat erial variances.
Calculate
. el z Solution
4 soo | 3.50
Material #00 400 | 2 ons: Actual for 182
kg. outpul
Msterat ¢ : 2.00 2.00 ?.00 Basit calculati
ka. eutput
'
taterial ¢ 1 2.00 Standard for 180

sis panei
4.00 3.09 . ' ty.
oe . 3 3.0
8 9.00 Material Amt. 1
__. Totat
8 16.00 ; ty.
Rate”
ae 8a 3.00 18.00
;
z zg , kg
18 1,620
Solution , kg.
16000; tt 88 3,740

rns
(B.Com,

: Ain
Hons., Dethi} 34
Materi . A (ogo 3,609 | 110
30 5,360

e:nesr
terial Price Variance
© 4 120 | 200

o nitata utapti
: \ 3 5,200
Material 4
B . ~ 3.50) x2 | 200 | 8B
Neri «1 * 3 (A) Total - _~ 5,360
=e (2-2) Nil » 20

e
- e | 182

al neat
Material ¢
= (4-3) x3 Less: 103s 5.200

s
3 (F) 180

dr nic
-

namnne
MPV =f
Material U sage Vari cS 2 (A)

pos
= (5Q - AQ) x SP
182
¥ 5,257.78
lance 5,200 “Tec 7
.
Material 4 - .. output = = zg
= i -2) 6] St. cos t of act ual
witeral B 1) «2
- 2 (F)
=(2-
2 (F) Variances AC)
aterial ¢ 2 Calculation of actual autput -
“(2 - 3} x4 . 4 {A) Vari ance = (SC of = 2402.22 (A)
ftv
*
o S 4, Material Cost 2 (5,257.78 ~ 3.360)
AP) « AG
- these S0~ 40) «59
Material Mix Varian
ce Variance ~ (Sp - 180 (F)
Material 4 “9 Material Price = (20 - 18)* 90
=%
= (3 *- 2x1 A 440 (A)
Matezial 3 =% 1 (F) Material 110 2k
= (Ld * - 2p x2 = (30 - 34) * 260 (A)
Material ¢ 3} x 4 mi 1 (F) Material 8 mpy ==
oS
2 (15 °-
2 cutput ~ AQ)
« SP
Revised standard q rantit t Yois ca tc ated
MP
Fa) 3. Material Usa
ge Variance=
(SO for actual
182 2g £18222 FA)
= (120 xo184 - x0)
as fo lows:

.. $ vtandard quantity of material Material 4


item
x Total actual quantity aye
Total standard-quantity 182
7 110) x ga -= @ 340.00 (E)

wov ~ 2157.7@)8
= 120 * 755
Material 2
Stand, .
and Variance
: rene castingi iiiraeiene Analysis
aime! a Management Accounting
4. Materi al Mj Varian
vate i ce = (Revised 0-
SQ - a. 3,50
AQ) g@
« SP
= ]| iance = (5P - AP) x AQ 45,000 (f)
9. Material Price Var -%
A = (% 10 - % 9) « 35,000
| Mate
| rial 8 = (120
am- e110)
n x 30
=_ 0g0G " = % 42,000 (A)
‘13 B = (& 5 ~ & 6} * 42,000 = 53,000 (A)
9. Material Yietg Variance = (AY
.. SY) * St. material c= (F 6 - % 7) x 53,000
a e per
cost unit a
shee MYV « SP
= (182 ~ 780) _ 20
185 Variance = ($4 - AQ)
= %5 7.78 (EF) (or Quantity} 56,000 (A}
=“ 3. Material Usage z 10 =
A = (30,000 - 35,000) « =? 10,000 {A}
B = (40, 000 - 42,0 00) * z5
MCV & 102.22 (A) ~t upto fal
4 G = (50,000 - 53, 000 )x = 6
(8)
4 Muv = 2% 78,000
Hl PY b rn te q
|

J — _
F 260 (A) ;
5 x 5P
= (Revised 5° - AQ) {A}
MUY & 157,78 (F) 4. Material Mix Var
iance
= (32,500 - 35,0 00) « 10 =F 25.000
[ct J
A
MMV % 100 (F) ng =@ 6.007 (F}
Oa
Be a - 42,000) x §
— (2.3) taera Variances) =-& . 0 (F)
7.00
€ standard material — (=3 ~33 sa0|
. } x 6
cost to produce " f=
3 One tonne of chemicai x js may = % 15,333 (A)
2 eae 8@e syerig,
300 kg. of material A@ z
as follows :
quantity is calculated
4 * Revised Standard
During a period "00 na C@e 6 per kg, A 130,000 30,000 - 32.500 ke.
33 Total AQ cq 1,29,000
35 tonnes of materia 4 4 chemical X were prodiiced from the usage of * “otal $0 00
42 tonnes of materia
= 9,009 per tonn at a cost of 4 0 kg. 1,30,08
130 = 62,
,0000 59,093 _1. “kg.
a B at a cost of 2 6.000
per to 8
1,30,000 49,000 = ae C = 420 ,000
53 tonues of mat B= 720,000
us
Calculate Material vari enial € at a cost of 2 7,000 per tonne 5, Material Yield
Variance (MYV)
unit of output
lances, yield) x *St. cost per
j Solution
(B.Com. Hons., Det = (Actual yleld - St.
hi} a0*
« 8,000 = % 66,667 (A).
Conversion Rate j - [100 - ieee)
ate is 1 tonne= 2,600 kg.
~~
|| T
Material Q
!
St oR
wa
Notes:
andard for 700 tonnes *Working
}
~ Actual for i % 8,00.000
otal standard cost
.
i
| | 08 ton
st of output
108 Tonnes
i
of, 0 Pu ‘Total standard output
“ee
* Std. cost per umit
= % 8,000 per tonne
3,00,00 f

|
B
35,000 1,300

=
2000 . ‘
Actual output
100 Tonnes
x 1,30,000 = ~7y7 tonnes.
c 42,000
3,25,000
Fatal sa x Total AQ ‘="7,20,000 le

ane
3.00.00
53,000 2,52,000 2. Std. yield =
Total ee
Less : Losssf | 3,71,000
20,000 Check MeV % 1,38,000 (A)
9,38,000
-
Toh O00 9,38, 000 MUV & 78,000 (A)
Calcutatigon ‘ : Mo
1. Material
of Variance es MPV & 60,000 (A)
Cost Varianes

MYV % 66,667 (A)


= $C of actual output
e
~ AC
MCV = 8,09,
90,000 - 9.38, 000 = © 138,000 (a) MMV @ 11,333 (A)
Management Accounting
5 tandard .
Costing and Variance Analysis
eee
3.52
>
Toblem 3.7 (Material Variances) 3.8 ‘Material Variances) is made up of:
Problent mical D
. ;
. of 1 00 kg. of Che
and 40 k cost for production & 4.00 per kg.
standard price of % 2 per kg
riial
of mateter A at material kg. @
al Bat a standard 30
s
price of & 5 per kg. were 2
. 80 kg : te ri The standard
e
ur 100 kgs of 2 O ma
of e used to manufact . emnical, Chemical A - 40 kg. @ % 3.00 per kg.
During a month, 70 kas. ma te ri A
al priced k 50 kgs. of materi % 6.00 per kq.
. 70 kgs. o f ced at @ 2.10 per g. an d erial B priced B 80 kg. @
at € 4.-5050 per kg. were actualt y used andpri the out put of the chemical was 102 kgs.
Chemical
. Chemical C fro ma mix of :
Find out the varia D was produced
material
eriances. {LC.WA,LA, Inter) 500 kg. of Chemical at a cost of % £88
Solution {na batch, “ 140 ke
55
Chemical A 229 kg at a cost of ¥ 1,0
Basic calculations : @ 2,8 5)
Chemical B 440 kg at a cost of
act ual cost per
Sr. cost of 160 ki g. of output A ctual cost of 102
k,
g. of output Chemical C e to the v ariance in the
|
factors contribut {B.Com.,
hi)
Hons. Det
mix and the price
7 ,
| R
Material Oty.
f Rate |I Amount | ty.
| Amount How do the yield, t he standard cost?
: | ate Chemical 2 aver
* 100 per kg. of
£9. z 4
pe por
A
sl required.
z
14 Ch emical D as
1
I 70 I
' 2.40 Solution for 400 kq. of
5
f
|
160
; Yar ian ces are © alculated kg.
a 4a a ee ee ; 4500 | 285 Basic calculati
ons : Actual for 100
- ! for 100 kg.
120 kg. p 120 kg.
i Standard Amt.

ene
- (360 : 472 Material
Qty. z
St. cost of actual output 38 x 102 417.60

St. output price = % 360ee = 100 kg = ba


% 3.60 211.20

Calculation of Variances 572.00


: Material Cost Variance = St. cast of actual output - Actual cost 900.80

ae at
Material Price Varia fice ==vist
(St. we
price - Ac tual price)
: prici ns
= Actual i C= “hag * # 6,50
=. 4.80
‘ = (2 - 2.10) » 70 == 7} i 488
> 2749 2 t 4.20; B= po

|
pet kg A
= (5 - 4.50) x 50 =F 25 0) Actual price
mical 2
per 4100 kg Che 440
Actual quantity 3920 Cee ~ 88 kg.
Material Mix varance
i = 44 kgs
= (Revised St. Qty. - Actualatta)Oty.)J +x* SP.OS. 140 B 5
A= “ = 28 kg.
‘ = (80 - 70) x 2 =% 20 (F)
= (40 - 50) « 5 =%. 50 (A) riances
Calenlation of Va = 7100.80 (A}
- 900.80
MMV =2- 30 (A) Variance = $C - AC = 800
Material Yield Material Cost AP) x AQ
i Variance = (Actual
(A i
yield i
- St. yield) x St. output price riance = (SP - 5,60 (A)
Material Price Va
2%
= (102 - 100) x % 3.60 = & 7.20 {F) = (4 - 4.20) ¥ 28
A -% 8.80 (F)
gi
Note :. 8Q giv
.
i sed St.
en 1s Revevi : iH
Oty because the total of actu al quantities of materials onsumed
B = (5 ~ 4.80) * 4488 =e 00 (A)
44.00
=t A4
c= (6 - 6.50) «
s and
that of standard quanti tie i
~
is equal. py = < 40.80 (A)
Check
MCV & 4.80 (A) - AQ) » SP a iy)
Variance= (SQ at
Material Usage 28) * 4
i
. ‘ A = (30 - “2t 20 (A)
B = (40 - 44) «5 t)
n ——».

“8
Lp
an
MPV = 18 (fF)
MOVE 22.80 (A) c = (80 - 88) x 6 80 (A)
uv =%
7 \ 1
. MMV Z 30 (A) MYV & 7.20 (A)
5Stitandard Costing
i and Variance Analysts Management Accounting
ee
3.54
Materi . "35a
aterial Mix Variance + (RSQ -~ AQ) «x S. §P ¥ 1,05,000
“2 160 x 5,606 Kg. = & 98.000
A = (32 ~ 28} « 4 Le
cost of actual output = 6,000 Kg.
= © 16.00 (F) Standard
Bow (425(242> - 44) x 5 =f 6.65 (A = $C of actual output -
AC
“= %17,500 (A)
Material Cost Variance
- . } MCY = 98,000 ~ 4,45,500
= (SP ~ AP) * Aa
OS = 88) 6 = 16.02 (A) Material Price Vatiance = 6,500 (A}
A = (20 - 21) « 4,900
as
3.000 (F}
MMW = & 6.67 (A)
=
RSQ (Revised
{Rei St, Quantity) is computed B = (10 - 8) « 1,500 2% 1,500 (A)
as belo We: C = (5 - 6} * 1,500
4 _ ibe MPY = € 3,000 (A)
= 32 kg
* 20 180 50 = as!
“q5q * 8¥ = 854 kg Ad) * 3P
iso (SQ for actual output -

5 it Material Usage Variance


“769 * 0 = 424 kg 37 « 20
A= [2222.00.50]
6,000
Mater 1 al ¥. Yel, d = ¢ 20,000 (A)
= (3500 ~ 4500) » 20
¥ arlance = (A Ys ¥) x St output price

MY = (100 - 1065): « 8 = % 53.33


(a) B= [= x 5,600 - 1500) x10
. 6,000
St. yield «2d, - -% 6,000 (F:
y 150 7 -00 = 1064 = (2100 - 1500) x 10
ot. dutpu
ce (2289 00.500)»

soso
pric
i e = O0 + =
Problem 6,000 (A)
3.9 (Material
2% 500
2h 500 (8)

es
= (1400 - 1500} x 5
Var iances)

PK Chemical MuV = 714, 500 {A)


rec PXE by mixing three ra
: s Ltd. M anufactures .
of P, KE, 125 kg. of raw mated
erials are used, In March 60 b tch materials, For each batch“fh of f 100 kg.
of 5600 k q of PXE. The Standard ard act atc. hes are = (Revised 50 - AQ) x SP
- te are as f to prod uce an output Material Mix Varlance Zi5-000 (A)
ee ual particulars f or June Prepared A =, (3,750 - 4,500) x 20

u
allows : = 7,500 (F)
Raw materials i | B = (2,250 - 1,500} * 10

-
i_____ Standard Actua az Mil

spaee
5
| Mix, % ped
1 Mix % | Quantity of c = (1,500 - 1,500) *
MMY = % 7,50 0 (A)
% Price per kg. | Price per kg. t raw materials i
wi z

gat seh
A ot « St, output cost per unit
purchased kg. = (Actual yield - St. yield
)
| 50 Material Yield Variance
B : 1,05,000
30 ~ z 7,000 (A)
MYV = (5,600 - 6,000) *"G apg
c
20 quantity of material
given. It does not give
of material purchased is rial ¢, it has been
varia,
Calculate @ ailail varia In the ques tion , q uant ity units . In case of mate
Note: 7,500
is thus assum e’ 4 that quan tity actually use dis it
tityy is
| onty 1,200 kg. while

LT Cree
consumed. It quan
Solution neces
@ 6 were op ening stock because purchased
assumed that 300 units
Basic calculations consumed quantity is 1,500 kg.

rR
s)
Standard for 6,000 kg Problem 3.10 (Material Variance helow :
output finished product are given
required for 1,000 kgs of a
Material | Kg.
The standard material input St. rate per kg.
_ | z Rate
: quantity
Material. v
A oo
3,750
c Amt. kg
20
450
75,000 P 40
3 | 2,250 it - 400
c “ Q
1,500
~~ 22,500
12, 60
ote reine |
000 250,
———— 7,500 R
co Total L 7,500 1,100
Or 1,05,000 400
115,500 Standard loss
1,000
‘Standard’ output
g
Management Accountin
Standard i and Variance Analysis
lard Costing
3.55 3.56

Actual produ clio na B period


i was
+" te ops
20,00 0 kg. of finishe d product fo T which
c
5,000
- 4,300) x“
go = 2 43.656 (FP
=
pri
* 22,000
———
t he actual qu 3 Htities |
used and the prices =
| 23,000
of material
Material
paid thereof were as under:
t 26,363 (F}
Quantities Purchase price’per kg HMV = . a
St. qutput price
(Kg.} = . Variance = cay - SY) *
P Material Yield 00,0
10,060 (v)
8,00,008
a 8,500
19
- 23,000 * 20,000'
_ {20,000 22,000 20,000
42 s
R 4,500 65 40 = % 36.363 (A)
Calcul ate (1) Materiaal Cost Variance (i?) Material
eri (v) Material Vid tate
Pri
Variance (iit) Material Usage Variance (iv) MYY = {20,000 _ sn 00)
latce
Material Mi ix Varian Price

Present a reconciliati
on among the variances, (ECWA Inter) Reconciliation
Solution = MPV + MUV
(i) MCV 000 (A)
Basic calculations : 39,500 (A) _ 29,500 (A) + 10,
+ MYV
= MPY + MMV (A)
__* Materiat ndard for —20,000 kg output !| (ii) MEV (F} + 36,363
- Sanda Actual
wat f for 20,000
. kg. output (A) = 29,500
(A} * 26,363
39,500
standaras
terial variances) . The following
. ; : vig ; ei
Amt. (ka ie
problem 3-11 (Ma ic raw materials
Amt.
ndi ng Cw o bas
e by ble
duces an articl
Philips Lid. pro
4
8
soon
900 - io 1,80,000 10,000 uP for Taw materi al. st, Price per ky.
ee
.
have been set Standard Mix 4
: 3,20,000 ; 8,500 ser oop Material

Satta, Soe.
. ae 3 3,00,000 |! 4,500 i 720
40% 3
A 60% produced 1,700
“ oo
2a 0,000 ! 23,000
5 00 aan
39,500 B Se pt em er 201 1 the company ber 2o1t
js 15%. During mon! th of Septem
s in processing c hases for the
. LOSS
" !
The sta nda rd Los sto ck and pur
ition af
— wee

output. The pos


| 3,000

ai
|

kg. of finished Purchases during


is as under : Stock on

ech etter
Calculation of Variances on Sep. 2011
—_ Stock
Material 30-9-2011
()i Material

Cost Variance 4-9-2011 Cost <
= 5C Ac Kg.
| Kg.
Kg.
300 2,400
7 339,500 (A)

|
M cy
(i) Material Price
Pri Variance
Vari
= 800,000
= (SP - AP) x AQ
- 8,39,500
re 5 3,000
1,200
ck
P = (20 - 19) x 10,000
50
The opening sto
= 7 20,000 (F) B . Ad
th od of iss ue of materials. Hon s. Del hi)
assuming FIFO
me (B.Com.

|\
Q = (40 - 42} x 8,590 = 217,000 (A) erial variances
R = (60 ~ 65) x 4,500 Calculate all mat price.
at standard
(33) Materi
aterial Usage Variance - (SQ - AQ) . .
__ 23.500 () is to be valued
Solution
ons ¢
P = (9,000 - 10,000} « 20 = % 20,000 {4} Basic calculati 100
. * 2,000 kg.
Q = (8,000 - 8,500) x 40 = % 20,000 (A) quantity = 1,7
00 * “ye
of standard
R = (5,000 ~ 4,500)" 60 (i) Calculation
2 0,000(F)
£3 00 * 4% = 800 &4.
Material A = 2,0
0
= 0,00(A)
HEV $1 = 1,200 kg.
(iv) Material Mix Variance = (RSQ* ~ AQ) x SP Material B = 2,0
00 x 60%

9,000 ption of materi


als Consumption
(ii) Actual consum “Chasing steck
P -{ 23.000 te 10,000] «20 + Purchases ~—-—"~ = 330 kg-
\ 7000 _ = 11,818 (A} Op. stock 5 kg.
Material + a = 1,190 kq.
35 ka.
q (23 , 000 « 8,000000 ~8 > seo) x 40

;
\
4% 5.455 (A)
A
B, 40 kg. + 1,200 kg.
50 kg.

22,
4
andard Castin
g and Vari
Te ance Analysis

(i; CalentatyUon of actuat


: price per kg. ee 3.58 gement A Accounting
Management i

Material q , 2400 3,00 Problem 3.42 {Material Variances)


800
= € 4.25 Bet,
00 ;
i
rrovemt te M :
L200 ® 2.50 4 XY? Ltd has established the following standard mix for producing 9 gallons of product ‘A’:
_(y) Calculation f
oS 0 standard and actual c
yp , z
Matenai ost of actual output
5 qallons - Material X at ¢ 7 per gallon 35 q
ao,
Actual 3 gallons ~ Material X¥ at = 5 per gallon 15 d
| dey
Rate Amount 2 gallans -~ Material
i Z at % ? per galton we 4 ‘

4
bene AG ? 7
Bes
i

4 Fg
poNw


3.200
aie cow 4.00 me J
; : /
A standard toss of 10% of impul is expected
;
to crcur. Actual input was
4
:
i
4.25 440 Material X - 53,000 gallons at % 7 per gallon f
B L.20¢
3,600 h 3378.75 ‘4 Material X - 28,000 gallons at 7 5.30 per gallon ;
4
“00 120 Material Z - 19,000 gallons at % 2.20 per galion 4
I pono
Actual output fer the period was 92,700 gallons. :
ioss 3 “40 2.50 2.875 :
“1 300 Calculate: {i} Material Mix Variance, and (#) Material Yield Variance, (B.Com. Hons., Delhi} t
Jog a

of woo
: Calculation - Solution
| Ta) vaten
erig CostVariances
Variance at
= Standard cost — Actual cost 700 6,513.75 Basic ic calculati
calculations
-
MCV= 6,8C0 - 6,513.75 ° . Standard Cost Aztua: Cost
GO) M — Rate
Price 1 Variance = (SP - AP) role.
aterial Price x ag -= (286.25 (F aty. Rate amt. Oty Ame.
Gallons z z Gallons z z
A = Ia - 4) « 35] + [4 4.25) .
= [3 ~ 3% 407 + (93 - ~ 2.50) x * 1,150)
795] St
== ¥& 7500
198.75
975.09 x 50,000 ? 3,50,000 53.000 7.00 3,712,000
Y 30,000 a) 1,50,000 28,000 5.30 1,48,400
fe) Material : MPy -
Sage Variance = ($0 ~ AQ} x sp = $376.25 (F) z 20,000 2 40,000 19,000 2.20 41,800
A = (800 ~ 830} x 4 : 1;00,000 540,000 1,090,000 561,200
B = (1,209 ~ 3,190) x 3 “f 120 (A) Less: Loss 10,000 7,300 .
(d) Materialsal MixMas Variance = (Rsq AQ) « SP MUV =*___
OF 9030 _(F}
(4 Output 90,000 92,700
92,700
x .
: = (808 ~ 830} x 4 Material Mix Variance = (Revised SQ ~ AQ) « 5P
= (121d ~ 1,190} x 3 - es (A) ¥ = (50,000 - 53,000} x 7 = & 27,000 (A)
asa 2.029 muy==
2866ee (F)r ¥ =- (30,000 -- 28,000) « 5 = Z 10,000 (F)
ue =Toa9 * 800 = 808 & Z = (20,000 - 19,000) « 2 = © _ 2,000 (fF)

2020
’ - g. .
Mav =%= 9,000 (A) :
Material Yield Variance = (AY - SY) » St. output price *
(e) Material viela ¥ “20090 * 1200 = 1299 kg.
.
a Hance = {Actual yield ~ St yield} x § % §,40,000
we t. Gutput price = (92,700 - 90,000) x $0,000 gallons
MYV = (1,700~ 1717) , 808 aes
170g ~ = 68 (A) MYV = 2,700 « 6 = & 16,200 (F).
St. yietd «48 - Problem 3.13 (Material and Labour Variances)
_
Tae x 2020 = L727 Kg,
Check From the particutars given below, compute: Material Price Variance, Material Usage Variance, if
HCY = MPY 4 MMV + Mey
Labour Rate Variance, Idle Time Variance and Labour Efficiency Variance with full working details. Ai
One tonne of materials input yields a standard output of 1,00,000 units. The standard price
286.25 (F) = 376.2525(F) + 22 {A) + 68 (A) of material is = 20 per kg. Number of employees engaged is 200. The standard wage rate per }
employee per day is 7 6. The standard daily output per employee is 100 units. The actual
tiag
Management Accoun
ee
Stand ard Costing
i and Variance Analysis 3.60
3.59 per ka.
% 1,350 + 90 kg. = £15
t of output =
quantity of materitial used isi 10 tonnes
t and the actual price paid isis ©z 21 per kg. . Actu al output {b) St rate per uni as follows :
culated
obtained is 9 00,000 units JA 50 ual input is cai St. 0 wtput price
3
is T 6.50 per day. Idle time wed fee
nea ie days worked is tim
above
wages paid
and actual rate of (C {c} St. yield for act yield - St. yield) »
or and included in variance = (Actual
eis 1/2 day
-A. Inter} Material yield st. yield} * 15
‘ ko -
Solution , 2 435 (A) = (90
3 = 99 ke ‘
/ tonne = 1,000 ka. gt. yield
Material -
KE. gg kg. = 110 kg.
output = WO
Price Variance = {SP - AP} x AQ woke
for 99 kg. of
» {2G - 21) x 10,000 kg. = = 10,000 (A)
(a) Actual input of chemical B
3 ~ Actual input
Material Usa ge Vari lance = (SQ - AQ) x SP ut af Che mic al A = 110 kg.
(e} actual inp 70 kg. = 40 kg. of Chemicets
— = 110 kg. - gst of actual mix
= {9,000 kg - 10,000 kg} x F 20 = % 20,000 (A) . Hen ce t he actual ©
. (A)
Labo“ur Rate Var ianee en as = 650
= {St. rate ~ Actual rate} « Actual days Yariance is giv kg. of
ih Material Cost 50 + zes0=% 2,000. the cost of one
be 2 1,3 is & 60 6. Thus
A and B will @ % 15 per ka.
Note . of Chemical A
: Actual days 50 di t of 40 kg.
. " rloyes a The actual cos as follows :
Labour : Effici ney Variance == (St.200 employees «80
x eee .
d is cal cul ate d
days - Actual iy)3} * St. rate
sat Chemical 8 use per kg
9g
| Y 40 (A}
a
z 2,000 - 2 60g .
Idle Time Variance = Idle
le daystyes x StSt. 1 te d 70 kg.
en r :
= (200 employees x4) xx % 6 ‘4 culated as unde
= 600 {A} Revised St- aty is cal 55 kg.
(A) 110 « 50 =
%
q Chemical A =
Prob
Problem 3.14 (Material Variances) % = 55 Ke
4 chemical B= 110 » 50St. (ky ~ Actual Oty.) * St, Price
ne akilogram
One
podutt ki ft ten : fect ‘ ¥ reas
of product :i
requires two chemicals A and B. The following were th 5 ance = (Revised
i
Material Mix Vari = % 180 (F)
(1) - 40} * % 12
ny sar mix Chemical ‘A’ 50% and Chemical ‘B’
chemical A = (55 = 225 (A)
x & 15
‘octet ' :
price per kilogram of Chemical ‘A’ =
~_ Chemical B =
(55 - yo)
2045
miey =
and
dh acta put of Chemical ‘8°70 kilogram * St. Price
Pang GEN EL - Actual Oty}
(Standard ary 1)
fe) Stand pce per kilogram of Chemical We e Variance =7 120
| Material Usag * zig
{2} kg. - 40 kg}
5 Monn normal loss 10% of total input ° chemical A = (30 » 2 15 = % 300 (A)
t) Materia cost variance (50kg. - 19 kg.)
total ¥ 650 aver
Chemical F = 2% 490 {A)
vos yleid variance total 7 135 ad se Qty.
UV ce) * Actual
ts oe required te calculate : ce - actual pri
_ = (Standard pri = 120 4A)
ce Variance =
(2 Mn mix variance (total) Materials Pri -2 15) x 40
chemical A= @ 12
Actual loss of actual input (3)
= 350 (4)
ai usage varinance (t |
(& 15 ~ = 20) * 70
; (a sett input of Chemical ‘a
(3)
_. Material price variance cout uu. a price
i per kiogram
i of Chemical ‘8’ chemicat B= = 470 (8)
(C.A. Inter)
\ _uey
al Input
Actu
Actual Loss of = 110 kg.
Actual output put isi is based (4)
The following solution input
ci .not
given inWorkin wo ti Actual total
that actual output is 96 kg ased on" the assump pilon tput = 90 kg.
4 : . i s sha wn as bel ow: : Less : Actual cu
(a) 20 kg.
St. cost is calculated below : j Actual loss a ve,
a kg. (as calcul ated abo
aty. —~ Inp ut of Ch em ical A = 40 ate d above}
Price (5) Actual z 20 {as cal cul
} Amount kg. of Chemical 3 =
XQ. z
.
z (6) Actual Price per MYV
Chemical A
4 D MCV = MPV + MMV +
50 600
+ 45 (a) + 435 (A)
Chemical B 50 ~~. 15
.
560 (A) = 470 (A)
.
—— __150
7 0 1,350
\
Standard -loss 10 -
Output 90 1,350
Standard Costing an
d_Variance Analysis5

Management Accounting
Problem : co
ene (Heater and Labour Variances) a
esti & standard cost on ¢
stimate d as under SC on ‘Materia
nai’l’ . [ 00
and ‘Labour’ ' for the making * = 230 (F).
of a unit
met of of 2 certain produc
t are
Dept. B =_ |{035 - 5,= 800 | x 5,800
Material
Labour Efficiency Variance = (SH - AK) « SR
Dept. A = (8,000 - 8,200) x 0.30 = 60 (A}

AW
— 5,868) « &.35 TO (Fy

AL
Dept. & = {6.g00

Check
+ Efficiency Variance
Wass
tua 4
dle a 5 Labour Cost Variance = Rate Variance
Dept. A 400 (F) = 460 (F) + 60 (A)
req ! T é€ d G analyse :
t
Ma erial and lab our VdTiances
1] i
Dept. B 300 (F) =230 (F) + 70 (F)
Material Variances
Materj al
:
Problem 3.17 (Labour Variances)
15 hours per unit and 20 hours
¢ ost t Va Tla
Jannce
e = Sta two products Mand N are
hours for manufacturing
dard
id Cc OST - Actua
u cas Standard rd wage rate
e identical kind of labour and the standa
sper unit respectively. Both products requir
= (SP x SQ) - (AP x AQ)
units of ¥ were manufa ctured . The tota
= (% 1.50 « 80 k per hour is € 5. In a year 10,000 units of
M and 15,000
000. This
9-)~ (2 1.75 « bill came to 23,00,
«=% 5 kg.)
of labour hours y worked were 4,50,500 and the actual wage
:

Material 120-2 131,25


prj Kee Vari lance :
hour and OG hours paid for @ % 7.50 per hour
@
7 per
included 12,000 hours paid for @ %
= (SP - AP) x ag
r variances.
hour. You are required to compute the labou
—_
ois
the balance having been paid at ¥ 5 per

aterial Usage Variance == ‘sr
(a ~ aa AQ) x gp ,"
:“S178 » {B.Com Hons Delhi, LO WA. Inter}

Siti batat
eo Variances 7
: TEST)
abeur Cost Vari = 50 Solution
lance * Standard cost - . - Actual cost.
. Actual cost : Labour Cost Variance = Standard cost for actual output
~ ASR = SH) ~ (AR
mz
. ws
x aH) ~
°
Standard cost : = = 7,560,000
200ur Rate Fer produc t M.= 10,000 units « 15 his.» %5
Variance
=. se ~ AR)ea«xn AH hrs. « 5 = % 15,00.000
") For product N = 15,000 units x 20

pie a
: fsee)
"
L abour Eff
Effici } Total standard cost = 2 22,50,000
iciency Varianee
== (SH (SH
ist ui).x

stele
veo - SR,
*
= (18 ~ 16) x 1,25 “f " Total actual cost = = 23,00,000
‘roblem 3.16 | (Lab ~ % 23,00,000 = 750,000 (A)
our Variances) Labour cost varianc e = % 22,50,000
= (St. hrs. - Actual hrs.) » St. rate
“ee ° Labour Efficiency Variance = % 2,500 (A)
(4,80,000 - 4,50,500) = 5
Fron the . fiolt 3 Wi ng ' a late la IOUT va lances |
|
"a
= (St. rate - Actual rate) x Actual hrs.
for dep. a riment
4 and B,
A :
direct wages \ Dept. A Labour Rate Variance
9,400] + [ (5 - 5) x 4,29,100}
an I

ae hours produced = 2.000 Dept. 8 | = [(5 - 7) « 12,000} + [(5 - 7.50) « = %47,500 (A}
8,000 = 1,800
Staae aid rate per hour
30 paise 6,000 )
ual hours worked Problem 3.18 (Labour Variances}
35 paise weekly wage rates of labour force engaged on
- ,
8,
°1200 5,800 The ‘details regarding the c omposition and the
Solution a job scheduled to be completed in 30 wee
ks are as follows :
Labour Cost Variance sC (CTA. inter) Actual
Standard
= SC ~ ac Weekly

i
Weekly No. of
Dept, " A == (3 (8,008 x 9.39
- 2,000 - Category of No. of
. < 0.3 )
“D = workers wage rate
(% wage rate
.
L abour Rate Variance ept. Boe oe 35) ~ 1,800 : soe (F) workers workers
per worker
per. worker"
~ (SR ~ AR} x AB “< 300 (F) 70 = 70
75 zt 60
Dent. A = (030 _ 2,000 \ Skilled,
% 40 30 = 50
460 ( z) Semi-skilled 45
. 8200) * 8,200 &nT ~% 30 8d z 20
. Unskilled 60
ounting
Management Acc
eee

Stan dard Costing


i and Variance Analysis
3.63 -% 3,600 (A)
= (1,350 - 3440) * 40 2% 3,600 (A)

ieev 41= 6,20(A)0


Semi-skilied
The work i $ actually y c completed ini 32 weeks.
| Calculate the all all t labour varia neces.
Unskitled
= (1,800 - 1.920) * 30
(B.Com. Hons., Dethi)
Solution
Variance 2 (AY - Sr) x SOR
Basic calculations: Labour Yield
5,760 = % 16,200
(A)
, —— x@ 2,635,000
a|l-
Standard
Actual Ww 5,408
categoy of
Hoots
workers oF 1 Rad Weeks
: | ° Ami. A mount
im of workers rkers " ate check
: LEV
ats
No. ohof weeks
» Ne | (No. of work Lev = LRV +
z 6,600 (A}
% 6,400 (A) + *
————____.. |
Re
es ‘* of weeks) ; .
(2)
z 13,000 (A) =
:
Saiiled
. 7 5k Ws
45 «30 2
2,250 | £0 1,345,000 7
Rev (Or LYV)
Sewui-saii Ds 232 = 32 = 2,240 70 1,56.80 LEV = LMV + (A)
(F) + * 16,200
wwirsailted
6 ‘300 = 1,350 40 54,000
BOx0 x 32 %60 30 48 oc (ti) 8 6,600
2 6,600 (A} =
,
54,090
Unskilled
| O « 30 = 1,80 20
hou cae - 51.200 a company
3,400 {Labour Varian
ces) department cf
2,43,000 2.780 problem 3.19 ho wr in a manufactur ing
Caleulati pet
is 25 wits is % 6.
ion of Variances ee oulp ut9 gx’ labour hour the time paid
wage rate per
LL
{SC The standard st an datd despite 3° of
abour Cost Variance
= (SC - AC) wo rk e rs, Th e
40 un it s of nv"
% 6.29, = 6 and
< 5.70
« 2,43,000 - 2,56,000 employing 400 depa’ rtment pr
oduced, 1.
ly pal q were
= T13,000 (A) we ek, the rat e ac tu al
In a 42 hour hour ly
Teasdn. The
Labour Rate Vari ance= (St anderd ra rate - Actual ti ate} x A ctual
(3 abnormal (CA. Inter}
was lost due
ce dard time
-@ d 60 ¥ oTk e4r s.
9600
Skitled = 70) x« 2,24 30 an
ve ~- 50)
(40 = & 22.400 (A) respectively tO
40,
= & 9,506 (A) vant variances.
Semi-skilled =
Unskilled = (30 - 20) « 2,560 Compute rele
= % ,600
25 (F)
L abour ae
Efficiency Variance
Lay 6400
= © (A) -
Solution
Basic cajcul
ations +
= 100 + 23-4
= St andard time - Actual time} x Standard t
pe unit = 4,160
Standard hours
rate. hours
Skilled 2250 + 2,240) x 60 a? oufgut = 1,049 unit s x 4
- 600 {F) s for actual 24,960
Semi-skitled
i-skitled =| { ne ~ 960} x 40 Standard hour = 4,160 his *
Feat
of actual output
ano (A
t
= & zr 22,800 d cos
805-2560) +30 $ tandar
Unskitled = follows : ;
LBV = 6,600 (A) Actual cost 1s
calculated as
Idle
Effective z

eae
hour ©

itt
L :
abour Mix Variance " No. of Actual hours hours a 2,604
s { Rev sed
$ s tandard hours
time ~ Actual time
ut x 5 tandard rat
ate
359 6.20 7,360
= 9,600 (F) workers paid* mA 6.00
eos
Semi skilled (14 0 960) x 40 = 19,200 (F) 420 4,197 44,
oe
464
eee
40 63
Unskilled (3,920 2,560) = 30 —=219,200{A} 4,260
426 2,394
iM 9,600
= © (F)
ee 30
i
60 2,520 _ 3,990 _
i 210
Revised standar¢ i 4,200
Total
time is a
calculated r $ Under.
!
. 42 hours.
St. time of grade af workers «
paid is No.
Revised Standard time x Total actual time . “Actual hers
Total standard time Variances
Calentation of Actual cost
e St. cast ~ = % 432 (F)
2,250 . Variance
Skilled = 5400 * 5,760 = 2,400 weeks Labour Cost = 24,960 - 24, 528
(SH ~ AH) x 5 {F)
= 1,020
ency Vayiance = %
Semi-skilled = 120 Labour Effici 5 (4,160 -
3,990) * 6
3.400 « 5,760 = 1,440 weeks * AF os
2 (SR - AR) {t6 - 6} * 1,260]
Unskilled _ 780
Lapaur Rate Variance
2 (6 - 6.20) * £20 +
5400 * 2760 = 4,920 weeks ¥ + {(6- 5,70) * 2,8
20)
« © 672 (F)
Labour Re vised E ffic wency
ey Varianc
Tl nee = ( St. . fime -- Revised st.f time } x SR. = gh (A) + Wil + 756 F)
Skilled = (2,250 - 2,400) x 60 -= 2. 9,000 (A)
a
St. andard i Costing
i and Variance Analysis Management Accounting
rr
3.66 .
a
Idie Time Vari
ariance = Idle hours x St, as under :
rate l output are calculated
= “SH St. pours for actua
= % 1,260 (A}
Check 80 hes hrs.
vane
Skilled =——— * 300 kg = 60.
400 kg
a cost
Labo j
. Efficiency Rate Idle ti
Y ; vanlance variance riance
vad ince io hrs .
Notes i ve
™ Unskilled =—~~— * 300 kg = 120 his.
* 1,020 (F) + 672 (F) + 1,260 (A) Ad ks
it may be on nance has been calculated as
a sub-var
~varrance of labour cost = |dje hrs x St. rate 750 ia:
| cula ted
ulate b- ayria The @
a8 4 § sub-va a f e fficiclency var
vanlance. Alternativel
y Ideal Time Variance x 2 hrs x © 25 =F
iance, Skilled = 15 workers == 300 (AL
E ro blera « 2 hts * € 15
Unskilled = 19 workers
3.20 (Labour
== 1050 (A)
Variances} 1
ITV
A . group of 10 0 skiskilled a : iency variance. Now LAY and LAY
heer
labour effic
8 hour day. 20 workers wer @ ex
unskilled
to produce 400 k d as a sub-variance 0 f
he standard hourly wage rate wes fina
BRT in au 1s ¢ alcu late
"Idle time vari ance
actual hours.
ded ucting idle hrs. from
heen calculated after « SR
= (RSH* - AH*)
: Labour Mix Variance = 1.000 {A}
of € 22 and =
age rate ctive ly, prod hours were wasted for the entir
Twouced = (50 - 90} * 25
due ¢
0 Dower failure and onl 18 cfrespe was
Skill ed =% 809 {FP
y 306 kq BXT
.
@ group
Unskilled = (100 - 60) * 15
You OU gre
are required
i to compute ; LMV = % 400 (A)
(1} Labour cost varian ce: (i ) Lak . s - die hrs.
iy = Original actual hour
ur Labo
. anceLapo(vi)
varlance; {¥) Lab our mix: &;vari(ii) anced : va( fi) Idle time variance: (iv)
rate urvariyiel usa 4 * Actual hours
= 90 hrs -
riance
Labour
Skilled = 120 - 30
Solution (B.Com. Hons., en ge Unskilled = 80 - 20 = 60 hrs
Total = 150 hrs __
s)
* RSH (Revised standard hour
. 80
= 150 x7 = 50
Skitled 240
.

j ed = 14 50 « ».
340 = 100
; kille
Skill
9 400 120 2?
Unskilled
| 160 6
: 80 18 - 2,640
1,440 Labour Yield Variance
= (AY ~ SY) « SOR
~~ AHAngin 240 4,400 200 4400 =

4,080
400) x"
—~|
{300-150 x——a (F)
tS. = Mo. of workers x 8
irs.
et
| wo 7 8

)
4,400 « 30 Problem 3.21 (Labour Variances
Stand . {of actual output (sc) = —2-—
ani cos O = % 3,300 . They are pal d at
400 , 15 women and 10 boys
Labour cost varian g of workers nor mal ly consists of 30. men —* 0.40 .
hey A gan * 0.60, Boy
as Man—* 0.80, Woman—
ce =~ AC
: = 3,300 - 4 080 standard rates pet hour gan g is expected ta produce 2,0
00 units of output.
hours, the
k of 40
A in a normal working wee women and 5 boys.
abour Rate Variance consisted of 40 men, 10
; = (SR ~ AR), « AH 34 December, the gang s were produced.
ee During the week ended tiv ely . 1,60 0 unit
Uren iled = (-5
(25.~ ee
22) x 120
bo es paid wer e @ T 0.70 , = 9.65 and ¥ 0.30 respec
=F The actual wag
to abnormal idle time.
36¢ (F)
=F 240 four hours were lost due iency variance, (iv) idle
variance, (il!) Labour effic
LRV = ® p ate (i} Wag e var iance, (if) Wage rate va riance) and {vi} Labour revised
iance, (i.e., labour mix
Labo ur Effici
ici ency Varian:ce. 120 (F) Cal cul
= (SH* -- AH) position var (B.Com. Hons., Dethi}
x SR
time variance (¥} Gang com
renin, = (40° 120) x 25 var ian ce, of Lab our yield variance.
skilled = (220 - ac} x = % 1,500 (A) efficiency
15
=@ 600 (F)
ERY > z 900 (A)

counting
Management Ac
a

Stan dard Costing


i and Variance Analysis 3.68
idle hours are
: = 11440 brs.
Solution Actual hours after wen = 1600 - 160 = £360 his.
:
Basic calculations: women = 400- 4020 aS
- © 180 hrs
poys 3= 200_- 7¥1,980 hes.
220
Total = 2.200 - : — the pasts
Actual computed of
Type of Standard —
.
yield yatiance aie
worker Rate rs.*, Amt. of labour mix
variance and
labour
of 2200 . 600
- 406.
Hrs." z Amt. z Rate~
z RSH fo: the purpose hrs in the ratio of st, hours
Now
5 hours i.e. 1980 « SR
_ = 238 (A)
rT ® of 4,980 actual = (RSH - AH)
Men 1,200 6 1,600
6.70
1,120 (v} Labour Mix Variance
= (1.080 - 4,440) « 0.8) = 108 (Ff)
Yomen ' nen 0.65 Men - 460) « 0.60 ee
0.60 400 260 Women = (549
* 0.40 at
ays = (260 - 180)
600 9 03
Boys 400
2.49
160
—200
40
60 tuv = 108 (Al
> 200 a
1,480 . RSH) = 5B
Total 2,200 “1,440 at :
Variance =
(Rif 2% 96 (A)
“H ts. = No. of
workers x Weekl .
:
Revised Efficiency
Men = (960 - 4,980) « 0.80
-t 36 (4)
y 40 hours {iv} Labour
omen = (480 ~ 540) * 0.60 at 16 (A)
; « 0.40 aS
_ Loe = (329 - 360)
Boys rev == £148
(A)
148 (A)
SC of actual output -a000 1
* 1,480 = 7% 1184 Variance ©
for actual output
: Revised pfficiency
Standard ho urs (SH) in place of Labour St yield race
400 may be calculated (Actual yield - St. wield} + -
x es 6 Bo Labour Yield Variance =
Mena Stop 2608 = 960 ys =>, 1,600 = 320 tapout Yield
Variance 2148 (A)
2,000 : Evy = {1,600
- 1,800} *
1st
7,906 units
.
Women <«~ we = 480 ;
209 1-600 units
Variances 2,000
Calculation of =5 200 x 1,980 = 7,800
riance (Labour
cost variance) output) -> (Actual - Standard yield ‘
= (SC of actual

(i) Wage Vari
c
_ ual cost) (A)
=
1,184 - 1,440
= 7256 : - 416 (A)
=a (SR age (A) = 160 F)
:
Wage Rate Vari + 148 (A)
= 160 1A) * yog (A)
Check
atiance ~ AR} « AH
Lev = LRV + LEV
[
(ii) Labour or
|
M
en = (0.80 -
0.70 (i) * cy 46 (A)
(ay Ley = Tv «LAY
(F
1,600
70) =x 400 == © 160 (A) ) for the
- 0.65)
60
= (0.60 = © 29 Variances)
x oo aterials and Labour actual performance
Women
Boys = (0.40 - 0.30)
and
=@ 2 (f) to pudgete a
s-
Ltd. relating cost variance
~
inv = €160 () data of A Go.

a7)
cn]
rect. about

o
fo ll ow in g als an d di .
m the ct materi
(ii) Labo
as
ance = (SH for actual output - AH)
x SR 5
Mar ch, compute dire 1,50,000
ur Efficiency Vari 0} x 0 80 month of
for March : 495,000
Men = (960 -- 1,60 = 0512 (A
Budgeted data
=
. (480
= % 48 my ! to be man ufactured sed on sta ndard tates) 4 40,000
Women
200) : on Ynits al requised (ba n 8
Boys = (oon - 48 (F) ct mat eri se
== Units of dire (units) z
oO iz v= %616 (Ay pur cha se of raw materials ned : oul
cy vari
planned materi
Note: Here Idle Ti jatlance is treated ag a sub-vari of labour efficien labour Average ynit
cost of dire ct
goo $ 29, 92, 500
cy vari ance may into idle time variances and’ yield ee Thus howss pet unit of finished
efficien e segregated mix variance Direct labour
(iv) Tale Time
nie mee, Hance,
= Standard tate hours* cost (total) 1,60,000
ance per hour x Idle Direct labour of March : 43,441,900
= % 0.80 7160 21 at the end %
Men Actual data nits actually
issued)
45,10,000
me 3 manufactured %
Women = = =? Units actually (pu rch ase cast based oF actu ally purchased) 3.20
Boys =% on
, a «2 8 fa) Direct material
cost
cos t ba se d on ani its
urchase 1,25,000
ITV = %160 (A) Direct materi
al cost (p
direct materi
al 6. -*
un it co st of 33,75,000
Caleulation of idle hours Av er ag e Marc h
bour hours for Inter}
Men = 40 x 4 hrs=% 1 Total direct la co st for March - UWA.
labour
Women = 10 x 4 hrs= 2 ‘0 Total direct .
Boys = 5x 4hr=€ _20 ‘
Total 220.
a
Standard Costing and Variance
Anatysis
Management Accounting
Sal
Solution
Material Variances
; thead Variances)
a : h
{a} Material following data for the mant
system and showed the
Cost Variance = (5¢ AC)
operates 4 standard costing
= {1,60,000
' uni its « $B x 3330")
Standard material quanti t ¥ per :
~ 43,419
AT,90G = & 1,17.900(4} Budgeted
.
unit of finis Actual
hed product is - 20
oo) mater 4 22 -
= (4,395,000 +
atetial Price Variance
= (SP ~- AP} x All
= 3,30 units
j No. of working days 4,300 4,000
o
4 Man-hours —_ = 0.50
. = (8 = 820) » Overhead rate per hour me m
———~~~ wee
= % 105,900 (4) Hours pet unit of output = 1,800 _
Materiais i
¥. Sage Variance= = (5 {50 i
- AQ) x SP Fixed overhead incurred 425 —

.
; No. of units produced
|
=| 166.000 x 3.40 - 43.41,900 Calculate:
xB.
(b) Budget Variance
our Variances ; {a) Overhead Cost variance (d) Capacity Variance
“ oe (c} Volume Variance (Le.WA. inter)
(a) ie our
Lab Cost
ost ¥ arian ce " (fy Efficiency Variance
(e) Calender Variance
= SC ~~ AC

T1,60.000 * 19.95*) — 33.75.0900


fl

St at =F 1
- Tate per
:
unit &
Solution
29,92,500 + 1,50,au, 000
{b} Labour Rate Variance = g 19,95
.
= (SR - AR) « AH
“4
Basic Catculations +
~ :
w sate per hom . 900 = 400 umts
= vo(26.60 . - 27) « 125,000 _ 4,000 houss

iy!
z 1. Budgeted output 10 hours
= z 2,000
ctue = 92,500 + ; " = 4,000 hs. @ = 0.50
2. Budgeted overhead @ 10 Ws. = £250
{co} neabourate
r Effi
percien
hour
cy Variance = (33,75 Oe tonne emres a . output = 425 units
= (sH ~ AH)
‘iy = e
SR o 3. Standard hours for actual = = 2125
:s ”° = 4,250 hrs. @ = 0.59
= (1,60,000 « 3/4 - 4,25 ) * 26.60 =% 1,33,000 (a) 4. Absorbed overtiead 0.50 0 = zt z.150
= 4,300 his. «
pH 3.23 (Overhead Varian &. Standard overhead
ces) ”
22 days
om = 4,000 hrs. * 59 days « O50 = 2 2 200
6, Revised budgeted overhead
the fallow J ng nf ormati
inf
N :
allo
oy compu E e Fixed
ead Cost, Expendi Overh
level ar capacity is 5,000 hour
s. Bud
hen overhead rate- is penditure
¥ 19 net stand Vari
and Volume é Variances.
apacity utilised is 4.40 geted fixed
dard 22
55° 4,400 hrs.
0 sta 4,000 *
ts. Actual fixed overhead Z 52 O08 Nour. 7, Revised budgeted hours

tt
000. Actua
{8.Com. Hons., Dethi)
Solution Calculation of Variances - Actual overhead
ce = Absorbed overhead
1. Fixed {a) Overhead Cost Varian = €425 (F)
Overhead Cost Variance
: 22,125 - 1,800
°= (St. hrs. . for actual outp - Actual overhead
ut = Budgeted overhead
, Actual ov (b) Budget Variance = €200 (F)
« {2,000 - 1,800)
= (4,400 x % 18) ~ 52,000
?, Expenditure Variance
ron me
= € 8,000 (A) = Absorbed overhead - Budg
eted overhead
ar {c) Volume Variance = €125 (F)
= (2,125 ~ 2,000)
Bi Ligg eted
geie,
fi oveerh,
f ad
rhe ‘a ~ Actua
ACER!
l t o ove eThead
= (5.000 « = 10) - Revised budg eted overhead
- % 52.600 = Standard overhead
3. =® 2,000, (4) (d) Capacity Vatiance = © 50 (A}
.
Volume Variance
= (2,150 ~ 2,200) ~ St. rate
(St. hrs, For actual s - Budgeted hours)
output ~ Buddg = (Revised budgeted hour
geted hrs ) * St. overhead (e} Calendar Variance = € 200 (F)
{4,400 ~ 5,000)
x % 49
rate .
= (4,400 - 4,000) « 0.50
dard averhead
(f) Efficiency Variance = Absorbed overhead-~ Stan
= % 25 (A)
eeeVariance
P.O. Cost 7m -= : Exp
ottend
o ituta
re seVarian
: ce
a (2,125 - 2,150)
0 + (aVolu me i ce
Varian
.
_ °
Accounting
Management
” e e

1
4.72
and nVarigance Analysis
ard osti
3.72 f
.
Stand ance
rdC ~ Check Expenditure
Variance +
Yoluma Vari

(i) FO Cost
Variance = (a) + 3,000 (F) Cal endar
Variance
Check 0 (F} = 1.090 .
2,90 Efficiency
Variance +
(if) Cost Vanitiance == Bud get Variance
i + Volum 1 capacity Variance + {F)
Variance 0 (F y + 2,400
{il) Yolume
000 (F) = 90 ° (A) + 1.50 ~ uting the
acl y Variance
vane + Capacit
ws + Cale ndar Vatiane 3, ; Ww . comp
(ii) te
volume teih. fies cnet Variance
Efficiency 8 an ces) a unit for
erhead Vari @ cast re co rd s of
(EF) = 25 (A) + 50 (A) + 200 (F)
llected fro
has been co : 25
éwing data for a period
25 (Overhea firiances)
ad variances - 6,008
you the foll
owin. g data: é fixed overhe ing days
.

td. has fin nisheded dgeted work 4


Budget
Actual (July) Number of bu day .
50.000 n-hours pet .
(0 units) & 1,50,000 .
os in units nen Budgeted ma t man-howr pe
or : geted). 27
f ee % 30,000 Output (bud
'

eads oT budgeted
Fixed overti 29 ad cost 4S . 6,300
were 31,500 Fixed overhe
ng days . In Juull y, the actual hours worked
}

g days
Ww

et ed fi ot
.
ki 2 1 pe r ho ur et of work
in 0.9
Budg is . y, Actual qumb
ix ea ead rate (C.A., Inter,
Adapted)
Calculate ariajanc urs per yda
= +,56,000
nces. Actual man-ho n-hour {in un
its)
solution Per ma
Actual ou- tput ad incurred
.

nasi al fi xe d overhe : ces *


lculations : - Actu Variance,
asic ca
_ 30,000
hrs. over head varian (bs Calender
+ @ 1» 30,000
xed
Budgeted hours Catculate fi riance. r}
3 Ll 1 ti
cant « Ac
utal outputa (a} Expenditur
e Variance, (a) Bificiency stya Variance. (LC.WA. Inte
aa
actual (f) Fixed Co
St. hours for mtayt
putpys
= a out
utput
, Variance,
(c) Capacity Va
30.000 ,000 hours.
— me riance,
* 22,000 = 33 e) Volu
= 2000 & 1.20 - :
days =
= % 30,000 + 25x % 1 =
:
St. rate per
day 2m Solution = % 1,50,000
ad = 33,000 hrs. ations * - & 1,56,000
Recovered overhe 33,000 \ Basic Calcul
1= iven)
andard overhe
ad = 31,5‘ 00 h TS.s. x © de= F 31,500
.
\
1, Budgeted
overhead (g .% 1,935,090
head (given ). x % 4}
= Budgeted ho
urs ys * 9.90 1,70,100
ed h days) x Acutal days 2. Agtual over his. x 27 da
= %
Revised budget ours (for 27 (6 ,300
.
Bu dg et ed d i overhead * zy =z 6,000
s
,
; 4,. absorbed (6,300 hrs
* 27 day
30,000 hours
| averhead + 25 days) = 2% 1,62,000
37 = 32,400 4. Stan" da
rd
pe t da y z 4,50,000
"95 i
5, St. gverhe
ad rat e * 27 days) ati.
, x € 1
=? he ad (€ 6,000 his * 35 days
)
ed overhead = 32,400 hrs
1 qudgeted ov er
0 * (6,000
Revised budget ° = © 32,400 6. Revised t har, = & 15 0, 00
of Varianc
i Tate pe
sat. overhead
-
Calculation overhead -
Act
st vartenee 2 = Recovered ual overhead (F)
\
Calculation of variances averhead ~
Actual over
head
6,000(A)
(7) BO. Co
- & 31.0 06 = @ 2,000 Variance
. = Budgeted #®
= © 33,000 - Actual Expenditure - 1,56,000}
Variance overhead . \\
{e) = (1 ,5 0. 00 8
(ii) Expendit
ure = Budgeted overhead . of) St. overhead
.
30,000-= % 31,0
00 1,000 (A} == % (Acts Ho
. _ St No
days * ‘vate pet day
= % , covere d ov er he ad - B
erhead
.
.
«= \ of days
- 2% 12,000(F}
ar¢iance= Re udgeted ov riance
(iil) : tnis VY
Vol 0 . (b) Calender va * 6,00 0 ed ov er head
e (27 - 25) overhead ~ Revised budget
00 « *¥ 3,00
£ 8,100(F}
¥ 30 ,0 {F}
ac te ven -~ erhead - Revi .
_
riance
ov
jsed budget overhead geted *,Standard
riance 62.00)
Tdar
apacity Va
(e) Capacity va = (4,70,100 - er1.head ~
=
00
oe - & 32,4 erhead
=
-
= 900 (4) standard ov 17,010(4)
-ciency Variance andard, overhe
ad * Absorbed
ov eZ
(v) Effi
’ = Kecovi
erhead00 ~ St Variance - 4,70,100)
ov () (dj Efficiency
per j € 1.500
no
=%33ctual a = 34,5 = (1,53,090 averhead
- Budgeted 2 & 3,090(F)
}« St. rate
.
Varie . ta averhead
(vi}Calendar ar Variance Budgeted days = Ansorbed

ay”
+ (27 - 15) «* 1.~ 200
t
riance
= € 2,400 (F} 42) Volume Va
)
0 - 4,50,000
= (4,53,09
,
Standard Costing
i and Variance Analysis Management Accounting
3.74
Uf fixed Cost Variance eted o
over headma
7 ~ Absorbed overhead - Actual overhead = Standard overhead - Revised Budg
Capacity Variance {Revised}
» (1,53,090 - 1,56,000}
Frac) = © 17,280 (a)
4,000 *« Aa
= ADIGA
= 1,20,960 ~ 4( 144,000
Cherk
(G) Fixed Cost Variance "
= Expenditure Variance + Volume
Variane St rate
f
2.910(A} = 6,000 {A} + 3,090 {F)
*
.
. | Actual working _ St. No.odays’ © pez day
i) Volume Variance = days working
-¢ alender + Capacity
i + efficiency Calendet Variance
variance
3,090{F) = 1,44,000 2 25,760 (A)
2 (24 - ) 25 days

Check
nse Variance + Volume Variance
manufacture one unit (i) Total F.0. Variance = Expe
80 (A)
of pri duct is 4 hours. 44.680 (A} = 2,000 (F) + 16,6 iency Variance
Th April, the com pany ance + Calender Variance + Effic
etd ee day $ of 840 machir
y worke
Volume Variance = Capacity Vari
of output. The actu (it) 0 (A) + 6.360 (F)
et Ours p per
16,680(A) = 17.280 {A} + 5,76
da y atl d produced
a fixed overheads 5 305 u its
were % 1,42,000.
Compure
Overhead Variances)
;
ae() Efficie ney varian 3.28 (Materials Labour and
i ce X is made available to yo.
Jl) Calender variance
(if) Capacity Variance
stan dard and actu al data in respect of chemical
{iv) Expense variance 4 followin g
Chemitals Ltd.
{r) Volume variance
(vi) Total fixed overhead varia
: the tecords of Naulakha Total
nce,
Standard Data : g g
(B Com Hons Dethi, CA. inter) .
Solution Materials 9,000
20 per kg
450 kg of mate rial 4 @ F
3,600 12,600
360 kg of material B @ © 10 per ka
Basic Calculations
:

et
Fixed overhead (= }
, B10

sea Labour : @ per hour 4,800
Working days 1,42,000
ee 2,400 skill ed hours @ z2 1,200
c rs ‘ae
7 1
120 machines x 83 hrs x 25 days 1,200 unskilled hours @ os
:
oa
a0 kg Normal loss Tatai 18,600
Output in units
24,000Ohhrs =: 4 h 15 '
~ 720 kg
. ot
aues ”
= 6,900 :
Actual Data : z z
andard fi xed overhead Materials: 8,540
rate (per hour) = 8,00 % 19 per kg
0
24,000 41s . ” 450 kq of material A @ 4806 12,510
Computatian mate rial B @ = 11 per kg
of Variances __360_ kg of
Total Fixed G verh ead 810
Varia
i nce =i ecovered
o ve thead
Labour ; @ per hour
~ Acku
Actua
1 ial ver hyead
overh
5,400
= (5,305 units x 4 hrs « % 2.25
6) - 1,42,000 2,400 skilled hours @ XZ 1,500
= 127,320 - 142,000 = % 14 680 (A @ % 1.25
Expense Variance 1,200 unskilted hours __
= Budgeted overhead - Actua
= 144,000 ~ 142,900
l overhe :
§0 kg Actual loss Total 19,410
13,4 10
Volum
olu € ¥ Varian
a
lance
’= Recover ed
cover
= F 2,000 )
o Vi erh
Thead - Budgeted y
overh 'é ad
160
760°
You are required to compute
:
= (its units x 4 hrs x % 6} - 1,44 (d) Labour cost variance;
,000 (a) Material cost variance;
= © 127,320 -1,44,000 = & (ey Labur rate variance:
Efficiency Variance 16,680 (A (b) Material price variance:
= Recovered overhead - Sten {(f} Labour yield variance.
dati ‘overhea
ne d Material yield variance;
{c)
°= (5,305 units x 6 h ts
x % 6) ~ (20,160 (i) From the following -data, calculate:
_ = 127,320 - 120,960 = & 6,369 (F) ure variance;
meee ‘ (i) fixed overhead expendit
Managem
Stand lard Costing
i and Variance Analysis
3.75 - 3.76
(it
(if)
Gi) Fix ed overh ead
Fret ¥ tbed Overhead ,800 hrs
* 4 hrs, = 20
volume i
variance;
ation of Abso 5,200 limits
thead cost variance *Note: Calcul tn al output =
Std. hrs of ac
z 10,000 = Re 9.50
-
Actuat pet hr. = 4 hrs.
Fixed overh ead for 3 Std, rate 5000 units * 00 10,4
Units of production th z 10,200 . * 9.50 = %
ly ad = 99,800 hrs
"on 5,200 Absosbed overhe riances)
d gverhead Va
Sta ndard time
i for ane unit
nie
ial, Labour an
.4 hours
a y
Actual hours worked m 3 (M at er s :
proble ‘eimea’ reve al q i
d for product
om 20,100 hrs,
sap eos t car 4.00 ii
(B.Com. Hons., Delhi) s —~ ;
Solution
andard material 6.00 i
a :
(a) Material Cost Variance
/
> kg of A @ & 2 pet ko 18.60
% & per kq
“fem
= Std. cost of actual output - A ctual cost 1 kg of 8 @ per ho ur ) 12.00 |
la bo ur (3 hours @ & 6 rect la bo ur hour} 40 .009 0
Direct @ * 4 PF 1 di cos t per uni t
—_ 720 | ig Okg.] le ov er he ad 3 hours’ al st an da rd an d pu dgeted costs
7G - 12,510 = z 790 (A Var iab Tot
month © € March
‘Simca ’ i a the
,000 units of rd are aS fol
lows:
to produce 10 dard costs ca z
mel Price Variance = {SP it is proposed if the st an
AP) | in fo rm at io n contained 40 ,000
pone as eo on the
“ ma te ri als — 60 ,000
omen, O- it) x 360 Di re ct
o . = &ton @ % 2 par kg
360
A 20,000 kg 4, 38 0,000
@ z 6 per kg
erial Yield Variance = {AY B 10 ,0 00 kg
@ % 6 per ho
ur ) _1 000
,2 0,
0,000 hours labour ht.)
~4,00,000
ay ~- S¥) xx § SOP oe a la bo ur (3 4 pes dir ect
000 hours @ =
Dire ct
= (760 ~ 720)
720
x 12.600 = &% 700 (F Variable overhe
ad (30, Total
°
{d@} Labour Cost Variance >
ults are 41,800
(Std. co st of actual output - Actual cost} The actual res
s —
Direct material 56,560
kg @ z 7.20 pet kg
f
A 19,0 00 1,82,400
= 5.60 per kq
[4 4,800 + 1200,
_ 720 ky
0k + « 7760 kg ~ (5409 + 1500} = q 56 6.67 (A B 10 ,1 00 kq @
hours @ * 6.
40 per hour} -1,04,000
. la bo ur (2 8,500 3,84,760
Direct
he ad s
suites Rate Variance = (SR - AR
‘ Variable Over
wean eee sea a (a)
= % 600 0 anits.
nskilled = (1 - 1.25) x 1290 tion was 9,00 iances:
) tw Total $300(a) Actual prod uc
From the abo ve inf ormati on, calculate the
follow ing var

‘our Yield Variance ce an d usage


= (ay - 5¥) x SOR
{a} Material pri y
labour efficienc
ges rate a nd
{760 {b) Labour wa ov er head variance
- 720) x £600
6000
0
= % 333,33 (F) al vari able
head (i) Tot
(c} Variable over
. 720k ance.
expenditure yari
{ii} Over head
(B.Com. Hions., Detht
() Fixed
Putatter an Expenditure Varia
efficiency variance
oe overhead - Actual Overti ‘i (iii) Overhead

(ii) Fix
the ad tL Overh
verhead
est vi olume Varian: on= a one)
‘oot Overhead - Budgeted oe ; Solution
é ances
tin 00 « 050)" - 10,000 {a} Material Yari * AQ 2
0 ne = (SP - AP)
{i) Material
Price Variance
0) * 19 ,0 00 = & 3,400 (A)
A= (2 - 2-2
oe Ovethead Cost Variant
000 (F)
>= : eos
Absorbed f Overh ead -.A ctual tualGverh ; * 10,100 2 © 4,
, B= (6 - 9-60)
0 * 0.50) - 10,200)“ ms = & 200 (Ey MPV® 240 (fF)
a

1
tin 9
Management Accoun
a
Staridard Cost i an
nee osting and YariTiance
a
Analpsi:
lysis 377 3.78 eee
Me me 40
a ne .
(1) : Materipal ; ce = (SO - A
Usa age Varian Tatal hours paid for 850
Q) x SP .
4 +
740
1 00) « 2 = Output (i units)
,0 0 -- 19,0io}
A= (18,00 4
2 = 8 2,000 (A) Actual wages pai
d (} 240
3 = (9.000 - 10
+ 6,600 (Ay .
65%
'
Muy
x
le ex pe nses (%) 4,0 00
Actual var iab : i
= © 8,600 (A) nses & - sai d wee k.
(ni) Mater = MP¥ + PLIV Actual fixed expe No. & fox the
tial Cost Variance the var ian ces relating to Shep AA . Inter)
i
to work out LE 4|
-2 {A -2 You are required
a 40 (F) + % 8,600 {A) = * 8,360 (A)
4
E
Labour Var ian ce
(1 $ _ '
iance
iy Labour Rate Var Sol
olu tion
uti
= (SR - AR) x AH Labour Variances ‘
et Zid ib :
= {6 - 6.4 0 « 28.500 =F
1,400 (A)
. e per hour {5R) = = 0.90 i
40)
« SR =@l Standard wage rat = £18 = 20 units
Labour Effi; iency
fic Vari
: e
anc == (SH ~ AH) pet unit _ 3 7as
0 2% 765
Standard wage rate
uw) :
.
= B50 units * Re 0.9
9.00
= (27 ,00 0 - 28,500) « 6 = %
(27,00 standard cost of act
ual gutput ts = 42.50
L abour Cost Variance 000 (A} 2 % B50 + 20 uni
Guy « LRV + LEV Standard hours = © 740
. 11,400 (A) + 9.080 (A)= % 26
Actual cost hours = 2B.
cl} _
Vari iable
Var Overkead Varianc 400 (A) -
hour
= 2% 740 = 40
ag rs.
= 40 - 36 = 4 hou
,
*s Actual rate per
Total Variable 9 . e Ail .
head Vacance
Idle tim = (SR - AR} * = & 20 (4)
(St. hrs. » f [0 ver aduction « St. overh variable overh
(i) Labour Rate
Variance
= {18 ~ 18.59) *
ao
> actual pr . thead tate) ~ Actual
- (27,0008 = & 4,000 (F) erhead * SR
x
000 units @
4) 3 - ea n = (SH - AH)
“9, ‘
ency Variance = z 45 (F
Hows per on
" ce mit = 27, 000 standardid hours for actual
production (ii) Labour Effici 2 (42.50 - 40) * 18
(i) Overh ead Expendi iture Varian -
» (Statdarg ce = sc ; - AC
variable overhead (ii) Labour Cost Varian , - = 25 {F
(28-500 overhead - Actual = 745 - 740 ”
= % 10,00
(ii) Overhead 4} ~ 1,04,000 000 (F) j
i ces
overhead Varian
Variance Variable = 950 units
uas e for a
ard ts = 800 ETS
: and rd
|
(St hour Actual output = 4d his « 20 uni
.
duction (A - Actual how rs} x Standard overhe i unis
. ad
~ 0 ~ 28. 5
ct
ual pro6,0 i tate Budgeted output = 36 hrs « 20 units = Ped
~ (27 ,00 8,5 00) x 4 = ¥ 00
= % 200
Total Co
Total St c. “) Standard output = 4p his « % 5
st Variance = | (Variable)
Cost Budgeted averhead
- Gast - Total Actual (A) = = 240
* F 9.25
units « % 40) - z (Va ble)
i ria
60 = & 24,760 | Actual overhead -25720 units
3,84,7
Standard rat e {ya riable) pet unit x % 0.2 5 = & 212.50
Probl HK (Lab . = R50 units y
er h \ (variable)
2 SANle. o Madour
and Ov ead Variances) . Absorbed overhead =_
:
720 units <2
9.25 = © 180
c o s t i
Z Per ate s a standard sys tem . Th e following d the week
|
Standard
:
variante overhead - Actual oveThead
st eno rt cos tin g
g data are availa
ble from = st. overhead = % 60 (A)
t for Shop No. 5:
|
Budget Variable
fe Variable Overhead
wee ly
/
per h (i) = 180 - 240
) Standard casts our b ased on no rm al work are |
rhead - St. yve
rhead.
= Absorbed ove = € 32.50 (F)
z i . rhe ad Eff iciency yanance
ne
per hour
1 (ii) Variable Qve
= 212.50 - 180 ove rhe ad
Wages aa , rhead - Actual
Variable expen 18.00 , ad Cost variance
= Absorbed ove = & 27.50 (A)
. 5.00 (tii} Yariab le Ove rhe = 212.50 * 240
=
Fixad expenses ‘
, 25.00 ces
~ 0
48.00 , Fixed Overhead Varian = & 25 + 20 uni
ls = & 1.25
_ Standard output uni e (fixed) per unit * = 1,000
per hoon: ur—200 units » = 25
St. averhead Tat = 4g° his .
{5) For the week ecac d
ae ed 25th November, 201
1 - ad {fixed) 129 = % 1,062.50
. Hours hand
ek Budgeted overhe = p59 units * ©
, (fixed)
.
36
. Absorbed overhead a & 1,000
ed for work , (fixed)
Non-prod (waiting : Actual overhead
uctive hours .
Tk): 4
:
Sta adard Costing
i and Yeriance Analysis oe Management Accounting
360
3,80 0
(i) ) FaFixed Overhead Budget Varirlance = Budgeted overhead ~ A = {b) Labour Cost
@t3 = % 21,000
Standard: 7,000 hours
ae .
(1) Fixed Overhead Volume Vari = 6,800 hours
°
ariance °=” Abvotbet ore
Absorbed °
his
overhead ~~ Bud getedov seat
Actual : 6,400 + 400 idle
3.25 = % 22,100
6,800 hours @z
(iit) Fixed Overhead Cost Variance 1= ite Absorbed
hewiheso “oaks
overhead
as - Actual overh Actual
eadan Overhead Cost
~e " {c) Budgeted (ar standard)
= 67,000
2Problem 3.331 (a
Salem 3.31 (Materi 3,500 « 202% 70,000 % 1,88,000
"£6250 (F) Total variable expenses
* Ltd. ' operat al, Labour and Overhead Variances) 4,800 « 40 = F, 1,92,900 3,500 units
Actual - es a standardra i
costing system. Following information 3 Fixed expenses 4,800 units 6,860 hours
Qutput 9,600 hours
© Supplied by th
*Materials ¢
Hats consumed (3,600 units @ % pe enpany Hours
52.50 each} - rate -
Direct wages Standard fixed overhead 40
+ 4,800 hours = %
Fixed expenses Per unit = @ 1,92,000
2 1,92 ,000 = 9,60 0 hours = © 20
Variable expenses 88,000
as Per hour = ;
9 600 hrs hours
se 0
62,06 3,500 units = 7,000
Output du nm actu al autp ut == 4800 units
i g he p eriod
i was 3,500 units of finisl ed Standard hours for
geads
For the aboveyep e rlod, rj e We a oh
standard s
Calculation of Variance
production
i Capacity was 4,800 units and the break-up
ance = s¢ - AC « % 14,000 (4)
Cost Vari
{a} Materiat = 1,75,000 - 1,89,000

Material
' cos { {one uni ance = (5P - AP} x AQ = ¥ 9,000 (A}
Direct wages (By Material Price yarl = (50 » 52-50) « 3,600
mes oe ea
Fixed expenses Variance = (SQ ~- AQ) x SP
= € 5,000 (A)
Variable expenses 40 (c} Material Usage (3,500 - 3,600) * 50
:
: 2 SC - AC = © 1,100 (A)
Labour Cost Variance

paiSma
(d)
' . = 21,000 - 22,100
The stand
ndard wages per tyunitwane
6,400 hours Were actu Ay
is ba sed on 9,600
, hi ours Of the above period
Wier :
= (SR - AR) x AH = % 4,700 (A)
a ort sauna the shove period and
in addition wn n Do
(e) Wage Rate Variance = {3 - 3.25) « 6,800
Were paid to compensa
73.25 per hou,
0 break-down of machine and overall.
al wage Variance == (SH - AH) * 5R
= 1,800 (F}
« (7,000 - 6,400) * 3
rece nes
rate was (f) Labour Efficiency
Tou are requir ed d to ¢ calc } late :. = Idle hours * 5R = % 1,200 (A)
Idle fime Variance
the f ‘alle
l wing
ng Vall1 ances
= 400 «3
:
(g}
(a} Material Cost Variance ~ Actual Expenses
{c} Material Usage Variance
(b) Material Price Variance
Variance = St. Variable Expenses « € 3,000 (F)
(a) Labour Cost Variance {hy Variable Expense - 62,000
= 70,000
: Fixed Exp.
(e) Wage Rate Variance Fixed Exp. - Actual
(f) Labour Efficiency Variance t

re Variance = Bud
get ed = € 4,000 (F)
(g) Idle Time Variance () Fixed Exp. Expenditu = 1,92 ,000 - 1,83 8,00 0
{h) Variable Expenses Variance , actual output ~ Bud
get ed
(i)y FFe Expenses Expenditure Variance = (St. hours for
Variance (i) Fixed Expenses Volume Varia Fixed Exp. Volume over head rate per hour
xed Expenses Capacity Variance
(fj) hour s) * St. = % 52,000 (A)
{} Fixed Expenses Efficiency
Variance = (7,000 - 9,600) x 20 hours; * St.
m) Total Cost Variance
Variance = (Actual hours - Budgeted
(CA. Inter) (k) Fixed Exp. Capacity overhead tate per
hour
= € 56,000 {A)
Solution
(,g00 - 9,600) x = 20output - Actual
Basic calculations :
cy Variance= (St. hours for actual hour
Fixed Exp. Efficien head rate per
(a) ‘Material’ Cost (i) hours) « St. over = % 4,000 (F)
(7,000 - 6,800)'* % 20
|
Standard: 3,500 units @ % 59 0 4
~~ (Total SC - Total AC)
Acttal
ctaal : 3,600 units @ @ 52.50 Total Cost Variance
= 461,100"
=. %© 1,85,000
1 ae v0 (m) = (3,500 units x <
116) ~2
2 % 55,100 {A)
= 4,06,000 - 4,61,100
St ndard Costing
Sta i and Variance Analysis
Management Accounting

Tota Cost At tual


== 1,89,
89,0 000
00 + 228
2,100
00 + stat
’0 ; : :
quantity
= ¥ 4,613,100 _ -Qateulation of revised standard
+
ss 88,0
a
88,0 009
Reconciliation * : :
Total AG
Totalde Cost y, aNlance
~Foiat so * Standard Quant
= Material ¢ .
.
. ost Variance + La’
+ Vatiable Expe pevised Standard Quantity 4,400
Feed Expenses
55.100 (A) = 16,000 Exp. arance » ‘riraned penance
Expenses 7 Pense Vatiane et
Val ume Vaziance 4,400 Cs
aay
000
.
» 2,000 = 2.208
{A 4,400 * goo == 880
Note = dle um ® Valance has been } + 1,100 (A} + 8,000 , * 1,200 = 1,320 B=to00
aTT
’ (F} + 4,009 {F) + 52 A=%o00
a
: O OG (A)
capacity varia
nance effi
and efficienc i . treated a 5 a sub-varianc
y variance .
are sub-variances Ne
OF
nou
volume
cost variance, . Also fi x
vari é d expenses cateulation of Varianees | sales
Tlances. geted
= Actual Sales ~ Bud
= ces} 1. Sales Value Variance %19,760 (F)
= 1,335,760 - 114,000

ii
SK Ltd. Furni ;
& following information relating to budgeted
te ishes th
.
April, 2077 - == (AP - SP) x AQ ta
geted sal 2, Sales Price Variance A = (18 - 15) * 880 . =? 2,640
Sales for Fn (20 - 20) » one
tity
5 and: actual
Product
Sales duan
ts) Selling Price 5,280 (A}
'
(Uni = 4
* 2,640
Budgeted Sales: =(B- 40)
2,640 (A)
C
Total = a
Per Unit e
1,200 15 dard sales |
800 = Actual sales - Stan
: Alternatively, Sales Price Variance = © 2,640 (A)
A ct ° .
se
= 133,760 - 1,36,400 i
a = (AQ - BO) * 2%SP 4,800 (A
4 ” 8 nce = (880 - 1,200) » 15
ves a 3. Sales Volume “Varia A ve poe tl
. 40
2,6sao *038 4 ean ~ 800) « 20
als the following variance s:
rate
.
.. ,40000 (F)
rotat == ©22225,6 iF) !
us Sales Quantity Variance
: C= (2,640 - 2,000) #40
Variance
Mix s Vari B
“Sales Price Variance, and (iv) Salels Sale
(1) Tota ance
§ tion
Solu dard sales.- Budg=eted sales '

Alternatively. Sales Volume Var. iance== Stan


1,36,400 - 1,14,000
% 22,400 (F)
.
{B.Com Hons. Delhi) '
EE ebeletons ce = (AQ - RSQ) * SP
=f 6,600 (A) i‘
4. Sales Mix Varian A = (880 - 1,320) * 15 ; Nit
Lo
i
+ ta80 - 880) = 20 .
Quantity Rate = ¥ 17,600 [F)
a e
= (2,640 - 2,200)
« 40 i
“eeu
= Amount | Quantity C Total = 714,000 (F)
Ba‘
a é e - |
_ 15 18,0.00 ws
5, Sales Quantity Varian ce= {RSQ - BG) x SP 1,800 (7)
a as ance: « 15 =?
880 i8 15,840 ~ 1,200)
A = (1,320
a 2,000 io 16,000
| 2% 1,600 (F) i
B = (B80 - 800) * 20
a oo
: £0 17,600 8,000 (F}
: 0.000 2,640 os x 40 = 2%
ap “ 100,32 C = {2,200 - 2,000)
= 1,14,000 4,400 - Jotal = £11,400 (F)

°1,33,760 | |
ae C .
Faleulatios of standard sales :
Check Vatiarice
Product TO = Price Variance + Volume
Actual Quantity (i) Sales Value Variance{F) (A) + 22,400
;
(F)
~ ~- Budgeted Price = 2,640
wee FAY 19,760
vanance
Stendard Sales
= Mix Variance + Quantity
|
(ii) Sales Volume Variance
A
22,400 (F) = 11,000 (F) + 11.400 {F). .

880
ue 5 To _ .

¢ 20
. 13,200

Problem 3.33 (Sales Variances) a: ucts are given below :


2 640

17,609
ai cere ny

period in respect of three prod


wwe.

40
er
"440
0 4 The budgeted-and actual sales for a
+48 ,600
° 136400 Budgeted figures:
ting
Management Accoun
a
.
5 tandard Casting and Variance Analysis 2
- r . 38h 333 [F)
: x 10 = 2
% 750 ~ 150]
Prod uct Pre , B= ES
Tod uantity Valu : : 1
eon)
.
=
; 2 :
: E soc s00| oS =
« |t5en : :
5,000 2,250
105
A
.
1,000
¥ 2,222 (F}
Bco 750 7,500
15
a 30 am ee
20,000 * Quantity Varian
check
ce = Pri ce Var ian ce + Mix Variarce
yalue Wasian 2,222 (F} +
+ 222 {A}
Actual. 4,900(F} = iu {a}
~ a Value .
ne Price
es Variances)
Product
problems 3.34 (Sal
ntity
qua
da ta gi ve n below : .

um e) ya ri ance
tz from the antity {sub-v al
: ances
following yari ce 4, Sales matg-N qu
compute the rgin varian variance
1,200 7,200 4. Total gales ma 5, Sales margin Ux .
me variance Z
° 5 6,300 9, Sales margin yolu
,

14 eT andard 7
.
a

price variance St
c _ 500 8,400 3, Sales margin Budgeted sale Act ual sale ’
Bu dg et ed Actual cost per
2.200 21.900 product price pe" unit
price per suret
(CA. Inter) quantity
s.
- wantity z UT ite sone
Neoe
tilconul
5 oluCa ate sales varian- ce " < et
30
uti (units) (units) - 4e 45
ed 240 400 Bo
1. Sales Value Varia nce = Actual sales - Budget x
25 a :
ed sales _= 1,900 (F) 200 (CA. Inter) A
= 4b, 080
-- 20,
21,’ 900- SP) AQ set
y 169
2 a) ates Pri{ce Vari:ance == (AP we
. —
4
5} x 1,200
A = (6 - 10) = 21,200 {F) . gotution kb
B = (9 - x 790 = % 700 {Aj sque 5 is used here
: sons
techniane ©
) €
c= (14 - 15} x 600 © 600 AA(A)?
=Y Basis calcula
(Quant _
ee
e
S d Budg et ed Pr of it
___& 200 (A) tual, Standard an Standard Budgeted
+ ance = (AQ - BQ) x SP Calculation of Ac . fi Star dard ia
Sales Volume Vari - Actual. quantity profil +
» 5 Actual i prafit
A = (1,200 — 1,000) quantity
j a
) profit
- 00 (5x7
geoo
* 10
t . ti00 ~ 750) cr eas00
-
0 - 500) = 18 %2,000 {F y
= (AQ - RSQ) x SP 4600
Sales Mixi Vari:ance tl
4,
"= , 1,6foe09
|
: 6,400
As [200 _ 250050 x 1,000} x 5 2% 444 (A
2,2 8
ett
10 ..
{70 - 250% 750] x Catenla’ Revised St. profit
» €1,333 () St. profit per unit
2,500
B= = ‘
1250 Product
Revised 50
:
fe

|
z
: fr units a
2,500 ' : 7,200

* 15 =,¢. $67 (A) i ; 20


“ 22255 0 * x0) { ity
“- [as
|
240 . 360
600 .
a _
_ = 22 (a)
z
x
400
~~
nce = (R80 - B
: 2,400

Sales Quantity Varia


5. Sal )
10
ee
.
" x 160 = 240
x Sp . 600
a)
, 9,600
. ‘ 400
(22% 1 .
As
2,250 x 1,000 ~ 1,000} x 5 \
=% 556 (Fy
.
ting
Management Accoun
St andard Costing
i and Variance Analysis
3.85

Es Total § ales Margini Variance


i = Actual profit -- Bud Budgeted profit
Solution
Budgeted Profit
, Standard and
= 7,000 - 6,4
ulatio n of Act ual Standard | Budgeted
Sales Margigin Volume Variance
i = Standard enit
profit - Bolus
Budget ed . profit Calc Standard
Actual St. profit quantity | profit
Actual Actual per unit profit
; = 10,000 - 6,40 Product quantity | — pro
fit
Se
5

Sales Margigin Price


i Variance
i = Actual profit ~ Standard
ater mnie a
profit
profit
7 gx (5% 7)
per unit
/ = 7,000 ~ 10,00 zg
Sales: Margi i
argin Quantity (Sub-velume) ane
Variance “me 5,120
5 409
evised standard
evised sta arto profit - Budgetec srafit
5,780
. = 9,600 - 6,400 = % 3,260 (Ft
17,280
dale: $ Margin
g M ix Varian ce = Standard pA prohit - Revi: sed anda
standard Pp refit

Check ~ 10,000 - 9,600 = € 400 (F) Total


ee ee
Profit
Revised Standard St. arofit
Calculation of unit Revised
St. profit per
'} *° al sa es Ma rmgin
gi v. ariamce # = Vi Volume 1 an
4 variance + BP Fnce i
Variance
. __—
Revised SQ z re
, 300 (F} * 3600 ‘F + 3,200 (A) Product units eee
_ 3,200
i} Yolume vatiance = amtity Vallance + Mix varlance . _. 4
1,308 6.508
2
3,850
PProblem 3.35 (Sales Variances)
3,250
3 2a
4,950
- 11,530
C aoe
6,500
Product Budgeted
7 sales Budgeted §.P. Actual sales Actual sale fit
Variances - Budgeted pro
per uitit Calculation of iance = Actual profit
untes Total Sales Var 280 = ¥ 270 {F}
(1) Gross Margin = 17,550 - 17,
vee eee Budgeted profit
‘:
A Ae eee me e
“ = St. profit - .
ue
1,280 2x 650 12,350 Grass Margin Sal
es Volum e Var
iance
= 16,250 - 17,280
© Z 1,030 (A)
- 2 i 3.900 = «50,700
(2)
profit
Revised St-
2 St. profit -
Sales Mix Varian
ce
= Z 1,300 (A)
(3) Gross Margin = 16,250 - 17,550 eted profit
: profit - Budg
Cost data are as under : = Revised st.
= om es Qua ntity Variance <8 270 (F)
= 17,550 - 17,280
gin Sal
(4) Gross Mar
- St. profit
ee cost per unit. z ®: 13 = Actual profit
ce % 1,300 (F)
ctual cast per unit (5) Sales Price Varian = 17,550 - 16, 250 =
.
z cost) * Actual Qty
it ; . 12
. 13
: - Act ual
You aye requi
“quired to calcu ate h
the f fo owing f ‘or the
h perio id = (St. cost 00 (A}
ce * £1 ,3
- (6) Total Cost Yarian = (16 - 18) * 650
;1.6 moss Marginin %Total Sales Variance 4. Gross Margigin Sales Quantt
00 = % 7,800 (A}
oan 5S Margin Sales Velum= Variance 5. Sales Price Variance *y Variance = (10 - 12) * 3,9
e .
ss Margin Sales Mix Variance 6. Total Cost Variance
B
= (13 - 13)* r (A)
€ z 9,100
Total
(CA. Inter}
; wit
ol Ratios) partment. Each
Problem 3.36 (Contr et te r, in one of its de uni ts
ities, Good an dB Good and 600
ilc o. Ltd . pr od uces two commod tim e res pec tiv ely . 1,000 units of wer e 10, 000 .
Ins tion this production
10 hours 2s produc n-hours spent in
takes 5 hours and ch. Act ual ma
oduced duting Mar
of Better were pr 96,000. (C.5. Inter)
rs at
Yearly budgeted hou ios.
ious control rat
Compute the yar
counting
Management Ac

Stand lard Costing


' and Variance Analysis 3.88
3.87
ual owlpl!l 190
St hours for act
Solution Budgeted howrs
*
o
(b) Activity Rati
Mon thly budgeted hours == 9,600
9, b rm + 12 months
= 8,000 hours
750 = 85.23%
“90 7 0
x 10)
Standard’ h ours For act
ual output
PB
= “on « 5) + (600 = 11,000 hours
A ctual hours worked 109
= 10,000 hours Actual how's

_ St. tts. for acutal output


_ 1,000 - {c} Capacity Ra
tio , = Budgeted hours
(a) Effici ency Ratia Actual hours
100
x» 100 = 110% 600 = 68.13%
10, 008 x 100
= 380

(8) Activi
ny Rat
.
_St bsBudgforetedcualhoursoutpat x 100
sooo * 100) = 137.50% Check
Activity Ratio = Efficiency ratio
* Capacity ratio
.10% eee
85.23%= 125% * 45
Actuat hours
(c) Capacity Ratio x 190 = 22,000 x1 = 128% 8520
~ Budgeted hours
00
KEY TERMS vice should cos
t |
wh at ea ch product ot ser
Check which zells
termined cost
{tis a pre-de dard cost of
‘ stan da rd Go st
s. whick the stan
st an ce the cost in
Activity Ratioio = Capacit ¥
um erations 8°
Ratio x Efficiency Ratio

\ under given ci rc to co ntra t


efficiency of
the op
137.50% =
4 sp ec ia li ze d technique rm in e the
sting is ts, to dete
Probl
Problem 3.37 (Control Ratios) - ‘ Standard Co the actual cos
ee ~ uc t is compared with im me di ately. -
d actual cost.
pr od n
; each
me di al ac ti on may be take anda rd cos t an an ce im such |
ratio,f acti ¥v ity
is 96%, find out its efficiency * that any re between the st ding the total
va ri
ra to1 of a compa
pany i
15 44
104% and its capa C1 it ¥ tatio
di ff er en ce ey b- di vi ective

||
is the DY rr
| Variance This ivig variances e so that co
is the pr oc ess of analys off st an da rd performanc
Variance Anal
ysis asons for
(€.A. Inter) n identify re
nagement ca ect materials
Solution a way that ma dard cost of dir
ce petwe en the st an
This is the differacential cost of direct matenals usedbour cost specified for
taken.
| action can be
Activity at i
Tatig = E cut
C Lapacity ratlo
ati x Ef ficier cy ratio
t1 | |
: Materi al Co
for
st Va ri nc
the, output achi
e’
eved and the
he tw een the stan
dard direct la
sp ec if ie d ff er en ce
2. i is the di curred.
Yariance This
‘. Efficiency ratio « labour cost in ad.
Capacity ratio 96 * °° = 108.33%. , Labour Cost the actual direct ed Ov erhead and
actual overhe
achiev ed an d n ab so rb ntrolling
| the activity erence betwee for us e in co
Problem 3.38 (Control Ratios)
Prob Va ri an ce 3 It is the diff us ed by management pa city Ra tio.
overhead Cost are commonly Ratio and Ca
. Cert ai n co ntrol ratios fi ci en cy Ra tio, Activity
Calculate {a)a Efficiency ratio, (b} Activity rati
atio, and ( c) Capacity ratio From the followi \ Control Rati
os
t contrat ratios
are Ef
figures : wing Three importa
| operations. S |
ee QUESTION
Budgeted production EXAMINATION
88 units
Standard hours per unit Questions
Objective Type
‘Actual production
75
i units
Actual working hours ion.
600 e statements of cost reduct
1, Tmie OF Fals of cos t con tra {and not
{B.Com. Hons., Dethi) technique e variance.
costing “is 4 of ‘abou rat
Solution 1. Standard 3 sub-var iance
riance 18
2, Labour ynix va favourable.
St. hours for actual output :
e variance js always un budget varian
ce
=
75 units x 10 hrs, =
750 4, idl e tim ys favourable. al to overhead
B udgeted hours lume ya siance is alwa en cy variance t 5 equ
4, Overhead vo fi ci
88 units x 10 hrs. = 880 overhead ef
variance plus
i

expenditure
4. Overhead
(2) Efficiency Ratio St. howrs for actual output
head. erheads.
for variable over e ° ¢ fixed ov
“Actual hours *108
ri ance an ges onty in cas labo ur time
variance. (Favourable).
50 6. Catendat va
ri an ce is a su b-variance of
al qu an ti t y va riance i s & 1,800
ition ya and mate ri
Gg «1G = 125% 7, Gang compos ,000 (adverse)
i ma te ri al cos t variance is 21 go o (A dv erse).
| 8. Whef wil l be z
price variance
then m: terial
sCosting-and ¥ariance Analysis Management Accounting

te SNe
. 3.89
' 9. : Consumpti
ption i
of high ials i
“0, Revision. variance 5 alae materials in larger proportion resutts is

acme
by the following
tame. 1. Pa r ie difference between actua? cost and adverse material mix variHance, 14, What do you understand (B.Com, Hons,, Deiki)
- i. False, 2. Fails se 3. True, 4, Paise, 5, Fatse, 6, True, 7. T, standard
tevised cost. (Bb) Activ i ty ratio (¢) Capacity ratio
(a} Efficiency ratio made the followi ng statement—

-
standard co sts
er discussing budgets and ax d are set up and
Il. Fill in the blanks 7 /. True, 8, False, , 9 9. True,
«
10. £,
10. False] 15. Recently a conference speak have diffe rent purpo ses
same things, they

-
f a
I te sh e
e “Budgets and standards are not the etween therm.”
relationship exists b
eeeteees ne IS} an By hypathet she i eal ways. yet a specific
ical hou
hourit wh
whic
; ih fepresents the amount used in different

a
of work which en budgets and standards.
{a) Identify differences betwe (LCWA. Inter)
© . Neto mt ‘ . Varlianic betwe en budgets and standards.
e isi a : sub-variance of stete
eeceee MAL (b} Identify similarities
bye B : Vartance arises due
to reasons like sl
_ “ Practical Questions
5.7 When UNRCHEstern overhead: ns ¢ st variance 4.
is sub-divided actual production of product
6. The most into a “e regarding the standard and
The following particulars are
ET

is more than actual cost,


MEO , ij e 1.
it is 5 kg
: cal ne ial per unit
Portant purpose of standard a oe " Standard quantity of mater B45
vee enees vatian costing is too ttrete
ce arj ises only whe eeeree COSES,, cama| nse.
— Standard price per ka. . 400
n more than one type of uced
product, matate
erirtal is us ed in the Actual number of units pred 2,200 kg.
manufacture of a used
8. Tf act ual material : Actual quantity of materials % 4.80 per kg.
;
cost isis 50 50 units
Per uni : t. the materialOst cost at %
variance is 6 per unit and standard material Price of materials (8.Com., Madura}
vati ance
variance (if) M aterial usage
cost is
boteiesee, variance Niet een eee
Calculate {i) Material price
Lo Ma
is th i © 440 (Fy (a) © 1,000 (A)]
fans. (i)
actual
ng qua_nti ity co
nsumneded an andd sta @ % 10 per kg, compute the
standa
eedrd squa
ea ntity + sified for material consumption are 40 kg.
, rane ave to ue - 2 Given that the cost standards
(Ans. - 1.1 °Stanvar
daian is , the2, res
rd ce h our variance when actuals are:
Labpon
oursibilieffiej
ty of |ue © 10 per kg. (b} 40 kg. at % 12 per kg.
— (a) 48 kg. at of = 360.
cy, 5. Favourvab i
@ 12 per kg. (d} 36 kg. for & total cost
abllee 6. 6, Control - 7 7, Mate {c) 48 kg. at (C.4, Inter)
Materiraal mj fe BMit, -
aterial usage, £9, {c) F 96 (Ap. (d) Nil.
Purchase mana ger.
a) Ans. MPY (a) Nil, {0) Z ao {A},
(A), (8) Nil, (c} & 80 (A). {d) % 40 (F)]
ay Type Questions MUV {a) = 80
ne : and
Defiain
2a Expl explainthea) sith * concept of Ess
,
‘standard cost’ and ‘stand facture of 1,000 unit s of
product 2 is 400
brie: fly as a in pe costing’ (B.Com. H ates for materi ai for the manu
gnificance of standard costing 3. In a factory, standard estim pro duced it is found tha £825 kg af materials
“YOM. Hons. Belhi) units of product Z are (B.Com, Calcuttc}
chnique of cost “at2 2.50 pet kg. When 2,000 nces.
3. Dis tin guish het wee control. kg, Calculate material varia
n standard cost and est cost, are copsumed at % 2.70 per % 62.50 (A)]
169(4) MUV
4. Define & tandard
imated om
, ‘om.
Hans. Dethi)
: [Ans. MCV % 227.50(A), MPV @
costith ng. In what t y pe of industries is Standa ri a
Hon| s. Dethi)
5. Poi
costing ém ployed
? 4Aalculate the mix variance Rate @
: nt oy t the diff Quantity (kg-)
erences between histor (8.Com., Material
ical costing and sta
stannda
d rd costin g.
Calicut) ; 2
A. 10
6. Discuss the adva Standard ;
7. Dutiine pri ntages
limitatiations of stan
: and of standard : (8. €om., Keraia) a 15
3

!; heads:
the primary objects casting werd esting (B.Com., Kerala)
a 8
2.50
its uses under each of A
/ rod of performance - Hescribe briefly
: the foilowing Actual
VF
2,50
(a) )AsA a measuring 8 (B.Com., Calicut}
:
; a | has; ;
earnin. gs budget
an aid
(c} AS ‘
aid j A preparing (8} As a basis of inventory valuatio n,
' (2) As a guid
guidee inin fixi [Ans. Mix Variance 7 2 (A)]
fix ng selling prices,
and tabour.
8. Bistinguy
quish betwee
(B.Com, Hons., ‘Beth one unit of produc t shows the following costs for material
9. nt standard costin g aad budgeta t) 5. The standard cost card for
(B.Com. Hons., Dethi)
What is Ty control.
stan d ard d ¢
,
aterial 4 pieces @ @ 5.00
Ostii ng ?? Brin g Ou lear ly
t Clea the , tionshjp
tela bet Wwe e, Nn stan
£ da' Yr d
costing and budget
g. Explain ary 10 hours @ % 1.50
Labour March, with the following
{a ) B asi c
factured during the month of
5 st andard, (8) Id eat tandar d (B.Corn. Hons. Deihi}
5,700- units of the product were manu
al stay (C)
* Curre mE ft stan
dard
' and (d ) Expected standard.
iTW. Sta
Expndi
lai ve Concept of standard
hour 1 (ec (B.Com, Calicut) material and labour costs:
Costing 7 and and | 4.95
budgetary contzol; : are
inter-related b ut not “bor, Hons, Hethi) Material — 23,000 pieces @
inde’ penden 1.52 aa
3. “Caicul latii ia. Labour — 56,800 hours @ % (B.Com. Hons., Dethi)
on: OF fF vari in es : . yatiarices.”
B.Com labour
Calculate appropriate material and
‘ aIC
(Fi: MQV % 1,000 (A)
In standard co SCL:
ans, (i) MCV & 150 (F): MPV @ 1,150
ti ig is not an end
Tid in itsal f but a means ta foe
ntd." ne
Discuss, {A); LEV € 300 (F)]
(LOKA, Inter) &. ii) LCV % 836 (A); LRV @ 1,136
\! :

on
Accoumungy
Management

Standa: rd Cos ti ngiG and


1 Vari ance A nalysis nce (ii"} Material
quantity yana
ce ¥ ari anc e (i!) Material
Material pi on
6. Fora Vate
uninitrn of Proms mpute « (ij
A, the standard data is gi n bel following co cost variance
.
ea = 40. From the d {i¥ ) Ma te ri al Actual
. kg. @ F 40 per kg. ce , an
men neon mix varian
. his. @ % 1.00 per hour | Standard Unit Price
~ , Quantity
. eo Material z
Unit price Kg.
F2 Quantity
Actual data é
Kg. 3
oe 5
, tual production 100 unit 5
10 WW
Materials:: ne kg. @ % 42 each
a A
5
20 A5
260 brs, @ & 1.10 per hour B 6
on 20 qn tral
c Foo yea
n z (Al. ov)
= 70 (F}. (a
o
Calculate: (A). Gi) Bnarachigasa
n}
{ans. (i) = 20 (M.Com.
po kg. of & 128
>: os
Material cost varia face a 4
price vatiance rials. For every
Materialial usage ver ~ three cw mate star derd and
actual
v : nt « . taboour ‘P’ by mixing kg © FB, The
.
ur cost cot varian c pr oduc t af 5,60 0
he wees May mpany manufa
ctures s an output
April there Wa
:
2 ne em na
6. Labcu r nn y Me
11, Philips Co
efficiency var tiance (Adapted)
in at er ia ls are used. In
(A kg. of Taw Follows: ee
stand . , 1,5. % 396
}. 6 % 40 (F)] April até as
particulars for
*. The material’ and
ard costsi on for the maki of a
‘labour’
aking of a unit
. a
estimated $ under: certain prod uct are Standa rd
Raw Matevial
pest
80 kg. at % 1.50 Price per Kg.
nh apour 18 hrs. at * 1,25 mh Mix z 42
: completion of the .
tion ofof that 75 k g. of material costing Z 1.76
producvie unit, it was f
16
g. has been consy
d and t t the a time tak €y wasoun16d hours, the wage rate b g 1 75 Pp per
Your a d atethamat eri al and ab e bein @ 1.50 per ho 12
are required to
bie. calcul .(F
Our VaTiances (B.Com.,

Ho Hans., Dethi) ur
18.7 (F), LRV & 4 7,900 Kg.
Exbtn MP V- %
{Ans, e vateral wg (A) V & 7.5 0 = consumed was
8. the follow!wing data. calculat
, MU
‘ (A), LEV 2.50 (F}E ill | yantity of materials
am . Hers. pethiy
is ix variance . eee assuming ac
tu al ip.t
Material
_
ma material va ri an ce ¢
M ay& 16
,900 (A)
Actual uc anPri Calculate all1) 5,600 (A).
(A). MY Nill
Standard ,600 . MPV @
maty = 14,000
quantity [ans. MCY % 19
(A)
| fey x i: ity
quanti
unit —
= 45 rn
94bec
: 14
riances-
material ya Acrual Rate
¥ 35
40 .
1 | Dang z 12
in g da ta , , calculate Actual Oty. ee iene
40 foll ow ;
39 11 12." from the St. Rate —_— Zz 1.20
Due to shortage of x it w. : eee St. fity-
was- decide d te reduce its cons um ption b y 10 and incre
i ase the consumnt 7,500 kg.
of ¥ and Z by 6 and 4 respe motion Materiat 05 Te
ctiv (B.Com. Calcutta) z 1. 3,300 kg.
: g,000 kg. % 3.50
fAns. MMV Z 4 (A)] x @ 2.45 kg-
(Hint. Adjust
Adj 2,400
standard qunati 3,000 kg. = 3.30
Y
9. Fro m the following calculate: compute,
s that you
kg. er,
2,000. LC.WA.. Int
Z
gf ! s variance
aridyeu
(a) Pri @ variance
atlan ' {5 } a : relati on sh ip = 1.440 {A} :
note of t he 100 {A}: May
Sai ‘9 é@ Vi variance . {c) H 1X Varian
fiance - ar d (d) Revised usage variance:
Material algo write a (A): Mpv & 2, 0 (A); mRay % 430 (AY
Standard
Actual MMY % 1,41 and cost
- :d p roduct
~ rd cos t system. Standa
Oty. da
R
Qty d uses 2 stan
Ant Rate Amt. praduc t P an
=2 :
Ltd. ma nuf actures are 35 foil OWS!
10 20 5 13, Bitla & ‘to , af prod u ct
3 15 for 4,000 ka.
20 10 3 specific ation
69
ot 5 65 60
“20 15
~~ SO 75
oe ‘ wo _ JG
tan
20 (A), (4) 70 by, (3 @ 10 a a

ns. (a) a) &2
‘ 80 (F)]
OS
.
Costing and Variance A natysis
Standard
tr 4.94 icipated in p
r duction
+
ber.
a nee ea
3,98 10% is an t nt h of De cem
ee an da y d loss of l a b le for he mo
- Méterial A st a v a i .
Mey in kg. Price 360 per kg
wen
The follow
ing pat used at ¥
A best 6g 0 pe r ka-
m
= z= 180 .ks- of
R 800 2.50 kg.
200 22 0
‘AB’ is 369
ka.
: 2,000 pr oduction of
tnput
280
4.00
1.00 B00 The ac tu al
(pb) Material
Price Varian
ce

_volo Output 1,20000 “a Ca lcul at e:


st Varian
ce
~ (dj Materi
al Mi y Va ri an ce
{B Com Hons
Bein. LO.WA.
Inter]
1,0 ne 3,006 Material Co ce
indicate . lo age Varian 9,200 (F).
aterial records ee Material Us 0 (A} iq Mu
y
Q
d Va riance Mpv € 190,40 iy a 7 5200 iil.
(dy MAY © 4906
al Yi el (A) . (E}
Con sumpti= on in Januar ie) Materi {ans. (4 } wey 2 1,20
0
¥
A 1,57,000 kg. @ % 2.40 re is:
ical raixtu
rtain ch em
38,000 kgkg @@ % 4.20 of 4 ce
cost
£ n 25 pet kg-
1 10
ay pe at z
material A
36,000
Actualal & finished Production 35% wed:
for the m onth of January is 2,00,00 at < 36 pet
kg. od there is
00,000 kg. - Cal Calculate matezj
terial varitances, material B ti on. Du ring 2 peri
65% in produc
(8 Co Delhi 16 is expected
[Aus. MPY- Z 4.500 (F), MUV = 79,m 500Hons(F}, MMV ae Inter} A standard
loss of 5%
z 27 per Kas and
3,600 [A ), kg . of ma terial 4 at
‘ 425 r ka-
2 at 2 34 pe
material 8
14. Preco L td. { ). MC 2
is prod ucing
iI a ‘ standard ’ 000 ( FY]
of -
standa rd | :
mix’ ty usin g 66 kg of ma terial Xa 275 kg .
ass of Xi
st ndard price of
tion is 30%.e The sta
nd 40 kg.
Gg of material
s 36 5 kg.
The ectuat mixt produc
¥. Th 2 wa
= 5 peper kg. and that of ¥% 10 gutput
ure ard yield wer 8 per kg. The actual Price Varian
ce
¥ 80 k } Material
as follows: Calc ul at e: Ab
ri ce
an |
% 4.50 per
k riance al Yield Va (B.Com. Hatis., Delhi-
g. @ ri al Cost Va {d) Materi
® 597.97 (Al
¥ 70 (a) Ma te
*
@ & 8 per k 4 Mi x Va riance
& to ) i)
(A
Actna’ yj kg. is 115 kq
@ yield (c) Materi
al
z yo (Ff), to
2,97 4A)
tans. (a) = 37
(
q t
ale ulate t
material vanlances
follow s . iz]
& 2 is 3s price per k&-
% 180 (F): MO of product
{ a 5

(twee
i2 }
fAns. MCV 2 30 6 a Standard: mx 4.00
: MOV % 50 | Material
15, Following are tha
MV? Kgs xk : 4.50
larticulars
of a pradu ct whe te two types of materials Berd,
in respect
: A and 8 are.used: gh Y 9.50 for a
95 Cowd production
Material
__ ___ Standard Act ual z scra p va lue- Actual ri al du ring
n0 mate
input
Rate Tons " 39 2 io n .is 10% af inpu
t. There is
pu rc ha ses and cons
umption of
yal
boss in produ
Rate ct
mixes. Act
a eevee pe
The gtandard 5 kgs. of A2 from ga
-
Tee a - =
% 42 (F)
A Pa vmonth wa s Price per kg.
10.00 140 we re +
9.50 the month Material 6.50
7.50 60 kg 4.25
———__. 9.00 x
4,206 Y 9.75
Less: gement:
to the mana
8S: Toss
4,700 Zz pr esentation
s fot
2,600 following
variance
ance
ulate the . price yani
ired t0 calc ) Material (c.5. Inter)
You ate re qu
ce
{ti
te ri al yiel d yariance.
cost varian (iv) Ma
T Material 61 2.50 {A}.
ic( } Materiai
al mix va
riance eo (A); (iii) *
(ii) ? 2,325 .25 (AY
Prica Variance ha) Material Mix Varian ce, (iil) Materi e vanance 40 6. 25 (A }-
5 (A ) (vy = 24 81
Material usag [Ans. (i 8, (iv) & 4668.7
€ Material Yield Variar ce a
:
a } Materitial Usage Variance (v) . information.
x 1/8". From the foll owing
x6
is t size of 6"
i.
oe Madras) les of sta ndan
%-50 (a); (C) & 20—~ ry2 «AG)(d} 8¥ 3030 (A)]
. {Ans {
( s P jastic ti materials:
16 he standard ard +‘ost of .
, (a)ture2 20‘AB’ (Ay
is: (2) es Lt d. ma ke
tain chemi mix 49. Modern Til ired to catculate direct
40% materi erial A ata %cer400 per i . 4
you are requ total. Foo
Dba
. variance in Tet
60% TR aterial B at z 600 p er kg.
{ij the cost
.
Adds
Accounting
Haenagement

L} bone
2
Standard Cos.
G tin iG a nd d fanian
V ce Analy S15i

i 3.95
d ‘ 70,400 houss
th @ cost vari
{it
ne
}
ti an ce sub-dividevi de d into (5} us ag e Hours taken:
1
(ii) the usage vari Price; and . o
ysed into o 400 hours
-
:
st whet analou mlx;
i (5) yield. * Production
an
A yste dard ni k 0 comp nd re qu ired to 4 s
ow produce an output Idle time 40,800 hour
thick is as fall re feet of tilIles 1/8"
put of 20,000 squa e:
. “
du an ti Total time ut. Calculat
ect
Dir et 4
Ma te rt al s .)
60 4! tz 5.20 per ho ency varian
ce
ut}
Labour affici (8 Com., Meer
gs
antity (X ce Per Kg. (2) Pri made @ 56,1
” payments (b)
4 600 va ri an ce ti me va ri an ce :
2 2,00 0 (AN)
cost Idle . td)
(ce) = 2,180 tai
0.90
(a) Labour (d}
B va ri an ce = 4,000 (A),
6,160 (A) {b)
400 0.65 rat e
{ct Labour {A ns . (aj) @
Dec Cc
w. 500 0.40
During eight mixes materi ces.
ecember, sed and actuat bout Varian Actual
ere proces umed were: different La
rials aterials cons 22, Find out
Direct Mate
Quantity (Kgs.) Standa rd 1,208 units
Price Per kg, (@) - = 8,g0u

satladeaal
a 1,000 units with bonus
5,000 Gutput :
Wages paid
B on ent -
%G per unit
40 hours. y % 2,400
ay
Rate of paym 3,200(A). Le
2,900 50 hours y 7 80 0 (A). RY % ho ur ¢ 200)
4 as
Time taken
+ [Ans- to al rate pet
Actual | prod
oduction for Decemb
er was «ono ‘ & 120. Actu
29,000 tiles. (B. 6 om Hons Dethi, €.5. Inter) rate pet hour
(Hint. St. nt OF bP”
[Ans . MCsV 6”220 (A), MPV % 175
% (F ur amployme age of ea
with a labo w
55 (F}: * MtYY 450 (ay) s
{Hint . ve re d pe r tile (F). MV
z
in 30 we ek
Tne standa t completed
Ar ea co completed at iv es . is actually
lee 6" 2" x 6" =, 36°
; le d to be
sk il le d op es wo rk the
un operativ and .
sc he du Th e
23. A job is 2M 60 . ille d #2 4. es
sq. ftft= ye 1 x 12" = 144" d operatives 36 and unsk hd 7 unskilted bour
40 semrskille -skilled =
ea per sq. d la
No. of ti iles required per
Ar
2 60, se mi est d for angk il le
14 + 38 = 4
‘ e" skilled i ; gkilled and = 20
No. of ti iles requi1 vhslbes
ite operatives si a labour forc
e 0 ULC.WA, int
er}
f hs20,000 sq. - ft =z 2 20 (000 « 4 ==
ited for 80 000 we ek s wa th
in 32 s av er ag e s re as on s.
LEV 24 ,4 40
wage rate cost due to
va ri ou
® 10,240 (A).
actual weekly (AH
St. o om 8 MLxes = 80,000 x g = § 40,000 tiles the labour a.g00 (A), LAV 19.200 (Fj, LREV = 17,760
ce 0 ,

pe e the yariance in fa ns . LC V %
ctual output= 6,20,000 tiles.] Anal ys tay % hour ly rat es
20 WeThe standard at standard :
. of manufacturing a prod uct shows th
Material cost .
e fol ing standards: n an d 5 boys paid ga ng 15 expected
wome 40 hours, the
Material Is per kg low 10 men. 9 ing week of
Guantit Hy andard Pri ce St consists of novmal work
Amount
of wo rk er s usually ly. In a #
24. A gang 0.70 respecti
ve s heing
4 z : 0.30 and & hourly rate k
ys: actual
z
of Z 1.25. % bo units
time and 0
of ou tp ut . n an d 3 96
men. 4 wome
BE

4,000 units abnormal idle


40 kg.
ie 3,000 to produce is ted of 13 du e to
B co ns los t
19 kg. ‘ week, the
ga ng he urs were
ectively. Two
5
.
ina ceTtain er)
c 20 1 00 and € 0.65 resp (LC. WAL Int '

a
50 kq
= 0.85
ee z 1.20 8 58.90 (A):
re produced.
Ley = 94 (ADL(A)MV. EREY z 3 (Fl \
:
Material i cost pez unit (Total) 4,500 ~-pf output we ances. 24
270 (A), LRY @
T he standard material labaut vari
n put mix is 10 0 ki g. and tl of the § nished prod , . | Caleulate
appropriate
Ans. Lev irry € 43.10 month ate
:
k g. The actual: resutts fo
r i he standard output : uct is 96 | engaged du
ring the 4
: - period aie : ' cormposition |
Materials used tual labour
. on and the ac tinskilled
semi-skitied
comp osi ti i
dard labour
a 2
Ka. @ % 80 per kg. 25, The stan
+ skilled 10
40 \
‘ono
000 kg. given below 30 ?
, 52 per kg.
=
6
p 3 |
. ers in a grou
tg
5
qumber of work hour)
2,20,000
of the
@ 2t per k g. 12 i
(a) Standard 15
Actual output
pe t
wage tate (=
; tne product is 2 20,000
OM k . 24 i
You are required to
: ca Cuia @ the I aterial
1 (b) Standard of workers
employed 2
F I ice , Mixix and ¥ yield variTlances,
{c) Actual
number
group 6 2.5
nth in the urs of work
.
dunng the mo standard ho
{Ans. MCV % 45,00,006 (A HPV & 15,00,000000 (A),}. MU
| V .100,.4.000 Inter)
(.C,0 wage ra te pe r ho ur (=)
gr ou p pr o du ced 9,600
ri an ce , La bour mix
. & 34 ee (d) Actual hours, the efficiency va LEWA. Int
er)
24. The st }, MMV &% 19,00, (A). MYP = 15,00,000 A} month of 20
0 working
variance, Labout :
(revised)
a hours per unit at % 5 per he “ Busing the Wage ra te
riance € 27,
400 (AY
rd cost of Product is: 10 show in g
actanualda data is:is: u ations © evised) va
our
Th @ Required : Calc tal jabour cost variance. efficiency (A)]
(Ay Labout ce = 2,708
Production’ 1,000 unit
¥
(> _ va ri ance 4 d To
Tate varian ce = 3,300 : To ta l la bo ur cost varian
fPdup (F)
\ [Ans. Wage riance z 3,000
5

Labour mix va
. i
‘ eon
ae4 CR] ‘
oa Lk CF)
Management Accounting
ee

f ow:
Frat the © foll
ingsis data
ogether wy th ad Raly . prepare unit castst statement showing :
at % 2.80 per uni
t.
18,000 units af B
is of va fla prim e
: lanc
nte
es
Materia's B % 1.20 per ynit. e Routs the
* 88,500 units of Cat . For 2,500 of thes
Product A labour he' urs during the. year the stan dard rate.
Standard Product B The Company wor
ked 17,500 dire ct
t he wages were paid
at
Actual B00
5
kg. @ % 5.00 Z 12 per hou r whi le for the remaining and Effi cien cy Variances.
Labo y 90 k
company paid at and Labour Rate
(B. Com Hans Delhi}
fabour - 106 i @ T 3.00 Price and Usage Variances
80 kg. @ F 5.50
Standard
:
Calculate Materials z 4,000 (F)}
80 hrs. @ % 2.09
(A), ERY F 10,000 (Ay, LEY af April.
2.8)
Actital 9. @ z 80
500
16 his. @ & 2.89 100 (A). wu %
G2 hrs. @z% is
[ans. MPV % 19, the month
16 hrs. @ & 266 on ind ica te the fall awing for
of Con wes t Engineerin g Corporati Unit cost
[Ans. Product 4 - Standard
faiG total (Oma, inter) 29, The records
uta cost
¢ % 3,180- Produc! CB; e 314.80: = 4.80
Standards
Actual total Cos
Cost J 33 4 gallons @ F420 z 5.40
Tot es vari A ;= iepro
. ance : 351 UB: % 316.40: Direct material 4 hours @ % 1.80 |
z 1.80
Material : a - a our
% 109 F) uce = 1.60 (A) Direct Lab hour
his ‘an (4) MUV Z 0.60 per labour
Factory overhead cost z 12,00
z g
. 20 Tatal manufacturin
(F) MOY & 24 30 (A (Aa) acy rio ia)
A}, inning oF anding wor
k-
Month of April Activity 00 units wi th no beg
27. The f ollowi of B: ERV % 2 8 O (F) LEV? 5 66 i Le
oye oe (A):
the month © f Apri l, has bee n 6,5
in one of the was the camposition a
: of workers OTs i inj a factory durin
“40 (F)] (i} Production during
were as beloe e tuction departments me gang
standard composit tat month in-progress inventor
ies.
position of workers ay ‘ge lon
@ rate per h ‘
{it} Materials: @ € 1.08 per gal
32,000 gallons

bs fives
Skilied ~ workers
: Two
our
tate of z Purchased
Semi-skitled - Four wa i at a standard 25,600 gallons
Unskilled rke:sTs at a standard rate of anne hour each Used in production

e
"
12 per how Y each

coe pets tr =
: Fo ur work

as
ate standard
at @ star dard Tate of Fg
(iii) Labour: 20,000,
Output per hour each
stion, however
of ‘@ gang was four Hours worked Zz 175
units per
rate
+ tire
he ac in
Average hourly wage
hour of the

of worker :
Type
(iv) Factory overhead z 12,500
cost incurred ory overhead.
Total ove rhe a 4 l variane e for fact
Skilled our va vari ances and only tota (B.Com., Hons. Delhi}
per ho variances . tab
Caleula’ te material
2 Op
(fF), LCv = 100 (F),
Semi-skill ad 3
% 992 (F ), MPV & 912 (F), HOW z 4ao ial. acy € 800 (Ay)
wh Unskiiled
5
[Ans. MCV Lav Z 1,000 (F). LEV
% 900
e€ Gang was en “ ° variances
:
was possible, ine age:
ea for 200 hours during th 10
ad expe nditure and
vol ume
eulate fixed overhe

WES,
e month, which inctuded 12 hol cat
gag during the month da chine by. bre. akd OTS when no
30. Fro m the fol low ing Z 1,0 0,0 00
onth of November,
own . 810 uni ts 0. of the Production
Produ ct wer @
rhead for the m 5,000 units
_ Budgeted fixed ove
: I record 2 d as
You are Tequired outatput
for the month *
of the
:
Budgeted production
ot fa) =
tota i
the standard unit labour cost of the product: 54,000 units
{ variance ia lebous omputeUw ting the month; and fc the month : (M.Com., Madras}
vari lances and T2concile anal vse the varian
i cesct: in (b)(b) to abecomput e the Actual production for month :
F 1,20,000
ance 7 8,000 (F}]
) ve in sub- ad for the [A}; Vol ume Vari
fAns. ie Ley z Actual fixed gvethe Variance % 20,000
28. The following st anda2200 (A
). DRY z q 2 0
4,20 (A), (ROW, Enter) (Ans. Expenditure
tis have been set fo man 440 (A), Lyy 2
Direct Materi
ais u ‘ 1,740 (FJ owing calculate:
m th
Cost Variance
‘{a)-Variable Overhead
2 uni in af 4 at 4 per unit
Variance ©
Efficiency Actual
Variable Overhead
3 un ts of B at z 3 per unit 8 (b) Budgeted
15 nits of ¢ 320,000
at z I Per unit
|
9.00 | 3,060,000
26,000
15.00
Overhead = 30,000
7
60,000
Direc la
. Dour 3h
: is. + Output {units} 75,000
(M.Com., Bharathidasan)
@ FB Pp er f ur
Total Standard
! PrimRe Working hours
(B) 2 20,000 (F}j
Cost
5 24.00

" . E wy mar ufa | 56.00 fans. (a) 2 60,000 (A):


Were ag fol ows:
y.
tecords of @ compan
cfured-and f units
sold 6 00 , of the pr O duct
is availabl le from the
i urin g \ the
deri
12,506 unis Actual
hy yi |ear, . Di 1Te:
CC tna ter: jali costs
low ing inf orm ati on Bud get
32. The fol
OS)
RUS Of A at & 6.4. 0 per unit, 12,000 °
a % 10,000
0 .
for Jun e co g
*. Fixed overhead r o
2% | J 10
490 anvo
aeons” -
Management

Standar, ‘d Costing
t and Variance Anal
nalysis

13.000
tiont vin
ucar
i
June {units} ka- 16,000 27,300
Prodew
St 2,000 z 32,000
ha ime per unit (hrs.) 2,100 . Materials: tity
Actual
in June (hrs.} : 10 Total quan 2,000
urs worked = nt
comput Is. a Total amou 2,40 3,000
Qe; Fixed overh ead cost varian
ce 22,000 Labour:
;
€ 3,000
: pols(t)
moui i
Volume variance
(ii) Expependndiiture .
. Varian
ce
Total ho ur s
2,009
titv) Capaci _
nt
Gd } Efficiency variance
Effici city var Total amou Expenses: 2,406 2,200
5
iAns. . (1) z4 1500 (A). {i @ (B.Com. Hons variable
erhead
Ov z 2,400 :

€ 2,00 0 (A ), (ii t) i,
10000 on(F)s.. De(¥)th %
Ad apte
500 ta Total hours a
rom th pr
.
ep are v , ) . (H Y 50 0 (F), (iv) % 1, ing varian
ces:
e Yariance
following items: is nt the follow
a
is af partic ular de Total amou rial Usag
Variab . ® Overhead anance analys compute
for te
partment a month: quired tO b) Ma
ncy Yarian
ce
You are re ppecie
Va riance (d} Labowt ans®
Material handing (a) Material
price
er he ad Budget Yari
Rate Variance (A Ov Inter)
Idle Time {c) Lavout Variance (CA.
Efficiency )
Re-work (e) gverhe ad (4) z 250 (A
° 825 : Cost Variance cay , to? 500 (A). {g) z 400 (A
ertime
Overti prenmitu (9) Overhead = 2,000 0 {A).
1,300 (A)- tb) {e) z 200 {A}. (Fy t 20
Supplies
: (ans. (4) z
Dept. Y
fi yariances:
ixed Overhead Item st overhead Dept. X 2,050
—— calculate
: : g data,
supervision the foltowin 4,000 z2
ays = wn 0.50
Feat of plant pacity z 4,600
; normal ca
han
epreciation of equipment
i
,
wh en working at 4,060 1,550
a
5,000 Hours. te
pourty ta 4.450 3,750
Overhead production
for actual
Lcw.a. Inter)
1,159 a z 2,020
Insurance lo we
.350 Hours al urs Adopted.
ils — Ho ns Delhi, ov € 950
(A)
Actual deta (B. Com Ho
10;200 Qverhead De - y —
pt 800 (AY
av _ nit (F). ovy 2
Normal c apacity
y i 10,000 stand ard fix
houedrs, _ b budgeted rattueat = le7.70i per standard hour Tf for variab overhead (A). opy
= 20 {A) ogy = 259
and @ j per standadarr d hour ndfor overhead, Ac ocy = 29 for Decemb
er . 2005 :
vel is 8,. 000 Standard h hours. nabl . X —
e
d [Ans- Dept a company
j e kd
Vari WA. Inter) e from th
[Ans. VO Cost Variance % § 50 (A), £0 Cost Vari (LC 2,200 (A) in formation
is availabl g8,q0e
20 0 e g following
FO Expendiiture Variance 2 quta nc ‘
34. From (A), FO Volume riance © 2,000 (A)}
(a>: pieces
foit awwing prepare overhead variance analysis of d 29,000 24,750
factory
the over h ac ti
a par cular de partment f i s purchase
Material pieces
i = 14,250 ora month. Actu
al med 19,0 00
Budgeted p: ‘ g 40,009
od ucti 1on Capa ¥ 10,000
ci t
hour s, budg et ed ab. ~ 44,000
standard h our fo ad Actual production | overhead
absorptionn rate & 1.7-70 0 per
t factory overhe . evel 8,00 standard hours,
curred
averhead in
-000
:
(B.Com. # ons Deelh
Oc v (A), OBV tar
lhii} Fixed factory
. [A ns . % 650 (E), OV % 3
40 0 (A} uced 1809
bud Units prod
75 0
the
get an d capacity ia ‘
» De te rm j ine var ing d
neces fro m. the follow g data _ : pr ices ate °
Estimate d er he ad
standard ra
tes and 4 per piec®
pactory LaOv al tate—~
Estimated0 bour Ho m5 5,000 Direct materi t unit
wag _P es pe
ec
irect py
Actual
verhea d Expe nses 5,000
dard in pu t—10
stan t hour
Applied 0 verhead £ xpenses
~ .
t 2 6,500 Bi
ate—% 4 pe
produce a
pnit—-2-9
hours

posobe
r |
z 22,500 i

{Ans vari nce % i


500 (A), Cap
:
aci (LCWA. Inter)
{Hint—s
» Budget
0 =ia
ci ty va ri an ce & 2,, 500 ( A)) | e followin’
t. overhead rate | 25,000 + 5,
pa
00
ci t 25 pe r ho ur; ; ActuActuaall hours Ts 22,500 +; i 4,500:
2 3 Compute. th usage:
l Ca ity Varian ce = (4 : price and Hons. Be
Ltd. produces a singgle product t - 5,000). » %
es {a} Mate ri al
36, DCW a single mate
. ri al. It ; 7500 2,500 (A= rate and ef fi ciency:
e;
(B.Com.
e M
e vat janc
It furnishes th {b} Labour
fro m
in -
:
followin: iq informatio ni: ex nditur
pe
. : ” Fi xe d overheads ri an ce .
its (c ) va
Gutpet un Act ctual overheads
volume
-tt
8,009 6,000 (a) Fixed
mo,
. unting
Management Acco———
Standar, —
—_—
ar rd Costing and Variance Analysis
3.101 - 3.402 Sales Mix
Volume Variance, (a)
(Ans. (a) MPY Sal es Pri ce Variance, {c) Sales
- 7,60
600 {b)
s Value Variance,
( A ): MUV
2,800 (A);
4,000

0 fF). (2) & 51,000 (Al


( A ):
Calculate {a) Sale
(b } ERY t4 ‘ 959

(6) € 4,000 (4),


{ A); LEV Z
39, @) FE. m the following data, catcula id) & 400 (ayy Variance. (€) Sal es Quantity Variance.
00 (wv. (c) 42, 000 (A) | (a) & 9,00
te the following variances: (b) & 3,0 pro duct for the
on 4 [ans. (a) © 45.000 {A}.
. 3 single
Material cost variance acturing and selling
if} Materi ‘a Material
i prici e variance: get and actuals of a compay manuf
aterial quantity variance: bud
) Material mix variance 42. The details of follows : Actual
&) Material yield variance @; , year 2011-12 are 25 Budgeted
64,000 i
Materials 80,008 110
. 100 t :
Sales (units} z 5.4 kg
unit 5 kg :
Selling price BET g 7.50
|
/unit) zg 8.00
Raw material (dt. 4
o.8
al pet 9
Price of raw materi z 2200
pef wmt 20.00 -
Direct labour hours z
Standard loss : 10% r 5.00 z 4,80
9.60 i
Wage tate per hou
unit
zg
z 40.00 z=
Actual output Variable overhead per out the
: 180 units two by ( b) working
Fixed ove rhe ad pet unit
act ual pro fit and yeconcile the
any has a acit budgeted machines working @ h budgeted pro fit
and
th.ctuThere fix& ed ove malads capare
norrhe j Calculate (a) the
fanufs per month The per day of 25 davs
tion is " : 1,44,000 var ian ces - variance
t of produc following (vy Labour-rate
In Aprit 2006 ne uni
Ne standard time requited toa (i) Sal es pri ce var iance
(vi) Lab our -ef fic ien cy variance
units oF out pany wor
, the comactual 24 da ve
‘ ked overheadys of 8 40 machine hou ts per day and prodated § var ian ce ove rhe ad variance
“WO6put, fixed tii) Sales yol ume (vit) Variab le (LWA inter}
Compute: The s were % 147.000 1305 var ian ce variance
q = % 3,046,000 (F)
e rhe ad
: — tin) Material pric (viii) Fixed ove
i ) Effie; ge variance 000 - 20,00.00
(iv) Material usa Vari ance © 23,04, 00 (F):
0,000 (A). “iy € 1,79,2
{i) fficiency variance: (i!) (a) Pro fit
y variance: fii (Ans.
A Capacit i (if) F 4,0
[Ans. (a} (2) & 760 (A): (i nance; (li) Calender variance i (B.Com. . Hons - Dethi (b) (i) £ 6,40,000 CE): 1,02,400 (A): vi) @
2,56,000 (F):
(vi) & 3.07,200 (A) (v) & 600 (FY
(vu = 25,
b) (i g 6,36
3 0 (F):
(F
¥) & 440 (E); tv) % 220(4} {vii) = 12,800 (F}:
B of Messrs. KY
(i) g 17,2 8
280 { Ali} i
(ii?) g 5 2760 {A)] ts A and
f cloth wa of two produc
ur hours @ Ss &pre-determi
40 per hone as 20 skilled the month © f December, 2011
. actual sales for ee
a nd 30 unskilT led c l
lab o t h wa
our Mod uce d wit h the help of 30 43. Budgeted and .
Pout Hows paid @ & 17 per h
skill Ltd. were as follow
s: ce/Unit ‘
Catculate :
rs paid
paid @ 8 12 pee hour f per hour

Budgeted Sales Price/Unit


Actual
Units
Actual Pri
z
mixmi
(i) Labour r : . yield variance Product z
(b } Details ° of overhead and (ii) Labour
fixed Variance, : a units
5,000 f yr. 5.00 -
5,
duc tio n hours and productio: n for a peri 6,000 5.00 1,500 als i
B Pro K
udg eted hours 2.00
he on
ndard ho 00 7,500 gy
Sta 10.000 us | 2.00

reer
1.90
perrh unit of output
StandardG £jfixemsd ove oot 10,000 1,750
(overheads per h 5 hours Y
Work out from
' Actu = 1,920 unit
= io
= 4.50 per unit
respectively. CSA
het ual ho at io n its % and ¥ were z 4.0: 0 and
VG
ixed overheads . pro duc ts
c,
alcuiate : Budgeted costs for :
® 94,000 following variances
the above data the

a
variance
(*} Fixed overhead . ps (vi) Sales mixture
Lg ian ce variance
{i} Sales value var
COSC vari
volum .
® variance ance; (ii) Fixed overhead expenditur
ature variance, aand {iii (v) Sales quantity
itt) Fixed vatiance
(B.Com. Hons. mead {ii} Sales volume (A);
i) LMy z @ 1,782 (F&F 7a2
(A}, (iii) = 1.000(F) (ivy
{Ans, (a) (
‘A:
price var iance
(iif) Sales
ee
30 (A); (fi) EYV & 189 ©) Gi} = 5 50
data is given
{®) (1) % 2,000 (F); (ti) % 6,000 (E}; (iii) & 4,000 {A)] fans. (7) = 450 (F),.
41. '
Following
the following dala.
ivity rat jo from
eff ici enc y Tatio and (8) act gq units
44. Calcutate (4)
Actual Budgeted pro duc tio n 16
unit 8 units
Standard hours pe! we
n 2,000
. Actual pro ductio (LC WA. Intet.)
Actual hours worked {b) 89% (Approx.
}j
{Ans. (a} 40%.
Stand lard Costing
f and Variance Analysis
.
, 3.203 cot
45, From th @ fotlowin ‘activity ra
‘A’ 4 data, calculate and efficiency
ratio’
products
‘8. Standard hee arity ratio’ product ‘4’ ot : A factory manufactu res two
The bullget ed andand actual production i manafactare were as ours and product ‘8 10 h ours,
m December :
as follows : :
Actual
; Budgeted
Product ‘a! production
eee Production
125 uni
units 100 uni
Product “2 2«C«*«*<“‘<CS*# 30 units
units
Total_ actu al hours worked were 660. 24 units
Te 669. (B.Com., Cali
+ Calicut)
!Ans. Activity
ctivity ratiorati 80%, Efficie .
ney ratio 665%]

— meaning, i
Absorption costing
Product costs and period costing — meaning,
Introduction; Marginal
and income determination, pistinction between absorption
characteristics determina-ion; marginal
and income absorption and
characteristics
Difference in profit under Key terms;
costing; of marginal costing;
. and marginal
Advantages and disadvantages
~ costing; questions.
solutions; Examination
Problems and

ascertainment and
Introduction the two main techniques of cost treatment of
are 18 regarding the
and variable costing between the two also fixed
Absorption costing of distinction costs and
' The basic point alt yarable mamnfacturing produced includes alt
. income determination. absorption costing,
. overhead. In je cost of goods
fixed manufacturing the cost of products are included in the
. costs aze included in variable manufacturing costs to
manufactusing costing, only charged directly
. But under variable pariod costs and ‘
manufacturing costs. costs aze considered a5
but fixed manufacturing ‘
cost of products costs (Le. administration,
Profit and Loss Account. and marginal costing treat non-nanufacturing
costing e income.
Both absorption patiod costs. ation of taxabl
lin g and dis tri but ion overttead) as te ma l rep ort ing and comput pos es.
sel d for ex managerial pur
g is mostly use ie, it is used for
Absorption costin d for internal reporting
is ino stl y use
Variable costing
sts
AND PERIOD co
PRODUCT COSTS will not be incurr
ed if
ary for pro duc n and which
tio
tor y av er ted.
those costs which
ave nec ess e of the fac
Product casts are dir ect mat eri als ,di rect labour and som inv ent ori abl e costs
of led
n. These consist duced. These are cal
there is no productio bed by’ oT ‘attached to’ the units pro
‘ab sor
Product costs are in the cost of inv
eniory.
and are incurred
n
even if there is
included ary fot productio Such costs
because these are ts whi ch are not necess ch these are incurred.
tho se cos icd in whi salary of
Period costs are enses in the per of the period . Ren t,
are written off as exp Loss Acc ount
no productio n. The se
are charged bo. Pro
fit anc entoried; 1-€.,
time period and costs are not inv
are incurred for a period costs These are generally
expens es are exampies of selling expenses
company execut ive s; tra vel inistration and
of inventory. Adm
are not inc lud ed in the value
these
costs.
tieated as period
Costing
Management Accounting : us Variable (Marginal)
Absorption Costin ig Vers
based on
determined overn ead rates
ee

cost may be char ged on actual basis or at pre


3. Fixed
Recorded as an asset in
the normal capacity. ory overhead.
form of inv entory or qver-absarption of fact costs are treated as
i 4. There may be under selling and distribution
Balance Sheet ¥ in the
Non -ma nuf act uri ng costs ie. administration, but are charge d to Profit and Loss
5, ucts
s. Thse s are not incl uded in the cost of prod
period cost le costs.
These are not inventtoriab
Account of the periad.
ting
under Abso rption Cos
Income Determination ion costing}
Tacom e Statement
(Absorpt

Petiod Cost Rectrded as an axpen


|
the Profit and Loss Account
of the current period Sales
" xWKEK
Production Costs :
ed
Direct material consum
eax ex
:
Fig. 4.1. Accounti ng treatment of
Preduct and period costs. Direct labour cost
RR

It is]
to note that coms
“OM: onerits of produ ct cost and period
i cost are different in ab Variabje manufacturing over
head £EAEK
costing
sting and variable eesti
we absorption overhead EXEKE
Classification ‘ csting.
. - Fixed manufacturing
al inte pro
productd and Cost of goods produc
ed
T
A tof
18 So because product cast | period cost is impertant from of vi
the point
‘s carried forward to the next accounti part determinati
of view of asprofit of the on.
nished stock whereas period
‘nod cost is written off in the accounting
nets
unsold goods ~
Opening stock of finished period's
it is j od in which Add: iaus
production) KX KK
tcurr ed. (valued at cost of prev
e for sale «KR
ABSORPTION COSTING Cost of Goods Availabl
shed goods
anyThis ereis a Less: Closing stock of fini
technique under which total
cost ( Le, cast of current period) KEKXE
nae fa nate
vari manufacturing cost as well (valued at production
Ting cast) is charged as producti
mann ‘ words, in absorption costing at Cost of goods sold
Ting costs ave ‘absorbed’i } absarption of Axed ARNKEK
ove cont of the po Ada: {or less } Under {or over
eads are absorbed on actual head
manufactuting over
MEEKER
c
mal capacity. Under/ costs MK KX
Selling and distribution
KXKAR
pend. Closing stock Add: . RKRKX
is: Administ ration cost s
oe aeatation ovethead Total Cost
MMR KR
also} . Absorption costing is a ——— ne
)
sting’ or Full Costing’ Profit (Sales - Total cast

.
Direct materials
the followin g data:
Direct labour Zen Ltd. supplies you 48,000
fatiabie factory overhea Charged to cost of
Charged as expenses 22,000
Ixed factory averhead goods produced when goods are sold
Direct material cost
13,000
Direct wages
2,009
Variable overhead—Factary
—Adm. and selling 20,000
—Factary 3,000
Aik settin g@ and7 Fixed overhead
—Adm. and selling 1 .25 000
adm. overhead Charged as expenses Sales
. on costing.
a when incurred ement undet absorpti
Prepare an income stat
Fig.§. 4.24. Absorption
i Casting Approach
Characteri so
' “aot ba ie aain characteristi¢s of abs
' :
‘artahie Tha
. a
products, tantifacturin ig cas
‘OSiS ar id fixed 4
manufactur FV] 0. s
cost are c har g je d t 0 O:
cost 0) f
2. Stocks are also
valued at total cost i.e. variable plus
fixed
2d cost,
iable (Ma rginal
) Costing
4.4 ting Versus Var
Absorption Cos
_ Management Accounting : t
25,000
units _ 5 000
ble) cost of 109
Inco me
Marginal (varia _ 30,000
Statement (Absorption Costing)
Fixed cost Total cast

will appeat as follows :


{A) Sales unit, the cost 25,250
out put is inc reased by one
If 5,000
Direct materials (103 * 250)
Harginal cost “30,250
a wages aa ~ ; va22,006 Total cost
ized cost
cast.
ch is its marginal
actory overhead —Variable 0%
» add iti ona : unit ig 2 250, whi , thi s ma y be
ing om lume af output
L cost of produc reas e in the vo
Cost of Production | oe 1 Thus the additiona 1 th the inc dea lt wit h as a
fix ed cos ts also increase inc rea ses in fixed costs are .
Adm. aiitu seuing iv
overhead ~ 0
Higwever, whe re Suc h oR Decision Mak ing
tion capacity. sed in ¢ hapter
rease in produc ana lysis’ discus
~-Variabie _
the result of inc ren tia l cos t
known a5 ‘diffe
lt 8,000 16,000 4
part of what is
(B) Total Cost vee 0 | variable
le Costing syst em in which
1,13,000
12,0! 00 Meaning of Yariab nd on 4s “rh e accounting l age inst the
DY C.L.M-A. Lo period cr e wri
tten off in ful
iab le cos tin g is defined cas ts af the
Yar and fixed
profi
plion costi ing, fo of Purpose of determining the amount of profit/loss, d to cost units decision making
”.
Under absorpti a costs are charge l value is in
St atement isi prepared. the foumat i is as foilows : , an Income on . [ts spe cia
Statement co nt ri bu ti
ch Income aggregate
of Variable Cost
ing y ‘be summed Up
Characteristics al cos tin g technique Ma

orect mara
mar gin
oT mechani sm of
Di t acteristics and
to Ch arged as The essential cha costing all costs,
af
Charged . T™ variab le
Var iable 1
|
+ cost of goods
expenses when as fol low s -
ed and vati ab le el em en ts
e into fixed and
var iab le
alsa 5e gregat d
factory
overne e
produced goods are sold 1, Seg reg ati on 9 £ costs into fix
Semi- yat iab le cas ts ate
ad I
7
classified inte Fix
ed and variable.
d te products
of yar iat le costs are charge
al
. elements. ts. Only margin
as products cos
2. Variable costs
~~]
.

|| charged to
andy
iod ¢ osts and are
Fixed factor Charged as the period .
oyernean when
- produced during cos ts are treated as per ed .
| expenses as per iod costs.
Fixed they ate incurr
selling and 3, Fixed costs od in which va lued at variab
le
ineurred co un t of the peri are
adm. overhead g Pro fit and 10 as Ac
an a fin ish ed sto cks
Costin work-in-progre
ss
saat invento ry. The
4, Valuation of and variabl2 cos
t of
en ce he tw ee n sales value y of
cost only. is the di ff er on @ st ud
i . Contributio n s is based
ig. 4.3 Variabie Costing Approach 5, Contribution. pr od uc ts OF department
y of
sales. The rela
tive profitabilit ot departments. ge
the products d by & two sta
ibu tio n’ ma de by eac h of
cos tin g, pro fit i 5 calculate nt . The
‘contr fit. In margin
al OF departme
Meant ng of Marginal i Cost and Variable Cost | sting and pro each product
§. Marginal co is determined for and such a total
of
; . of alt, co nt ri bu ti on eth er
e same as variabl e cost. Margiginal cost is are pooled tog
cost i “at
tae ginal
Mar fis th
approach. First me nt s the tot al
additional uni it of prod uct. It ;
rt
of producing an various products
of de pa
this fund is dedu
ct ed
oe panased ofcostall diirect costs
|
s, contributions of nd, Then from
var iab le cost is called ‘Ey
defi: ned maigiafginal cost1 ‘as
val Londen Pea8 Or a all pr od uc ts
n ch a The CIMA
r
output beewhi any given volume
of contributions io oF loss.
:i ggrega te cost.
ume of out put is me amount at sed by ane unit !
|
cos t to arr ive at a profit
It nel if vol see or dec rea fix ed
t nt of is one unit of voduet which would be avaided if nen r tt not produced oe
An
inet tantcospoi that variable cost ine epi eti
per unit remains unc
hanged
tive of the le vel of activit ye
Example ~
A company
t Manutacture
Tu s nits
i of a pia' duct
duct iyet mont .
per month Tot al fixed cos; t per month is % 5,000 and marginal
cost per unit is-2
‘is- 250, The total cost per m i "
Harem hci
47
(Marginal) Costing
Sa
[Stes of pradut a
in ct A
.ghsorption Costing. Versus Variable

_ les Sales of product 8


Sales of praduct ¢ - §olution
Less Income Statement (Variable Costing)
| cost ae Less
Marginal cost of B é
[ote
2 Marginal cost of ¢ 1,25,000
| Contribute ~ofof AyA
ionon t _. (A) Sales 78.900
i
Contribution of B Direct. materials 22.000
Contribution of ¢
Ie Direct wages 13,000
Total Contribution Variable overhead —Factary 2,000,
| ~
| —aAdm. and selling ee RAD
Less

ediniaadsins ab
(B) Variable cest aval
Total fixed cost
(€) Contribution (4 - B) 20,000 |i
Fixed overhead --Factory
8,000
—-Adm. and selling
__ 2 8000
Fig. 4.4
. Profi
rofit ascertaiainment
i Marginal
in
(P) Total fixed overhead 2.000
i costin g in a multi -pro
p IC duct
d c ‘Ompan
¥ Profit (€ - D)
Income Determina
tio
inat i n under Variable Costin same. This is because there are
g costing and vat iable costing is the
income statement u Comments. Profit under absor ption there a je opening and/o r closi ng stocks. praiit/loss under
nder variable
i . However, when
costing is prepared in the € f
tolicwing
it no opening and closing stocks
sae eee form. at : be differ ent.
In come Statement
(Variable Costin ) the two systems may
© osting and Variable Costing
Sales Distinction between Absorption as
ption costing are summarised
een variable costing and absor
Variable Fae actuating
costs The points of distinction betw
follows:
between absorption costing and
~~ulrect material _¢
consume : Basic difference
~~Direct labour 1. Treatment of fixed and va riable costs. costing. only variable
oe
.
of fixed facto ry overh ead. In variable
con
~~ Vari manufacturing overhead marginal costing is in the treatment charged to Profit and
treated as p eriod costs and
até: Oe of goods produced
re costs are charged to products. Fixed costs are
_ : cat
Cost ning rs the period.
stock ofaieinished
fini goods
ee (valu ed at variable — Loss Account of to product. The fixed
i cost of Previous period} and yariable)-are charged

se in absorption costing all costs (both fixed pre-determined on the
. at a ctual basis or at a rate
is absorbed in units produced
cnt Stock
goods ofsaefinished g
oods (valued at current variable
cost) factory overhead
| me~— basis-of normal capacity utilisation.
: Tlddte adm., ” sellj ing
and di
cat Variable Cost Example
, mh oreread
tess) Fe ws :
costs (Pon
ontributi systems is computed as follo
_
~ Total variable cost)
|
“we Product cost under the two ption cost fariable cost
Wet Prana Toduction, ad
m., selling ve
* Absor
i and dist.} per unit
tens
per unit
=
x xOOW
gz
Or z
Tlustration 4.2 50
50
From the ¢ ata giv Direct materials 40
en in [| ust tation
i 4.1, prepare 2 anar Income Direct labout
40
Statement unded r marg 25 25
inal costini g.
Variable factory overhead oo
_
Fixed factory overhead 115
1a?
Product cost
goods are
stock of- work-in-progress and finished
n stock. In variable costing,
2. Valuatio‘of
valued at variable cost only. includes both fixed and variable
are valued at total cost which
In absorption costing, stocks ¢ han that in absorption costing.
ble costing are lower
costs, Thus étock values in varia
\
4.9

Costing en
unting” le (Marginal) :
48 Management Acco Absorption Costin
g Versus Variab
ig g Vern Yor
abintion Castin {Absorption Costing) ee
(b} Income Statement Total
3. Measuremen t of profitability.
itabili In variable
i costing, ' relative profitabtti
ability of products cts Or; “4= Products
departments isi based on y of relative contributi
:
ion by respective products or dep
art z me Y+2)
The manage
on sa stud
r:ial deciision
aget are thus guided by contribut nl.
ments
x Y g
ee Ft
a
A


ged
i ? i di by profit figures ids
in absorpti Ion Costing,
" € relati ¥ 2
ie P rofitab RLY
tit 15 which 1s aiso
\ a gu 1]ding
=
40,500
3,000
30,000
7,500 19,500
9,000 ___ 1.500
Direct materials 9,000 60,000
4,500
Direct wages 16,500
39,000
From the followir i ormation prepare an Income Statement under:
wang i
Prime cost 6,000
1500
( a) ) va ia bie cos tin
i 9 (b} Abs orption
i costin
i g } Factory overhe
ad
3,000 1,500 17,400
__ 4500
Products —Fixed 3,900
9,000 "33,400
x 49,500 40,500
—Variable 23,400
Cost of Production
Z
=
- 3,900
z 600
materials
30,000 Selling overhe
ad 900
Bi irect 7,500 eon 1,500
___ 3.006
11,100
9,000
Direct wages ong 500 —Fixed 2,100
6,000
14,100 97,500
factory overhead
—Fixed
000 1,500 Variable 27,000 56,406 14,500
1300 1,900
—~—Variable 2.900 9,000 4,500 Total cost 5,000
4,600
16,000 “709,000
‘ $1,000
ae the mi
~
.
Selling overhead —-Fixed 1,500
soca 600 Profit 32,000 t

2,100 3,000 ee maiginal costing and


absorpuon
Sales
—Va
rable 32,- 000

61,000 abo ve tha t to tal profit under the clo sin g sto cks of finished
be noted from the is because there ar e no openin
16,00 g and
° Comments It may
Fixed factar ‘ were apportioned to O pro prod 500. This
same, ie, %
11,
and fixed sefling overhead
j
: ble bases,y overhead
produc ts X, * and Z on costing is the G
equita —
B. Com.,} wor k-i n-p rogress. RPTION COSTIN
STING AND ABSO
goods oT
VA RI AB LE CO valuation.
PROFIT UNDER stock
Solution DIFFERENCE IN of difference in the
stem s Ma y be different because
o sy
{q) Income Statement (Variable Costing) Profit under the tw is summarised 25 follows :
: ti on in thi s regard
Posi
Products Total equal to sales absorption and variable
{a) production profit/loss under
x Y (X+¥+ 2) op en in g and ¢ iosing stock,
z are 10
(i) When there two systems
z
< z costing systems
are equal.
stocks then also pro
fit/loss under the
cks is the same
to closing and closing sto
g stock is equal
(A) Sales 32,000 61,000 16,000 1,09,000 {ii) When openin ed the fix ed cost ¢ lem
ent in opening

Variable costs: > be equal provid


will
Dizect materials 30,000 3.000 amount.
7,500 sod 40,500 -
more than
Dizect wages 9,000 9,000 19,500
(b) Production mo
re than sales ie, when closin
g stack is
by vatiable
Variable overhead: : ‘
per iod is more than sales, mor e tha n tha t shown
9,000
4.508
When production
during 4
ab so rp tion costing will be ad included in closing
Factory 3,900 17,400 profit as per of fixed over he
Selling 6,000 3 000 opening stock, the a bs or pt io n costing a part for m of closind stock.
2,100 -.
1,1“ 00 because in peciod in the
(B} costing. This is nex t ac co un ting
Total Variable Cast 22,500 54 000 12,: 000 88,500 stock value is cat
ried forward ta
Contribution (4 ~ 28) 9,500 7 4,000 3
s than sales is more than
en opening stock
000 0,500
sel i
factory and {c} Production les
on
al of fixed ie., wh tion
ool: fixe d cost {Tot
Leds sal es,
t shown by absorp
overhead} n
selling 9,000 is less tha
during a period be more than tha
rofit a 11,500 When production own by variab le co st in g wil l
s sold is higher be ca us e 4 part
stock, pro fit sh g, cos t of go od sol d in
= closing rption costin cost of goods
tin g. Thi s is be cause under abso is add ed to the current year's
cos ing period
from the preced
Jaf fixed-cost sto ck. ting
‘the form of open in g and absorption cos
ect on pro fit of variable costing
eff
qo summarise the
: = 4.11
4.10 ———
Management Accounting Versus Variab le (Ma
rginal) casting
7 Absorption Costing
For instance ,
tor is als o ‘ignored.
a ee

ed costs, time fac


——.
oS

tor. By ign ori ng fix twic e as long to c omplete


2. Ignores time fac be ide nti cal but if one job ta kes t of the other.
] variable cost of Two
jobs may higher than tha
caution =Sales - ofit in variable costing Profit in ab H
of t he job tak ing longer time is ed wit hou t incurring
e cos t n cannot be ach iev
___Preduction ag the other, the tru costing. Pro ductio g to do with
Production <> Sales va ria ble not hin
HiLeet sige
ower ~~ This is not disclo sed by
tes an illusion
that fix ed costs have
o iable cos ting crea
ADVA NTAGES AN D DISDvn fixed costs but var
ADVANTAGES OF ihVARIABLE COSTING in industries
sa production. iable c asting technique
; fic ult to app ly var construction
Advanta
ges. lic ati on. it is dif
ked up. Thus in shi
p bui ldi ng or
advantages are claimede& f [or 3. Difficulty in app
: :

costing: The following variab le costing over total absorpti on


0 £ work-in- pr og re ss aT 2 loc
valuation © f work-i
n-p ress, there
rog
where large stocks s are not inc juded in the . ther e may be huge
ov erh ead letion of contra cts
contracts, if fixed
i, Help in man ager
ig ial
d de Cislons < Thi @ most
0 impar tan le on the comp , is employed.
eac h year. whi al absorption costin
advantage
on
that
3 snit tend
coners to m anaseme or vaniabi 6 COSti
‘oOstiAG G L Is the as: SSldsiCe
n i anei
gement may be losses in be avoided if tot
bddlc
may valuable decisions. Informat s can
i pro fit portion
atio ns in ive industrie s, the pro
4
profits. Such fluctu ind ust ri es. in capital intens tec hni que,
i ne cy shelow c se ake cap ital int ens ive cost ing
or buy, introduction of a Oe prod 4. Less effective in is large. T he variable
of sparenl olateselling prices beloSha
w cest feuct s Laeeeb intenance atc.) increased
a yex make
@ mostbu y itabintodartin ienn rec iat ion , ma ust nie 5. Wit h the
prof le product mix “at et. th tis he has ibe
been sa af fixed costs (like dep less effective ins
uch ind
ted scope.
discti
usse
nd @ cost, thus proves , left with a limi
which ignores fixe le cos tin g 1s, the ref ore
var iab costing qives
compatitio n, variable
2 ustrie s,
automation in ind
Cast ¢ a ntrol,
G reat er cont:
is possible.e. ThisThis
ces ar e fixed by
ntrof over cost
into fixed and variable, the man isi so because by classifying er e pri profitable. Tt
costs of pricin g. Wh tio n is
variable cost, pro
agem the cont duc
" concentrate more on
3 5, Improper basis pri ces exc e ed plus some
¥ contuollable ond on that so long as cost or Var jabie cost
controlled only & variateb be the impression m ade at variable product
attention
and thetat too, to fixed costs which
limi which may y 0 £ too mu ch sal es bei ng
ough in cert ain
cir cum sta nce s
the top management
.
te a a limited extent. ignores the danget ove ra 1k lasses. Aith tota l cost as
yy
4. Simple tech nique. Vari i . ma y Tes ult in
the long run mus t cove r
ariable F “osting
-OSting 1§ com
:
to operate because contribution as it cost, prices in
ir. allocation cpnottionmn. simple less than total
the complicati
plica : it avoids
which is, in ‘ ct,, involved and may be sold at °
arbitrary division of indivisible fixe can not be earned.
:absorptio n of fixed
i overheads
xed cost otherwise profit
4. No und :
er and over-ab of SOLUTIONS
under- oF over-absorption
absorption
of othe In variable costing, . there there isi no problem of PROBLEMS AND
.
5. Constant cost per uni .
i
takes into ac
temain the same ter untitVariable costing
. Problem 4 ending 31s i Dec, 2017.
. foe which data for the year
af the ‘volume ¥, cutout
tutput. it therefefore sup pli es you the fo lowing
of varying 206 . per ue
Dim, ne tte nd have as it\ ignores s fi fixed co xyz Ltd. ts
6k ; no telatio units, Sale 5 1,000 uni
Which are incurred
on Production— 1,100
size of production.
evanable thecostin g stpck.
at Progress and fini _ There was no openin
of stocks. g, stocks of :
ooode bein
cost vated only at variab‘e cos ts. Thos no fictitin ge eek
a valuation
arise due te nished
{
uring cost per
unit 2,200

AL AH
can Variable manufact
in unsold acutious Profits
i {t otal) 0.50
th absorbed and capitalised | ring overhe ad
rhead pet unit
prevents steck val uation of some p his is because variable : ned I Fixed manufactu
administrati on ove 400
forward incosting, fj rosting Variable selling and
1
& carryin marginal

AR oR
Steck valu ation portion of current year’s ‘
ation ov erhead (total) 15
is, therefor “ed costs administr
7, Aid to profit .
Te more realistic and . unifor m. Fixed selling and
be presented planning. To aidan profit pl anning Vari riable costi
.
Selling price per
unit
to m
enables data to
management Co show cost volume
.
Graph form ot bn 2 way as Prepare le costing.
the
to facili Tesentation in eve charts and piofit-volune
hone relationship
ume charts are also
used
(a) Income statem
ent under variab
orp tion costing. if any.
veto future
to other techni ance.
{b) Income statem
ent und e r abs absorption costing,
under varia ble and
we planning
(B.Com Hons. Dethi)
8. Valuab
ctstiny adjunct ques. Variabl £ costiingng isi a valuable dif fer enc e i n pro fit
adjunct to stand
st ard fc) Explain the
: and bud:
Thee contro!.
‘ re
ad t ges,
Disadvantages,cialis main
va disadve nates of variai bie
costing are as foll

es
? 2

a
and ai ets In practice noWever, it me be diffe ee se
cannot rabl e comp
tly oten ified Certa
€ stric classts, as fi ixedal “costs ae eanaed nay pce eal et i a
Orot vaiab
variale
ble, ee
e.g, smenetiee o
staffe
. “bonu
bonus
s ee
to woke
worke
n rs, etc,
iz
&
‘2 2
Costing
able (Marginal)
4.12 M ing Versus Vari z
Liens atte skbrenpetontcott z
EPA: Aeupting 500
sanage| igi’
AO
Solution
ran peek et cee po
- 700
soak wet . tab 700
:
ne Costs
usin g
oy cpus Waniable aangfafactcturing
4 BAP BE 700
SRR {4h
Stat
ended et Dee, cea
Income, Statement
_ income 1200:
4,006
fat ihe yoar
"
an «Fixed mand administration 400°.
av £00
a rketing and
: Le: _ Variable ma r
(1000 units @
74 a 2H ministration
, ing and ad
ales
. 5) vt
Foe
_ 15,000 Fixed matket
Variable manufacturi (i100
@tr :
" Ting overhead
(1100 .u units 700 ‘ Required : ts hased on
come statemen
man
Fixed
ufacturing overhead ’
Cost. of - Hh 2206! (i) Prepare in
units@ &2) tsa (a) absorption
costing an d
yeat. /
Les : 58. C goods produced a cos tin g for each . Delt)
@ %.9)* le yn the answer ig. Com. Hons..
_ 300
(b) variab
;
stocks (200 units a Byte oes |
en ce
“Cost of losing reason s: for the differ
goods sold (i), Give
2 nat
us
Add: -Seli and-adm. overhead:
it Selling C7 9,000 “hg met oe
year I . “Year Ul
mae
—Variable : ith sk
(10O00: uHits.k - F050)
0 ; oe
golutio —- 1400
1096
deo
Total cost Basic data 1000
‘production un
its ee)
Profit (Sales - Total Cost) | 7 .
:
ee
g epst peryitie
eevee s C72 22
eae Sve Sates units Og
alued at manufactiurHtining pst at Pe unit
“Closing
5 4
tock is v, e, z 5 4
ni ng st ac k units - . 5 etinge fe
dee
: ‘Ope MEAG SRBA
SPUR
k ymits ie be 1
Closing st serge dad
ption casting):
fe aheeat
e Statement: (absor
ao aate
Setéthedlt WATS déiting ) NT EES SS
en
nig Fist fee? i568 -
ed
TES
SATE
GA 40 Torres
Sales (1000 units @ % 15}

manufacturing cost (RGRORIS QE CHE 79631


Variable manufacturi 100san runner
Farable Closing Stock (100 thee HE eee cd
8 ce eertne
7,708
700 Manufacturin
g cost - Var
iable
Co : _— Fixed
- goods produced
Add: : tee jable selling. : a nd adm; overhy : ~ -Rdggeitars
{1000-upits@ %-0,60)%
:
fs : m;: vetoes 5005)
Sf ™ .
:
cot. OT ad i ntnaeockA de”
sost of sadevariable sc tgaglapobu
T MP ce H TES

fection (Sales~
cont ribution s-Total
cost)
UGEf wine
Hing
! =0500} * coe ag ed ad s
Fixed overhead eG nb roe: e Oia ack* BAiEG GG
Less: : Less: Closing st IO ne
wdvelongor sfdeirs¥ evens
| agi 5 qistian ads
oa oY
—Manufacturing
—Selling and adm
amhigtehanasr
tochangat & :
savanna umetoeouie
Profit ee 298 jay —Variable
aasizinimgsBibbaswitha
noctestetni on iat
maments. Profit unde: 1 osting is € 5,100 and under marginal }
costing tatalicosts-:
ra
a & S06,
ng Tae hi The
difference } cost)
(Sales - Tota
of ) 2 zi 0 in p profi tisd
i et o ovet-valuatio n tl of cwI sing stock
alu in abs
d OF Tp Hi On cos! fin bPeSb5y Le.
Ate, Net Income
sonidos aidanay + fn} 2@1
4
Problem 4.2 pies: mogicads tah « dao metete gmeonk fat
an]a fd}
Inomatese emorom * Working Note = €-1400 + 14
00 units
= & 1,20.
tBot ia} : yeat inl 00 units
dalimy tabaMULEn TG
-Toblem
a
bootas Eees siofda
» int ef
diab .ols
ratlir
misigei“ d (334} Cost per unit = 7 12 00 + 10 = % 400.
“ZED. tinted & sells sells itsit products
at 1 per uni
unit. The c
Out actual Cost per unit
yeat in ll
. goo units @
= 4
= % 240.
;per rate 1i" First-InnFirst-each year uaby
. 1
k value in yeat
Oompa
costing sys
te . fix ed man tio n Closing stoc s @ 71.28
Nea! m. A new ad all oca = 200 unit
utingig ov overhe at et
UL:
dividin t fixed rhanafactd to-its
+
neat cost by the Costing stoc
k tin year IT
following eaaite ine data are rel ate years of 0 ration:
: uction co sts. The
0 | Ope
y
‘ ea Year If
Sales (units) in
: n (units) , 1200
P, troductio
1,409 1,006.
4.14
,
Maiandgement Accounting . rginal) costing
Versus variable (Ma
Absorption Costing
TTT entre me Statement (Variabie
ee a mooree
e
income
§
Costing)
ing}
Solution Statement (Absorption Cost z,
Income A
{A) Sales
Oo 1,00,006 |
Variab
ue le Ranufacturing cost . 71,500
. ;
* Opening stock Sales @ © 6.50) 46,400-
; c ory (11,000 units
leeost of G oods Available
i * 3
Variable Cost—fact
y (11 ,00 0 units @ 7? 1.50) 88,000
for Sale
: Fixed Cost-—factor ; 8.000
Closing stock - a
: 96,000
units @ 78)
Opening Stock (1,000
oe of Goods Seid
" Add: sale for - 16,000
(e)toarable marketing = Cast of Goods Available g6,000
market By
{z,000 units @ %
and adm. cost :
‘oe variable cast i 3 Less: Closing Stock 1,500
@ 1.50} 78,500
en wution: (4 - -BY = of overheads (1,006 units @
: ion
ixed cost— Manufacturing 1,5
Less: Qver-absorpt __ 40,000
ee : : 88,500
stration expenses
(0) Tota Administration = ::
7 Selling and admini 11,500
‘ fai Fixed ee
Cost : %
Total cost
-
Profit
oven COMME (C ~ 3)
1,109
: 1,100 {Variable Costing)
408 Income Statement zg
nat"R
: : 700
1,00,000
for differeuce in profit. rofi
Reason for difference in profit 71,500
is nal costing is the . of stock
diffe tence in t he valuation under absorp tio n cos ting and ts @ & 10}
an Sales (10,000 uni 0) 6,500
year I. . Profiti under t (11 ,00 0 units @ & 6.5
abserption
, costi Th Variable cos ts @ € 6.50) © 78,000
costing. This § isj bec, ai: :
Add: Gpening Stock
{1, 000 uni
13,9090
ilable for sale
a
stock is Cost. of Goods Ava
costing is tagher ae 45,000
m
inai l3 Mor a {2,060 units @ @ 6.50)
stoarg
ck ae Less: Closing Stock 45.000
than cl Osi
Ss n G stock. t of production
Variable Cos
- Variable Cost) 45,000
Contribution (Sales __ 25,000
+ Factory 10,000
Less: Fixed Cost stration 40,000
Your com pany A Selling and admini
80%. Opening irven sec peodacti nection capacit 0. + -

: “pening it rroduces
*
of finished goods on r500
os
unis and normal Capacity utilisation is Profit costing and variable
31-12-2011, in profit under absorption in the value
-

11,000 units while it. sold it 1,006’ units.D ting the year ending
e in Profit. Difference of stocks
Reason for differenc 11500 - 40,000. This is d ue to the
difference
10,000- =
uni its, © 8 per umit
at total cost ie.
nly '
is = 1,5 00 ies , % sto ck ¢ are val ued
costing tin g,
and adminiuisstrtraatition tems. In abs orp tion cos
cost only i6., % 6.5 0.
produ et at F 10ingper anit, overh
000. The coma; under the two sys yalued at vatiable
' j
ompany sells its in var iab le cos ting, stocks are
, whereas
Pre;pare [ ncome Stat ementsae “under Ab
for differete
. nice in profit, sorp jon Costing iq andani Variab
i le Costing, Explai Problem 4.4 duct x
j n th € Teasons a per unit of pro
"
ing is the standard cost dat
(3.Com. Hons. Delhi) The fol low
49
: 8
Selling price
Direct mat eri al 5
2
Direct. labour
y ove rhe ad 000 units pet year)
m al output of 36,
Variable fac tor
a pudgeted nor
ad & 5 (based on
Fixed factory overhe
rhead 7 6 -
Variable selling ove ye ar were & 1,2
0,000--- There
Fixed selling ove
rhead per
ts 0 f the product
and sold 1,500 units
duc ed 2,000 uni
com pany pro
During a month the g, and (ii) variable
was no openin g stock. (i) absorption costin
e an inc ome statement undet (C.S, Inter)
to prepar
You are required fit if any.
difference in pro
costing. Explain the
ting Wereus. Variable (Margina
l) Costing
Cos
._ppsonptionsn-
4.46.,

;
Solution fit os
“26,000
pifference in pro pa
orption’ costing Zz 3,500
ca Se ! Grofit as’ per abs g 83 500
~ ARABS SSEeRaAE Pants
Lage 8 Altirs
Profit as per mar
ginal costin 7
ck valuation
‘Difference erences in sto
-
enc e, in pro fit is due t 9 the diff
erence 2. This
differ
Sales (1500 units x % 40 Reason for Diff expl ai ned” below: £10,000
st em s as
Production costs ; under the two sy in abs orp tio n cos ting | t 7,500
Material (2,000 units « % 8) Stock valuation
in marginal costing "2,580
wow (2,000 units « % 5) Stock valuation
_ Difference
retractor
ath overhead {2,000 units « % 2} 43
capacity utilisation
per years Normal

sdetsbiitndacing
ee fon overhead (2,000 units « 2 5) * ood uni ts cos ts are
: Problem ‘4.5 t y of 2,90, 1 unit. The fix
ed
production capaci production costs are © 11 pe are < 2. 7¢,000
secs: Of goods produced 3 t xYZ Ltd. Rasa variab le d fi xe d “gelling Goats
Ae) phate 0% . St an da rd 3 per uni t “an ne , 2017, the
BoP Bea an reckoned 23-9
riable selling
|e %
costs ar d: on 30th Ju
year jus t ende
20)*
haa stock (500 units x % per yea l- Ya _ In the nt or y on 40th June
forne goods sold 3,60,000 The closing in ve
|
it selling price is uni ts. her than
un e # 35,000 hig
¢,0 00
: eT absorbed overhead* per yea. The and sales ‘aare
1,5
for the year wer
was 1, 60 ,000 units uc ti o n cos ts
production variable prod
ts. The actual
Add; seting expenses ~ ¥, atiable 2(1,500,1
+ units. Ae z 6.,agetid2 énc2 was 20,000 wai
(2-1-4 nths)~ the standard, g method.
( “420,030 + 12° mo
Fixed yeat:
.
“otal cast of sales Q Gateulate the
pro it for the
od , and {b y by variable costin (CA. ‘Inter, Ad
apted)
meth
by’ ‘a bs or pt ion coasting
Profit {Sales - Total cost) (a) the profits.
differencece in
“Working ‘Hotes:
(ispExplain | the
3 sissovs gheed Costin g)
1. Closin:tg stock is valu ed at production cost per unit ije,
ee“60 t {Ad sorption a
prod Solution -- Income Statemen 36t h Jun e an? nn
anding
2. Fixed:‘fa ator y ove hea
T dp er month (36, 900 x 2 5) + i 12 months - for the yeaT
Less: Fixed factory 9 vi érhead‘ absorbed
, (2, 000 units
i = ze ay? al8
i -30,00,000

“ oH. ae units @ 7 20)


gales (1,590,009
hoo
wT
447 ,60;000.
.

,000
45000; --°2-17,95
. _Income Statement (Variable €costing) ts :
Production: Cos
gnits @ Fee. ,
abn Hi amit
Re ers crt a sib i igi
Variable :(460,000 oe
3,20,000
sate i. 500° “Unites . Rath
B ih ani att? 020,62
ivadlo6 pried Add: Inerease. 2 ee 21, 15,000. mea
units-@” cas] 1,30;000..
: venatle ‘production’ ‘costs Fixed (460,000 a
Produced
arect mat Cost of Goods ts, @,: % ysyts. _ 22,45,000;
Direct meee
s gu gning st or e 19,000,: uni for als e
(00'S syns dd available
paory overhead {2,000 -units ea Cost of Goods
Tg tin is stab dans
noc brads r oats
Brabijeie gilt 41 ghiOOOj oi =ent
x 9,000 nits)
= tiable production cost : 30,000 e
stock (S
Less:. C losing stock| (500i units x % 15) - ST Gt é Less: Chosing
thee
Cast of Good
s satd: ‘
| neda U3 gam sneny
tiable selling overhead {1,500 ietion dr
: units «x @¢ 6) ; “7 8
dd : Ur id er” sh ea red" foie: -proak Sak gaa
Total Variable cost ‘a s”
. “fai gy 31,
31,500
st oF good
ne ohfgtat co 7
pet
Contributi on. {Sales an .
Total: varighle
Less: Fixed wdastsales— .~ Factory gost) mre ‘ep
eo ban “ baarhava api
apiban bsxid duction costs
:
@t .
s (150,000 units
Add: No n- D ro
m = aa
variable setting ot
overhead_ ee ee its
1eN r6gy Totp.086rav0
Selling eee ~~ tec
2

wi — overhead ae .
ot st
Fixed selling co
MOS. F mew a ot, oe so
afin fia 2S . otk orgs, ft
G adi ie , afin:
clin OO0,§
UG0,5 soubor gvnegaras
ensg ares adt dipkhO0
Tat
ante t OR8. Eo
ves
Total cos
re éost)
Profit (Sales - we
Daroreads
Sonar Gp
}igads q. (i)
#3 wary u Jremsjeds
(Sah J IE ai smeu7! ie% oyseray
DIS TSG : ATey. bons
byDONHIDS
vostSthoF
“UOT Hot}
as
ak
Wis Tide
ia
i ant sre tth ed zsnis! “a patess
- log Hib ei
4.18
nr oo
erent ccont ing "2g ps
gbsorption. Costing Versus Variable
(Marginal) Costing
4.19

1 gFeta
“Workin Notes: ‘OunEthig
-
. HE
for the two years under
. z Production overhead ar . ai 10.50. Prepare Income Statement
3,66,609 = 1,80,000 wait @ absorbed at a pre-determined ; Selling price in each year was.% (M. Com. Adapted)
rate based on normal capacity costing:
2. Opening stock j s=%2 pacity, fe, {i} Absorption costing, and (ii} Variable
units. 9 ie, + 1,5 . : .
unit,“kisie,10,000
% 13 1,50,000 units + 20,000 units Solution
per 2 414%
(Vatiable + fixedy- ~ 160,000 units, ft is valued ‘at Basic Data ane
Incom (Varia. ble Costing) Year 2017 Year 2018
for ee eutement
_
ve year ended 30th June, 2017 2,00.000 150,600 "
— Production: units 1,60,006 1,80,000
Sales (150,000 ounits @ % 29) ~ z Sales units 28.000 68.900
e 29) Opening inventory
aDt: production cost (1
Variabi 30,00,000 oy 68,000 38,000
Variable sellin 4 cost (1,50,(1,60, 000 units @ F 124% 3 5,000) EA 000 Closing inventory een
000 uni 17,95,000
UO units @ % 3) : :
4,50, 000 ; Income Statement (Absorption costing}
Add: * Open;
Upenin, . creek {10,006 units @ z 11)
“22.4 5.00 or
cost Year 2017 Year 2018
me
0,000 3
ods Available for sale
_1,1
rn
_—_— 60,0,000
_10,16,80
: 18,90,000
Less: Closing stock (£27 95.000_ x 4 .
79.55,000
Sales (@ % 10-50 pet unit) 0N
2,460,000
vocas
, 1,60,000 units * “2°00 units} - Direct materials (@ € 1.60 per unit produc
ed) 3,20,000
2,24,375 - 00,0 00
3,00,0 5 00
2,25,0
bom ; cost af goods sold i Re. 1.50 per i
unit)
——
21,30,625 fp Direct labour (@ 2,40,000 1,859,000
Atribution, (5 .
Less: Fixed cost (Sales ~ Variable cost of goods sold "he 375
Variable factory overhead (@ % 1.20
per unit}
6,06,000 £,50,000-
) Z 3 per unit)
—Production 69,375 : Fixed factory overhead (@ 19,95,000
3,60,000 14,60,000
Profit ~ Selling Production cost {@ ® 7.30 per unit) . 436,400
2,70,0 os 7.30 per unit} 2,04, 400
£0,000 6,390,000 - Add: Openning inventory (@ ¢ 16,64,400 15.91,400
ve ee renee
__ Reasons for Difference ; a 277,408
oR 4,96,406
nae as aS in Profit: Less: Closing inventory (@ % 7.30
per unit)
11,68,000 13,14,000
Profit er ‘ Sold
per absorption casting. 2,59,375 * - Cost of Goods. .
_ _ 150,000
Add: Op,B. stack under-vatued in variable costing (% 1,30 units @ ¥ 3)
Add: Under-absorbed fixed overhead {50,000 _. 11,68,000 14,64,000
-30,000 - 1,109,000) 20,000 . Total Manufacturing Cost,
hess: C1. stock : D70 a7e
costing (2
Profit as per cee valued in variable Selling and Adm. overhead
164,375 - 2,26,375) 19.375 80,000 90,000
ariable costing pet unit sold}
ve
Tear
; . —Variable (@ = 0.50
75,000 75,000
— . —Fixed 13,23,000 16.29,000
Problem 46 °° JT ae
Top class co 7 _ Total Cost 3,957,000 2,612,000
“488 company supplies you the follow!
owing standard cost per unit for one of it : Profit (Sales - Total cost)
-
in material 7
its products,
7 ’ Ieome Statement (Variable Costing)
wect labour
< ;*
pable factory overhead e me a
a
£1.20 .
Production a sen uted ; = 3.00 16,80,000 ” 18,90,000
Capacity is 2,0 - Sales (@ % 10.50 per unit)
Ber unit is Ra sdinieraiee matte selling and administrative
and sales hae ar fixed selling and materials ({@@ %€
ect materials
se Direct aucee} van
"Units produced in yoor one year~ 2018 are as follows 9. 7000 Re Heat. Production 1.60 p.u. proauce
3, 0, 0 2,408,000
2,25,000 4
- 3,00,000
Unizs .solg in the: Direct tabour (@ % 1.50 p.v.) . :
year 2017 2,00,000 (@ 7 1.20 p.u.) ae ; 2,448,000 1,860,000
Inventory. ! Variable factory overhead
8.60,000 6,45,000 &
1,60,600
Units prec, Dec. 2017 Production cost (@ & 4.30 p.u.) 1,20,400 2,92,400
\ 68,000 Add: Openning inventory (@ % 4.30 p.u.)
Units sotd in R year 2018 1,50,000 980,400 9,37,400
Year 2018 \\,
1,80,000 (Contd...)
, 8
ing
4.20.8 ActsBis g Versus Varia
le (Marginal) Cost
* Miffagenent ‘shedeptiom bastin

;
"Eegey TE Sidsiny: inventaty (@°F 4.30 prey
og Rie AO. (B)_ Marginal co
st
waaiGns “oft ‘of Goods Sold
UFRE GER
fas Oy “at

Contribution (A-B)
UES

"8, 88,000 ©
.
adm. overhead 80,000 0
2090,,000 s: 1,20,000
Total c tiable setling and ~——— Fixed “overhead 1,20,000 10.00
7,6,68,000 12 1,06,000 .
(Saible)
Cnt niribuOsttion(varia 8.64,00 00 _facto ry 4,00,000
8.33
es ~ Total variable cost 912,000 10,26, 10
—_—_
Fixe ry
) See 6,00,0 Sat _-administrati
on
2,12,C00
* overheadoe —Facto 6,j0,000 a |
as 2,12.000
___75,ve00oon
(Fi
Total fixed cost 2,00,000 |, ALOT»
Selling and 0 . (D}
220030
a t. fixed(9 cost
ain __ 6,75,000
eo
.75,000 profit (C - B)
0 223,56000 2
=220,00
rofi ed 98
(Co ntribution ~ Fix ey
— - eaacaan
2,37,000 351,000
pee. PaiPontint =o© = 9,238 unit
s
6,111 units
Comments ns
tetiarte TEASE
Year 2017—Closing stock is iviore ‘than opening, ‘ock (pro = < 102 pel unit
$ more than_sates), profit shown by = 100 + 2H,
terial “post js
s
absoration: He isting ,is more than that of marginal costi
sting. a "it “60 % + 5% = & 105sper: cupit.©
Notes? & 100 ert
rial cost ig ait
~ At 80% mate
Year 2018 ——Closing 2 196 per
stock i$ le ss than opening stock (sates areI more th an production}, profit shown 200 - 2 =
by mar ginal; costiri ig is more than. tha at of absorption costing. e “Ak 60 % sel ling. price is = 2 & 160 pel unit.
lh - 5%
ice is % 200
a .
:
: “pactory ov head's 40% = 12 per unit.
Sk+E Che" 1s currently workiin at 50% capacity and Broiprod uces
2a neon
1 wan At 50%
.
Fined ~.% 30 x, per unit.
Taw materi ost increasesoeby"y a 2% and selling price’ “falls wae fo WOT ing raw
mate’
Tal --“_wariable.
42) = = 18
= (19,000 units
ee over he ad s
“total fixed’ t will decrease,
ie.
At 50% Capacit
tity worki ind'the Sfoduct costs F 186 per unit. ES nd j “gold at i 20 a per
i is . factory. guethéad-pet ani
at B0 %: “f xd r decrease, he.
unit cos Lore: 180 is mad ‘up--as -follows: 984 uit will furthe in total wh
AY 8 0 “fixed overhead t will increase
gait t twill U
peraani not change -pu
= 100. vacae-oveheat
= 30° initioion
hae for
‘ Factory overheads cae ve F 30. (40% Fixed). EBtation sie
in which unit
3

Z 20.(50% fixed),:
Spgs phy
Administration ovethieads tae
sting system,
ve hi cl es . It uses2an “detual co April, are:
motor h and
emblesand. sells,
Prepare a } farginal Co st Stat ement showing the estimated profit of the.»
evbusinéss when: dtvis lating to Merc we
operated at. BOR b and 80%. Capacity.
ili s
Also C calculate.
i break: Cals] po oints ts” ak: thase-
eyLeven:
“ABC ‘Matets “ags oFon & month ly
basis- Data re March
Ap
° HARGIS these |
levels.
etealeul at ed
costs ar un nai
feet (BLomi-Hons., Delhi)
'
299 JetoT,
Solution
os oe Un acdauave
:
t
2 tyM5%arginaCoslt ——St—atemen
seind: Gitar nttoryle
Beynning |Inve
coe capapity.
oo
me Production
3 TA
kee ee - Ones {asnef Be
1, "000 ST capacity
es pa-
fa
ye leso.ts
i y,
teri po 2,000 wits) “ Variaplerrost data: nek unit produc
ed
“Per unit Total Per uni ee
aea Per unit pee gyiit sold
- vinta | —— ele ren re
Sales 268
iA) 20,00,000 196 23.52,
00d F199 49,000,
10010
0 2 send
Material , cost : les2é 08.
age
02,
teay
8f-80,000
26801 abel OSn p55ch ad
“ 00,000
¥ 9 S05 di4
3,0a0,000 0°
| siesta
scl “aatiaaeJo
.
arialienfactAtroryi, rm
wap Oe! -Regtlvans ss
mensai 3:ABC: ‘yptbeosndeda6°“
gi sm
(i) Rapp uaNRurEes® 1resting 144 gid agsu
00 op jE ci
1,80,0
t”
.

isveeel atta! ga
18
4

° “tbs th noi s2508Rh iy at atsaio Hb tot! Dethi c.A, Int


er;
iinssa0 een (a} (B.Com. Hons
differences betw
: DHF
{it} ‘Explain the
10
00,000 ut) 1,60,000
mw ;
: (Contd...)
4.23
4.22
able (Marginal) Casting
——_—————
Ff rption Costing Versus Vari
Sutin «= (tt! — ae 18,50,000 ~ 12,50,000) than vari able
ER
t by 2 6,01 0,000 (i.e.,
ihe

TR
h abso rpti on costing shows higher profi ing show s high er profi t than absorption "
Basic data In Marc . in April. marginal cost Difference in
:
costing because pra duction is more than gales use sates
€ 31,20,000 - 96,70,000) becain the two months.
are more than production. 4a
Productinn sc: i costing by % 4,50,000 (ie.,difference in inventory values -:
|
uetion units arch profit is exactly equal to °
Sa ~ :
na €s units 500 i Notes: costing inventory
Working cost while in absorption
opening inventory units 350 is valued at variable manufacturing
In variable costing inventory .
lesing inven tory units 0 gaiuation is done as follows: - :
of 30 units:
150 For April, Closing inventory
Income St
we, a atement {Variable costing) 3,00,000
cost (30 units @ = 10,000)
Variable manufacturing 1,508,008
units @ = 5000}
Fixed manufacturing cost (30
Ion
March
me 4,50,000
Sales
® —_—
(A) nes under :
Var:able manufacturing costs _84,00,000 2 . . pet unit is calculated as
ae
.24,80,000 Fixed manufacturing cost
Ada: Opening inv entory 50
.
ri 3
@ % 10,000
e
.
per unit
00.000
0,98,000 q S. 20,00,000_____z 5,900 per unit
Co '
st of goods avait 400 units of production
ClosiUlasing i Hable for sale
——
Less: 50700,000 ~1700-000 4
@ & 10,000 per unit 55,00,000
For March, inventory of 150 units:
ress.
15,00.00 E
Variable co : ofmventory
goods sold —12-00,000 1 z
Variaht st 35,00.000 3.00.00
able distributi 15,00,000
(8) Total Vatitbe wean “ost 16,50,000 *2,00,000 i i
manufacturing cost (150
units @ % 16,000)
15,60,00 Variable
@ € 4000) 6,00,000
ee
Fixed manufacturing cost (159 units
000
{C) Comtributy ton (4rast~ B) 45,50,000
3 71,00,000
.
Fixed cost: --4 “38.50.000 §7,60,000 j
aa

. ay anufacturing ~20.00,000 ___57,20,000


20,00.0 , % 20,00,000 = @ 4,000 per unit
OU, 00,000 cost per unit = 500 units of production
(2) Total fixed ¢ 0s arketing —_,00,0006,00,00 Fixed manufacturing
—stt.o00
oneNet facom e(C~ __26,00,009
;
ED) 12,50,000 —£5:00,000 Problem 4.9 —
The production costs
are a8
" 31,20,000 — for % 5.20 per wnit.
no Statement produces an item which it selts
an ___¢ome (Absorption Castin: } | A company . .
_ :
a: follows :
year © 2,25,000
(4)
M arch , Fixed production costs pet
Apri per unit = 1.25
1

Sales tr Variable production costs


.
.

‘ .
vavietle manufacturing cost 1.24,80,000 1,50,000 units pet
yeat
-
.

of the company was as follows


sone ig
Normal level of activity age
xed manufacturing cost the yeat,
os the Position Statement
40,00,000
At the beginning offs —
1

sooo ‘
00,
70-00 an 20,00,000
1
January Year i
Position Statement as
at
: z
Add: - cpeni pening
imventory
Cost af ook 3 availab
inve
os _ 60,00,000
z 27,500
~te.o00 -
*
Fone
sont Goods
. i le for sale g¢,000 Finished 75,000
Less: Closing
sold ro
:

goods sett
Cost oF* goods
09,000
81,00,000
Capital: . 22,500 Other Assets
(8) P41
00,000 Retained earnings 1,02,500
ution cost—Vari
Add: Distribut; 49,090,000
4,
76 ae 1,02,500 ae
a
—— 15,000 of fixed
of variable costs and
70,000
«3 nae: A Marketing cottFnad 10,50,0¢ i
0, G00 10,000 units (@ 12,500
‘otal ‘Coase *-
-6,00,000 15.60,000 *Finished goods represent
6,00,000 .
Net
meComme (4 - 3)
Income aoe es
“65,50,000 ! casts).
:
:
1 and II is as Follows.:
The information for the years
98,10,000 : Production
”48,50,000 --- “Sales
costing ‘rewards sal les rnd ,
. 26,710,000 Year , (units)
when sales aie Marginal walle absorption costing rewarda 1 Production, This means that .
whi (units)
;
or production, marginal
tore than Tot Costing produces higher otafit :{ 125,000
L§ mor
VIC e versa, when " production
e than tales, abso i n castin
Diio
a 160,000
now!s high (J r
G show i pr‘0fit.
o : I 80,000
a
1,00,000
ing
(Marginal) Cost
n Co st in g Ve rsus Varia ble
to

You are required to:prepast: 2.


aa 4.10
ilable, from the
recogds of M/s.
Income Satine Of Saat ahs
problem ate ava
owing cost da ta
2 ahe foll

Fi)on a stat 3 theemenass


ts i tke bs yo

ee ?
vari
Pf en iu m” :
Selling price per
unit
unit
Soluti Variable cost pet
Seg ed cost per unit
Soa Fix :
Basic gata : : er +
‘ . . dh: Normal output iads are 45 und Total units
ail fit] . for fout consecutive pet Period [V
Year J, ava ila ble iod LT -
Years... per
iti ona l dat a I] > 000
Production uni
ts Other add period 20,
1,25,00 Period I -
40,006
30,000
4,20,300
Sales writs
729,000 80,, 000 4,190,009 420,900 -
Va et eae -
4,20,000 +,10,000
Opening inventory units. ; ' tL ASE Opening stock 1,900,000 90,090
4,209,009
ning inventory uni Produchon 1,090,000"
~
629007 , Coatingng. and
Closing
. 35,000 58510007 Sales’ -00! 30,000 s, ot vier bat
inal
Marrggin
bothh Ma
fi nS a e per-unit diff er ent pe ri od go urrechiments.
Pre-determined ned fixed overhead rat ing profit Zor
e
a res
15,000
J+ Glowing stock em en t sh ow s. if an y and alsd giv (ICWA}
at ad
to prepare a st over he
absorption of
Hoos. ya
tp 96 om You are required hacs, showing under/over
= Fixed overhead 25,000, Absorption Co
sting m at :
.
2a ;
put In units 0, O0@units “1,5 ,
nal Costing)
vee

em en t (Margi nn a
oe
Solution : in co me St at a
: TH
Income Statement (Abs orption Cas
C istinng) ) ’ Me Periods =*
ption °
ye eiRl? Fo¥
oO
-

* Total
—_—oeoore
eo Vv cs t
Ye
=
oe
Hr ” cope
: Rouhae Gar ae
leant. ‘ 1 e0
~ i eta: es 2.32,00,000 z
Sales (4 uni
@-* A pe Gap aren yd
egg a ago0 |
(4)
:
(#,00,000
0 units Ayre
520-000"
UGo04 aafs Ot, 000
ost @
0 pp oe
25 F141 pu
Je15n6,25En
0 Lon aay 1,41,20,000
ixed cost @ F 1.5
Fixed 1,00,090 1,51,20,000
pO ae “gic j1 1-87.5 0 Op. stock
Add : Opening
Pening stock’
stock — Variabl
Variaht e cost: ype eters 1,20,000 een
: 43,750 t @z 36
Production cos isl 20,000
Fixed cost 62.5 Total
15,000 “Eee BoDOe

Less:. Closin: a stock — Vari al ble cost - eh ate


ck
sithp ume wey G08 F¢B vgh flys
sto
c&oap: sts soot i a {-} 43,756
=) TAG 18750" -- Less : Cl.
sold {B} — 48,000,000
Cost of Goods
FSS BARTS VEEN
: 00
but ion (A - 3)
Contri
27 5 ohn! Less: Fixed cos
t
Add + Under absorpti - ad @UU.25.5 7 2 1k 000
CST DRLAS pg
. an of fixed overhe HGU
SF
Cost of Goods oad Profit +:
(B) 85,0 F diag ca>7 Ean
TEESIG- THER
qe0 YO
gagoe
Profit (A - B) ont 1s9y tsq ating 000.4
. 'HONS00 v2!
eanet HE EIN
. FRO,000

te os
in: TU E SPAT dn om at an ad s we ak Sie IA ni ps
*"SN ot e. tinse tnfactory overhe 1G SS
.
Under absorbed
In absofp
wv tic
eh enn i acosat
: $ are abs orbed aS¢ 310
ann ana
fF 50 pet unit, ge m le of £15
3
ad is th
ele atement
A? actual amount
var verhead,
_ income Staie tt it (Variable Cost
ing)
wud boas
jatigg Sales (A)
&
2
Fear: dydinin
sinceiss hon
i est
g “ Gpistock @ 48 48 eet
@
Production cost

AOBE v2?
ne ble cost of- good:( prodtidedisy
‘Varia
Fa 00, 000.» AB "2AO40,000
a

0) ER
:|
Less : Cl. stock

Te age fi 50 . | SERSb260z2002 Basha; 1


s 1.25 Pu, = apsjic
) tore. at
sorbed overhe
Less : OV et ab
~ shat ee aE Sin sa140 290 is ‘ome
Con: tin th ~ B tieat
“Aad : Unger absorbee ovet
on . 3,95, 000 | sasoas
tes ed coe
reo.
(atin g) nt “Sse
Zasae 225 000
_178,000 70,006 :
“eu
a
— — Menten ct anagement Acc
Absorption Costing Versus Variable
(Marginal) Costing
:
Comments . oi
Unting:
Period | ~ T in marginal
nal costing. How would you treat variable cost and fixed costs
absorption castin
g. §. Define marginal cos tand margi (CS. Inter}
costing? e when the number of unitsso td exceeds the
exceeds variable costing incom
6. “Absorption casting income (B.Com. Hons., Delhi}
agree?
number of units produced”. Do you s production.” Comment.
whereas absorptio n costing reward
7. “Marginal costing rewards sales (Le WA. Enter)

ption costin g for exter nal reporting. Give reasues far your
ed in absor
8. Which costs are to be inventoris (B.Com. Hows., Delht)
4 answers. as regard s valtia tion & finished gaods
marginal cost and absorption cost
9. State the distinction between \C.5. infer)
KEY TERMS inventories. limita tions of ma:airal costing.
4 ibe the thasec terist ics and
costing. Descr
i| cests (vari
ariable and fix a 10. Explain the concept of marginal (Bcom.. Hors. Delhi)
dministratiye. expe ed
| admini } are treated as Prod (C.5. Inter; A.cam., Hedurat)
in its managerial uses.
11. Define marginal costing. Expla

Practical Questions

"
le
(a) Absorption costing. ardiai variab
prepare an income statement under

f
1. From the following information
z

Esa
o

me
a

2dn
5
=
oO
casting.

aa

ec
3 a
z

=s
: 5,000
1,560,000 Adm. overhead —Fixed
; Sales —Variable 42,000
Period fo:
50,000
1i nventoried.
: Sts~
These are
In

absorption voces which ate not included int j , Direct materials 20,000
; h cost of 20,006 Selling overhead —Fixed
‘osting at 4 ing it inclu products ig. these a Birect labour 15,000
[ fixed factory sty administration sellin 40,000 —Variable
~~ ——— 2g it a Includes a ory overhead,
1
*
adzistration celing
.
j " . Fa

distribution overhead ta Fixed factory overhead


5,600
[a . and distributio N overhead, a Variable factory overhead (Adapted)
| ace |
EXAMINATION Quzsrtons | . (ans. Profit% 13,000 under bath ca} and
(b}]

31st Bec. 2017.


is given for the year ending
0 : -

True or False ? bjective Type Questions 2. The following information


Opening stock ~ — 1000 units valued at % 70,000
1. Absorptj 10: € 50 per unit).
7 '
(including variable cost of
2. There an pening 18 2 total cost technique
3. Vari .
. er or oy
"
Variable cost —? 60 per unit
Anable costing is more wide ee of everhead in varia _ Fixed cost (Total) — F 1,20,600

| Production — 10,000 units


Sales — 8,000 units at = 96 pet unit,
i le costing.
(a) absorption costing, and (5) variab
: Prepare income statement under . (B.Com., Hons, Beth, Adapted)
costing,
00;
8. Variabl
€ Costing j
|
(Ans. Profit under absorption costing = 1,46,0
:
9: Selti ng is used main
inter d nal reporting. Profit under variable costing % 1,30,000)
c Ng and distribution costs iy arefor treat i
for the yeat ending 31st Dec, 2017:
ed as p riod
10 nn. ca: st undde.
er bo
bot
3. The following information is given
- In absorption costi semi-vari th vi ariable & costin ig
cos and d abs
a orp
OTPttion z
{Ans. 1. True; Costing,
2, False: 3. False, a costs must be segregated int
vari
into fixed andNd variable 10,00,000
§, F. False;
+4. True; 5. Fatse: 76. 7, Tre: ¢} t Sales (@ T 50 per unit)
tee ? 8. True: r 9 4. T dTue; 19, ements,
Faise] 2,90,000
Direct material 3, 10,000
] Dig: tin, g t lis te
Essay Type Questions Direct labour 1,206,000
h
Variable factory overhead
be Weele ;
2, What do mar g inal
costin : 2,40,000
0 mean by marginal costo!
yeu absorption costing, :
. usaf Ng.“Discuss its ; Fized factory overhead
3. Distina?: efelness and limitations (3.Com., Hons., Delhi) ead (fixed) oo 60,000
: Miguish between-marginal cost Selling and adininistration overh
B.C ., rata; Delhi: MeCom bangle
.
(B.€om 20,000
hat are thé-ch ay - terist
# charac oo:
ic of marg § and total costing,
“costin
™ . Ore
Selling and administration overh
ead (variable)
ng?
inal costing? (B.2.M,, Bangalore)
. (B.Com., Hysore)
. nal} £os ting .
.
4.28 sa Costin and sel js caicula Lors-
Feb.
Management Accoungi psorption (* pany which makes jan. 19,800
——— :
During the year 24,000 units were produced but only 20,000 units .
were sold. There was no opening stocy 7 9. Th
o data given DEOjow relates 10.2 °° 5,000
10,000109
5,000
40050
Prepare an income statement under— /
‘ in uni
(a} Absorption cesting, and : 50 400,900
(6) Variable costing sate cto in oats 2 1,00,000
eet
10
. . pri en cost per unit z 10
Explain the diffezence in: profit,we if any. =0
(Adapted) variable Pr cto overhead
Hon ox unit 59,000 5 0, ant on the
[Ans. Prefit—absorption costing % 1,20,000: variable costing @ 86,009}
Clasing stock valuation in absorption costing @ @ 40 per unit, variable costing
Fixed pro quetion overhea ate) : inal costits: Come wea, Inter}
@ % 30 per unit] Fixe Pernined 2a Pts (fixed)
4. Add Cello Co. manufacturing ball pens is working at 40% capacity sting, and (py marg ; ne 3,90,000-
producing 10,000 penis pet year. The rine and d eribution
cast elements for each ball pen are given as under ; ser (0) absorption co san? 299,900. t. 5 z 3,90, 000:
sell come statement Un von costiNg re jan. & 1.00.0
Material sca austen]
— %2
Labour ~~
prepare [ans ee nal casting mths ig the same vACE
6 results:
Overheads —F 10 (40% variable)
: mt for the two mor
Qverall P 30)
Each ball pen sells for % 40. The selling price falls by 3% if production
is at 50% capacity and by 5% if
worked at 90% capacity. The fall in selling prices is accompanied by
similar fall in material prices.
You are required to find out profit at 50% and 90% capacities using marginal
costing approach.
(Adapted)
. {Ans. Profit at 50% capacity % 57,500; 90% capacity
% 1,42,500}
5. Using the information given below prepare operating statements for
the months of June and duly using;
1) Variable costing and (ii) Absorption costing.
Suggest reasons why the two techniques disciose different amounts of
profit.
Selling price % per unit 50 Monthly costs :
Direct material cost 18 Fixed production overheads = 99,000
Direct labour cost 4 Fixed selling expenses— % 15,000
Variable production overheads 3 Fixed administration expenses € 25,000
Variable selling costs are 10% of sates revenue and normal
production capacity is 11,000 units per month.
Sales {units) Production’ funits}
June 20,000 : 12,000
duly 12,000 40,000
(B.Com., Osmania)
fAns. June : Profit—Variable costing"%61,000; Absorption
costing = 79,000;
July Profit—Variable costing = 1,01,000; Absorption costing
% 83,000)
(Hint ; There is over absorption of Axed production overhead in June of
F 9,000 in absorption costing.
In duly, there is under absorption of = 9,000)
6, 4 Ltd. produces a product which has the following costs :
Variable manufacturing cost —~ & 4 per unit, a
.
Fixed manufacturing cost — 2 2,00,000 per year
Normal capacity ~~. = 2,00,000 units
There are no work in progress inventories.
In year i, the company produced 2,00,000 units and
sald 90% at a price of €7 per unit. In the year II the
company produced 2,10,000 units and kad sold 2,715,000 units at
the same price,
Prepare income Statement for two years based on

(a) Absorption costing. ~
{b) Marginal costing.
. (M.Com., Delhi)
[Ans. (a) Absorption costing profit yearI € 3,60,000, I
(5)
% 4,40,000:
Marginal costing profit year I % 3,40,00
0, I 7 4,45,000).
oununy
Management Acc
a

t and profit.
in changes in cos
lume of pro duction results re pro fit s of changes
brief, variations in
vo on fu tu
of the effects
7 influences cost. in ined CVP analysis 8. “the study x.”
def mi
a | CIMA London has t. sales price,
qu antity and in budgeting and profit
in fixed cost,
yar iab le cos to manageme nt
lys is ig ex tr emely us eful fit :
g of CVP ana on the net pro
An understandin the following
It exp lai ns the impact of
planning.
selling prices.
(a} Changes in
volume of sales,

ANALYSIS
(b} Changes in
in variable cost,
(c) Changes one of these
fixed cost- of change in any
(d} Changes in probable effect
helps in deter mining the
* Infact, CVP analysis g factors.
factors on the remainin
ALYSIS
BRE AK-EVEN AN p. It 15 interpret
ed
to study the CYP relat jonshi
que
ly used techni
lysis 15 @ wide ining
na Break-even ana se- ned with determ
chev t, 4 pasts
aren
:
Margin of safety:ve
i Cal ions in b
ow as wel l ag broad sen k- ev en ana lysis i 5 concer a nd no less.
cha rt: Key fac tor ; : ane eof incidence; Bre in na rr
ngrrow sense,
br ea
e there is no pro
fit
sum + Profit vo
lu me ak en In its d sal es wh er
of production an e.
g.
Narrow meanin
rmulae: ; Use 1s cof Break-eyen ana
ys i, ‘ ie., that level veru
break-even point, cos t is equal te total sales ye
of fo lys bable
10 determine pro
Mary ms and solu
& Proble
questions, tions: 3 Key terms; Examinati
ion poi nt tot al pr ea k- ev en an aly: sig is used the am ou nt oT
At this broad sense, termine
When used in also helps to de

warn
me an in g. on /s al es . it
Broad producti
given jevel of
Introduction

profit/loss at any earn a desired amount of profit


.
Thy ta _

SHE ARE
Se Volume profit
d CVP Analysis
:analysis (CVP analysis) volume of sal es
S Our study of cy j Xtension of the
Principles : ofof variab in g Br ea k- ev en Analysis an ons -
as begins hon vari le aderly eight ass umopti
fevious chapter. whey :
left our stu ve variable costing i Assumptions v the following
CVP . i isi follows f we . has ed on

ome
erived and applie directly ron Tomt variable costin in th analysis is ents.
d j aria le costing . T h i $ means that principle The break-eve n. int o fixed and
vari able compon s im direct propor
tion
able cost va rie
1 are appticable h. ere, s d
Every companyer mu 5 . 1. All costs can
be se par ate
an d total va ri
ee profits nS constant
planning, M
to stay in busi pet unit remal
9, Variable cost
profits will be € Varlous plans to in: c
to the volume
0f pr od uc ti on .
nt.
ed cos t -rem ains consta me changes. does not
-3, Total fix
doe s not ch ange as yolu the sales mix
lin g pri ce Der uni t of mu lt ip le products, ri ou s 9 roducts
4, Sel uc t or in the ca se
d, the sale of va
y one pr od s are be in g sol
.5, There is onl several pro duct °
er words, when on
change. In oth ne d pr o portion. lume of producti
ot r words, vo
pr ed et er mi
CosT-T-VOLUME-PROFIT ANALYSIS ays be in so me
an d sal es. In he
- wilh alw o n
tween producti
c VP a T al y SS hronisation be
6. There is sync
1s a Pp Ow e rf al
factor taal of profit planning.
hasic of sales.
$ of busi Ress operations: equals volume
It Studies
ations:
the Inter. relationship of thr change-
(a } Cost of oroduction . ee
vi ty per worker does not
7, Pr od uc ti price level.
in the general
{&) Vol ame of production be no © hange
/sal es, and g. There will
Cost E quation riable) cost
(c) Profit, and Marginal n sa le s an d the marginal ( ya
Co nt ri bu ti on
fference betw ee 5 contribution
is
lie r, co nt ri bu tion is the di (C,,) oF 9 sg margin. Thu
As stated ear marg in
contribution
so known as
of sales. It is al rm ul a :
fo' Llowing fo
calculated by the le cost (Cc =5

Contribution = Sales. + Variab
Cost-Valume- Profit Anal Management Accounting
ysis en et
5.4

= Sales - Variable cost


Also,
i
Contributi
(ii} Contribution
on = Fixed cost + Profit
= Fs Pp } i
. (e 4,000
= 11,000 - 7,000 = =
or
it
ribution per unit) + Prof
. Contribution /
= Fixed -- : Lass (Cc a Fw L)
= {B.E. Point ( units) = Cont
cost
oo
Fro In :
this, the following margina (iii} Contribution 000
‘Owing marginal cost equation is developed : —— - 2 (2.0 00 x27 ) + 3,000 = % 17,
S-V-FeP | V RATIO)
founTfd anycut thre e ieof the@ abov
PROFIT-VOLUME RATIO (P/ ress the
abovee fou r fact
four factors
ors | in th @ equation ratio (C/S ratio), exp
i are known, . the four as contribution/s: ales
ourthth one can he easi easj
ly The profit/volume ratio, better known
PsS-V-F relation of contritobsalues.ti||on = © _ g-V |
Contribution
SS
Pe(C-F P/V rati
holicaily,
P/7 12° = Ses
is
Fe (-p
| Symbtiootn,icwealy,
have
Example: By transposi
mee hP p/¥ ratio
{i} C=5*
Sales = % 12
«
Vatiable 0 . C
a cost= = 7 : 000
sus. ixed cast = % 4,000 (i) S* (py ratio
Sales = @ 10,000
Example:
8,000
C=S-7 Variable cost = 2
2000 2
C= 12,000e ~
Pleo 7,01
7,000 = % 5,000 C _ S-¥5 _ 10,000-8,000 10
Then P/V ratio = S = = “19,000 16,006
FP = 5,000 - 4,000 = z 1,006
a. Profit is ¥ 1.000
= 2. 400 = =o 20%
f sales fi gure isis notnot give centage, P/V ratio 10
oj Ne
n us
b uf contribu
i tion is given then Salesal When expresse d in per d from any giver: leve
l of
s can be determined as fol ows
: con tri but ion can be qui ckly calculate can be cal culated
When P/V ratio is give
n, the contributic n
When i f onl y sale s and P/¥ ratio we ye given,
e,
gales, In the above exampl
fixed cost (0) 5S = 5,000 + 7,000 = = 12,000
F} ts notee
given but_ profit is give as under:
gi n, then : C= S « P/V ratio
x 20% = = 2,000
C =10,000 sales
wy tibution to change in
t he change in cont
Fs5 (000 - =
hen variable cost (VY) is com put ed by com par ing se in con tribut ion
not sven. , th then: P/V ratio may 4 iso be will me an increa
; 4000
‘in sales.) Any in crease in profit duc tion. Thu s :
VeS-¢ (ox change in profi t to
cha nge
at all levels of pro
ain Oo nstant
are assumed t o Tem ee
Vs 12,000 - 5,000 = % 7,000 because fixed costs rT]
1

The conce . .
but ion Cha nge in profit i
pt of contriributio
a Change in con tri
:
man agement decision-making nis extremely helpful in the study of of b break-even a . Change in sales —_ |
nalysis and io = ~ change in sale
s
Hustration 5.4 | P/V rat ere

Calculateel ¢ Ontribut Example : Net Profit
ibt ion
in each of th Sales
WLigq
iiig inde
i penden u Year g
ns
a Fixed cost * 8,000, profi
t F 5 500 z
_ “pense ato 1,000
ais1) ha
Vari n cost ©¥ 7,000 20,000
, sales 2. 11,00 2gti 1,600
| 22,000
sotutin 1), Gontr
| ibutio n per unit
it z <7, profit F ing3,000 2012
: , B.E, . Point be,2 ago units. - 1,0000
1,60 5-4 a9
in profit
P/V ratio = Change 20,000
22,000
() Contribution = Ea
Fixed cost + Profi
rofit ~ “Change in sales
= 8,000 + 5,600 = 13,600 \
.
= £0 x 100 = 30%
'
2,000
Management Accounting
Cost-Volume-Profit Analysis
5.6
ratio. thereby
which have larger P/V
Mustration 5.2 s mix, Le, sell ing more of those products
(c) Changing the sate
P/V ratio.
Calculate /¥ ratioratio iin each of the following independent
P/¥
situati ions. improving the overall
“ able cast % 60, Contribution F 40
EVEN ANALYSIS
he sass t 26, variable cost @ 15 METHODS OF BREAK-
:
be lowing two methods
ne ‘ vane vost to sales 84%
ana lys is may pe perforiied by the fol
000; Sales % 25,00; Fi Break-even
{v) va I Sales % 50,000, Totat vost & 40.000 e800 (a) Algebraic calculations
ar II Sales % 60,000, Tota! cost = ;,000 (b) Graphic presentation
n Analysis)
Solution lculations in Break-eve
“ Algebraic Method (Ca of output or sales at
which total vast
Contribution ‘0 poin t. The brea k-e wen point is Le ugluma This is the min imum point of
_ Break-eve n .
G) P/V ratio. COntnbution 40
sale s. It is a poin t of no profit and no loss it beg ins.
is exactly equal to prof
Sales arable cost + Contributi
Vari i on . ~60+40 ~ 190 7 4°% 4 and after this point
total cost is recovere
production at which n poin t is:
i) P/V ratio « Contabutien _ S-¥ ula to caiculat e break-eve i ot
20-15 5 The fundamental form | a ee fouifa

el
a
+i) } 0/4 P/V ratio=ratios 8p
100 — Variable cost to sales ratio “

: Break-even point (n
units) = Fontrib ution per unit
S-

= 100 ~ 86% = 16%, cates _ :


I3 Total fixed cost
F+P
(iv) P/V ratio= Contribution —— 5,000+8,000 13,000
Se pomt (in Rupees}
Contribution
a
~ 25,000 25 | Break-even

5,000
i .
get* —_15,000-10,000
in profi
(v) P/¥ ratio. Chat | {in Rupees)
= P/V ratio
60,000 - 50,000 — 70,000
= 50%
sales Lor Break-even point a
*Profit is the ‘itee ne \ ee
@ between sab
es and total cost. Example
Uses of P/¥ ratio .
following data is given:
= & 12,000
)P/¥ : tatio
e
ic i 3$ G one
he af theee Qst important
i " ratios Total fixed cost
= % 12 per unit
ati to watch fal
1 business,
i It is an indicator of ‘the rate
Selling price
a wh n profit 3s being earned. A high P/V ratio indicates high profitability
= % 49 per unit
and a low Tatio

such as sales
: aréas
Variable cost
1 é SLOMers
. ie product t lines, .a ™ ethods
thi of
be com a .
wun » Classes
fhe help
of cu. i
p roduction
c
! s : etc . may ; also: Thus :
of profit-volume Taio. The P/V ratio is alse in making the =5-V
Contribution
used '
per2
%3 9 unit
212-
D Calculation of break-even point 3 5
C = 73% 100 = 25%
(b Caleulation of profit ata given level of sales. = —*100
; P/V Tatio
(c} Calculation of the volumne of sales requir
. . .
d . Fixed cos t 12,000 = 4,000 units.
(a) Caleulation of profit when margin fate iver. * gen Prof
sie . . = __ fred per ae
units) 3
{e) Calculation f Break-even point (in Contribution unit
is reduced, of the volume of sales re
J tT ed tom aintain the present level of profit A if sellin ig p nce

x Sales
Improvement
gy
Break-even point (in Contribution
in P/V Ratio
|
As
‘ P/V
: Tatio
z 48,000
‘08 in dicates
i th e
ae x12
Tate of P rofitabi
‘ li t ¥ t any improv
i ement in this tatio without
uy fixe d Increase
ore would result in ora profits. As a note of caution, erroneous conclusteas may
drawn by here refe pe
ence to P/¥ ratio. Th
T a tefore, this ratio
m
should n o t be us 2 d ‘in TN . ola
} tion, aos
the functi
P/V ratio 0 1sis
variab le cost. i .
Unction
gap between sat Q S and ari able
vari cost. andThis can be St
of sales es by ean be improved by widening the Also,
Total Fixed cost © 12,000
= — = © 48,000
" Increasing the selling price
Break-even point (in gj
fo
P/V rati’
2tg o 25%
(b} Redhicing the variable cost
|
ounting
Management Accan
Cost-Valume-Profit Analysis .. ae

5B
Verification
sing figures
Calculation of Mis
point may be ¢ verifiverifie
Break-even paj

Total cost as follows : Example + = z 30,000


Fixed cost + Variable cost Break-even point
Total cast Given:
_ = = 1,500
© 12,000 + 4,06 Profit
= % 6,000
j

O units x ¢ 9)
= F 48,000 Fixed cost
The sales value and total cost of variable cost ?
at break--even point
i are exactly equal What is the amount
ADDITIONAL CALCULATIONS -
fit
Solution Fixed cost + pro
In additio nm oto the : . Contribution = 1,9 00 = z 7,500
point, the above formul %
I +
.. ne onacal cul atievuontieof eee aiso j z 6,000
inakin 9 certain additi val mula can
be used in
. These are :
. . Fixed cost
te
1, faleulati ton of profit at at didifferent sal ev en poi nt = Contribution
Break-
: manctien of sales for desired
pr _ veumes 6,000 sai
- Finding missing figures 0 x Sales
_ = 30,000 = 750
Example
. z 3 75 590
30 ‘ 900 =
The Follow n
data is g i oY
: 00 x
given : = 7,5
“ing Sales .
Fixed cost 7,500 A
(total)
- : 22,000 unit es” soo Contribution = yyap9 © 100
Selling price _ z “ per
.
P/V ratio= “Sol
Variable cost per unit
~
cost = ino - P/¥ ratio
Variable % (of sales)
10¢ - 20% = 80
Calculat lon of Pr 1 Fit at dif fe rent Sales Volumes
Variable cost =
What will be e ¢ the profiti when 500 « BG % = € 30,000
sales ate {a) F 60,0 sales} = 2 37,
= 1,00,000 ? le cas t (8 0% of = Zz 24,000
Variab = & 49,000 x 80%
break-even sales
P/V ratio = £ Yariabie cost at Fix ed cost
§
= 3 “ sales = ¢ 30,000 -
t at Break-even = 2 24,000.
(a) When sales Also, variable cos = = 30,000 - §,0
00
= % 60,000
ntribution
Contributi = Sales x P/Y ratio .
mai =% 0,000 x 25% Example : ¥ 10 per unit
= 4,000 units @
= F 15,000
= Contribution - Fixed cost Sales
= = 1,500 units
= F 15,000 ~ Break-even point
= % 3,000
| (5) When sales Fixed cost and (b) profit ?
= ¥ 100,000 © 12000 = € 3.008
am ou nt of (a) variable cost:
! Contributia nos What is the
Prati 00,
1,00,000 « 25% = % 25,000
= % 25,000 - & 12,000 = Solution Fixed cost
0 t
Caleutati
ulation of Sales for Desired Profit © 73,000, {in units) = Contribution per uni
$ Break-even point
Continu 1 na¥ the same figures GD what will . % 3,000
= 6 a b @ the am
of fa}
000; (b) ) = 4 15,000 2 the @ ount of sales if it is desied to ear na profit per unit
.
1,500 > Gontribution
+ red
.
Sales
DdecosstP/Vi
for desired profit = [e profit —_& 3,000 * = 2
ts
units
ratio unit = 7,500 uni
Contribution pe!
Contribution
+ @ 6,000
(a) - <22,000 = 272,000
= Selling price -
per unit
25% {a) Variable cost E19- 2270 8
© e = % g,000
= 4000 units x
+ @ 15,000
() - ©12,000 \ Contribution at
sales of 4,000 uni
ts
= Contribution
- Fixed cost
= F 1,08,000, 5,003.
25% (b) Profit = = 8,000 % 3,G00 = Zz
-
Cost-Volume-Profit Anal Management Accounting
ysis

a
5.18
Example ;
Calculate (a) Break-ever point by 10%
Civen: if selling price is reduced
(b} New break-even point by 10%
Fixed cost if variable cost increases
{c) New break-even point (B.Com. Madurai Kamaraj)
Profit earned ~ po int if fixed cost increase
s by 10%
,+ : d cao (d) New break-even
Break-even sales z «o-oo
What is the actual sales ? Solution
_ 80,000
S-V _ 2,00,000 - 1,26,000 x 100 = 40%
Solution
, . ry 00,000 - ~* 5 60,000
= >
P/V tatio
£ Intribution
ibuti at break-even point is equai
to fixed cost 30,000
F
= % 75,000
Thus, = P/V ratio = 40%
P/Y ratie £ = Sy = 20% | {a} Break-even point
y
S 40,000 ,000
2,00,000 = 10% = ~ 1,80
by 10%, new sales =
Fixed cast + Profit (6) When selling price in reduced
Actual Sales 1,80,000 - 1,20,600
_ 60,000 =i
PIF ratio 3
tt

:
= "7, 80,000 1,80,000
8,000+2,000 10,000 New P/V ratio
39%, * Jo, = 50,009 F __ 30,000
Example ;
= Pjvratio 1/3 = % 90,000
New Break-even point
Selling price ~ 2 150 per unit by 10%, new variable
cost
Variable cast (c) When variable cost increases
-% a per unit
TL = 4,20,000 + 10% = F 4,32,000
Whar Fixed cost ~ % 6,00,000 (total)
68,000 - 3%
1s the break-even point? _ 200 ,32,000
2,00,,000- 1,32 ,000_= 990,009 100 = 34%
————
oo
- os New P/V ratio 2,00,000
What is the
selling price 00 anits?
Thus, B per unit if oréak-even point is 12.9
' nits? 30, 600 (Apprex).
"8 a = % 88,235
New Break-even point
Fixed cost = % 33,000
cost = 30,000 + 10%
s by 10%, new fixed
B eak-even point =
(d}Sf fixed cost increase
* * *
Contribution Per unit
fected at 40%
P/V ratio remains unaf
33,000
~ 10,000 units. . 0% = © 82,500.
t69.
When 150-90uni
break-even pomt is 12,0 00 =
gs
units, contribu New Break-even point
tion is calculated as und
nder :
12,000 = —20 0,000_ Tllustration 5.4 brought
Contribution unit if B.E. Point is to be
g par tic ula rs, find o ut the selling price per
& 6,00,000_ * From the followin
Contrikution =
12,000 units = 50 down to 9,000 units :
== 75
C= §-F Variable cost-per.unit
= € 2,70 ,000
50 = S$ - 99 Fixed expenses (B.Cam., Calicut)
= z 160
S= 50 + 90 Selling price per unit
S= 2 140 Solution
Thus, setlin § pricice e j
is % 140 when break-even poin
t is 12,0 000 units. d cost -
Nlustration 5,3 Break-even point = ContFixe
ribution per unit
The followinwin
g g info
i: rmation ton isi
~
a 2,70,000 .~
e
given : - 9,000 units = 7
Sales Contribution per unit
o = €2
Variable cost = §& to aooe 2,70,000
. ge o. = % 30.
Contribution per unit = “Da
t

Fixed cost_ |
='. % 30,000 ‘! . 9,000
‘ :

ieee mnecmmne IKE


ounting
Management Acc

5.12
Cost-Volume-Profit Analysis 5.11 tage of sales.
ms ar as a percen
exp res sed 1 n absolute money ter
safety may be
AL present the ¢ dbw flion 1s z 25 le, 1¢c0 -7 5). In order to bring B.E. Point t at 9,000
*
units, point. Margin of
|
Thus, Break-ever point ~
| M/S = Actual sales a
tcontribution tin; shouldi in
b b roug
at hi Et 0 x .: Th js
i e, n
means that selling
i price should be increased oy z 5. 7 Th WS, el
eel
en ae : nets ae
Itustration 5.5 Example + - _ Campary X
Company ¥

You are given the following data : ee 60,006


zg 420,006
Fixed expenses % 4,000 , 40,000
Actual sales z 40,00C
Break-even point € 10,000 20,000
Less: Break-even
point < 80,000 :
Calculate 20,000 —
Margin of safety 39,600 = 56.000 * 100
(i) P/V ratio = 730,000 iat
a5 9 % of sales i
(ti) Profit when sales are @ 20,000 Margin of safety
2 2 335%
;
(itt ) Ne Ww b a
eak - even point if seliin ts] price 1 reduc ed by y 20% (B.Com. r Dethi} = 665%
tae
15
margin of safety
Selution eee n gne ss of a buses. When uat ion .
indicates s0u es. I such a sit
margin of gafety a serious fall in sal safety
At break-even
‘ s
K-ever soin
saint, contribution is equal io fixed
i cost thus when sales
le are % 10 00. The size of the can still ak e profits a fter ress! on. A large margin of
s the bus ine ss in fim es of dep a mat ter
large, it mean af survi val of sales may be
ss 5 tan ds better chance in of saf ety is low. any loss
the bus ine When ma rg
400 1902 4,000 low fixed costs.
CCc usually indicates
() P/V ratio
5 io,cq9 * 100 = 40% of a serious Concer
n.
it. This 18 show
n velow -
is dir ect ly related to prof .
(if) When sales are 7 20,000, contribution will be— Margin of safety ee * Pre fit /vo lum e ratio
of safety
ofit = Margin
20,000 x 40% = 8,00C "pr
4
| pa M/S P/V ratio
P
Profit = Contdbution - Fixed cost
m ‘ = 8,000 - 4,000 = a
(tit) mn een es pet when. selling price is sedued , oe M/S = pj¥ ratio

on
|
Thus a
I = 20,000 - 20% = € 16,000 18 40%, then
and P/V ratio
Variable cost = % 12,000 | if profit is 40%
Contribetion = 16,000 - 12,000 = % 4,000
M/S IS = 30 % _ 254 °
70%
C 4,000
New P/V ratio 16,000
= 25%
es are given te * Actual sal
es
5 When actual sal ratio * pi¥ rat
Pro fit = M/5
Fo, 4,000 M/S is known,
then
New Break-even point
P/V ration 25% = 7 16,000.
When profit is not known but
wes

ratio
Cash Break-even point
pan/s x P/F
. = 10%
: whi ; . P= 25% * 40%
When break-ev i is calcul costs
-even point
point i culated only with those ose fiked
fi
Profit
such a break-even n point point. This a ich are payable in cash,
known as cash break-even and per unit
other non-cash fixed vaste, are excluded from the fixed costs in computing cash that depreciation
break-even point = ¢ontribufion to improve it :
Its formula js M/S in units may be taken
fcllowing steps
ee

Also the (2)


isf act ory , fixed cost,
of safety is not
eat ce, (c) Reduce
When margin (B) In cr ease the selling pri od uc ts wi th latger P/V
Cash break even point = __fash fixed costs‘nit_ lume of sales. the sales of pr
crea se the vo bY in cr ea si ng
Contribution per (a) In e gales mix
cost, (e) Improv
Reduce variable se the break-ev
en point and
: ratio. wa ys to red uc e P/V ratic, rai
MARGIN OF SAFETY (M/S) is. al
\. price. reduction
. “The effect of @ saf ety .
: : of
: Margin: of safe ty
may be defi
rence etween actual shorten the margin
words, itit j efined as the bydifference
point: : In other words, which actual volume of as ne sales theat break-even
xceeds break-even
is the amount
Cost-Valume-Profit Management Accounting
Analysis
5.44

This is illustrated below


Solution
Example : z
Fixed cost % 4 0,006, Variable So per unit. Act .
Su . 40,006,000
reduced 3 f-om
: 5 to F‘ 60
cost F
. Actual sales = 75,000. SUPPOSE Price js Sales
MUU. 4,00,000
sales) -
Less: Margin of safety (40% of
” Break-even point ~6,00,000
| Before A -
price reduction price deci fi Contributian = Sales x P/V ratio
008 « 50% ai 3,00,000
Contribution at B.E. Point = 6,00, there is no profit at this
eling price per unit £9 of contri bution is fixed cost since
ariable cost per unic ("4 A+ break-even point, the entir e amount
cul pred cost (F} i paint.
ontribution
: (§ - ¥ ' . 10,000: Thus, fixed cost = % 3,00,000.
Os25 cost
10 Profit = Contribution - Fixed
00 = 10,00,000 * 50%
P/V ratio aie Contribution on sales of & 10,00,
| P /V ratio = % 5,00,000
7
a 3 wa G7
000 - 3,00, 000 = z 2,06,000
Profit = §,00,
reak-even e d as follows :
point
= These caiculations can be verifi
1/3 —
1 76 Profit % = M/S x P/V
sales .
Actual sales = 40% x 50% = 20% of
” 35,000 « 20% = 7 2,00,000.
M/S (Actual sal
sales ~ B.E. Point) Profit = 10,00,000
15,
75,000 ~ 30008
40,000
: 75,000
bs - 650.004
a
= 45,000 = 15.000 MWustration 5.8
pointThus
has bygone
y reducn i ng price
i from % 75 to o€ & 60, situations :
60, P/v
P/v rati
ratioa has
has redu ed hom W/a ten
reducced eac h of the following independent
istration es P from % 30,000 to % 60,000 and M/sF has ¢ ome down Calculate break-even point in
un oe
from F 45,00
ee Oe ratio 50%
0 toreek
Z 15,00
even
0 (i) Fixed cost & 10,000; P/V
% 15,000; Cont ribu tion % 3 per unit.
(ii) Fixed cost
Calculate may gini of safety in
: each of the follo
Lowi
wing ind
i i i - (iii) Margin of safety - 20%
un nak fee Point 40%, Actual sales F 40 100 ble cost to sales ratio = 60%
spendent Suations (iv) Fixed cost % 9,000, Varia
: B reak-even
Sales point in of safety 30%
(iil) - 40,00075%units, Break-even point
sbi i
25,000 units {v) Actual sales % 50,000, Marg 70% of sales.
in of safety 20%, Variable cost is
te id ratio 40%, Profit % 35,000 (vi} Profit = 30,000, Marg
Actual sales = 4,00,000.
ortribution per unit % 20, Profit z 15.000 (vii) Margin of safety % 70,000, units
Margin of safety 2,500
Solution (vit?) Actual sales 10,000 units,
('}Ty Margi
Margin of safety = Actual sales - BE Solution
Point
(i) Maret = 7% 40,000 --40% = % 24 F 10,090
Fixed cost ee 20,000
) Margin of safety = Actual sates
- BE point he
(i). BE Point ee e
P/V tatio BO
sit) Mana _ = 40,000 units - 25,000 units =
) Margin of saféty = 100 ~ BE Point = 200 ‘ % 15,000
ie an ails d cost___, =3 = 8,000 units
(ii) BE Point = ContrFixe
(1) Marcin of safety - lL _ 35030 ibuti on per unit
80% af sales
P/V ratio * “Te, 7 © 87,500 safety in % = 100 ~ 20 =
40%
(iif) BE Point = 100 - Margin of
- 60% = 40%
(¥} Margin of safety = Profit _ © 15,000 (iv) P/V ratio = 100
p/¥ ratic a F20 = 750 units . Fixed cost = 9,000 .
2 ,5 0‘
=F q 22,50
a BE E Point = P/Vratio” eo 240%
Mustration 5,7
of safety
in
The profi t/ volu
y me tatia
ati
(v) BE Point = Actual sales - Marg
of Esc OTS Ltd
td. ig @ 35,00 0
= % 50,000 - 10% =
i 50%
Tequized to work out ; t he net . profit Margini
safety ¥ isisi 40%
of s
40%. , You are
and the
and t he break-even point
int ifi sales volume is ¥ 10,00,000
Management Accoumting
Cost-Volume-Profit Analysis 5.15 5.16

P/V ratio 40%


(vi} P/V ratio = 100 - 70% = 30% ; Margin of safety 50%,
(iii) Fixed cost % 2,00,000 o to gales 35°
70,000, Variable cost rati
: P
&
30,0 00 {iv) Margin of safety = t7 10,000.
Margin of safety = P/fof¥ ratio = © 7,00,000 P/V ratio 20%, Fixe cas
d
{v} Actual gales % 80,000;
30%

Margin of safety = 1,00,006


Actual sales = = = % 5,00,000 * Solution
Margin of safety% - 20% Fixed cost + Profit
(i) Contribution=
BE Point = Actual sales - Margin of safety = 37,000 + Profit
84,000
% 47,000
= §,00,000 - 1,00,000 = = Profit = 84,000 - 37,000 =
Verification “eee = 100 - 80% = 20%
(ii) P/V ratio
ratis
Profit = Actual sales x M/s ratio x P/V ratic Profit = Sales » M/S ratio * P/¥
* 20% = % 6,009
- = 30,000 = 500,000 x 20% x 30% = 1,090,000 x 30%
(vii) BE Point = Actuat sales - Margin of safety . 2,00,000 _ z ¢,00,000
(ili) BE point = Fixed cost
on = © 4,900,000 - 70,000 = % 3,30,000 P/¥ ratio 40%
tit) BE Point = Actual sales - Margin of safety in of sajety
Marg
eaoomneniin
Margin of safety e keiaiak-
eea
= 10,000 units - 2,500 units = 7,500 units o = —Ketual sales t + Margin of safety
Margin of safety rati BE Poin

x
~ Suppose Margin of safety *

sesiidizperes ernest ates al


Cateul
Ot ibution
contribution ini each of the following independent situations :
argin of safety % 15,000; Fixed cost € 25,000; P/V ratio 364 . xe =__*__..
5,00,000 + x
; Also calculate profit in this case. "
.
(i BE Point & 40,000; P/V ratio 40%; Profit € 10,000
ee x

(itt) Margin of safety 40%; Profit % 30,000 _ 50% = 5 9q,000 + x

Rees ce
io a ratio 40%; Profit ¥ 50,000; BE Point % 1,00,000
x = 0.50 (600,000 + x)
salts, v) Margin of safet y 4,000 units;
i ution % 3 per
contributi per unit;
t it: Fixed
Fi cost = 30;000. « 2,50,000 + 0.5x

. x ~ 0.5x © 2,50,000
{i) Contribution=.{M/s in & x P/V ratio} + fixed cast
x = & 5,00,000
= % §,00,000
= (2 15,000 « 30%) + 25,000 = % 29,500 Thus margin of safety
safety « P/V ratio
Profit = Margin of

oi”
at
Profit = M/S x P/V ratio = F 15,000 « 30% = % 4,500
= §,00,000 « 40% = T 2,00,000
a. rofit =~ Fs % 29,500 - 25,000 = 2 4,500
Verification /
ontribution= (BE Point » P/¥ ratio) + Profit
+ Margin of safety
= (40,000 « 40%) + 10,000 = F 26,000 Actual sales = BE Point
00 = z 19,00,000
= §,00,000 + 5,00,0
(iil) Contributions = ———P2ft___ 30,000
© 40% Profit = Actual sales ~
M/S tatio « p/¥ ratio
| Margin of safetyin” © 75,000 0,000
«x 40% = z 2,90
= 10,00,000 « 50%
{!v) Contribution= (BE Point « P/V ratio}+ Profit
(iv) B/V ratio = 100 - 35% = 65%
0) Contribut = (F 1,00,000 x 40%) + % 50,000 = % 90,000 B/¥ ratio
Profit = Margin of safety *
. on= (Margini of safety ini units
ion= i x Contribution per unit} + Fixed x 65% = % 45,500
= 70,000
cost
= Contribution - Fixed
= (4000 units
= i
x € 3} + € 30,000 = % 42,006 “
(v} Profit
Illustration 5.10 ~ ual sale s = P/V rati o) ~ Fixed cost
= (Act
- 10,000 = % 6,000
Calcul
r ~ Profitit inin each
each of the following independent cases = (80,000 « 20%)

ixed cost % 37,000; Contribution % 84,000 \


(it) Variable cost 80% a of sales, Marg i TL of safety yatio 30% ir Sales zg eu, O00 *
1,00
miter
Management Accounting
Cost-Volunte-Profit Analysis

Hustration 5.11 Fixed cost 4,80,000


% §,06,000
BE Point = Py ratio ™ “son
(v)
Short Questions
y ‘
@ Margin of safety 60 io, Fixed cast 2 2,10,000, Variable cost tatio to sales 70%
. % §,00,000
.
or % 10,00,000
sales Capacity sales = i
actual
amount of
termine the
city = 10,00,000 x 72% = % 7,20,000
Sales at 72% of capa
( it) BE Point
i z 40,000,
fo Fixed cost zg 15,006. W hat is the P/V ratio :
d cost
= (Sales « P/V ratio) - Fixe
= & 36,000
4 Fixed COsy z /
42,000, )
Actual sales z 48,000 ' M arg a of safet ¥ z 8,000.
0 Profit
wt . 8
-(7,20,000 » 30%) - 1,80,000
atributi | ygg 2 70,000
i —— x 100 = 16%
(vi). P/V ratio
(tv) Find out th
the BE i
Point 4
when P/ V ratio 40%, alg
Malgii of safety 30% ais Py 0fit zg i2 000,

. Contribution 7,00,000
(¥) Break-even point
poi occurs at 60% of capaci and ifP/V ratioratio isi 30%. What is the Sales
amount of capacity
.
i sales when Eixed cost is sales80.000
pacity
amaun
the amount of profit 80,000, Also find
» of capacity sales. fi = _ = ¥ 5,00,000
Margin of safety = sya io

{vi} What is the a MOUr


unt of ma gin
T of safet ¥ when rofitit i z i i 2 ? 0.000
Pp 1S 50,000 contribution
and sales g 7,0¢,000. Also 1 :
= Contribution - Profit
Fixed cast 26,000
- 50,000 =z
(vii) Catculate the a . P profit eak - even z 5 0,000 = % 70,000
mount of actual sates when * 20 ' 000 ' b vi point
1 fi -
Fixed cost 20,000
| Se 2 & 2,00,000
E Poi _ Fixed cost 10%
(viti) Variable COST 0 . and
g 1 of
Margin
ma safet ¥ 1s
1 4 40%, W hat is the amount 0 fix e d BE Point P / V ratio
. t 25 80% sales

vil) P/¥ ratio


Fixed cost 25,000 _ . 50%
Solution BE Point 50,000
wil /
25,000,0 29,000 2 F 90,000
(1) P/V ratio == 100 - 70% z= 30% fo Fixed cost + Profit _ 25 + 00
A’
=F /¥ ratio 50%
BE Point = Tzed cost _ 210,000 Sales
P/V ratio 30% ® 7,00,000
= 100 - 30% = 20% |
(viti) P/V ratio ,000
,000 - 40% = = 1,20
Sales - SE Point int 7,00,000 7,00,000 Break even sales = % 2,00
100-M/s% ~ 100-60% ~ 40% ~ © #7-50,000 Fixed cost
ratio
= P/ Vial
BE Point

$45, de ioe ceca td


Verification
RE Point = Actual sales - M/S x P/V ratio
Fixed cost =. Break-even Point
000
= 27,50,000 - 60% = % 7,00,000 = % 1,20,000 « 20% = & 24,
OR KEY FACTOR
(i) BE Point == Fixed cost, LIMITING
ieve
P/V Ratio
is not always easy to ach
ss is to earn max imu m profit. However,it an und ertaki ng
The objective of a busine of factors. For exampl e,
= PW ratios Tocheest _~ 18,000
BE Point 40,009 * 10¢ = 37.50% this objective bec aus e prof it ear nin g is affected by a variety
led labour and produc tio n capa city , but may be
rs on han d, amp le skil imu m qua nti tie s
may have sufficient orde ty of material it needs
for the manufacture of
max
and prevents
(i)
ut BE Point
i = Actual sales - Margin of safety quanti of output
unable to obtain all the or which limits the size
= 48,000 - 8,000 = % 40;000
ch cou ld be sold . Thu s, material is the fact mes a bus ine ss is not able to sell all
whi eti
P/Y ratio Fixed cost 12,000 ng fro m max imi sin g its profit. Similarly, som
an undertaki is the limiting factor.
BEPoint 7 40,009 * 100 = 30% In such a case, sales undertaking,
that it can produce. tor in the activities of an
may thus be defined as the fac Examples of
A limiting or key factor will limit the volume of autp
ul.
{iv) Profit = Actctual i
sales x Mou
Margin of safety -% x P/V ratio
in time or over a period
g ratio x P/Vratio _ 40%te x
r poi nt
oA
ctl cle: OE Profit”
30% = 1,00,000
which at a particula
limiting factors are : me
(it} Materials
{i} Sales machine hours
BE Point
Poi = Actual Sales ~ Margin of safety l (iv) Production capacity or
=/1,00,000 - 30% = % 70,000
(iii) Labour.of particular skil
_Y) Financial resources.
Management Accounting
Cost-Volume-Profit Analysis 5.19 5.20

sa e is| to indi
imiting g f factor techniqu . ANGLE OF INCIDENCE
The purpose of the limiti break-even point
. ail cases where alternatives are oie. indicate the most profitable course of action
in such and total cost line al the
e is form ed by the intersection of sales line g earn ed once the break-even
o : This angl at which profits are bein
Contribution per uniti of key
aioe is reached wh key factfa or—When a key factor is angle sho ws the rate prof its. Therefore,
position {see Fig. 5.1), This is the rata of earning
‘ most profitable r the angle. the greater
product A which unit of key factor is maximum, , Foe in stance, if a choice point has been reac hed. The wide
e an angle as poss ible .
between reducing yields a contribution
a of & 20whichper unit, product B ation oF & 15 per
lies yields a contribution unit and product 8 will be to have as larg .
which the aim of management when sales are expanding
profitable. e is of part icul ar imp ort e in boom periads
anc
denc e with a high
The angle of incidenc e of inci
:
If, however, , product A takes 3 k q., of material (which is a limiti therefore, a large angl
with margin of safety,
5 kg.. t .
the respective contributions per kq. of mts ould he’ factor) and product 2 takes Taking in conjunction our abl e posi tion .
g., et s and extremely fav
margin of safety indicate
Product4 = % 15 +3kg = %5
BOINT
Product B = % 20 +5 kg. = % 4 COST INDIFFERENCE
profit of the
ge l of out put where
the total cost or the
th methods
Product A, which which gives
gi
e greater contribution in terms of per unit of timiting fact
. actor will be refe rs t o that leve alte rnat ive
more profitable, Cost indifference point level may be calculate
d where two or more fixed cost
are equa l. Suc h a e invo lves high er
two alternatives the use of one machin
es are considered and r fixed cost and higher
Illustration 5.12 of production oF machin whil e the othe r machine involves towe sation
and lower variable cost per unit help in a cost minimi
s
The following data at is given : cul ati on of poin t of cost indifference l of out put o
The cal le for a given leve
variable cost per amit. rnat ive which is more profitab e prof itable
rire sterile Product A Product B es the alte per unit is mor
exercise and identifi d cost and a hig her variable cost with a tugher
a lower fixe a, 4 machin e
sales. A machine with
ow the poin t of cost indifference and vice vers s are mar e that
ye nou @ 73 per hour
when actual sales are bel actu al sale
z P ‘s
cost per writ ig more profitable when
niable overhead variable as follows +
" @ & 4 er heu : fixed cost and a lower a for calculation is
fference. The Formul
Selling price ° : 0 10 the point of cost indi
Standard time oie —- ; Difference in fixed cost
on — ion per unit
State whi
which product yau would recommend to manufacture cea _wh mu nt (in units) = ‘pifference in contribut
Cost indifference poi
Labour time is the key factor _ oo Difference in fixed cost
setution d} Sales value iis t he key factor. (in 7) = Difference inP /Viaio
Cost indifference point

| Product A P Wustration 5.13 vides the following data -


e X, and X, and pro
choose between machin

east
Selling price (5) GMR Co. Lid. has to x,
oo , %
10,000
10,000

seg
Direct Material {uni ts} 24,000
7 a Gutput per annum 30,000
Direct labour level =
Variable overhead
%
Profit at the above 3C,000 16,000
&
Variable cost (¥)
3 Fixed cost per annum
3 e
Contribution (5 - F Compute :
7 : two machines.
{i} B.E. Point of the are equally profitabl
e
(2) Contribution
) per labour hour E62
62 = 2rt hrs, ~ B75sae
++ 3 hrs out put whe re the two machines the product.
(ii) Level of ls of out put of
table for different leve
(>)ayc Contribution
I per rupee of sales value~ ~ {itl} The machine sui
.= z 92 +; 100 =i; 754 #
Ten 110 | ,
“nian = 62 paise « 68 paise Solution cost + Profit.
Contribution = Fixed % 60,000
Machine A = 30,000 + 30,000 =
+ 24,000 = F 40,000
(a) ‘Product
labo A is recommended:
; ‘amet when lab ime
abour time isi the f | tion per Machine B = 16,000 6
+ 10,000 units = =
unit - A = = 60,000
Nessie contribu C or contribution per

at Dm at at pt
imore
fs ththe Akey than that of product Z 000 unit s = B4

oy BES atei
sales vale etis + 10,
. (0) When
rupee of le factor, , Product B isj recom ' j B= F 40,000 ~
= C
Break-even point = FO 00 units
: per unit.is
ate quantity
isthan the that
key factor, prod: net B isi more profitable because
: its : co ntribution
mt) A = % 30,000 = < 6 = 5,0
highér of pro Prot “ B= % 16,000
4,000 units”
= ¥ 4 =
. Cost-Volume-Profit Analysis Management Accounting
5.22

Cit} Difference in FC (iv) The margin of safety.


Cost indifference point = p rofit is being made.
indicating the rate at which
Difference in C fv) The angle of incidence, level s of sales . ( This can be shown only on a specia
lty
nt of cont ribu tion at various
30,006 - 16,006 14,000 (vi) The amou
en chart:)
= ? = 7,000 units designed ‘coxttribution break-ev
6-4
t
fae
- At 7,000
‘ units, . b both the machin es willi produce th Construction of Break-even Char
chart arte as follows :
(tit) aching B will be more profitable betw truction of a break-even
een break- Point
"poi nt ard
and poli
pointt ofof cost indifference The principal steps in th e cons line which is drawn and spaced
-€, between 4,000 units and 7,000 units The X-axis is a horizontal base
“ i, Select a scale on X-axis. re of the follo wing factors:
sent any one o7 Mo
into equal distances to repre
Ais more prof ié able When
ne sates
i on e more t an 7,000
' u Its

(i) Vetume of output (units)


Break-even Chart GRAPHIC PRESENTAT ION OF BREAK-EVEN ANALYSIS (ii) Volume of output (in rupee
value)
(iii) Volume of sales (units )
value)
Break-eyen ch art isi a;< QTa. pon}
hi c presentation
i of bre pa ix- even analysis,
tiv} Volume of gales (in rupee
th @ f, i This
ity (in perce ntage)
(v) Production capac
Fac t th a t tk @ chart takes it
eme left of the chart which
point t a at which the q mM
s is a vertical line at the extr
total cost line a Ti d
t i e Sales li line : inter
point, . A break-even chart not onlyonl shows the break- : sect ig the break-e
s. The Y-axi value.
2, Select scale on Y-axi
ven
at various levels but also sho WS profiti and ioss show cost and sales in rupee
ev ls of activity " even point On this Y-axis, it is usua l to
. is spaced into equal distances. xis, starting from an appro priat e point
280 p- This is drawn parallel to K-a
3. Draw the fixed cost line. wi tl be plotted as is shown 1 Fig.
5.2.
fixed cost is % 30,000, it
260 on Y-axis. For example, when
240 7
220 +
eo 200
a
o
» 180
S Angle of incidWence

Gost and sales (® '000)


a feo F

a
Cc

8
oo
>
o
rc

8
oD
‘ .
G ' ' Variable cost
z

&
‘ ‘
a '
o

ao
a)

cost
i| Margin: of safety: ' |

oO
Nh

Fixed
\ t

1 Fixed cost
t t

Qo
Break even sales}

=
20 Aciual sales | X-axis
i |
10 «20 «30 40 (50 60 70 80 90 100

a
Lot op Ng Jt {
'
Loy. 4
2 4 6 8 10 12 414 16 18 20 Steps 1, 2 and 3
22 2 Fig. 5.2. Break-even Chart
Units ef Production or Sales
in ‘000 ee chart by super-imposing it
variable cost is depicted in t he
thus «tea
4, Draw the total cost line. The
Fig. 5.4, Break-even Chart
co st line is drawn starting
from the point on the Y-axis which
byeak-even chart portrays the on the fixed cost line. Thus a total 50,000 (fixed cost being &
following informati ion ; ple, when total variable cost is
(i)vy BreaBreak-even Poipoint
nt:-—" represents fixed cost. For exam cost. point of Y-axis, to the = 80,00
0
= the point at whi i i is draw n fro m % 30,000, the fixed:
an we Brant/loss at different leve 30,000), a total cost tine w in Fig. 5.3.
ls of outa Profit the Y- axis. This is shown belo
el tionship between . variable cost is * made cost point on the right side of at the left {the intersect ion of X-
. fixed cost and nd tota
total! cost. line starts from the 0 poin t
“5. Draw the sales line. This
Management Accounting
Cost-Volume-Profit Analysis
5.23 5.24
¥-axi
100 f Example
is supplied >
The following data = 49,000
9 Fixed cost = 60,000
80 " Variable cost = 1,40,000
~ Sales 1,40,000 units
_
Zales/production
8 ro \ chart.
y BOE Draw a break-even
<3 oF units
~ Solution 000 and 70,000
% preak-even chart is at & 70,
2 sot point in Fig. 9.5
: The break-even ations :
the following calcul
This is verified by
2
s
= ggt a cost x Sales
Fixed tid
3 = poiint Cone
x tbution
Break--even
& 30 7
Fixed cast -5-¥ = z 80,000
Contribution 1,40,000 - z 60,000
= ¢
207 to .
; 49,000 0 = = 70,000.
= 3,900 * 140,00
10+ Break-even point
re Y-axis
0 pop j 4,40,000
| t t 1 T . 1:
80 90 100.
q
10
2 30 4 80 60 70
Fig. 5.3. Break-even Chart Steps 4 120,000
j
axis and ¥-axi s, where there ere is no production ‘
othe r at a =nil 1,00 ) and{Fixe
cost,000 exte the vepoi of maximum
cost to 230
d nds
or any ), s valu e. F r assu min
Z 50,000 the salesales li urthe g sales 000, variable cost

oe
:

5
a
‘3°
0,000 poipoint on the

_
:
. ine willtedbe indraFiwn fro m Q point at the left t 0 the-% 1,06
tight Y-axis. - This this isj illu stra
ig. 5.4 .
on
The sales lineine ini tersects the total cost line at at b break-even
output point representi
ating % 60,000 sales and 35® spoof gg :
Y-axis = 60,000 3
100 me )
: 8 | 4
!
90
2 40,000 : :
(= ‘000 )
at
a

sot —- — — — — '
X-axis
| 3 awe 2>
8 S8
saies

Break-
6 236 *
e8 e
82e8
ee
8
rs

3 b3
o
even point :
40 |
|
and

| (units)
Volume ot sales
Cost
wo
o

| even Chart
| Fig. 5.5- Break-
20
| ferent
Chart tribution at dif
| Contribution Preak-even and sh ow s the amount of con ri mp os ed on
. form of break-ev
en cha rt cost is supe
+—++—__}—_-+_
|
to do a This is an alterna te is dr aw n first and fixed but ion.
1) 30440 50 G0& 70 80 0 100X-a
xis le cos t s con tri
| such charts variab iable cost and sal
es line rep res ent
levels of opitput. In space between! total var mple.
line. The above exa
ig. 5.4. Break-even Chart Steps 5. the variable cost the figures 9 f the
in Fig. 5.6 with
This is now shown
Managements avers

Cost- Vokrme-Profit Analysis


5. 25
5,26
;

YanixIS 4,40,000
1,40,000
=

i
rheads
1,20,000- ing and dist. ove
420.000 Variable adm., sell

g
5
1,06,000 5
B
féak even point 400,000
= = 5
o
6

2» oo = 30,000
G
z _
@ 80,0005
e
ov

= 70,0 0 --- ro
“ 8
es a
5 $0,000
.
Fixed cost 0
60.0007 -i
5
8 ereeee : :
:
: o
40,0004 ZT
40,0004 7 =
&
~258 area -
- 4
20,000 ad
2
20,0005 Variable cost Fixed factory overhe re

d }
:
1,20,000 1.40.00
O° t
2 °
8°e8
i {
o
3
X-axis
70.900 40,000 60,000 80,000 4,00,000
8
3 ° a 3 S
S a68 28 {units}
S 2
$ Yolume of sales
8 8 § & Data
, . Volume of sales (units) oe Break-even © nact
with Assumed
Fig. 5.7 Analytical
Fig. 5.6, Contribution Break-even Chart

Analytical B reak- even {lustration 5.14


0

Chart 8 = 5,000
Thi 8 type of break-even chart can be con Fixed cost Sales volume
fixed and varivariable
structed to show greater details by breaking € 20 per unit :
i Selling price of the following
down
:
fixed facte it costs into sub-classi n cha rt and show the eff ect
le cost
overhead and fixed administration, selling and distribution overhead. Similarly, Draw a break-eve {b) 10% decrease in var iab
variable < casts mM a’ ¥ b e classi
5
rea se in fixed cost es vol ume .
:
£& Ried I nto direct materials, direct
(a) 10% dec increase in sal
variable admini strative,
labour,
. may b
variable factor overheads ¢
(d) 10%
eads. Even profit area
=
se‘ti . price
oa: IVE,
indicati ing appropriation of
8é.11N g and
profit te distr
pets ibuti
buti on overh
ay be subdi vided A (c) 10% increase in selling
ax, inter ‘est and
divi es, etc
dend payments, reserv
with on
chart wi
This type of bh
vee teak-even
will appear as shown
in Fig. 5.7
:| soluti its. This
decrease in fixed
cost
lt in 10% increase in prof
to #590 and would resu
EFFECT OF CHA NGE
IN
THE PROFIT F ACTORS ,|
| {a) Effect of 10%
A 10% decrea se in fixed cost wou ld amo unt

The break- fect the following £


chart can also sho w the -ef
ect of change in any of is shown in Fig. 5.8
affect profit sven 9 factors, which = 9g
(a) Ch a Calculations selling price Z5,000
int _ Fired cost g “310
0
cost Break = Contribution
(8) Chance n fixed | teak-even polh
able cost s
{c} Change ia vari = % 10,000 or 500. unit
= £5,000 - 10% = © 4,50*0
ce
(4) Change in waleey pri
olume. me |
T of-a busines: i New Fixed cost
ile profit 450 units
varihlé eect an be inc teased j there is (a) decrease
in fi
when j
int 2 x 20 = % 9,000 at
1

asz inih variable cost, and/or -(c) increase


decrease -
or m (b) ¢ 4500
s yolu
saleand/
i
xed incost,
iincrease % 10
The fist three, i lé
c } & se
will in pric
sell the effe
have ing ey
ct and
of (d)
{d@} New Break-even POI!
pro (b) and ¢} facto
ie,ng (a),
i €€,easi
( rs is lowering the break-eve
n point2, ed cost
and thusus incr
and calerlations :
fit.it. Th The separate effect of each of these
~ shown in the following charts Profit = Sales -
Variable cast - Fix
TUMBLE
ee

Cost-Volume-Profit Analysis

7 20,000 - 10,000 ~ 5,000 ~ % 5,000


New profit = % 20,000 ~ 10,000 - 4,500 = % 5,590 20,000

A
16,000
Ss
20,0004

Cost and sales (2)


e€ New Lotai
&
12,000 cost line
Additional profit e
_ 16.090 “ New 6.E.
€ re ra
a point
o 8,000
5E B.E. paint “
? \ a New total cost tine
2
oc
12,0004 a
on
Ja
z <—New B.E. point 4,000
ra
~ a“
a 8.0004 we
T 1,00C
Oo
a
Ta
0 ggg «= 400 808 a0
i Zé.
)
4,000 New fixed cost tine Volume of output (units
varaliable cost.
in vati
In
wini g of 10% decrease
-even chart sho
Fig. 5.9 Break-
3 >
qT i I T I
200 400 600 goo 1,000
Volume of output (units)

Fig. 5.8 Break-even chart showing the effect of 10% decrease in fixed cost

(8) Effect of 10% decrease in variable cost


20,000
A 10% decrease in variable cost would amount to 7 1,000 at the present level of output and it would
increase profit by 20%. This is depicted on Fig. 5.9.
Calculations 16,000

Cost and Sales (@)


Fixed cost
Break-even point = <—.—— x Selling price
por Contribution
12,000
New variable cost = 10 - 10% = 7 9 per unit
“BE point
New contribution F 20 - 9 =F it
3000
New Break-even point = ae 20 = % 9,091 (Approx.)

New profit = % 20,000 - % 9,000 ~ T 5,000 = F 6,000 4,000

(c) Effect of 10% increase in selling price


500 g00 1,000
"A 10% increase in selling price would add % 2,000 to sales revenue and: would increase
i the current profit 400
200
by 40%. This is shown on Fig, 5, 10 ils)
Calculations Yolumé of output (Ln
price
10% iincrease 10in selling
New sales figures = 20,000 + 10% = % 22,000 . . wing the effect of
k-e ven chart sho
eve
Fig. 5.10 Break-
New contribution per unit = 22 - 10 = % 12 ‘ ~
Cost-Volume-Profit Analysis
fixed
cost and profit. The
s. The ver tic al axts shows fixed profit is shown
on vertical axi d vertical line and
2, Select a scale the sal es line on the left han
bel ow
costs are market nd vertical line. These
e on the right ha profit.
Ne w break-even point = on x 22 = = 9,167
given fix ed cost and
' ( Approx.)
above the sales lin are plotted for the

fen wane
. Poi nts at bre ak-even point.
and pro fit es line
_ = 417 units fA 3. Plot fixed cost gon al lin e # hic h crosses the sal
ted by a dia
New profit o
(6) Effect z 22,000 ~ 10,000 - 5,000 = 2 Noo points are connec
. ~
10% increase in sales volum , 00
Example: Fixed cost % 5,0 20)
A 10% iner ease ini sales volum e would i
increase profit by € 1000 (i.e,Be at th @ tate of contributi On of = 20 ,0 00 (1 000 units @ =
Sales & 00 0 un its @ % 10)
P ie runit or 100 uni it }. I ficrease sales vi c tlume WwW. will ve ak-evi t
10 ,0 60 (1 8,000
Variable cost ©
ase in sale i have no effect on the break. even TL p POL
i + {Se e F; ig.

20,0005 : 4 8.000
5,900
5,000 &
6,000

seasiatilth iniate
4.900 5

-
:
16000 Current profil a
4,000
New profit
2,000
2,000
j 12.000. BE paint

fa
o
gleilyn
feo ajaofrso
&
9 8,000 4 7 =x 2.003-] 185
et
: ! o
| |
.
a
| !
Oo
oa
Po4 a
4,000-
Current sales | |! New w sal
sales a
WA 3,000

catitti ph
it-volume Chart
a { T T a Fig. 5-12. Prof
20 10 400 ;
600 800 1 ood 1 100
-
Volume of output (Units) Calculations 20,000 ~ 10,000
g-V
200 — x 100 = 50%
5 = 20,000
Fig, q. 3. 168
5.1 eak-e-even chart
4. showit1 q the effect of 10%Yo increase in in sales volume Profit-Volume ratio =
«= © 5-000 x 49 g00
; Fixed cost = '
Calculations preak = io
tatio
P//VV sat = 0%
yeak-even point
= 2 5,000
New wa
sales fi gure = 1100 units 5,000 - 10,000
Ve= 20,000 -
en uni x % 20 =
Pro fit s = 5 - F-
ten vat cost= 1100 units x F 10 : +1000
n Analysis mmarised below
-
= 22000 - 11000 - 5000 - ‘coo,
Uses of Break-eve n analysis are su
s of bre ak- eve
ant use
PROFIT-VOLUME CHART Some of the ymport ak-even point.
1. It hel p in det ermining the bre give the desired
profit.
The! profit-v“volume ch 7 ng the sel lin g price which will ot or ! eturn on capita
l
profit gra. ph h portrays the profit
tativei presentati anc i 2, Tt help in determ
ini
volume to ean 4 desired pr
the bres at different levels of sales
is an alte art of es
ermining the sal
and
the data
can be constructed from thename ofbasic which ina pie preak-even chart. Such a chart
illustrated
facts from
3. It help in det
-even chart can be d awn, els of output.
c onstruction:
: employed. rev enue at different lev
of Profit-Volume Chart ing, the costs and
4, It help in deter min t prefit able sales mix.
ining the mos
5, It help in determ of each product line.
Th f a

ative B rofitability
e ollowing steps
ferent matkets,
should b A take M1 to co nstruct
u a profit-vo
fi lume chart.
. det erm ini ng com par differentiation in dif
Select aa scaleseal on herizontal axi
1, Selec axis in the profit-v e giaph graph represents
a olum part
6. It helpin
of change in selling price of of price
salest. This horizontal line ‘own ee salesee line, divides the graph into 7. It studies the effect and foreign market. :
‘ two s.
ket
eg., home mar
Cost-Volume-Profit Analysis
5.31 5.32
not
volume changes is alsa
: * 1
sell ing pric eg rem aining unchanged as affe cts dem and .
$ 185 rTaing change in prices
4, The assumption rega
@ impact of increase or decrease in ed and varlabie costs or
not remain fixed and
prafits.
9. It . studies th e effect on profits : ing pric es do sell ing pric e which
sell in
with low fixed cost and prons and break-even points of high proportion of variabt & costs true. In practice, on Ay by effecting a
reduction
put can be sold
.
Any increase in out
on
10. it rompares line, remain
would effect the sales
the profitability
ot th at product mix will
i of various firms.
. wens n management decision-making, ¢. g., in make or buy decisions only one pro duc t is being produced man ufa ctu red is not
, discontinuance of 5. The assumption
that of various pro ducts
ct line, acceptance of special job etc. not fou nd in practice. The sales
unchanged is also
dé. Tt hetp P ini determining
‘ c ined proportion. not always so. Sales
may
quirements at different level s of operation with the help of ca sh always int predeterm
. .

break-even charts, 9 casi re and sala s are syn chronised. This is by effe ctin g
6. It is assumed that
production match production only
Limjtati n of may be cap able of increase to
imitations of Break-aven Analysis fall short of productio
a reduction in sellin
g prices. l employed which may
eu It 1
Dreak-even ana is i : ign ore 5 the consideration of capita
- Alth e Ql le tool of management, the analysis completel y
witich this technique ss is an invatuab
froat Cat some limitations 7, The break-even of profit analysis.
h the a teste Thes e limit ations of break-even ‘analysie 4 ant facto. in the study management devi ce,
It ghanid
assumptions on whic nalysis is based and which are in effect, not trTue. anse Hom
“breakcerta in be an imp ort
ak- eve n ana lys is is a Very useful can be use d more
~even mitations, bre que
n alone the techni
Thus
In spite of these itt
*
” ed bec aus e the
It “must be appli
a Qiysis 1s ba sé a on a simp TMpiyt e d m ode: of f @ bu SINZSS whic h iS ur realistic tio ns
in m ind its limita
t
intell | igen f di, ;, crimina
with annding 7 t ' be used by keeping
i J cn
ti ton, with adequ ate grasp of assumpt \ ons under ying the ‘echn q ue
“ ne aing s a ic applications. The assumptions effectively. BREVIATIONS
RMULAE AND AB
of beak even analyse ave already a
SUMMARY OF FO
ta ted
Trated ar d Ww § di d
intende 0 study the unrealisti c¢ Nature of these d5§ umptioi ns.
1. The assumpti ption that all cost can be clearly separated into fixed and variabl cost is -¥}
. & components
but ion (c) = Sales - Variable (F + P}
is not | possible t 0 achiev
i : 4, Con tri + Profit
analysis, in practice, thereby resulting in in accurate break-e-even
@ accurately ar
+0} = Fixed cost
- Loss (F - L)
(C) = Fixed cost
. TI Ve assumption th at varia: ble i . or .
and that it gi s-V = F+P
mo

+
line chart is als o not alw cost per unit , remains constant Thus
of the varia ble ‘ tL Mo 1 = Strai ght s-vV = F-L
this tendency. Most of the va ‘abl In practice, many sts do not observe or
Sales in = » P/V rati
o
doubt, move i
riable costst, no orti Contribution = & « P/V ratio} + Prof
it
production but
Similarly, the not necessarily in direc on to fhe volowe
prop with the volume of Also
tnb uti on = {Sal es at 3EP in
tri but ion pet unit) + Protit
3.
ume ct Con
= (Sal es at BEP in units * Con
fi xed cost Contribution + Fixed cost
a assumpti on that remains c onstant iis also unrea listi
Tealistic, ' Fixed c osts ar in = x p/¥ ratio}
withi
dre constant only y within .
a limi ; tri but ion = (Margin of Safety C per uni t) + Fixed cost
imited range of out Con *
when additi
introd cod tend to increase bya sudden jump
or
= (Ma rgi n cf Safety in units
ditional plant and machinery is or Contribution
-
, Profit
Margin of safety in’
4
«Contribution = io}
or io (C/S or P/V rat
14,000-
/Sa les rat io of Profit/Volume rat C
2. Contribution .
io in % ="5 * 100
Cost line S-V¥
B.E. Point ibuti c P/V rat
1
2,000 4
Contribu ben . == = :
p/V ratio
(2)

Sales
\ but ion Cha rige in profit
10,000- Change in contri
Cost and revenue

Als P/V rati


e in sales Change in sales
/V¥ sate = "chang
8,000- B.E. Point | * agg
Fixed cost.
in &
P/V tatio = °E, Point
Fixed cost line =
Vo ratio
£a
8s5
1

. Profit __ x 100
__ Profit ,
p/¥ ratio = __Mar
Ss

in =
>=
o
L

gin of safety
sales
iable cast as a % of
P/V ratio = 100 - Var
2,000~
» 5,000
H8 19, 900 15,000 {
20,009
(BEP)
3, Break-ever point FE
Production (units) wi - Total xed cost
Fig. $.13. Break-even chart
more than one break-even point showing | i that BEP (in units) = Contribution per unit ¢
sales lines in a th cost and
practice cannot be represented by straight ines.
-
Manageinens aveuns-
Cost-Volume-Profit Analysis
5.34

ee nn
SOLUTIONS
Fog PROBLEMS AND

PHP 8) = Sais
F

res
c x
rate

wean aadeieei
¢ 15 per unit
problem 5.1 unit. ¥Yeriab le cost is
. . BEP = Actual sal es - Margini of saf pro duc t whi ch sells for = 20 per .
single .
Calculation of sales to earn a given profit A company produce s 0,000.
“ fixe d overhe ad for the year is 2 §,3
and
F + Profit
(in units) = = ———— (i n&)= F + Profit
Required:
3 profit of 10% on
sales.
C (per unit) ue needed to earn
P/V ratio {a} Calculate sa les val to 1,2 0,0 00 units.
s price per U ait
to bring BEP down (ca. PE TI)
Cal cul ate sale
= Difference in fixed cost (b}
profit 1s = 60,000.
3. Cost indifference point = of safety sales i f
«Marin ot St ifference in P/V yatio or contribution per unit (cp Carculate margin
ety (M/S} = Actual sales - Break-even point
Solution :
units of sale = x
M/S ratio = Actual sales ~ BEP (a) Suppose No. of
Actual sales 100 Total sale 5 = 20x, Var
iable cost 15x
+ Profit
le cast + Fix ed cost
OR Total sales = variab
00 + ex
“ 20x= 15x + 6,30,0
Margin of safety in % of sales = Margin of safety 20x - Lix= 6,30,0
00
Actual sales = 2,10,000 units.
x= 6,30,000/3
= = 42,00,000
= 100 - BE. Sales in % 00 units ~ = 20
Sales value = 2,10,0
Profit % 6,30,000
~ MWS/ S77 F = 1,26,000 units
P/V tatio 329-15
(b) Present B.E. Point = t =
Profit Margin of safety x P/V ratio.
“2 1,20,000 units.
Profit = Actual sales x M/S ratio = P/V tatio Required B.E. Point

7. Break even sales in %- 5:& Point Bp = = -

z 6,230,000
c

Total sales .
or = 100 ~ Margin of safety in % 1,20,000 =
8. Profit = Sales - Total cost
-or % Saies - Variable a Z 6,30,000 * 5.25
1 cost ~ Fixed cost 720,000 uni ts
units
Contribution of C=
= Contribution - Fixed cost
tribution
orvee et
of safety in & x P/V ratio New selling price = Vani able cost + fon
% 20.25 7 20.25
i =Sales - Variable cost - Profit = £15 + 5.25 =
~ Contribution - Profit
. C 5
= Total cost - Variable cost * rr = 25%
(c) P/V ratio = 5
to. overatl Bn at BEP in & x P/V ratio
i Profit— 60,000 Z 2,40,000
Teak even point (of all products} ——rat io onT
KF
es -_57
Margini of safety sal
Total fixed cost of all the products
. Overall P/V ratio
Accounting
Management eect

Cost-Volume-Profit Analysis 5,36

Cost and © ontribution


Problem 5.2 Solution Statement of
Marginal Total
Per unit
z
From the followi
owing data calculate
é ( 10,000 umts)
Direct material per unit me enven pons
50,060,000
Direct labour per unit’ r 500
ed. overhead (Total) 10,00,000
ri 0,000 {A} Sales 100
re 5,00,000
ariable ove
overhead 100% on direct labour
rc 46
Materials 2,506,000
= 10 25
Labour , eT
Trade discount 5% ads 17,50,000
_ Variable overhe V95
Geéiiine the
ihe net
ne p profits, ie if sales are 10% fo above the break-even point (B.Com, C.A. Incer) 32,50,000
(B) Variable cost 425
Solution 20,00,000
8)
contribution (4 ~
200
12,50,000 _—
Marginal Cost Statement
Less: Fixed ove
rheads
profit. 125

Net selling price (% 10 - 5% discount)


—— New SP - we
new selling price 50:
Direct material , Contribution al ays - E15 = F2
Direct labour 3
Desired pro- fit
Variable Overhead : Fixed cost + —- .
4 desired P rof
it bution per unit
Contri
Variable cost | x.00 Sales to earn
price
Contribution (% 9.50 - 7.00} 790
2.5 duced selling
profit of 125 0,000 at re
Sales to earth a 13,000 Units
+ 12,50,000 =
= 20,00,000 the
Break-even point = ri. uo ~ sald to earn
250
l have to be
t, then 13 ,008 radios wil
B.E. Point (in 2} = 4,000 uni ee “ uni
d ta 2 425 per
price is reduce
Less : 5% discount mis @ em Thus if selling
n sales value at B.E. Point 2 game profit. 2 45,259,000
Verification .00 0 * 425 ) = 22,125,900
wee sales are 10% above B.£. Point = | | New sales (13
le cas t {1 3,000 * 175) = 32,50,000
Variab bution
Centbution (4,400
1 400 units
titsx« && 250
2.50) Less: Contri
Contribution
on (a % 11,000 = 20,00,000
z 10,
Fixed cost = -22,00,000.
Profit = TT . Less: profit

Problem 5.3
Electre Com pany sold 10,000 units last year at a price of {5 60 each. The cost struc! ture per unitae is Problem 5.4 ze
as follows : a is given :
The following dat 20 pet wait
11 per unit
z .

Materials ” 100 Selling price 3 per unit


me cturing costs
Labour
Variable manufa 5,40,000 pet
yeal
Variable overheads Varjab le sel lin g costs
yeal
= 2,52,0 00 per
. erheads
Variable cost Fixed factory av
costs
Fixed overheads 200 Fix ed sel lin g
to compute : sales. in rupees
:
_ Total cost 375, You are required
nt exp res s ed in amount of
of % 60,000 per
year.
Delt)
Due as to com petition,
titi z 4 25 al; Assumin1 ig
comin Gg year. (i) Break-even
poi
be sold to earn 4 profit gal es? (B.Com, Hons.
:
thei p price has (0to be reduced to for the
te that there
uni ts tha t must of 10 % of
ts shall be sold 4 toto net income
chai fi of
change in costs, ensure the same am ount of totalal (ii) Number
be no
wilt
; nd out how many uni sold to eat na
(B.Com. Dethi) units must be
Profit as last year,” \ (iii) How many
tuig
Management Accoun
t. Volume-Profi
fost:
Cos t. Analysis
5.37 5.38

Solution Situation B fit


= Contribution - Pro
Fixed cost
2M Lb
P/V ratios OTL
0
000 - 20.0 uni©ts 60= .0z 20
~ 60,0,
| oO 30% = 2 000 = 4000
5 0 Contribution pu. per writ
5,.40,00+ 0 2,52,000 g.p. - Contribut° ion
Fixed cost
(‘) Break-even point = eT
.
Variable cost as a %
of sal
P es = e
PF ratio -
2 -20
50_— 199 == 60% a
~ BS 792,000 z =—

% 40
of 70
.000 5
50

Untmits t oO be soid to earnsa a profit7S 60,000 situation C


(i) 00
0,000 + 4,000 = % 1,50,0
Yu

Contribution = F + P = =1,2 %10 9 - 75% == 255254


or
. . d profit 7,92,000 + 60,000
« Fi arian desire
= 1,42,000 units
- = ; = 100
P/V ratio
on pe
per unit
: =
6 gt .
t = € 20 * 25%
= %5
ion = § -¥ = 20 - 14
. but ion pet uni
Contri .
Contribut . tc =be20s -
= 86
ae
tribution
TotalCe conon anik
.50,
= Bl“ow -
000 = 30,000 units
(iii ) Sup pose units 9 . . d = c
ole t = “x” Ho. of units sol ! 5

Oo earn 10% profit ontribution p
ling pri
Total sales = .Sel ica x pnits = 20 x
i :
Total saloxes == Var
eeVaria ble cost +
. ed cost + Profit
Fix
’{7 Situation D
~ 10.000 ~% 15,000
eOx = 14x + 7,92,0r 00 + ex profit «C ~ f «© 25,000 = 25%
2,000 - 75%
Thus 4x = 7,9eee p/¥ satio = 100%
-
792,000 = 4 24,000
Xo=
: F+P
py ratoio =F 5%5a = & 1,00,000
na sales = ra/¥ ti a
Thus saies to faro s
m a net income of 10% on gal es = 1,98,000
unit “4 : ae 1,00,000 .+ 6900 units = % 16.67
s. ; ce per umi t =€
. Sellin g pri
Froblem 5.5
.
ae
eFill Situation E © % 35,000
&
- 19,000
riyed cost « C ~ P = 50,000
blanks
in the saat uations: 4
S for each of th € following independent sit
‘ 1 50, 000
¢ _ . 33.33%
Selli A B C
p £ qotal Sales= 5
5 ~ 7,50,000 x 100
1,50, 000
varias Frice per unit 60-
50 20:
76 - 30 /
4 of Sakes = 100
% - 33.33% = 66.
67%
of sales - 75
t t as d %
cos
No. ofvie uriats 10,000 4,000 - Variable cost as
cononttriributionsol - 5,0
and a profit of @
0,000.
Problem 5.6
' 6,000 .
80,000 25,' 000 ae wit h sales at ¥ 3,00,000 000 , calculate
: % 20,000 -
10,000 30,000 .
ed experses af %
90,000
y suffered a loss
of © 30,
Fi
ixed cost ’ 0-4 a
-—~* TF 00
~| 20.000 120,000 | - (a) A co mp an y has fix
the next per iod the co mpan
Profit {Lass} 5 2 /Yolume ratio. If in
30,0 00 - 15,000 Calculate the Profit (CA. Inter)
(CAT the sales volume. @ pro fit of Z 60,000 in (a) abo
ve?
“A. Inter) gin of safety for
Solution (b) What is the mar
. .
Situetien A Solution 60,000 = % 1,5
80,000
= = 90,000 +
Profit « . ed cos t + Profit
Tofit Contribution ~ Fixed cost {a} Contribution = Fix
= 20,000 ~ 12,000 = c 1,50,00 19 = 50%
1,50,000
pppoe selling price = x 8000 P/V ratio = g$ * 100 3,00,000
30,000 = % 69,000
6, Of units sold (5.P,
(S per uniti ~ Variable cost per unit) == Contrib s = % 90,000 -
= Fixed cost - Los
Contribution
Con tri but ion
In th e next period
10,000 (x2
5 *}-~ = € 20,000 60,000 _ ey,
P/V ratio = “sates
25* ae? 60,000
Eo, e @ 1,20,000
Sales +
. xX af % 30,000.
will be a {oss of
Tr as selling price per unit = %5 %-1,20,000, there
Thus at sales 0 f£
Management Accountin g
eee

Cost-Velume-Profit Analysis
5,39 the
rease of 10% from
tied to a wage inc
, the dire ct workers wall be enti ad are exp ect ed to inczease
During the forthcomi
ng year and fixed overhe
60,000 . yatiable over head
(0) Margin of safety = and the material cast
P/Viatio * “soy7 = 1,20,000 Ee | peginning of the year
respectively.
by 7.5%, 5% and 3% 4
ed to be computed : maintained.
or M/S = Sales - Break-even point The following are requir t P/V ratio is to be qi
e in the for thc oming year if the curren , se 2s to yield the
same
< 3,90,000 - f Fixed cost) . {a) New sale pric to be sold
yeat
du ring the forthcoming unit will not be increased.
that would requ ire price pet
\Py¥ratio! ~ 309.000 - eS = © 1,20,000. {b) Number of units , assuming that selling
(LC.WaA., Inter}
in the current yeal
amount of profit

Problem 5.7 atement ee


Solution Marginal Cost Sttatemern
a TT Next year
__- iacrease
(ibempauynite
A Compa avyaa
has a P/¥ ratio of 40%.js By y wh what percentage must sales be increased to off-set: re Current year f
z 123
7.5%
=
(8) 20% reduction in selling price 120 3
(8.Cam, Hons. Deihi} 10%
Material 30 12.60
Solution 5%
Labour 12. 174.69
ad
se — Sales = (100 units @ % 1 each) Variable overhe 162,
unit
Less ; Contribution (40% of sales); ue
40 Yariable cost pet 270
= When sen Variable cost Sell ing pric e 108
¥)
q price is reduced b Contribution {S$ -
New Sales 7 iene : 408. 100 = 40%
Less : Variable cost p/¥ ratio = § « x 100 = 379 * of the
- g inal cost is 30%
New contribution 30
50 ng yea r-— The current year's Mar thcamr g yeal.
omi for
ce for the forthc racto of 40% in the
Volume of sales to maintain
i the same contribution will be (a) New selling pri er to mainta in the current P/V
276. In ord
selling price of %
ce should be :
40 ~ the new selling pri
bution
, —lontitri
New conbution ~ New sales = 35 * 90> % 120 174.60 _ 7294
have to be i . 60%
Thus if theto selli
% 100 Z Onefg pricei ce iis reduced by 10%, the volume of sales wiil @ increased by 20%, fe. year z
from
Sales volume in the forthcoming 13,20,000
{b)
(b} If Selling price
q price isi reduced Profit in the curfent yeer: 14,00,008
New sales by 20%
Contribution (40% of % 40.50 lakhs) 2,20, 000
Less: Variable cost Less: Fixed cost
: “0
New contribution F 20 Profit
ea 0
2,20, 000
Forthcoming year:
In order to maintain
ti th 8 seme contribution, ‘ the vo ume : . ot sales
li snou a: Profit required
14,42,000
(14,00,000 + 3%) ion 16,tO62,000
40 Add: Fixed cost Desired contribat
ag * 80 = % 160 474.60 = 95.40
Contribution yer unit = 270 -
Thus if selling g priprice is reduced b y
;
20%, the sales will have to be increased by 60% .
16,62,000 17,421 units (Appro
x).
Problem 5.8 uired = “ga
Volume of sales req
The variable cost structure of a manufact ured by a compan .
product y duzi ng the curren { year is: as Coviem 5.4 e products are
F100, % 80
ander: ot
The unit sell ing prices of thes orti ons (quan:
es three products
F, Qand R.
and = 20. The prop
. © per unit ABC Ltd. manufatur g unit vari able cosis ate 50, € 40 50% resp ecti vely . The total
espo ndin
The corr are 20%, 30% and
Material oo and % 50 respectively. manufactured and sold
these products are
120 ,
e) in whic h . ‘
Labour 30 tity -wis - ty and the
© 14,80,00 0. r-a ll break-even quanti
Overheads w4 12 fixed cast s are
requ ire d ta wor k out the ove
(B. Com . Hans., Deth!)
rmation, you are
-
n the abo ve info —
The selling pri Give
ig the current year are % 14 lakhs
price per unitii is € 270 ai md the fixed cost and sales durin of such quantity.
and © 40..50 lakh s resnecti
pectively. product-wise break-up
:
Manageniecne ee

Cost-Volume- Profit Analysis


1,52,750
+ 92,000 + 48,750 = =

HetE re
= 52,000
Solution z 1,42,750
: it (SP) = A bee = E5875
.
selting price per unit (SP) = 5° Eo9 buckets

SARE RER
One ib of the assumpti
ptions\ of break-e vi ent analysis
Yas 15i that the company Toduc ¥ fal Th in .
muitl-pre duct companies, product mix does 23t change. In th2 recent probtem ’ ‘hee produ cts ‘ 5P t QandR Ree ale
beir
. ig it.
mantifact
neA rured, the t otal contribution
bri i from the thr
hree rod ucts is taken
pre as a contribu t ion of f one c oOmMpos-
verification Total =
Per unit ©
4,52,750
Sales ~ Variable cost == Contribution 58.75
Sales 52,006
P =@100-50 =%50 . 20.00,
Less; Variable cost 1,00,750
a «€BC-40 =F 40 38.75
Contribution 52,000
R =€5¢-20 =% 30 20.00
Fixed cost 48,730
Less: 12.75
Contribution pet composite unit: : Profit
P =50* 20% =10
be merged for 5bette
r
Q =40% 30% =12 problem 5.11 who want them to
oo
pla nts und er the same management
R =30% 50% =15 ee similar
A, Band € are thr er :
237 details are as und c
pperauion. The A
B
50%
Plant 70%
100% z
Overall break-even point = Fixed cost - ewe ) ani Capacity operated z
Contribution 37 = 40.000 anits z fin takhs}
(in lak hs)
Break-even point of P = 40,000% 20% = 8,000 units (in lakhs} 150
280
300 75
Break-even point of Q = 40,000 x 30% = 12,000 units 210
Turnover _ 200
= 40,006 = 50% = 20,000 units 50 62
Break-even point of P Vatiable cos t 70
Problem 5,10 Fixed cost
Find out:
for break-even.
Sultan Pl astic Compan iY makes Pp plastic buckets. " An analysis of their ace ounting Te vi eds:
L of the merged plant
(i) the capac ity
variate cost per bucket = 20 capa city of the merged plant. , i¢.a, Inter)
75s
tif) the profit at % 28 lakhs.
ret
ixed a cost % 50,000 for the year
from th e merged plan
t to give a profit of
{iit} the tur nover

Selling price per bucket oe Solution O% capacity.


ts at 10
Required : er of the Three Plan
"
Statement of Cost and Turnov aS (Figures in iakhs}
( Find the break-even point.
(ii) Find the number of buckets to be sold to get a profit of % 30,000 Plants
(iit) If the com pany y cai mH. anufacture7 ixe S
600 : bucket more per year i
with an additionai
additi fixed
i xed cost oO f =
Merged Plant
100%
2.000, w hat ould
shoul be the e s elting Brice to maintain the profit per bucket 3s t «i Uj) above.
b

Solution
{B.Com Hons. Dethi) Capacity 1,000
650
Turnover
Contrizution = Selling Price - Variable cost Variable cost
Less:
= FW) - F 20 = F450 per bucket 62 i| 162
Contribution nl
Fixed cost _ = 56, 000 Fixed cast 70 at
(units)
(i) BLE, Point (units) == 774.4 >a 7a)
...
Contribution ferunit 50 _ = 1,000 units
ged plant
(i) P/V ratio of mer
{ii) Required Sales =
Fixed
Desived
cost + profit % 50,000 + 30,000
= 50 = 1,600 units Contribution of merged
plant «100 = 3500 Tak
_ ©1,00 hs | 9c = 35%
* Contsibution pér unit t lakhs .
Turnover of merged plan
75
7 30,+000FC, + Profit +1600 buckets = % 18. Fined cost _ % 19275%lakhs = ¥ 520 lakhs
(ii Sal
ne fit per bucket== ¥.C.
Proas = (P/V Ratio
Break-even point
eam can plow produce 2,000 + 600 = 2,600 buckets
(2,600 x SP} = (2,600 x 7 20) + F 52,) 000 + (2,600 « 18.75)
Cost-Volume-Profit Analysis

Capacity of the merged plant at break-even point

a Turnover at break-even point Bs. §20 lakhs = 52%


100 =
Total turnover . Rs. 1,000 lakhs
Revised sales mix
(ii} Profit at 75% capacity of merged plant
Sales * 9,000
Sales at 75% capacity = © 750 lakhs. ~ _
Yariable cost z
Contribution at 75% capacity = 750 « 35% = © 262.5 takhs
Contribution %
Fixed cost = 2 182 lakhs
Less: Fixed cost ®
Profit = = 262.5 - 182 = % 60.5 lakhs
(iii) Sales to eam a profit of € 28 lakhs: 14,700 (approx).
; 14,700 46,226
x 60,0G0 ==
. _ Fixed cost+ Profit 182 +28 Break-even point = 49,080
cy ¥ 660 lakhs.
P/¥ ratio
vat
Problem 5.12 Problem 5.15 towing data -
cute
You are given the fol
(a) ABC Ltd. manufactures and seils four types of products under the brand names, A, B, € and D. The
8,060
z 1,20,000
; 13,060
sales mix in value comprises 334%, 414%, 16% % 84% of 4, B, € and D respectively. The total budgeted | z 1,40,000
sales (100%) are % 60,000 per month. Operating costs are as follows: . seYeas 2018
oe
Variables costs:
B 68% of selling price find out—
Product A 60% of selling price
C 80% of selling price D 40% of sething price ti) P/V ratia,
m)
Fixed cost is € 14,700 per month. (ii) BE. Point, 0. ica. inter co
= 1,386,000,
en gales ate
Caiculate the break-even point for the products on an overall Dasis. jit) Profit4 wh 4 prof it of = 12,9
% uired to earn
(b) Tt has been proposed to change the sales mix as follows, the total sales per month remaining on Sales req
in yeat 2018.
ofis af et y
60,000. (y) Margin
Product A 25% Bo 40% Profit
Sales
B Od en
C 30% | 1,20,000
Assuming that proposal is implemented, calculate the break-even point. (B.Com. Hons., Delhi) Sausnon z 13,
z 1,40,000
: 5,000
Solution ve oe 20,000
z
ence
Differa
: = 25%
5 40
an
(a) sy & x 04 —_
oO

t
Difference inprof A .
000

Products : sales
= Difference
c De Total ti) P/Y ratio
A B
* 000
30, acd
Sales mix 3345 % 41% % 16% % 84% 100%
(1,20,000 x 25%) aire

e = - z Q | Contribution
in 2017
Less. Profit
Fixed cost‘
22,000
Sales
Variable cost
29,000
32,000
, 25,000
17,000
10,000
8,000
5,000
2,000
60,u0G
~ 39,000 )| "

= Fired cost
+ Profit
. jbution ti on - Profit
21,000 = Contribu
Contribution ed CiosSt
14,700 o, Fixe
_ 22,000 _ 288,000
_ hess: Fixed costs Fixed cost ‘
~ Profit 6,300
=
c
PNpvF atiric = 299%
poiint
tii) Break-even
an

™ .

Break ; Fixed cost Sales = 14,700


feak-even, point = Toighution 21,000 * 60,000 «= © 42,000
.
ws
Ye aesee eno
PLOT

Cost-Valume-Profit Analysis

a loss of %
"problem 5.15 ts a nd has incurred
(it) Prafit when sates are € 1,80,000 - |
00 uni ts and 9,000 uni d at % 100.
Contribution ( 1,80,000 x 25%) ; in two successiv
e per io ds 7.0
ling price pet uni
it can be assume i
i
A company soid pe ctively. The sel
Less: Fixed cost oe 000 earned ¥ 10, 000 as profit res r
16,000 and
Profit one to calculate:
You are required
fixed cost
{iv} Sales to earn a profit of % 12,000 = (a) The amount of n (LOMA. Tater)
uni ts to break-eve
ri
ixed a
.
Desired profit — 22, 000+ 12,000 {by The number of fit of % 40,000.
a pro
— 75% = 7 1,36,00 of units te & arn
ratio ~ (cy) The number
Difference
Period II
{v) Margin of safety in 2018 - Solution Period I ¥
z
Margin
gin of safety = Actual saies - Break-even sales t 2,00, 000
9,00,000
Palen ou = 1,40,000 - 88,000 = FT 52,000 unit} 7,00,000 20,000
sales (@ & 100 per 10,000
{-} 10,000
Profit/Loss (-) :
Given below ar e the sales and profits of the two h alves of| the year : t 20,000
Difference in Pr ofi
; x 100 = 10%
x 100 = 2,0 00,000
snl Ist half lnd half “pifference in sales % 70,0090
* BAW ratio =
z 7,00,0 0)
rare z 1,00,000 1,20,000
iod | (10% of = 10,000
Contribution per
z 30,000 "38,000 1 % 80,000
Add: Loss in period
Fixed cast duri alf isi equat to that duri
uring the First half the second h alf. Selling price and per unit
pri
variai ble cost temai anged. Calenta the following uring
n unch —_
g: : te (a) Fixed cost
but ion - Profit (ar
© ontribution +
Loss}
t = Con tri
in 1) P.mn ratioi for each half and for the full year. Note: Fixed Cos
Fixed cost 80,000
ai ae cost for each half and for the full ea 10% = % 8,00,000
point = “p/y ratio =—-—
;: ea | for each half and for the full year . = (b} Break-even units
@ + 100 = 8,000
alf-yearly sales to earn half-yeacly
: profit of = 40,0000. uni ts to bre ak- even = 8,00,00
Number of
(v} nnual sal les te earn annual profit of 90,00 , LOT] ons. le i h f)

- "pf iatio
Solution es =
{c) Required Sal
Profit _ 38,000 - 30,000 8,000 3 _ x 42,00,000 or-12,000 units
(i) P/V ratio ~ siference 50 ooo 40% ,
= . go,000+ 40,000
,000 - 1,00,000
lt wen sales ~ 1,20 3 10%
For each of th will
-e 40%,
remain th é same ie. re
d é eshalf x year
P/V P/V
rati o}rati- o Profit . oh
we as compared
Fixe
(ii) tet es = (Sal ta % 60,008,000
- Problem 5.16 7 sales amounted
alf = (2,00,00 0 cost = 40%) % -10,030.0
= 00 00
= = 10,000
. 4 an nual fixed cast
s of % 14,00, 000 . In 201
higher than in
2016.
_ was.% 4,20,000
For full year, fixed
00 «2 = & 20,000 A company has
with © 45,00,000 in 201 6 and profit in 2017
y ‘bt eak-ever?
. < (i) At what level
of 5 ales does the
compan
of © $0,00,000.
(iii) BLE. Point = <>
P/V ratio 4 pro fit or los s on a pre cast sales Va lume
the company desi
reg te earn the
same
. : (it} Determine in 201 8 by 10% and
(B.Com . Hon s., Delhi)
aki in gelling price
uction
If there js.a red be the required
sales?
Ist half - 2%
0% 7 8 tO
“st
és
:
{itt}
profit as in 201
7, what would

BE Point for full | years = 7 25,00 O00 x 2 =
=
250
00 Solution
. | 4,20,000
100
0,00,000 ~ 45,00,000 x
100
;
= ate oo rati Increase in profit =

{iv} Required sales


P/V x
.
P/V ratio ; “Tyevease in sales _
5
, {MV ratio =
14 4
000 + 40,000
| _ 420,000. 499 > ~ wet “28%
Half year sales «« 10, = © 1,25,000
40% 15,00,000
\
20,000 + 90,000 ” wf Fixed cost 14,00,000 = 7 50,00,000
{v) Annual sales =
40% = & 2,75,000 foe
Break-even Point
= P/V ratio 28%
SO (i}
Cost-Volume-Profit Analysis
6,48

(4) When it
sales are % 89,00,000 000 and earned a prof
. “problem 5.18 Z 450,000. with $ al red
es of % 15,00,
eaten = 80,00,000 x 28% . . —o ed fixed: expenses of 1,5 60, 000 .
- Fixed cost -2 22;40,000 inc urr su ffe a-loss of F
‘a company had second half, it
; 14,00,000 the first half yeas. In the
of 2 3-00,000 during year.
for the first half
Calculate: m argin of safety
(ii?) When setling price is 10% less =“ —_—" ume rati o, bre ak-even! point and tha t set hin g pric e and fixed expetses
(i) The profit- vol second half year
assuming
es volume f or the
bese ‘eine 80,00,000 - 10% i} Expected sal the sec ond half yea. (CA. rnter)
ged during whole yeal.
: € cost (80,00,000 x 72%") aan remained unchan saf ety for the 4
margin of
even point and
New contribution es tit} The break-
14,40,000
“Ratiotio ofof variable
vari cost to sales = 100 - 2B% o>= 72% iO solution
f year
(ij) In the First hal
£ | 14,40,000 = Fixed cost + Pro
fit
New P/V ratio = x 100 = 20% Contribut ion
00 = Z 7,50,000
5 72,00,000 00 + 3,0 0,0
= 4/50,0
Profit in 2017 = (60,009,000 x 28%)
profit s} ~ 14,00,000 0 == 2 2,80,000 0
C_ 75000,.00000 oy
Sales to earn the desired 25 75, = 50%
p/¥ atic
Fixed cost + Desired profit 14,06,000 + 2,80,000 _€9,00,000
= % 84,00,000.
00
=
. 7 Fixed cost _ 6,50,0 -00,
P ¥ ratio 20% nreak PV atin 50%
reak-even point
Break-even paint
Margin ol f safety = Actual sale 5 -
90,000
9,090,000 = 2 6,0
The ; following( . ' figure
ig 5 relati
: ating to the perforr ka nee of a compan’
0 pany of the yi years I a T d Ii ave available
b = 15,00,000 -
ra that
half yeat
: the Tati o of variable costs to sales, and (i) the fixed costs are t the same
(i) In the second
for b oth the
Loss
= Fixed cost -
a) the Pro t-volu me i Contribution = z 3,080,000
ratig, (b) the amount of the fixed-cos
| 5, A 1,50,000
= 4,50,00 0 -
300,080 . z 6,090,000
(¢} the Break-even point, and :
t- Loss
Fixed cosroti
(d) the bu dgeted Bp rofiti for the ¥ year Tl, i
if the budgeted sales for that y year are 7 1 C tores a Expected sal
xpected sales vO
um
ume = P/V 0 50%
Total sales Totalal ¢ sts”
cost, ::

f% in ‘000 ) (= in 00) year


Year } 7,000
(ili) For the whole
800
Year II Fixed Cost a 4 50,000 x 2 =k 18,00,000
9,000 6,600 e
B.E. poin t = 50%
.
(LEWA, Inter) por P/¥ ratio
,000
- 15000 . 2 3,00,000
Solution
profit io _ 30,00.
Sales
hr. ofit Margin of safety
7
= [yr rat
50%
Total:
(7 in ‘000)
cost
° ¥* 'p/ Wratio
.
(7; in( “008. ) 5.800
( in ‘000)
f r
Yea I of 2005
400
1,200 Problem 5.19 pan y for th e first half
and second hatf
6,600 com
Year If rin g
~ aon _ ae.122,40000 The fatlowing cos
ts and sales of
a manufactu
vast half
800"
500 Second half
pence a 1,200 mo 06 are given :
irst ha
g
z
{a) P/V ratio = 60% 30,00,00C
= 3000 * 100 24,00, 000
26,060,000
21,80,000
(b) Fixed cost os eas x» P/¥ ratio) ~ Profit 4 Sales
Total costs
, , Year I = ( @,00,000 x 60%) - 12,00,000 = z 30 00,000 ermine *
You ate asked to det ratio of the firm.
Con tri bution / Sal es
{i}
{c} BEE.Point
PiWratio 60%, ~ © 50,00,000 (ji) Annual fixed costs. (LCW,A. Inter}
lift) Break-even point. sales.
wdgeted sales= 190,00,000
(a) Builgeted 10, ag percentage of
1
Hargin of safety
tribution = 100,00,000 x 60% = 60,0 (Ww)

Profit: = C - F “um
= 60,00,000 - 30,00,000 = % 30.00.000
es
Management Awuune
Cost-Volume-Profit Analysis
"6.80

Solution =
solution ofit
on = Fixed cost + Pr 4,090,000
{a} Contributi 2%.
Sales (%} I Half
Hf Hat f | = 2,490,000 + 1,60,00000 =f 2,060,000
24,00,000 30 oo 000 ference = 1,009,000 + 1,00,0
= ¥% 10,00,000
Less: Total cost (2) KM,
21,80,000 26.00.000 600,000 t {For .M, and M,)
sales @ % 100 per uni
Profit

. __2,20,000
- 1.80,000
uy ——
1,80,000 ™ C
ot
P/V vatio = 5 * 100
} P/V ratio {or C/S ratio) = Difference in profit 1,80,00
Differences in sales . ooo = A0%
Moe 420,000 x 100
i) Fixed Cost "a Total
sconond
sales for th.
the year = 24,0 a 1 16,00,000
4,00,00 + 36,00,000

ai
= 20%
M, = 2.00,000, x 100
aneation = 54,00,000 « 30% 16,20,000 2 10,00,000
ratio
ess: Profit (2,20,000 + 4,00,000) 6 20,0 B.E. Point = Fixed cost + p/¥ 2% 6,006,000.
Fixed cost
10,00,000
1 0 000 KM, = 2,40,000 + 40% =~ 5,00,000.
= 4,00,000 + 20%
the same profit
M, 2
(4!)
the models will earn
Break-even point =
P/Vratio 304% m & 33,33,333 (by Sales at which both
cost 240,000 — 1,090,000
Difference in fixed
satio * 40% — 20%
tiv) Margin of safety . Sales- BEP « 100 = 54,80,000
— 33,33,333
= 38.27%, = Difference in P/V
S ales 54,090,000
Lat, or 7,000 units
Problem 5,20 = 7 7,090,000
= 210%e profit.
Theat will earn the sam
. manu
: factures
' a duct
sing} e Preduc h; a LAG
L g d Marainal
g. i co
cost 9 TO75a of 2 7,00,000, bath M, and M,
teens is such that up to
" ‘ unit.
i Fixed
i costs are & 12,000 At gales
40,000 un : can be sold at % 1.50a unit,
7,00,000 .
Profit at sales af %
but any additior al sales must be made = % 40,000
a unit. There 1g a plannec Pp ‘ofit of z 20,000. Haw many units
must be made and sold ) - 2,40,000
x 40%
= (7.00,090 = % 40,000.
- #,00,000
M,
x 20% ) low upto 7,000 unit
s. When
Solution (B.Com, Hons. Delixi} M, = (7,00,00 0 thu when demand is
s is $ vitable a higher P/¥ ratio.
poi it and bec aus e it has
Planned profit {c) M, has a lower break-even
00 units, My, will
give largef pro
fit

Add: Fixed cost = = 20,000 demand is move than 7,0


: 12,000
Contribution required type of praduct.
Problem 5.22 and sell the same
32,000. the sam e man agement, make
Contribution pet unit
= 1.50 - 0.75 = % 0.75 and Comp any B,
both under as under |
-- Company 4 ount for June are Company B
Contribution from 40 000 units
Theit pudgeted profit and joss acc
iv
New contribution = oe « 0.75 = & 30,000 Company A z
eT E-O75 =7% 0
g z
Tote
itional
ribution of % 2,000 @ % 0.25 piu wilt ¢ a
contributj z 3,00,000
quired = 40,000 + 8,000 Bee feulre 8,000 units 3,00,000
0 = 48,000 units. Sales 2,00,600
Problem §.21 2,490,000 2,70,000
Variable cost 70,000 _——_——
. ‘Less: 30,000, 2,70, 000
A com Pany
any wants to buy a new machine . . - Fixed cost nl _——_—_—— 30,000
for tw to re place ane whic i having freq uent i j
break-down, ' It receceiv 30,000
offe rs ch is e
a models M, and My.. Further deta is Tegdar
whi
re din g these models
od are g. yen belo We :
H
Profit
:
.
Instdat led capacity (units) H, Your are required to 10,000.
make a profit of ©
rive 10,0' 00 for each.
break even point two companies wilh
ant 00
um4 (3) (i) Calculate the
ern
Estim, ated ead perak ann
tha
240,00,000 0 100 volume at whi ch ea ch of the
in sales volume.
city pe()
” prafit
increase or decreas?
The
Tequ
h product m Pont af the abovesingcapathisty 1,60 . _1,00,000
of machini e (M, or M,) is sold at % 200 per unit. . ¥ You are
{it)
{iit}
Calculate the sates
Assess how thei r pro fitabi lity will change wil th (C.A.. inter)
(a
ired:B to determine’ an .
; an me
} Break-even level of sales for each
modelel.
(5) The " level f sale ‘ models will ¢. ;
for di doth the
5 at
(ce) The mod Jel ° 5suitable for which different level of d arn the same profit
emand for the product. .
:
(LCA. Inter, B.Com Bens. Dethi)
ng vi
Management Accounti
Cost-Volume-Profit Analysis a ans

5.51 §.52

Solution
Contribution
» Fixed cost + Profit’ (b) P/V ratio

Ss o
3
ws

&
A= 30,000 + 30,000 = % 60,000
4 = 70,000 + 30,000 = 2 1,00,000
Profit _ 1.50.000 » ¢ 90,000
Margin of safety = piv ratio 25%
P/V ratio : = . 60,000
x 100 4 = "300,000 = 20% or 1/5 .
Alternatively, = Contribution - Profit
- 1,50,000 = ¥ 50,000
1,00,000 = 7,00,000
B= 300005 7 334% or 1/3 =e = f 2,698,000
« 90,000 + 25%
int
BLE. sales
= Actual sales ~
(!) BEB =
F 30,000 Margin
ve in oof safety 2,6 0,0 00 = € 6,00,000
= % 1,50,000 = 8,00,000 -
P /v ratio 20%
sellting
Cos and ai price
70,000 Problem 3.°%
Problem 5.24 ff the marginalinal cost
oe ae
Bea .F 2,10,000 ing the yea 2017. ety sales.
1B da grofit of @ 30, 00 duri find out the amount of ma °
{a) A company eamme O per anit resp ecti vely . acre, ales,
of the product are
%8 t cecurs at 50% o
i The break-even poin 75% of the capaca
cit ter
{ft) Sales to earn a desired profit . Fixed cost + Desired profit vari ane
able et
cost .
Also compute profit at
(b) The ratio of costs are
ee € 90,0 00.
s when fixeto d sae
P/¥ ratio Find the capacity sale
_ 30,000
+ 10,000
= 20% = % 2,00,000 .
lution

aie
10 .
7 we 20%
p - 20.000 + 10,000 2SoM
P/V yatio

oe
= & 2,409,000 , (a)
334%
Profit
_ 30,000

lpg
(iit) When there is an incr ease in volume 2 4 50,000
of business, a com pany with latger P/¥
profits. In this case com pany ratio can earn larger Margin of safety
8 would earn larger profits in conditions of heavy = P/V ratio 20% se
When there is decrease in volume demand.
of business, a tompany with lower break-
this case company A will start earning
profits when
even point is better off. In 70 = 30%
its sal es reach the level of 1,50,000 where (o} P/V ratio = 100 - 7
Will start at % 2,10,000 as profit
be better,
in th @ case of company 8. Thus, in conditions of lower demand,
company wil f 90.000_ = 3,909,000
_ Problem 5.23 Break-even point = AV ratio 30%

{a} If margin of safety is T 2,40,000 (40% Bs.300,000 _ ¢ 5 99,000


of sales) and P/V r atio is 30% of AB Ltd, calculate
Break even Sales, and [( 2)
Amount of profit on sales af Z 9,00,000. its (1) 100% capacity sales 60%
= & 3,75,000.
00,000 x 75%
(b) x Ltd has earned a con tribution z ty #75
aci
What is its margin of safety?
of % 2,00,000 and net profit of % 1,50,00
0 on sales of & 8 00,000 Sales at 75% cap 22,500
x 30% )- 90,000 = *%
Selution {CA, Inter) ' 75,000
Profi t at 75% = (3,
. certalit
Problem 5.25 ; m anufacturec an d sold and a id in 4
{a} Total sales 100 mark et, 20.0% P ocket transistors can be nee 4 be manufactured and 5¢
= 2,40,000 000x ——
Te =« & 6,00,000 In a purely competitive transistors
Tt js estimated re : 900 pocket
Contribution = 6,00,000 x 30% = % 1,80,000 profit is generated. P i, vat? istor r isi % 100 an d
per transisto
to earn the same The variable cost
Profit = M/S © P/V ratio = 2,40,000 monopoly market ete 00,000.
Fixed cost x 30% = F 72,000 © conditions is targ
= Contribution - Profit Pro fit under both the itive conditions.
1,80,000 ~ 72,000 = % 1,08,000- & 37,000. nopaly and compet
* the total fixed cos t is selling prices beth under mo (B. Com Hons Deiht:)
to find out the unit
(1) Break-even sales wa. _ 108,000 You are required
= % 3,60,000
P/V ratio §——-30%
{2} Profit (Sales x P/V ratio) - Fixed gost
= (9,00,000 x 30%) - 1,08,000'= % 1,62,000
Management Accounting
Cost-Volume-Profit Anatysis
. 553)

Solution
U : - Solution
nder Monopolistic Conditions per unit be X
5 : (i} Let the selling price 40.000 X
uppose x is the selling price per unit Total sales 3,200 X
of sales)
Sales 2 Total commission (B% 4,000 X
Variable cost s}
yx= 2,000
O00. « Total profit (10% af sale
Fixed cost + Profit 4,000 X
=@ ” © 100'= € 2.00.00 Total sales = Total cost + 80,000 + 36,000 + 3,200 H+
cvsired profit X = 1,93,600 + 99,000
a & 200-000 49,000
+ 7,200 x
» one 40,000 X = 3,93,600

=F+P‘ ‘
00x ~ 2,00,000 = 37,000 + 2,900,000 32,800 X = 3,93, 600
ke®l2 of 10% on salas.
y 437,000
wheat of F 12 gives a profit z
Thus selling price
2000 o * 218.50 per wait 4,40,000
Thus, 7 selling G price
pri = % 218 .
Under Competitive Conditions -50 per unit.
(ii) Sates (40,000 = 11) zg
Less: Marginal cost: 1,93,694
Su Ppose x isi the selling price per unit Material 63,000
Salas Labour (90,000 = 70%) 51,200
= 10,008x 64%}
Variable cost or % 10,00,000 =. factory Overhead (80,000 9,000
10,000 « 7100 (30,000. « 30%) 3,52,000
heFixed cost Administration 35,200
= % 37,000 Sales commission (4,409,000 * B%) 88,000
esired profit = © 2,00,000
or contribution
30% of % 90,000
% 27,000 i
|
10,000 - 40,00,000 = 2,37,000 -
Fixed cost % 28,800
46% of % g0,000
% 21,000
70% of F 30,000
10,000 °° © 123.70 per unit = - % 76,800
: total fixed cost ~
Th us sellij ng price = % 123.70 par unit,
Problem 5.26 .
76,800 , (or 34.909 units)
:
= 55.000 * 4,40,000 = % 3,84,000
bury Dri nk estim : ai B.E. Point = — * S
Cad
Based on the
Co. i
ated planning to establi ry co: mw pany in ind: W a to P produc: 2 Min eral Water.
following nan estimated annual sales ofidia“o0
y 0 pubsid
attl esia of t he
imates for the Indian subs
mi neral waterAL, cost
8,000 units, it incurs
studies
di p roduced
d the
A
¥:
unit . Ina per iod if it P roduces and sells
nmap uct at = 15 per s a profit of % 4 per
unit.
"A company. sells its prod
oat Per cent of total
sees If the vol ume is rais ed to 2! 9,000, it ear:
. annual cost that a loss of % 5 per unit.
te nt.
‘Calenlate the break-even poi
103.000 . (B.Com. Hons. Delhi)
is variable

Overhead — Factory 50.000 7 ™ Solution Profit/Loss (-)


~~ Adiministration 30,000 Sates
ma
30% z
The Indian B TORU
j ctio nm will
i ; be sold
iG b ‘y r
{-) 40,000
manufacturer 4 representatives wh oO will T ecelve
ve aa commissi
1,20,000
> of ssion of
the sale price. No portion i
At 8,000 units
of the British Offic 4
i jary.
It is required
to:
to:
Penses is to be allowed to the Indian subsid 3,00,000
+ $0,000
Ted
(1) mroceeds
Com pute ia
the hai
sale price per bottle to enable mana
At 20,000 units 1,20,000
ge ment to realise an estimated
1,80,000 7

Difference
Or
1 0% fa pro
profi t on ale
sal ———
.

{if) Catculate the break-even _ 1,20,000 2


is € 11 per bottle.
Dp e tanin si bsidia ry on the assumpiion | Difference in Profit
mm p' that t he sale price
ces
in Sale 1, 80,000 3),
P/V ratio = Tifferen * 2/3} - 80,000 = %
1,20,000
(B.Com. Hons. Dethi) ~ {5 «P/V rati o) < Profit = (3,900,000
Fixed cost
B/V ratio = 1,2 0,000 + 2/32? 1,80,000.
x Break-even point = F +
5, ‘,
Management Accounting
Cost-Volume-Profit Analysis 5,56

en point? isation are 70 fo, 80%


Problem 5.28 (a} What is the break-ev its of the firm, if the
levels of capacity util
The following is the budget of Cadila Co.: be the oper atin g prof
{b) What will 78,000?
and 90% respectively? ating profit of <
firm c an make an cper
Fixed Vari iable
is the ievel of activity a t which the sell s 0 nly 6,000 units in a year
due
me {c) What the Ara manufactures
and
z z in of safe ty. i f
Budgeted sales: {d) What is the marg
to market constraints? to @ 9 per umi:, sales will
se lting-price is reduced
2 :
the firm is advised that, if the to redu ce to seili ng price?
00,000 units @ % 25 each ~ _ 50,00,000 (e) Managemen t of
s to 7,50 0 units, Is it wor thwhile ent selling price can
Budgeted costs: go up from 7,00 0 unit units at the pres
o that sale of 7,500 the
the firm is also advi sed h alternative—recucing
Direct material (f) Management of on advestiseme nt. Whic ‘
9,00,000 if a sum of 2 5,000 is spent t —wil l gene rate Tore profit?
also be achi eved ,
7 5.000 on adv ert ise men
i7C.W.A. Inter) :
Birect labour :0,00,000 per unit oF apending
; selling price to % 9
Factory overhead
3,00,000
700,000
Solution q
Administration
overhead
voir Total fixed cost
Distribution overhead «00000 3,00,000 :
ao 9,000 = Fontribution per unit
Total (units)
EE
1
44,00,000 {a} Break-even point 30,000 ;
—28.00.000_26.00,000 30,000, = 4,000 units oF =
40,600
Budgeted profit = To 22.50 = “yeg
Compute the
t b reak-even point in the f oltowing i i i i
4,000 units
“ 10% increase is effected in fixed costs ne indspendent stations if
: 70,000 units x 100= 40%
in % capacity=
diy ton increase is effected in variable casts nd Break-even point
ae
t
(i) 10% / inves
increase isin aeffected in salesale price
pri i
which results in reduction i i Profits -
(CS. Inter) (b} Computation of Operating At 80% At 90%
se in fixed costs and 5% decrease in variable costs iis ottected effected. sol oy AL 30%
8,00C 9,000
§
olution
. 7,200
Sales {units} x pos
i) - ~ fite) od 7
70.000 80,000 90,000
(ti) {iti)
10% 22,500
10% . 20,000
im
increase tn Sales @ & 10 each 47,590
increase in : Re as% in- wd cost @ = 2.50 67,500
cost:
fixed costs variable in
nerense S.P. and Less: Variable
52,500 60,000
: " sane 42,000
5% reduc:
Contribution 36,000 42,000
cast
decrease in R588 :
_fion in Less: Fixed cost
“je s00 18,000
Sales units ve
od.
profit
z of. : 2
z t
Fixed cost + Pro
Selling price | 25.00 “ 7.50
27.50 r 5. Coptribution per unit
130t 25.00
14.30 12.35 (c) Sales requited *
cost ‘ OL 13.00

as
-
vB ibuti
auibution per
unit iC}
10 12.65 aod = 6,400 units (9
64% capacity)

cost (F) 19,80,000 18,00,000 18,00,000 7.50


fixed cost is = 36,000
BED. = F + C (in units 1 8 — ‘At a capacity of 64%,
165,000 1,68,224
42,05,600 wavae jpssee Thus :
:

B.E.P. (in 8) 41,25,000


an 13,050 4 .
36,000 + 18,000 = 7,200 units or 72%
capacity
~
variable cost _ 26,00, 000 Required sales = 7.50
unit = Total
1. Variable cost per = Sales - B-E. poin
t.
Notes
“300,000 7% 3 Margin of Safety = ~ 20,000
_ Units sald (d}
(6,000 = 20) ~ & 40.000 z 67,580
@ z9
(2) Sales 7,500 units
a 2. B.ELP. (in zg } is BEP in _units Reitiplied
mul by'y the respective selling
: 4 price per urut. 18,750
units @ & 2.50)
able cost (7,500
Less: Vari 48,730
Contribution 36,080
oA firm manulactures
manufactures | productXi twh,wh ling price i has th city
10,000-units, |The The vavari wore
cig
Tiable costs are ¥:2:50 is
priceunit.
sellingper ae a ™
10 costs i are " aiaited
€ Fixed per unit. Th ! acity Less: Fixed cost 12,750
capac vel : re — : at y% 30,000
Hy to:produce
upto the Profit
iscapacity
80% orutilisation
aha levet el of 50%; F 36,000 above that level but { % 42,000
{ if the levelel of of capacity utilisation
Cost-Volume-Profit Analysis 537
| 5.58
Management Accounting

product should
m possible units of each
5 18,600 direct labour hou! 1s. Maximu to the following ranking.
facto r i accordin g
Product Mix: Key hours will be utilised
Pio fitat at 70% fo CAPAC
Bp at y is & 1 16,50
A G as co mputed in
i part (b) above. It is thus not worth Wilh
hile ta
reduce the § elling price to q <9 per unit, because it Test! Its in reduction in pe er] fit by y ¢ 75) {i &.
3.750 prad uced accordin g to ram king. 18,600 labour Labour hours
be
= 16,500 - 12,750).
3,000
(f) Sates 7,500 units @ % 10 = 75,000 * 2 hes.)
Product C (4,500 units 15,000
fess: Variable cost @ % 2.50 each18,750 * 3 hrs.}
Product B (5,000 units nce howys
600
; Contribution hrs + 4 hrs. = 150 units) Bala 18,600
Product A (600 Totai
Less: Fixed cost 36,000 =u :
Advertisement 5,000 41,060

Profit 15,250 Statement of Profit


Total
: 15,250 whilewhi re . Products
on advertisement results ini profit: of % a
Spending7 5.690 : . é
results in. profit of, 2 12,7 50 {as calcu lated :
in part (e)]. Thus spen
'
ding more on advorisenene C a 4
ment willwil gene
tore
rate
mote profit. 150
1,500 5,000
25
Problem 5.30 Froduction (units) 22 24
=
3,750 1,596,750
Contribution per unit 33,000 1,20,000
Fro m the followi ng pa tticulars,
al fi nd the e mos ? rofitable p product mix 3 and pr epare
prep. a statement
ent o t- = 60,000"
profitability of that product mix: Total contribution
Profit 96,750
Less: Fixed cost
Product B Product C
Units budgeted to be eted units
vous cost is based on budg
‘Calculation of fixed units @ Z 10 = 7 18,000-
produced and sold 3,000 1,200 A 1,80 0
units @ 7100 = & 30,000
Selling price per unit = 60 55 50 B 3,000
7-H = € 12,000
Requirement per unit: " Cc 1,200 - units @
Direct materials 3 kg 4 ka Total 60,000
Direct labour i 3 hrs. 2 hrs
Variable overheads Pac 213 t 3 Problem 5.31
Fixed overheads 10 tio 2 10 of a company:
d from the records
Cost of direct materials per kg ts 7 4 4 The following particulars are extracte Product 8
Direct tabour hour rate Prod uct A
: ; z 2 = 2 z 120
t 109
Maximum possible units of sales 4 000 5,006 _ 1,500 Selling price (per unit
)
. 3 kg.
: 2 kg.
All the three p prod ucts are produced from the same direct material using 9 the same type of machines and
ni Consumption of material zy 10
z 15
labour. Di izect labour, which
ich isi the key factor is limited to 18,600 hours % 10
Material cost

4
z= 45
Solution Dire ct wage s € 6
“me zg 5
Direct expenses 2
-_ Profitability Statement 3
Machine hours used
_ Particulars Par unit of Products Overhead expenses= = 10
z 65
8B c Fixed 15
z 20
ze
Variable
is € 5.
Direct wages per hour material) when:
(i) Selling price of each product (both use the s ame raw
a
(a) Comment on the profitability Qalto vw
Direct materials @ § 4 per kg. 3 is in_units is limited; >
(i) Total sales potential \W Rakin
Direct labour @ & 2 per hr. 6 in value is limited: —>
- Variable overheads (ii) Total sales potential aad, =>
(#} Total.variable costs
: :
(iit) Raw materiat is in shost
supply;
is the limiting factor. Mac. HAT
8 3 {in terms of mac hine hours)
(iii) Contribution: {i)-(ii} . (iv) Production capacity bil of
ity which is 10,000 ka., and
maximum sales
7 7 as the key fact or, ava ila witl yield the
(b} Assuming raw mate rial out thi e product mix whic h
ws ) Direct
cont labour: hous pet unit 4h hrs. . being 3,500- units, find
ntia l of each prod uct (B.C om, Hons. , Delhi , CWA . Inter}
a Contribution0 per direct laboour
Hankivg ii + {iv)
hr (ii) 2
6.25 , 3.00 ‘od
pote
maximum profit.
iI : I
|
a
eee
Management Accounting
Cost-Volume-Profit Analysis
5.59 f 5.60
Solution (a) . ;
. Solution 2015-16
;
§ tatement of Marginal Cost and Contribution 2014-15
7 4 18 lakhs 2.25 lakhs = 75%
“FS lakhs
2. ae 3 lakhs = 90%
Per unit of : Capacity utilisation % . . *
A
B - — preak-even point in units 512,000
z z 3,00,000 :
(Eee so)
a
oo 6.30- 3.30
120
Sales 100
, $-¥

. . 6-3
= 1,96,000 units 1,60,000 units 4
Direct materials ee
10 1,60,000 « % 6.50
.
Direct waces 15 6 1,00,000 x 2 6
= % 10,40,090 ig
Break-even point in & = % 6,00,000 iH
Direct expen . TO
6 1,80,000 - 1,006,009 2,25,000 ~ 1,690,000
Variable o he ° 4 Margin of safety (units) = 65,000 units.
i
ag
vethead 5 = §0,000 units
Marck (Sales units - BE. quantity) 65,000 « % 6.50
contr 45 53 j 30,000 x % 6
ontribution
‘ginal Cost
59 Margin of safety in © = ¢ zaum ais4
55 = 480000 zg
P/V ratio “ 55% 57.5% .
Calculatian of Expected Profit
z of
Contribution per kg. : of material
. 14,62,500
27.5 33 10,80,000
Sales {Actual quantity « 5-P)
os tial
per machine hour 7,42,500
per unit):
Contribution 5,40,000
: 183 0034.5, Less: - Vani
:
Variable cost (Units « V.C. 7,20,000
B = 3 540,000
Comments: : of key factor i
Si Best position 8j reached when contribution per unit r is maximum.
#. Contribution
3,00,000 __9,12,000
(i) When total sale factor, B is mo fi .
canteibutio S potential in units is a timiting ey +
a larger Less: Fixed cast ——_—_——
con 2,08,600
a re profi
it is making
profitable as i -- ° 2,40,000
Profit a see
as compared tosen A. toses
nt i
if} When total ales ntotential i factor, still B isi more profitable as its P/V ratio
: limiting
value is a
is more than th at : of A. ial in
(ii) When : : Problem 5.33 Office in
Taw material is in short supply, :
more profitable as its contribution per kg. * of
materiat and south with its Head
isi more than that of product B. poly, A is ena
TO
has thre e fact orie s situated in north, east oper atio ns of each
A company mary report.on the

*
capacity isis 8timi i . The man agemen t has received the following sum
(iv) When Production
B is more profitable as it makes larger contrib ti Mumbai . ,
: hour than 4
machine y is timited, bution per
factory for a period : (& in ‘000)
.
{b) When raw material is a key factor ; contributi
Ais mor @ profiitable as its x - a
ee
consumption wt be e 7, oF materia ; is
higher than B uni IF 3,500
es ’ on ee atuf actu i red, total materialmanufacture 1,000 uni 000 kg. fie,
Over/ (Unter
3.500 kg). The lan
) of B.* 2Thus the product six i be . OF: mate rial can be used to
0 units (3.000 kg. +
Over/ (Under) udget
1,000 units of B
s 3,500 units of A and . yo
Budget ——
l m
Probble
” Fro 5,32
7 135 (180)
.
North - he 0
4,10 210 90
On the basis of wi g data, work out th e expected profits i Seeatt on’ 11,450 150.
the folloowin . East 330 (110)
break-even voi
of safety {i the cases) for Gn i).
it
{200}
point and margins etyt {in terms of quantity and vatue in both
. Capac
ev n aT
South | . 4,200 a
2814-15
« and 2015-16 or the years - for the period :
for the company ; 48 4 whole
Prod 2014-15 Calculate for each factory and .
(C.A. Inter)
(i . 2015-16 : - (ii) break-even point 7
Toduction capacity ts)
2 lakhs (i) fixed costs.
Production sale tn (nu n a
“48 lakhs oon ch . tion “4
7 lakhs Solu RA
Selling price per unit its) : - 76 a-25. z "000
i!

a
. ‘
Varia ble cost per unit 30 Calculation of P/¥ Ratio

|
‘ %3 TZ 3.30
deprecia: tion}
(j luding : aa

er:
“Fixek d expex nsds (inc j

.
North: Actual 400 ___ 180
wre
. (EC.W.A. Inter) Add: Under budgeted 1,500 315 \
- . : Fa
Budgeted :
x 100 = 45% t
315-135) ang = 180 |
Difference in- profit
i P/¥ zatio = — 1,500 - 1,100 400
; in sales
Difference

ng
Management Accounti
Cost-Volume-Profit Analysis
5.64 "5,62

. = ‘000
East; Act ‘Solution
= 50%
in 2017 P/¥ ratio 50%
ess: Cver budgeted 0 “ zati o = 100% - 50% =
we
Budgeted . 1,300 12020 Variable cost x 50% = <4
(00,000.
30 = Z g,0 0,0 00
Variable cost in 2017 iab le cos t in 2018 is % 4,00,000.
Thus var
ZV raid = Isa * 100 = 60% sale s quan tity has not cl hanged.
In 2018,
= 37.50%
South: Actual In 2018 P/V ratio = 62.5%
ratia = 100% - 37.5%
stUnder budgeted
Add: a200 110
m0 Thus Variable cost
4,00, 000
vr : io
ti 1.400 440
sure 2 F 6,40,000
ti} Thus sales in 2018 = "62.5%
nt tao = Soy 106 = 55% to contribution.
fixed costs 1 s equal
At break-even point, = 78.125%
alculation of fixed cost at 100% - 21.875%
rs sales
In 2046, Break-even 125 % = % 5,00,000
ixed cost = (Actual sales x P/V ratio) - Profit
s 6.40,000 * 78.
(ii) Bre ak- eve n sale
p/¥ ratio
a B.E. sales *
(ii) Fixed cost .
a = {1,100 « 45%) - 135 = 5,0 0,0 0 G x 37. 50% = = 1,837,500
fast = (1,450 « 60%) - 210 = 660
outh = {1,200 « 55%) - 330 So age cost
Problem 5.35 yelume is 4.000 units, aver
40 er unit. When
s, average cost is€ pr ofit-volume ratio.
cal ; Total Fixed cost = .
(i) When yolume is 3 ,o00 unit s. Fin d the
point is 5,000 unit company
culation of break-even point (in % '000) = ig Z 3.50. The break-even it can sell. 4 and &. The
two products that unit
Ki of Z 2,0 0,0 00. It has
h
The unit contibution is ¢ 1 per
(ii) ABC Ltd. has fixed c osts of A to 1 unit of B. ak-even
2 rate of 2 units an d B would be sold at the bre
B.E. point = med cost sells these products at many units of A
P/V ratio unit for B. How
jor A and % 2 per
is % 200,000.
point? | costs when profit
North = 30 ty is 40%. of sales, find fixed (B.Cam. Hons. Delhi,
Adapted)
454% = 800 (iii) HE margin of safe ,

Fast SS ao Total cost


60% = 4,700 Solution
Average cost
Output € 42,000
(i) X 4.00
3,01 anits =z 14,000
South = a0
55% = 600 4,900 units
= 3.50 __
= 2,000
— nas T otal 2,500
Difference 7,000 unit 100 0 uni ts = t2
anit = & 2,000 + = = 6,000
Variable cost per - (30 units « = 2)
00
ts * B2) = % 6,000
cost = 7 12,000
2% 14,000 ~ (4000 uni
Total Fix ed
POR itLtd. nahas furnished
i the following data for the two ‘years:
* or :
cost
= ed cost + Variable units « = 2)
At BEP: Sales = Fix
2017
= (6,000 + 5,000
2018
Cost at 5,000 uni ts
00 = %,16,0
ween Ratio (P/V ratio} on or 16,000 = 6,000
. Fixed cost
___ argin
Marg 2
of Safety sales as a % of total sales | 40%
ionfs é 5 BEP ‘ PFY ratio
are
21.875% PIV ratio
whe There
romper
has been
ouldt substanti
walaat' i savings
savings ini ; the fixed
i cost in the year 2018 due to th
6,000
. 3 = 37.50%
ales quantity level of 2017 in 201 i tad wie P/V ratio = 16,000 §
quired to ae
You ate requi calculate the following : , eee me |
or
A B Total
(i) Sales for 2018 in ZS , 3
(ii) Fixed cost: for’ 2018
|
a a
(i?} Sales (units) ti €2
(iit) Break-even sales for 2018 in Rupees. _* (GAL Inter} Contribution per unit z2 4
T2 _——— __—
Total contribution ——
Management Accounting
Cost-Volume-Profit Analysis ce- _— i
5.63

aru ae
5.64
i H
a He
site}. = < 4 - €alculate : t reduction in sales volume.
Contribution per unit {compo h 10 per cen t inc rease in selling price with a 10 percen erial costs Hl
(i) Profit wit a 10 per cent rise in mat
cost
Total fixed: cost 3,
iev ed to mai nta in the original profit after (CA. Inter) i
ed = % 2:00,000. (ii) Volume to be ach
pric e per unit. ive
y bud get ed sell ing
at the originall
BEP {composite) = Rs. 2,008,000
rn » 1,50,000 units. Solation . . units = % 1,750
= 7,00,00,000 + 40,000 _
EP Selling price per unit = 1,925
{in units) in the rati New selling price = 1,750 + 10% = ©
s
= 40,000 - 10% = 36,000 =unit© 6,93,00,000
atio of 2:
1
« a ; New sales volume 36,000 unit s « % 1,9 25
A = 1,50,000 New sales revenue =
-000 x J = 1,00,000 units @ lakhs
B= 1500 I {i} Calculation of Profit 693
.
:
50,000 units
~
50,000 x > = Sales 396 4
Alternative Method Less: Variable cost (440 ~
10%)
Sal 4
reion 297 :
At BEP. let uni
en Bex Contribut 202.50
nits = p2x
of Ats Less: Fixed cost 94,50
Profit
(2x x 1) + &
x2) = 2,00,000 + 0 {ii) Fi guies iinn % lakhs
4 10% = 290.40
00,0,000
4x = 2,0 4 Hew material cost = 264 + 466.40
New variable cost = 290.40 + 52 4 124 =
x = 50,000 (units of 3} 4
New contribution = 700 -
466.40 = % 233.60
funi
2x = 1,00,000 iP rat 233.60
m4) P/V

ew P/V ratio = “700


ie Margin of safety = 40%, Th :4 view
(iit) Thus 5 60% of sales
= ove ,
, 202.50 . 50
5045/7+57. .
aigin of safety (W/S) = Fy
Pro it
+ Prof
Fixed d cost t +e es Sao £ 779.10 96
: _ Fixe 223.60/700 79.1096 lakhs
| | P/¥ rati o
Bee . aratiio
4 Required sales = 44,520 cats.
ws ® PN to soft | = Z #,79,10,960 + 1,750
P/V.irattio Required gales ip units
ae ieee _ Pro/f = aratt
;
Fixed cost / P/V. ratio
Problem 5.37
_
o Of sales
sede q ae a firm is as under: en
or 40% 23 _ _Fr ofit_ —__20,000 .
The comparative prof
it statement of two
o
quarte
e
rs of Cuter
Quater II
Fixedcost ” Fixed cost -
60% _The comparative PT Quater I
;
:
2,500 70S
.
Fixed cost =
60,
000 = % 30,000 Tits sold” = 37,r 500
??
Probl
1 Disect materials
e seann ;
ent manufacturiring
t /
Direct wage®variable
. :
in resp
A automobile
ber ctu compan
pars different models of cars. The budget 'n, Fixed and = 75,000
95,000
Qm nn 118 for the month ofSeptem or
respect of
factory overhea = 2,75,000
?
s
udgeted output ~ 40,000 unit Sales 65,200
= 40,000 en
Zin lacs Profit 00
- — -
sed by 20% . The re was a saving of €, 5,0
Variable costs: . ct mat eri al pric e has inc rea
the sam e, iet erm ine
in the second quarter, the
dire ained
Materials 264 second quartet. The othenta r costs and selling prices of rem
profit as in ‘the Ist quar
ter.
Labour
mL
59 in fixed overhead in the
uld hav e been sold to mai in the same amount
(C.A . Inter}
Direct ex
. the quanti ty that sho
2 440 .
Specific penises 4
ea fixed costs $0 . *
located Fixed costs Solution
112.
80 202.50 .
12
+ 2500 units= = 35
.~~
Basic calculations:
= © 87,500
| Totatt costs
Pra
642.50 Direct material percunit, = % 62,500 = 2,500 units= 25
- Sales _5E0 2. Direct labour per unit + 2,900 unitss & 110
2100.09 © = % 2,76,000
13. Selling ptice per unit = % 50,000 + 2,500 units= <
20
‘4. Profit per unit
—~
|
ae Gost-Volume-Profit Analysis
ene a
peep
Management Accounting
eB 6.66
- . BS
4. Variable overhead per unit Change in tatal overhead
Change in out - Solution
put
Statement of Marginal Cost and’ Contribution
~ £495 ,0
+ 5.000 ) -00
75,000 25009 (for Year 2}
3,750 - 2,500 unit “0 7 z 20
6. Fixed overhead in ond ® per kg.
Tatal overhead Warter z 200
Selling Price (% 20,000 + 100 : kq)
Less; Variable o Verh Less: Variable costs—Raw material 108
*
Fixed overhead , ead (3750 units @ = 20) ee
Direct wages (30 « 150%} 45
22.000 20* 165
Sac in end quarter, Variable overhead
20,000 ~
5: vgs *
atement of Quant . ;
; Contribution
(To maintain th antity to be sold in 2nd Quarter
a Profit (€ 2,000 - 100 ka.)meet Fixed cost =15 :
ae © same profit per unit as in Ist Quart Balance contribution to . — 4
- e) . J
& Selling price = Total Fixed Cost = % 2700*
iess: Direct . 110 kg.
Quantity to be produced and sold = % 2700 = 15 = 180
Direct ween (F 35 + 20%) 42
ges * Working Notes:
contr Variable overhead . iM and Fixed Overhead
Calculation of Variable Overhead
per unit tos — a8? t _
Prof ion
rean83°nbut 2 -
c ained ;
* de maint
for fixed
Bal ance- font 20; Total overhead i in i 5,700
'
Total fixed ed cast eg t em tt 5 3
‘otal overhead year
- 300
Add: Saving in fixed overhead
20,000 6000
No. cf units to be sold = <——o costt
fedue 26,000 ;
Balanos fixed cost pu. = = 6,667 uni
Less: Fixed overhead in year 1 5608
3 ' nits 1,000
Increase in overhead,
Frazer Ltd. Variable overhead‘ per unit = Inctease in overhead = Increase in kg.
Manufactures and sells a
remained unchanged during the Product, the setting Tice - ’ = % 1000 + 50 kg. = = 20. :
Past two years. The following and raw material cost of which have
iQ are the relevant data : kg. « & 20 = & 3,900.
. Total variables overhead for-150 kg. = 150
Quantity sotd (i Year 1 . .
| Total fixed overhead = & 5700 - 3000 = & 2,700
8
Ba pea
100 Year 2
'
Sales vz mo
z
150
: Problem 5.39
he Wau: TT

.

A Company manufactures ‘a product, currently utilising


80% capacity with a turnover of € 8,00,060 at
. .

9 oop
2 ‘
?
Direct Wages % 25 per unit. The cost data are as under :
eqials

3.000 ?
Factory overheads unit, Semi-vatiable cost (including variable cast
Material cost % 7.50 perrunit, Labour cost %.6.25-per
‘ 5000 ;
1,80,000. Fixed cost % 90,000 upto 80% level of output, beyond this an additional
Profit 5,700 of % 3.75 per unit} €
. ; 2.000
Duzin b ‘ 2,550 Z 20,000 will be incurred,
overheads 2, direct wage. rates increased by 50%; :
+ Dut e .
. , Calculate : ‘
Required: - oe . / there was a Saving of & 300 in fixed factory fi) Activity level at break-even point...
.
What , quantity
7 Number of units to be sold to eal rm a net income
of 8% of sales
(ii)
Y (in(i k s-} the company sho uld have produced we sect
the same amount i (iii} -Activity ‘level needed to earn a D rofit of % 95,000:
of net to maintain
profit per kg. as it earned during year 1? sold in year 2 in order point is to be brought dewn to 40%
(Working te form
‘ (iv) What should be, the selling price per unit, i F break-even. {C.A. Inter)
Part of answer) activity level? . . : : .
~ , : .
~ (CA. Inter) ent
Solution
Basic calculations : i
units
1. No. ofrunits at 80% capacity = 8,00,000 + ¥ 25 = 32,000
«! 2, No. of units at 100% capacity = 32,000 units = 80% = 40,00 units

sl GUI RBA “cL RO oar , .


Cost-Volume- Profit
Analysis
Management Accounting
5.67 5.68
3- Fixed cost Component
in semi-variahle cost
aoa ge l
= 1,80,000 - (32,000 acquisition of 30% additional Space and plan al will
units x 3.75) = 60,000 It is proposed Eo increase the Capac ity by the
Fixed cast upto 80% verh eads biy my, 00
¥°1,00,0 per annul,
ca pacity = 90,000 + 60,000 Tes es lt | in Ta
increase of fixed over
Fixed cost beyend 86% = F 1,50,000 BY hart
chart an d determine
2 7 from the chart at what capacity
P 0 t th 2 f eregoir ig figures on a single break-even
Capacity = 1,50,000 + 20,00
4, Variable cost per unit 0 = Z 1,70,000 will be produced after the extensions have been made
= 7.50 + 6.25 4 3.75 = 2 17,59 utilisation the
h sam:ne profit as before
;
(B.Com, Hons Bethi)
Contribution = § - Yo ~
= 25 - 17.50 = 27.50
Profit at 80% Capacity =§- Solution
V.F
= © 800,000 .
- (32,000 units x 17.50
) - 1,50,000 = % 90,00
(i) Calculation of activity 0 t
level at B.E, point ube
F Rs.1.50,000 ~ 144 fi
e = ys . we |
ne 20,000 units
e ! o*

Capacity ievel = 20,000


units = 40,000 units =
50%, 24 es
o
ot
aa
(ff) Units to be sold to earn a oh “
net income of 8% on sates >
ond ete “ 1
Let thenumber of units to be 7
sald = x 4& 0. or 4
i.
ge
“cet
25 = 17.5x4 1,50,000 + 8%
oF 25x Oe o “e i1
=
25x = Sy y 1,50,000 oy aoe”
ee of “
+ 2x of EP |
i
x = 1,50,000 + 5.5 iw
~
“ ee, B
. Ca
3
*
” wo 1
\ ~
x = 27,273 units, a o
we
o
~

(it} Activity level to earn ot ae !


a profit of 7 95,000 _ o1 . New Fixed
I
Profit at 86% capacity is cost line
will have to be utilized.
F 90,¢00, Thus in order
Thus the Fixed cost will
to earn a profit of F 95,0
00 Capacity beyond 80%
Bopanna
be = 1,70,000. B 4
F+P o Fixed cost line
= eee Bs000 = 35,333 units
Required sates ~ = 1,70,
: 1
I
24 1
. 35,
= 333 units «
Activity level i== 40,000
1
units © 100-- 233%, 1
(0% 88.33% J
: 7 . 1000020140
L T
(iv) Selling price if pf. ° 20
point is brought down 82° «40 60 80
Let the selling to 40% activity level ¥
price = x 116.67%
= Units at BLE. Point Output (% capacity)
= 40,000 units x 49% =
16006 units
F 150,000
B.E. Point = ~ = a
x-17,50 Fig. 5.14. Break-even chart.

1,50,000 i xe a
16,000 units = ¥—1150 4 l akh Ci ing
indica
. ky. ‘art, , th w total L cost li ine statts iirom emPp point, 1
5.14
4 of break-e ven cial : e Te
In the Fig. the percentage
i} = 2 kh:
kakhs. In order to determine
at @.
presert! {1 S$
x-1750 « 150,000 75 the to al cost ' plus profit line sta ts ne.rom .7
tio '
the the 5 sa he profit and " with increased cost,
acity
capacity earn
16,060 Where
8 plus % 2 iakhs for profit} tuns para tel to the total: castintlinete
and xis.
lakhs (i.e, z 5 lakh for fixe
fixed cost ; :
from this pon to X-axis
8x - 1405 75 re] drawn
this total cost plus profit line intersects
1 sales line, f a perpendicula r
the
it as before "
8x= 215 hi: hi h capacity
4
utilisation 67% ta a earn the same profi
11 6.679
ar xe 26.875 | which shows the percentage
Thus selling price =
% 26.875 unit. Calculations
Pro blem 5.44 0 i e cost - Fixed cost
Present profit = Sales ~ Variabl
The fellowing figures relate to a 12,00,000 - 6,00,000 - 6,00,000 =% 2,006,000
manufacturing company : i e cos t
Contribution = Sales - Variabl
Ann
: ual sales at 100% effective
capacity z = 12,00,000 - 6,00,000 = = 6,00,000
Fixed overheads . 12,900,000
Total variable 1,900,000 = % 500,000
casts 4,00,000 New fixed cost = 2 4,00,000 +

6,00,000
Cost-Valume-Profit Analy
cen re front Analysis Management Accounting
+ 5.40

‘ . . sales at various assumed levels af


Break-ey,
cost + Desired profit
en point (in % capacity) = et Contribution When cost and seliing prices are
given per unit, then total cost and
(See Fig. 5.15). For this purpose
~ 100 calcu lated to get neces sary data for pl otting on BE. chart
outpu t are
as shown below:
Rs. 5,00,000 + 2,00,ee000 }
2 a table has been prepared
assumed levels of production
oe
a
6,00,000 x 140. 7,00,000 vario us
6,00,090 * 100 = 1165 Cost and sal es at

ne, sae
J
vie FL. 18 SP. & 26
1l6=% Units SBF 26 SP
162% i terms
«Mm of expanded capacity = 3 @Ttipu tz z z

139 100 = 89.75% 6,00,000 4,00,000 3,60,000 5,20,000


Froblem $.41 20,000 2,00,000 4,00,000
400,000 8,00.006 8,00,000 720,000 10,40,080
40,000 4,900,000 10,80,008 15,60,000
10,00,000 12,00,000
6,00,000 4,06,000 209,89,008
60,006 16,00,000 14,468,000
Selling Price per unit 4,00,000 12,008,006
80,000 8,00,000
Direct:
Dire ct lan
material cost per unit
i cast, TC = Total cost
me ad0ur cost per unit : ¥C = Variable cost, F C = Fixed :
a break-even chart as follows
Rae overhead cost m3 These ‘figures are plotted in units
per unit unit 8.6. Point = 40,000
per
Paden level of output ry When selling price is ¥ 20 units
per unit B.E. Point = 50,000
tou getateed wrec When selling price is © 18 units
eover y Yate of & ixed xed ove se,0 a per unit 9.£. Point = 25.000
rheads cost per unit 3 When selling price is ¢ 26
rs “:
|: Verification
(a) Draw a break-even char
t an & sho
samel chart
(6) In() theI sel show th w the break-even point. B.E, Point = F + C
= 40,000 units
ice ie impac t
on break-even point When 5. is % 20 = 4,00,000 + 10
: = 50;000 units
i : ‘ 48 Micreased by 39%, When S.P. is ¥ 18 = 4,00,000 + g
26 = 4,00,000 = 46 = 25,000 units
selling price per unit is
decreased by 10% io. ne When $.P. is %

Problem 5.42 summarised


Solution 1,50,000 units per annum. The
(C.A. Inter) product with a capac ity of
A company manufactures a single
year ts as under :
profitability statement for the t z
15,00,000
15 per unit
Sales : 1,00,000 units @ =
2000 Cost of sales : 3,00,000
Direct materials 2,00,000
fin % ‘o00)

Direct \abour 60,000


Variable
Production overhead 3,00,000
Fixed
1,50,000
Administration overheads {fixed) 94,000
overheads: Variable
Selling and distribution 12,50,000
Sates

_1,50,000
ne

Fixed
oS
Sa

2,50,000
Profit
and

evalu ate the follo wing options :


You are require d to sales, if the packing
earn a target profit of 25% on
Cost

(i) What will be the amount of sales req uired to


is improved at a cost of 2 1 per unit? subject to providing
hasing 30,000 units per annum,
400 Fixed Cast Line (ii) There is an offer from a large retailer for purc er, in this c ase there will
of % 2 per unit. Howev
a packing with a different brand name at a cost any way, affect t he company’s
expenses. Also this will not, in
be no setting and distribution price from this additional offer?
the break -even :
20°25. 40 50°60 80 i907 0 existing business. What with be increase from the present
made on advertising, the sales would
{ifi) If an expenditure of % 3,00,000 is 3 price of % 18 per unit. Will
that expenditure by
000 units at
Production/Sates {in "000 units} level of 1,00,¢00 units te 1,20,
justi fied? capacity utilization. Will the
be 100%
Fig. 5.25 Break-even a by % 2 per unit. there will
chart, (iv) IE the selling price is reduce (€.A. Inter)
fied?
reduction in selling price be justi
Cost. Volume-Profit Ana Management Accounting
lysis
SES

Solution oo A? 5,42

Basic calculations KEY TERMS y of


Total fixed cost tool of profit pla nning base don “the stud
P) analysis is a powe rful
: -_"=
price, quantity and mix.”
ne + 1,50,000 + 2,50,089
cost-volume-profit (CV . yaniable cost, sales
anne

Total variab| = 3,00,0 = % 6,00,000 chang es in fixed cost


+ 2.00
09,0
00 .0 00 + 60,000 profits of
Variable cost per =f 6,50,000 the effects on future que to study the CVP
relationship
a widely used techni
unit = % 650,000 = 1,00,000
unit. meee is. It is
Revised yari iable ¢. . = 2 6.50 Break eve n ana lys ble cast.
+ 1
Ost per unit = 2 6.50= 1,600,900 selling price and v aria
Seliing price = 27.59 differenc e hetween calculating the break
eve n point
per unit = F 15,00,000 . Contribution is the hel p 5 in
* 100,000 unit
eZ 15 the rati o of con tri bution to sales. It
P/V ratio shows de sired level of profit
, etc.
P/V ratio « S24, 1527.50 and also calcu! lation
of sales for achieving
is ne prof it and no loss.
15 = 50% nt is that leve l of sales wher e there
try:
“'? 1
Calculation of required sales Break ever poi tion of break even
analysis.
Break ¢ yen charts. It
is a graphic prese nta break even sa les. A larg
e margitt
and
between 4 ctual sales
Let i equir
4 ireed sales =x, at target
g 2 rofit is 25% af sakes
is € he dif fer enc e is a seri ous fall in its sales.
Required contri ntribution Total & Margin of safety. It stil l mak e profit when there
= Total Fixed cost + 2 't he business cat
line an a total cost
of safety indicates that intersection of sales
5% of x

C ~ _ 5,00,0 25% of x
+ 00 is an ang le that is formed at the itabilit y.
Th us P/V rati.o = <7 Angle of incidence . Tt
nce in dicates higher
prof

x even char t. A large ang le of incide ts the vol ume of output at
dertaking wh ich limi
line in a bre ak
e acti viti es of a unl
or in th
+ 2580 x
x = 290,00 Key factor. It is a fact
time. alternatives is
50% par tic ula r poi nt of cost of pTo fit of the
. a
a lev el of out put where the total
0.50% = 6,00,000 « 0.25% Cost Indifference Poi
nt It is
9.25x = 6,008,900 equal.
x = 24,00,000 ESTIONS |
2
| EXAMINATION QU

Thus
quired sales ==F ¥ 2d.
TeQUI

(i!) Calculation of break “even 00,000.


price for additi . Objective Type Questioncs
by : exist i) nal 30,000 units
tio
breaks Fixed Costs
are being absorbed
ee
in brief.
ts. Give reason
in activity
ven price, Fixed cost iv and thus onl y Teviised
{rue or False “Statemen
js variiable cost Ww will b é its
o = Fixe d cost.
Present variable cost
1, Break even sale s
in & x P/V rati Jume does not cha
nge.
. s that sales yo
lysis assume s is aqual to profit.
Less
Add:
+ Vayj
Tiable selling and dist. overhead ~% 6.50 2. Cost volume profit ana t contribution 0 n additional sale
pu. _) 090 eve n poin t ig reached, @ fter tha
3. When break-
R - Special packing cost (+) 2.00 ratio « M/S ratio
= Prof it.
mee variable cost 4. Actual sales x P/V tribution.
—_~ een sales and con
us break-even orf
ional = % 7.60 5, Profit is the diff eren ce betw i
or sales ma y be represented.
iii tse as price for addit 30,0‘ 00 units 7.600
= = 7.6 6, In a break-even char
t, on K-axis producti
o n
tribution.
(iit) Justific atio
i n of Advertising co 15 equa l to con
fixed cost
7, At break-even point, by lower ing fixed
cost. cast
Now contribution = 5-V
can be improved s line at d variable
=atig ~ 5 » % 14,50
6.50 g. Marg in of safe ty
for med at the in tersection of sale
denc e is
ribution = 1,2
'
chart, angle of inci
20,000 units x &
centri
9, In a break-eve n
Total
11.50 = 2 13,8¢,000
fess: Fj
5S: Fixed cast levels of sales.
line. us
6,00 it of joss at ¥ ario
Advertisement cost aoe t doe 5 not show prof in sales value.
10. Break-even char the re ig ac han ge
nt eee

’if
90,030 o does no t change
11. Profit volume tati also reduced.
Profit . aon uce 4, B/V ratio is False 6, True 7. True
Additional expendit is j 4-80.00 12. When sell ing pric e is red
[Ans, 1. Tru e, 9, False 3. True 4. True 5.
e 11. True 12. True]
ure on00 adverti,
cen

Le, % 4,80 000 less % 2,50,0 e 10. Fals


existng ng uate
existi profit . eee eats ause it results ini additional
onal profit ofZ 2,30,000
g. True 9. Fals
(iv) Justigi
fleation of pri
by T 2 per unit .
Price re i
. evetion ak-even point is <....
Contribution = § - y Fill in the blanks: 40%, then the bre
6.50 = % 6.50 fixed cost is 25 lakhs, P/V ratio is l sales are %....--
4. When tota
Total contribution
at 100% Capacity (1,50,000 units « 2 6.50) 5 lakh s and M/S is 37.5 0%, then unit is %...-..
is
% 9,75,000 2. When B-E, point 0, contribution per
Less: Fixed cost
en sales ate 10 00 units, fixed cost € 2,00 rea se fra m = 4,000 to = 2,000, P/V ratio
6 0,00 3. Break-ev 000 while profi t inc
_6,0 from 7 20.0 00 to 40,
i reased
. 0 4. When sales i nerease
As profit has as inc b Profit
3,75,000 0% of sales, the break-
even point is <
¥ ¥ 1,25,000 fro ra € 2,50, 000, the price 1S. vce
i reduction is justified
i: 5 % 2000 and variable cost is 6
5, Fixed cost
TS Tuulne-Pragi
t Analysis

Management Accounting
7. Fixed cost
% 4,000,
3. Profit is 10
% (d) Greater, iower or equal depending on certain
. Break-even (c) Equal to absorption costing
po int i factors.
0. When BE. point i
- . the fixed
cost . _ Product costs under variable costing includes
+ actual sales are & i (a) Prime cost only (b) Prime cost and variabie cverhead
fixed cost is 2 (d) Prime cost plus fixed as well as variable overhead.
{ce} Prime cost and fixed ovethead
the sates is qa
. 13. Cost behaviour pattern in a company is as follaws:
2: 4 ° : . + Machine hours 5,000 7,000
Multi : e GD ‘a,
itiple Choice Question
Cno0se 9. 2,000; 19, 25 ado: 7, 20%: 3. 254
the « s me V1. © 20,000; 32 T 37.50 .
Cost (%) 25,000 30,000
correct ‘ ’ Which of the following statements is true ?
1. To obtain the break on for the foRowing multiple cho a (b) Fixed cast per unit is @ 2.50
. {a) Variable cost per unit is 5
a Variable cost per ~even poi ik rupee sales value, total
unit POINE ice fen
(c} Total fixed cost is < 12,500
(d) Total variable cost is ¥ 17,500.
C} Fixed cost per unit_ £6) Contripar; xed costs are e divided
diy; §¥ equation ?
2. The b Ntribution mara; . 14. Which is a correct marginal costing
(aj P2S-V-F {op} S-V=F-P
1a) hee
fre 1s m3 Point
rofi Is the point at winch:
which (2) Proit/vtume ratio eM (ec) F-L=54+¥ (dj) S=ViF-P
{¢} Total rey, PrOnt no loss fixed cost is % 30,000 and P/V ratia is 4 O%. What is the profit?
3 : enue is ¢ qual tp tota (b ) Contribute
oe . 15, On sales of % 2,00,000,
Margin of Safety is referred to a cost (2) A of the abo 18 equal to tatal fixed (a) Z 12,000 {b) % 1,20,C00
: Cost
(c) = 80,000 (a) % 50,003
B (OJ: :
6 (a): 7 (dp
4 (4) £§ (by: s 11 keys te (By;
9 (ai 10 (eh
fans. (1) (d) 32 (@) 3 fe):
13 (ce): 14 (a) ; 15 (d)j.
situations, indicate whether P/V ratio will increase,
assumptions: Considering each of the following as independent

talline
°
(a} The Sale
$ Mix decrease or not change.

inci
of f the B Toducts
(b} Ir ventory Is consta 1. An increase in sales quantity.
qua thities nt
(c) > Material Prices
pr; and lah
char ge du
;
ring the
year 2. An increase in fixed cost.
(@} The behavioar of hot wales in variable eost per unit.
donot change 3, A dectease
5, Which of Sand vari 4, A decrease in contribution.
the following
(2) Total fired oon g isj NOT an asslimption
variable cost js s |jlinear ¢
9 breakeven eashout the relevant range S. An increase in selling price per unit.
:
Nalysis .
{c} General arice |
evel Cogs @ Total, Vari 6. A dectease in fixed cost.
6. When Sale change able cost dee variable cost per unit.
s Increzs s mat ch,
Product mix does nat change, 7. A 10% increase in both selling price and
Tom 10% decrease in sales quantity.
8. A 10% increase in the selling price per unit and
¥ 40
in,000 te t 50 000 ange
by % of5,000 s and 50% decrease in fixed cost.
? the P . 9. A 50% increase in variable cost per unit
- OD break-even chart, x. (2) Cannot » 10. An increase in the angle of incidence.
vO ratio is
axis represents ; 4. Decrease; 5. Increase; 6. No change; 7. No change:
(4) Sales in x ‘ ® computed. [Ans. 1. No change: 2. No change; 3. Increase
{e) Cutput ig units B. Increase; 9. Decreas e; 10. Increase]
(8) Sales in units
Essay Type Questions
8. Wher Sa
les volume increases (¢) Any of these,
(a) Break e vi . ’ analysis ?(8.Com. Hons. Dethi)
Point increases . 1, What is break-even p cint ? What are the limitations of break-even
©. The att tas 5 * increases
(¢) Total (B.Com,, Mysore)
(4) Total Profit in 2. What is P/V ratio? Explain its uses. ©
ernative that w, (4) All of these. "reases 3. What do you understand by (a) Break-even point, and (bi Break-even chart? (B.£0m., Kerala}
(2) Decrease Im. selling:* ould decrease c ; i N ! (B.Com. Hens., Delhi)
price Sntibutio per unit the most Is, A a2 4. What do you understand by break-even analysis? Explain its uses.
{c) Inerease j (B.Com, Andhra}
W selling price (b)
20%
5, What do you understand by contribution? How is it related to profit?
]
0. An MNCrease
- Increase
in Variable
of the following on break-even: point and profit volume ratio:
{| n fixed ¢
Ost results in d ; cest 6. State briefly the effect
(4) Decrease in vatiable cost
a) Increase ; . (i) Reduction in variable cast ratio.
(c} Increase an margin of safety (ii) Increase in total fixed cost.
11. When there . break-even point ~
ia Increase in Profit/vol (iti) Increase in price of the product.
& na apeni Increase Mj co
ntributilume
on
-ratig
, (iv) Increase in quantity of sales.
(B.Cain.)
+ Profit under iy argina (M.Com., Kerala)
l Costing wilt 7. Explain the ways by which B/V ratio can be improved.
{0} Less than In abso be 8. Explain the following terms -
rption costing (b) P/V ratio
(a) Key factor
(d} Angle of incideace (8.Com.)
‘(c) Marginal of safety
Cast-
ost-Volunte-Profit Analysis
24,000
9. What is meant
n by break. -@VerL analy.
Fixed overheads on direct labour
alys: 1S. Dis: : US.
i S $5 Th Pp tio ho fF thi &
aS51,
10, "Break even te technigq
he u
que ( BE oOHit + He ONS.
4%
vi analysYSIS
is 60%
1§{ funda menta a as @
fits.
Variable overhe
i ll ¥
e the net pro
ona ae Delhj
the conc ept,
7 statici anal ¥ sis.”
5 Dis cuss and
me, determin
& ex‘plaini
“Break
-even analysis i
. . the li Lm: Ai t ation,
Trade discount lu {C.8. later)
11,
si fi
is based on a simpli a, Inter = of
ev en vo
which is unreal} (LemExpla; 4,800]
ea k-
i
model of a bususiness abov e the br (i)
give plified
istic” 7 If sales are 15% and
20%
Profit 3,500,i) &
Plain an, dl 3000 units,
2 (M.Com, } your vj [ans. B- ep.
.
. {a} Break. -8ven analysj
ysis assumes news, information: z
Axed. Bri that variable co revenues are linear You are given
the following
4 2d
- Briefly describ assumptions and costs are 5,
these and that fixed 12
be two break may not be realistic . pet unit
13, The a there Points? Discuss . . . Selling price
is. alwa P/V 96,009
fect ofof price reduction
a Explai is . ys to red uce and shDethiy q .
the ma Tgin safety.” xplai few, point Hons.,
the break-even oon, Orter Yariable cost
14. Explai plain er
and abject veillustrate
with
break-
eee igures, (hc
om., Ban Total fixed
cost
Significance of 2 chart and t
tea
een pont to chchange. .
even
ihe
15, Enumerste
and state
Hse cane
he factors which would

Calculate :
and value. be % 4#.00,000 (B.Com. Punjat)
to improve the margin of
(C4 Inter) |
rate th steps requi
@ a] Break-even ‘units of safety when sales would % 1,50.008]
j sa “ 4
16. What
expan . ment by required Cideni ce and
margin: of city.
or
ow these in a b
“A. Inter } (ii) Profit and margin
(i) profit = 64,000,
Margin of safety

units. z 2,40,000
teak-eve

bein Bethi} [ Ans. {i} 12,000


(B.Com. Hons... = 20 pet unit. = 10 per uni
t.
Practi ical Questions
af Sales 10
,000 units @
k-even poin
t z 40,000
1.ww Pind out the brea Yariable costs
tal)
Variable cost Fixed cast (to Point in units
- z - Find out (} BE. -
d}
wee cost z $00,000 . ti) Profit
earned at Z 60,000 (B.com.. Adapte
3,00.009 earn a profit writs}
et Profit sequited to (iit) 11 ,0 00
z 1,608,000 5 (iii) Sales Gi) % 50,000;
Sabir
~ [ Ans. (i)
5,000 units: {
2.00.00, °
Ratio = 50%
% of gales. P/V
. 7, You are given: 0 wh ic h repres ent 40
2. 2Acom 730.0
tn ¢& 7,50,000} ty » & 106,00
50 Daily annually manufa Margin of Safe

a product, the selti [Ans. a)
is eid eper andunit.sells 20,009 0 unit units of B.Com., Osmani
t,
{b) Fixed cos
; ysisProfit
The onal earnedof 20,00
of cost Find out: 5, 20 0)
sang Pree of whic | sales {d} Profit
*
& 255 i>, (dt
,900 units is : {a} Break-even *,
(0) z 7,500. (c) and fixed
aterials cost
z 3,00,c00
i
{e) Tot al sal es, and “ay,% 15,000, The variable cos t pet a aris on€ 14cost.
Labour Cost anits. 15%
:= 1,090,000 of 4,56,000-- ch a BY afit of
¥ Overheads
0 u (Bee:
.
4,00,000 (50% variable) | CEA company b dget
for a producti
co mp
on
any fixes its
selling! price
to fet
to compute u cost is 7 2
pe 1 unit. The
point? en point
the break-even the break-ev
units and i
1) Break. : is
What g price affect
(i) Salts ty entsa profitunitsof pues
% 3,00,600.
{a)
(b) What is the
profit vo lu me rat io?
, how doe 5
the re vi se d sel lin
earn 15,000
(tit) Profirofit towhen g price by 5% sates at the
: ces its gellin ould be the
Yow: are sold. (cy if ie re du
lu me rat io? bu dg et. what sh om . Hors. Delhi)
and the profit yo of 10% is gesired more than
(f 10,000 : the (B .C
re ai owing unitsi[Ans. 6,C 00]
* () 10,000 uni
given the followi ng information: 50,000;
or % 5,00,000 (ii) @ 12,5000 (iif) 4 00,000] f increase
P /¥ ratio 49
.9% (d} % 34,9
(4) TE a profitprices? uni ts, ling price
BEP 86,207 cos t and gel
reduced (b) 23.98%. (c)
,
t = units If the marginal of Safety’.
Fixed cos . {Ans. {2) 68.182 during the
yea 2904-05.
amount of Margin
es 4,00 of Z 30,000 find out the (CA. Jnter)
earned a profit
BLE. Sal
2.000 g. A compa! ny 2 10 per uni
t 1 espectively.
fans. © 150,000]
Profit on
are Z 8 am
li ing pri.ce (per uni
Selell
t} of a product
20
= 49,000,
Calculate (a )} Sales
Fixed cost *
le ar d mary g a nal st
cost oF of sales
b New BE, Point ifs e Lling
wh en sal es = z 2,00,006, .
nt is ¥ 49,000
Price 1 1s re duced
fit
(i) Ascertain pro
b y 10% O-
Break-even poi
‘B.Co
{ a m s Dethi }
[Ans Sale: sz 1. 1,6 0,000. 000 , Int er}
& From t he fo 1 0 v i Tig lat
25, 000: Marginal cost % 20 ’ 000 a NewiW B. KaE P, Es Zz 3 6 000 t
Break-even poi
nt = %
000, Profit
= 2 10, ic a.
ed cost % 20,
dla, ¥ yo ny a T eT 2 T RETited
BI wT tatentate the
fix

UES
this peint ;
break 2 ver p point
nt and
an (
net
es. wh en (ia) * 60, 008 )
tii) Ascertain sal
le
s. (i) < 10,0 Ho;
1 te at
Selli : ee
fan
per unit ‘\ X25

e
Direct Brice per unit o
Direct abour costcostper unit
nateri al
Bo Le
b he
© on o's
Le
ZEN
LAT , ar
Cost-
Cast Volume-Profit
YT Management Accounting

#
Analysis
aoe —
5.78
» . The Profit/v
(/Voiume rati
Work out the break-ev a company is 50% aad its margi
of poin 8 for two years are
as below: Profit (2)
en
eve of mesafeisty%z is50 40%. You are
Tofit ifi salesIginvolu “4
The sales and profit Sates (3)
Point and the net p
Dethi) Tequired to . 17.
: lakhs 7 (B. Com, 20,000
Ho: » Hons. , 1,50,000
25,006
Year 2016 1,70,000
dé. (a) From
the followin g partic
[Ans.
chart and he 2 30 lakhs, Profit & 19° . .
ulars draw a break-even lakhs} Year 2017
Nari:able cost per unit ind out the b Teak- a p rotit of % 40,000. (d) Margin of safety at
even point. Cateulate: to ear n two
uir ed Variable costs of the
(a) P/Y ratio, (b) B.E.P. (c Profit made when sales are % 1,00,000. {f)
z ) Sale s req
ixed cost 15
(2) (B. €om .. Madurai)
a profit of @ 1,25,000
Selling price unit % $.08,000,
% 70,000; (c} 2,30,000. (d}
,
fans. (@} 25%, {8}
years.
(6 ) Wh at
127.500]
and @
fe) & 7,500, (f) & 1.12,500
should be the selling price if break éven point j st © be brought down
to 6,000
' unifats
{B.€om., 1» Ba Bangali the following information
furnishes you
[Ans. (a) 10,800 units; The Reliance Co, Year 2017
5 + F Ltd +, Manufactured and sold 1 (b) a 48.
Second half
+ ]
at a price of & 800 each.
- Th The cost First half
10,26,000
$e, Materiais 200 € 8,10,000
Labour Sales 64,800
Sua 100 21,600
z t remains the same
Variable cost Profits fo Mow ing ass uming that fixed cos
com put e the
50 are requir ed to
From the above you
Marginal cost 350 .
Factory overheads (fixed) in both the perieds.
2 .
1. P/¥ ratio
Tatal cast
= : 2. Fixed cost % 6.48,000
Profit 0 of profit o 1 foss
when sates are (B.Com. Hans. Dethi}
_ 250 3. The amount n-é ? rofi t of % 108,000. 12,42,000]
Sale price 1 §. The amount of
gates required to ear
& 140,400: (3) = 10,
800 Loss; (4) z
toh say Competiti ion,
Due ol
price had t 6 be reduced
— 8007 [Ans. (1} 20%: (2)
n
the ta % 750 f years have bee :
sa ta e the number of Elect comin year. Assumi PJ Utd. for the two
pricge no ch ange
‘or the of Profit .
sold gat the new min
ri ¢ [rons that would h The tra din g resu lt
mount of total profit as
that of the | ast’ year ave to be ice to ensure
19,
Year \
Sales z
z

nee ee
. (B.Com. Hons., Delhi)
14, : 12,000
The tradin g result S of a company 5,40,000
for two periods are as under: [Ans.. 2,125 Irons] 2O16 39,0009
6,00,000
Sales 2017 °

eT
ing =
Computate the follow

ae
Period . Profit
z
.
Period I 1,30,000 " P/V ratio;
6,000 aj
Calculate ; {a 1,50,000 i
« ‘ {) PV. Ratio; and (b) BLE.P 10,000 (iiy-Fixed costs;
~ (iii) Break-even sale
s;
000: and
15. The ok (Blom, Cati ety at a profit of = 48, {B.Com Hous.. Delhi)
ating results of a compan 00.000)
(6) & 1 ~~
(iv) Mar gin of saf
yea rs. % 4,20,000}
(Ans. (a) 20%, ‘ during the two iv 3,786,000 and
7 for the last Sales : {v) Variable costs {iid } T 5.0 0, 000; (iv-% 1,6
0,000;
as follows:
were
Year 2016
Profit |
i a [Ans. (i}. 30%; (i) %-1,50 ,00 0;
4 Ltd. during the two ye ar 200%. and
200 5
Yea = 2,70,000 : 6,000 an d profit of M/s Profit
= 3,00,000 a4, The sales turnover Sales
You are veauinited to calcutate: .15,000 z
Year z
(a) P/V Ratio i .
60,000
to meak-even point 4,50,000
2016 75,000
arqin of safety : at 5,10,000
a profit of % 24 000 | 2017
(B.Com., te Mad ras)
to catculate :
[Ans. You are required
16. The sal
es and profit duriring .
(a) 30%, (b) & 250,000, . (c) & g0,00
tw o;
below: 400]
: (f} Profit-Volume rati lose not gain anythi
ng:
Year 2016 @ years
~ .
are given
Sales at whi ch com pany will neither
(ii) Sales 0,000;
Year 29
eof
earn a profit of @ 4,2 (B.Com., Delhi)
(iii) Sales required to
= 20 lakhs
]
n sales are 7 7,50,000. & 6,90.000; (iv) © 1,35,000
1? 2 lakhs
Calculate (a) P/V-ratio,
;
{ 5) Sales Tequired
= 30 lakhs
to earn a profit of % § lakh t 4 lakhs {iv} The profit made whe {Ans. (7) 25%; (i } & 2,10,000; {iii)
’ Ss. ( Lew, LA. Int
{Ans. (a) 20%; (b) 7.3
5 lathes
a i
Cost-Vetume-Profi
t Analysis
wo
Be hw) yy Wei Management Accounting

ane
- Contribution
e:
‘ould be sold at bre
ak-eve4 n ‘pot int?t? You are required to calculat
(a) Brea k-ev en poin t,
23.
(b} Sale required to eam
a profi t of % 72,000. (B.Com)
profit is % 72,000. units: (c) 4,600 units]
t and depreci .
charging int ere (ce) Maxgin of safety when (b) 14,000
t to % 60 1 akhs
tation [Ams. (a) 16,000 units;
. a
Moun
tio of the co st 5 industrial containers and packing
Pany is 0.4, calculate its breaks 100 lakhs respectively. If the mark et
. Mm and
prod uces
Cases Priva te Limited” selling price. f F the present level of
“even sale 8.(B.Com.
Hans,, Detht) 28. The “Containers and propo ses to reduce the
to competition the c dmpany prepase d reduction in
RK Ltd.: nonin
(Hint : Lo Loss
lakhs; Fixed cost 2 4.4 crores) cages. Due units to be sold if the
24, fAns. = 11 crores
] indicate the number of
z utes thre, : 8). profit is to be maintained,
prove and = 75 respectively Pets & Vand 2. The unit selli ls 10% and (c) 15%.
selling price 1s:
ending unit variable € pricesare of= three information is available:
ese oe
al z
eclively, (quantity)
senate in' which there product: costs
30, % 80 and 3 '
The following addi tion z
The ¢
S are manufact
CO. The

fixed costsquanti are T 14,80,0 ured and sald a 20%,


alu

Calculate t he overallotalb reak-even


ro
30% and 50% 300,000
80,000, (30,000 units}
ntit Present sales turnover 1,80,00¢
.
Electroni {Ans.
[Ans. 0 Overall BEP = unj productwiwise
26,195y and the break-up of s
quantity. (C.A. Inter) Variable cost (30,000 units) 70,000 2,50,000
25,
that uch a Ltd., decides to eftect ee units, X—5,239 units, ~—7 Mee Fixed cost 50,000 (B.Com. }
p then 2~13,098 units]
volume sheen In the prices of its Toducts because it j Net Profit (2) 48,000]
It is anticip sleet may lead to greater Sales. fat ts felt (ans. (a) 34.286; (b) 60,000;
are
The direc tors wish at there ate no prospects . information:
The faltowing igk ta maintain net profits SO change in total fixed and Variable costs per uni Company led to the following
t analysis of Sultan Manufacturing
Fixed cost
Sales— ormation has heen obtai ined its books:level,
from Present A 79
Variable cost 1% Gin
i
= -
“Cost element
Variabt 10,000 units . (% of sales) -
ladle costs-—% per unit * 2,00,000 32.8
Fixed cost ®. Direct material -
28.4
How would % 40,000 Direct labour 1,89,900 —
the managegement procee
thes. 12.6
d to implement1 thi 58,400
s decision?
(B.Com. - Hon, Hons., Det Factory overhead 4.1
26, Graphi phics Ltd. Manufactures
wee :ales required
i to earn current a hi) Distribution overhead aA
_ 86,700
moun of profit z 3,00,900]
a doc on over head ine:"Y Toe
Follows: Producting machine
which has a vari. General administ rati
2 318. 50,0 00. You are required to determ
next yeat
Material able cost structure as Budgeted sales for the
.
Labour 40 {a} Break-even sales value
t=
eted sales yalume.
Overhead = 10 (b) Profit at the budg
anSeles nitsi {c) Profit, if actual sales:
price of = 99 = 4% (B.Com. Hons., Delhi)
drop by 10%,

ee
Undertae {i}
current year are expected 5% from the budg eted sales. 50, (ii) & 92,925)
a {if) increase by 00: (¢} ti) © 34,6
g& agreement, an inciea
se of 10%
i is Payable to all di Irect [Ans. (a) 15.00,000, (by 73,5
fortheomi ing year, whi 79 = 21%).
b workers fy, omj the P/¥ ratio is 100 -
expected to j be. ginnin — is 79% of sates. Thus ently the
5% and fixed overhead cove Vv 3%, Costs are by 5%, variable ove
7.5%, (Hint ; Yari able cost 7 300 per unit. Curr
¥
Yo Ware requiri ed O increase rhead vaste cook ers the selli ng price of which is to redu ce the
to caleutate: pressure proposes
30, Z Lid. manvfactures 18 lakhs. The Company
isat ion is 60% with a sales turnover of Z posi tion by incr easi ng the output.
capacity util the same pro fit
desires to mai ntain determine the leve 1 at whic
h the
selling price by 20% but put cou Id be made and so id,
out
Assuming that the incre
ased
objective.
to achieve the des ited
savy Company should operate
a are available:
(Ans, (a) % 97; : () & 47.433 The following further dat:
{Hint : (a) New SP = machines] per unit = 60. 0,006.
«90
= % 97 {f) Variable cost of z 19 per unit} Z 1,43
54 e cost {inc ludi ng a variable element an additional amount
(id Semi-var iabl . Beyand this
& (5)b New sales quantity = 444,200 + 4,090,000 tit} Fixed cost % 3,00 ,000 will remain cons
tant up ta 80% level (LOWA. inter}
2} Reina SUBplY you th 318 = 17 +113} of % 60,0 00 will be incu rred .
% capa city = % 9,640,000;
Sila (Ans. Profit at 60
= 8.471 units}
é following Tiiforma : #0
vat overhead
in the sam e prof it
Required sales to mai nta
=
Hable cost
Per .
unit -
d co: st = % 3,00,000 +
1,8 0,000 - 60,000
z 60 + 10 = 2% 79, Fixe 48,235 oF
. : 1,80,600
Selling price unit
: 2 (Hint : Variable cost per unit a 170 /24 0. Sale s to ea im the same profit = 19.
230 300 ; New P/ YP sati by % 60,0 00. Thu s new fixed
g 20 420,000, P/W ratio ty. But bey ond 80%. fixed cost imer
e ases
capaci
8,118 units or 81.18%
yao PRM ftin ay
Cost- Volume-Profit Analysis §.82
5.81.
You are required
to: which adds z
nt expansio n
out break-even point. to brea k- ever after pla
4,20,000 + 60,000 + 9,60,000 Find Id
Z 20,32,941 (or 8471
cost = F 4 20,000 + 60,000 Required sales = ia} to be s0
number of bottles ity by 60%. sent pre-tax
170
/ 240 Find out the increases capac maintain its pre
(b}
to fixed cost and
company will be atle to
units}) 50,000 at which the
jevel af sales (M.Com.,
31. (c} Find out the after the expa
nsion.
. é:
Furnishes the & foll owing g information telating to the half year @ nde d 3 G June, ? 2{ 0 5. even after expansion. at full capaci
ty
profit pasition plant opera tes
Fixed cost z 45,000 wher units:
Sales (d) Find out the profit (b) = 3,759,
000 or 15,006,000
Z 150,000 units; 1,70,000]
: ~ Adapted) 19,00,000 -tax profit ©
; and (d) pet
oF
[ans. (0 } 2,50,000
Profit
= 00
30,000 {c} = 6,25,000 ro om s hot el with
000 for a 300
' During the second
| oO half
alF af t h @ year , the pany
compan p projected l 2 to ro om s of % 15.00, ren ted . The Hotel operates
each room
has
t appiicabl
a loss of f 10 .' 000, . Caicu ate *
annual fixed cas age varia ple cost af € 6 for
Leela Hotel has
a) The break 2
even oO
point 2
ane ma cgi
30 per cent.
gm of saf ecy ror ca ae SEX I Wome
th ending g 3 su June, t 2005 .
35, % 40 an d aver e of
for ti 2 second half of the year assumi g that fi d costs and P/ of
es Tat
{b) Expectsted sales ntvcluin average daily rat t to an inco
m e tax
remain: consta in the second hal f of the ofyear. umin xe V tatio
465 days pet yea
t- Tt is subjec
rented, and
to : er of 790MS 10,00,000.
it is required terms of numb after tex of €
(ec) The break-ev
éven point ml margini of safety y for the whole ¥' ear of 2005 CA, in a net income
Int
ntey t
ea k- even PB oin ear n
the br t to Delhi)
Ans. {a) S0 000 z {a} Compute the Hotel mus
t ren (B.Com. Hons.
er of rooms
80,000; 40,000]
32. tThe bud geted sal es af three products the nu mb ms pel day.
of a company are as Follows: (b) Calculate r of 421 roo
{a} 44, 118 rooms pet yea pet day]
{Ans. rooms
year ar 736
Products
{B) 36, 134 moms per
_ Xx ¥ Z 7

Budgeted sales i 15,000 F 10,00 ,000


Budeeteb oon
in units 2 10,000 % 15,00,000 +
a
price per unit 4 0,000 70%
; 4 = 86,134.
Budgeted selling cast per unit z
24
‘4
2.5.
3
5 profit =
244
Budgeted vanable
g LE: fixed expenses (total)
: Rooms to earn given
i
2,000 9,000 7,500 (Hint z
|
the yet 2017 29,00,000
From the above ave informatio
i i
T you are réquired to compute the followi is budget for
) The budgeted profit, for each product: 36. The following = 20) 10,080,000
| units @
gates (1,00,000
ai
1
.
10,00,000
Dudgeted break-even sales - Variable cost
(c) m udgeted maryi terms of sales vaiue 400,000
!

ad inaryin of é }
Contribution
us. (a) @ 3,000: (0) € 32,000
2,500,Z 24,000 236.006: € eae 6,00,000
(c} _¥ 0 6,000;oF & 20%,
}
ot 40%; ; & 20000.
,0 or oon
25%] Fixed cost
here are t wo factoriz: under the ment d
The manage |
ggest
tovi2s rs ate available lesires te mer ge these two Het profit tion find out
: ed and also su
plants. The icllowing particula able: set of infor ma set s of ch an ges ate introduc
th e above two
Factory if -
Fr om
fits for 217 if the following
y I cent
Factorper The adjusted pro
Capacity operation 100 60 per cent
{a}
which plan
should be im
plemented.
Plan 8 20%
Sales = 300 lakhs Z 120 lakhs ‘Decrease in pri
ce
25%
Variable costs z 220 lakhs
Plan A 20%
Fixed costs % 90 lakhs Increase in pri
ce
25% ‘incre ase in vo lu me
10%
are requi . Z 40 lakhs . = 20 lakhs Decrea se in vo lume
70% Decrease in
variable cost
5%
cel eulate nt to be o cos t
(LC. WA.. Inter}
ld be the capacithety vn of ne merged pla
You
quired to cel (i variable cost fixed
) wat(if) wou perated fo F Increase in : Decrease in ed abo ve.
p;aciofty. breaking late ( ‘ what would be 5%
“wo B rans referr
the pu
merged cacap ofit on wor kin :
g at 75 per cent of the se in fix ed cost und er the 54. 167 %j,, BEP = 7,75, 384
eak-even points
Incr ea ratio
rat io am d br fit = 53 5,000, P/V .7 5 o,, BEP 2 8,68,57¢
{b) The P/V
(CA. Enter) pro io 43
(ans. {i(i) 46.15% or % 230.77 lakhs:s; (ii)dif Z 37.5 lak. hs] fit z €, 95 ,000, B/¥ rat t be implemented)
vi B — Pro B should "0
34,
Goa Chemi1 cals Ltd. produces the following in come statement for the pre Exct e pt as noted ,0 00 and thu s Plan A and
are ava ila ble :
the cost : ous year, pat
expected to follow the same is highe ratz 6,
00
following gata
evenue relationshi ionship for t he coming yea r ising Original profit duct. The
Income statem ent for the year
end 03 ern as in th s 4 single pro Z 16 per unit
ploceedi ing year.
, une, 2005 is
as fi
‘ollows:

. ma nu fa ct ur es and market
37. ABC itd 42 per unit
Sales (220,00,0
00,000 nits
unit:
% 5,00,000 ,
Materials
@ 2 5 Pp paise each )
Variable cost . 3.00.0 cos t 4 per unit
riable )
Conversion {va g price)
Fixed cost (10% of sellin 40 per unit
100,000 _4,0 0,000 Dealer's margin
Per-tax profit ee e
an 1,00,000 Selting price
Ircom e tax tax 20,000
Profit after — 50,000
Cost.
st Volume-Profit
Analysis
vee & Fao Hanagement Accounting
A
ee
Fixed cost: & 5 lak 5.84
hs
nsrese
er sales:
s 90,000 unit Ss. 15,006
pacity utilisation:
69 per cent. Direct labour 30,000
ad
There j
is> acute competi . Fixed manufacturing overhe 10.000
Tpetitz
ration expenses
fixed selling and administ
1
neasing the sales:: tition. Extra efforts are n ecessary t Q
20,005
sell, Suggestion
(a) By reducj S have bee Gross profit 5,00)
Ing sates pric e by 5 ger nN made for
cent. Het loss
oF ending inventaries.
There ar e no beginning
It required to calculate:
. if
on expenses
(i) Variablé se iting and administrati
the company de
sires to
Maintain
the Present
in in rupees
38, Tw firm (M.Com., Kerala) {ai} Contribution marg
head, _.
(iif Variable factory over
{iv) Brea k-ev en poin t j in rupee sales ~ ¢e.€om, Hons., Delhi)
.
we
(v} Factory cost of goads sold. (iit) % 5,000; (iv ) F 1,20,200; (v) % 80.000)
A & Co. 4 5,000; (ii) % 25.000; is
; . fans. (i) 0, its pr afit of ¢ 50,000
: es
BEC 8.
pany de cline from
& 9,00,000 to Z 7,009,00
| Whe n sales of a com tri but ion mar gin ratio.
41, (a) ine con
; ° of = 50 (900, Determ is % 10,000, What is its
ariable casts
; converted into a loss 25,0 00 an d its fixe d cost
Fixre
ed ocscos
tnts
80,
6,0 0 000.
s at prea k-ev en poin t in a company is @ (3.Com. Hons., Dethis
(b) Sale
Secoe
009 ribu tien ? (a) 50%. (b) % 10,0004
so
4,3 on
a
0,0 00 total cont [Ang.
0,0es
on 60 4,70,000 :
equi : is supplied to you:
42. The following information z
21. Sta
Cal
=teculate
cnaat newhich is tiBet
y vol ~ 4,500
| teume
n both the firms" Willwill ear
‘ sm demand for’, n ae
rode is tt
j |~
“ Fixed cost (total) 7,500
profit in condition
m ofof - Variable cast {total) 15,600
fans ean for the Product Sales {total)
: 00,000, 2. ()(7 Com |aher Units sold 5,000
pany g because ion
its P/V rati
atio is high
“) ‘company oAbe
nter} Calculate; (a) Contribut
s
A because (b) BLE. point in unit
/¥ ratio is low
er] {c) Margin of safety
wil
:
(d) Profit (LOWA. Inter}
.
39. Two :
U earn equal profit = tence in fixed costtaio ) ume of sale s to earn a profit of z 6000 (d} € 3,00 0; fe} % 21,0004
0 units: {c) % 6,000;
com
XY Ltd
and 2018 Difference in P/V (e) Yoi
AB Ltd.
For the yearar ended March, reduce and sell the [ans. {a} < 7,50 0; (b) 3,00 per
es which are sold at © 28
, (heir forecasted Profit and | type of product in th rane ®
the past seve ral years produced boxing glov the ade qua cy of this
O85 accounts are as felon Market, Sports itd. has for to consider abo ut
AB lta. (is nate made managem ent
recent years have
pair. Higher costs in
selling price. cost of leather h as
= 2.25 per hour and the
Sales seased from @ 1.75 per hour to year s Sixe d expenses have
was inc five
¥ The labour rate
to F 2.15 pe ¢ squa re foot during the last oc vari able overhead has
Less: . Vari lable cost of sales 2,00,600 2,456,000
gone up from % 4,10 year s ago. Over the same
geri
Fixed cost 2,50,00) of = 18,000 five her and one hour of
2,50 000 800 increased 25% from
the level tequires 4.5 sq. ft. of
leat
25, 000 2,25,000 pair of gloves. Each glove
ee

of © 3 per
Profi Tofit . increased 30%,
es 75,000 cost structure to break-
You are veqy: direct labour. charge unde rt the aew
25,000 fe that the company has to (B.Com. Hons., Bethy}
to eden once’ [© compute ; > (a)(a) P/¥ : Break-even point es 25,000 Calculate the selling price
as five yeals ago.
T profit in condition, of (i) 1 (b) demand (i ) nia
oa ber of unit s t 160/ units]
greate
State whi even at the same rum = 14 p er amit, §.£. poin
37.95, New contribution
c)
ich company ¥ isis tilikely =
we

low ing pric e es. The cost


gos, {Ans
o0g: (8) bigh de mand. (B.Com. B, (Ans. New Sell
the same manufacturin
g faci liti
40. The
mi + (8) 20%: 7 40%, (8) = 1,25 : » angalore, Delh; ts—A- B and C-—from
following J miscellansepus j £9,000; F 1,87,500 Gn 3 ) thre e pro duc
187,500. (e) (i) AB Ltd:
from th records 44. Taurus Ltd. prod uces
of A 8 information the xY td.; {é} ate as follows: c
@
cme Corporati Tegarding operations of 2
Lt and other details of
the three products A B
ation:
- 10 100
0,5.
. 017 has been made availabee 200 4d
Sales z Selling Price/umt @ 120 reo
.
Direct Materials used Variable cost/unit {®)
2,00,000 th (} 2.76,000 6.000
Fixed expenses/ mon 5,900 8,003
40,000 production per month (units)
Maximum
©ost- Volume. Profit Anatysis
Lost- seams —

"5,86 e in the factory?


|
d for manufactur took
u recommen
Total hours avaj
ilable f d
ae 206. data which product woul yo
the per now ty month
demand cannot (units} - 4g. From the foll
owing Per unit of ,
hours 2-400
y processing 2,000
be i nereased beyond 200 hours per
m “ee
' Product A Product B
* onth, 3
Ou are required to: , . 2
compu te the most
r : product-mix; and cture time (hes.) 50
Standard manufa
(a}
{b} the overall hee
el eae,
Ah put
com als : : £0
30
even sales of the com pany for
the month based on th e mix - Direct materi . 20
18
calculateg ated bo ur @ % 10 per hour 12
Direct ia
Inter) ad @"% 4 pet hour 240
Variable overhe
[ Ans * U} A Te, 000 (CA. 200 .
(Lowa. inter}
4 > units, B _ 1,600
he“nen
: Folla
§ a wia g fi figur
gurees te
units, C-—2 ’ 400 units .
( iy } 2 . :
6 ‘ 72,6 0c)
Selling price
eiate
rel ¢. + one
business year's worki ia g at 100% capacity or y are 50 ,0 00. or ]
level in ble in the fa ct ting fa ct
a
hours availa e time is the limi
Total machin
e
t is re co mm ended when machin uc is ar e ma de
[Ans. Produc A e prod
nd ¢. All the thre
oe i averhrad products A, Ba below imarele
hae
ana man ufa ctures and m arkets three by cap aci ty. Fro m the data given
overhead 3,000 — HE Bt Company tio n is lim ite d
hines. Produc ng profit.
from the same set of mac A. a and C with a view to Mme simisi

ee
. | Product
Direct inaterials riti es for pro ducts, Product
40,000 prio Product C
B
Sates 1,00,600
oa A z
Represent 2,508,009 = €
. aboveres figures
point ~ Ye ¥Wenfya your
on 4 break-even chart and getermine
trom the ch. art 3.25
4.25
t by calculation. ine the break 2.25
Ho ~even 0.50
unit 9.50
Material cast pex
(B.Com.
» Hons., Deihi) 9.59 o.71
- Har 7 ors Lid. have
arvana Tract [ans. % 1,25,000] 0.45
annum. Th per unit
pacity of 5,000 tractors per
\ an install
alle cd Capa
.
Labour cost 0.30 7.00
at about a6
ar ent of inst
perc : alled cap are Presently operating
-budgTheyeted pet unit 6.00
c o m
acit y. For the coming year,i they have Variable overhead 5.007
Production/s sale 5 as follows: Selling price per uni
t
Costs « . ows: uired 28
4,000 units. time req 20
Standard machine _ 39 that 3 kg.
erial, it is noted
. ® (cror es)
ials
syottage of Taw mat
mater s)
Direc t per unit (in minute s ext rem e pec tiv ely . How would
. the compan y face C res
Direct wages 8.00
In the following year to pro duc e one anit of A, 5 and (Le Wwa ., inte?)
material are requir ed
Factory expenses 4 kg. and 5 kg. of B, C, A.
nge?
cha time is key factor
machine
sare ettative expenses product priorities n priorities—When al is key facta:
A, B,C).
Selling expenses on + [Ans. Productio —When raw materi
. period ending
fa ual profit for the
Profit ’ :
fou nd te hig sur prise that the act bud get ed selling prices.
: ctor y expenses as well 2 t of Flex Limited g 10% mare than
the
as sellin 9 expenses are vari so. The Cost Accountan in spit e of real isin
valculate th € break-k-even able to th & extentis
- of 20 percent. same as budgeted
capacity utilisation percen
ntag
t e. 30th June, was the results: Actua!
low ing wer e the
(CA, Inter) The fol Budget
47. This Pree
<
struct ure of a cycle z
made by the Cycl
ycle Co. Ltd., is as follows: 8,25,000
a 5,00,000
“es 575,000
Sales 3,00,0C0
sales 4,50,000
Materials Yariable costs of 1,06,000
1,00,000
Labour Fixed casts 1,00,000
- on as to why the
Variable overheads necessary explanati
20 Profit
Cos t Acc oun tan t in preparing the (B. Com, Ho as, Delh i, CA. Inter)
to ass ist the
2 You are required and incr eas e ir. sal€s- sales is fully
the sa me gespite in profit du e to
increase 10
nied overheads profit remained [Ans. Increase se in cast.)
pra fit due to increa
in
sant offset by decrease
® to increase in sales
ce © 75,000 (it ) due
Selling price
3
in profit (i) due to inc rea se iy. sell i ng pri
iable cos t 2 1,259,000 and (8)
This te = 209 (Hint: (A} Increase in prof it (i) duz to i merease in var
based° on the se
(B) Decrea
5 manufacture of of one lakh c volume = 1,00,060. s in
The com Company expe fixed cost % 50,000
]. in the volume of sale
to competition th veles
per annum. :
due te inc rea se in
equ ati on to ind icate the difference Z 1,00,0 00 in bot h
the otal poh ie to Teduce se iling prices » but th ting based profit a profit target of
cyclen mae What level of eo Heil have
ae will have to @ reached, i.e, how 51, Use the marginal cos e competiti ve situazon to teach bat h the sit uat ions. Also
8 wHl have to be a pur % 50,000 in
€ to get’the same amou at of profits if: many ¥
a pure monopoly and 400 ard fixed cost as ,
le cost per unit as % conditions sespectively
listic and competitive
( a } the
he seelli
li
cases. Assume yatiab
: ng B vi 1ce is
i I educed bd A] fo
& 150 und er mon opo ., Int er)
to he ¢ 250 and ULC.WA
assume selling prices
b the selling pric
e is reduced by 20) Ho?
.
(B.Com., + M Madurai i profit of ¥ 1,00,000}
units to get the
[Ans. (2) 1,25,000; (6); 16- itive conditions 3,000
007 Monopoly 1,600 units, compet
[Ans.
Management Accounting
Cost-Volume-Profit Analysis
5.87 ee

52. ¥ 20 pet unit


statement by produrt lines of Adarsh Multiproducts Co mpany for 2027 is ag 7
:
The budgeted inc 1 +
follows -
ame
Direct materials 100 % of direct wages
Variable overheads % 1,20,000
Products Fixed overheads € 100 per ust
B c Selling price [ans. B.E, Poin: 2,40,000]
A
of a factory:
. - : < given the following data for the coming year
Sales
2,00,000 5,00,000 3,00,000 55 Aca
unrts) 30,000 units
Variable casts: Budgeted output © 7! *5 Ga 2 4,00,000
- Fixed expenses 10
Cost of goods s old g
Variable expenses per unit
. 7,10,000
Selling expenses 20.00 on oop 25,000 zg 20
- 000 90,000 45,000 Selling price per unit ces to 18 per unit what
will
Fixed costs: the galling price is redu
| k-ev en char t show ing the break- even point. [f (ic WA,, inter}
Overheads Draw a brea
re ae5 30,000 t ? BEP © 900,000]
vn Administrative expenses 12,000 7.000 45.000 be the new break-even pain (Ans, BEP < 8,00,u00, New
28 000 30,000 18,000 i cast far any given venue
come before tax ing Ltd. concluded that ts
, 25,000 of cost -val ume- prof it relationship, the Flem to 60% of sales. The company’s
neome tax @ 40% 56. Alter a stud y d costs plus vari able cost s equa l
10,000 22.00 d as 7 1,00,000 of fixe
Net income - 11,200
16.800 15,000 i200 of sales could be expresse ,000 of sales.
range of volume was from zero to = 8,00 er sales ling to the graph
tion ship. Also draw a prop
All products are manufactured with the saz iliti h whic h will illu stra te this cost volume rela
femived are allocated among the mroducts in nlopontion to their budgeted Prepare a grap k-even point.
lee Pied t. Determine the brea also has fixed costs of appr
oximately
to form a break-even char as that of Flemin iq Ltd.
oper atin g @ plan t of the same size
able caus e s of the diff erence between
{a}neTo compute the budgeted break-even paint ~ A Competit or 0 sales. What are prob
pai of the com _ k-even point is ® 3,900,00 M.,Coum.}
(b} » the budgeted te eee aealuct F ae to ascertain the eect on huyeted ines provided, % 1,00,000 but this brea Flem ing Ltd.
that of the fans. BEP © 2,590,000}
this break-even point and
ee . i Me ior budgeted sales in rupees romain the came. A and Cin equal ee ont form the following data:
Brawa break even chart

ited
| woe company of the shift suggested in (b) above on the budgeted break-even point of th 57. (a) z 5,000 |
Fined cost
€ 20 per unit
Ans, {a} } BEP z ate ; ioe
Fi Cost : 210,000, P/V ratio 32%, {b) Profit ¥ 83,250 ae Selling price % 10 per unit
ne )
90 }, (c} New BEP @ 6,02,150; New P/V ratio 348754) Variable cost ° _
26,000.
53. A retail deate in garments i5 Cll Lfes 1 tly selling g 24 t 000 shirts annuali y- Show profit and margin
of safe ty when sales
are = nn
in fixed cost.
He su ppue
pti s the follo Ww ing of 20% de crease
chart to s how the effect (a) BEP = 10,090; (4) ¢ 8,000]
(b) Draw another break even ~ fans.
Selling price per shirt
;
Variable cost per shirt
Fixed casts: :
°
Staff salaries for the year
General office costs for the year
"50.000
Advertising costs for the year
“0-000
40,000
O)' cauat aceon tant
As a cost accountant of the firfirm
m i you are required
: i to answer the 2 followi
followin i
Gi) Aan ny ee and meraie of safety in sales revenue ord nate of ahivte cote
cia Assume that nee2004 shits were
were sold in a 1year. Find out the net profiti of the ‘im
.
ff
vt
wn re auc eving
selling commiss
commissiion
on ‘s on i how many shizts
shirt, its would
wo require
a shit ie at Uke
for t te ea 2008 an adestion
iti al staff ‘ala
salary of € 33,000 is anticipated
shit ant ay eo Pe, : y¥ 15%, what should be the break-even point in ‘oumber o
iAns.
rot the ioving data lit Aas.
oso eagn
{i (i) "BEP op anes ot 16,000 shirts, M/S % 3,20,000 B 00
"000 oror By ite :
. (ui) 21,250 shirts, (iv) BEP % 5,98 000 or 13 00 ny
54, meen ph, and determine the break-even point: mane
= 10 per unit
g
Management Accountin

counted cash flow,


ital ¢ mployed, dis
ve con sid era tio n of return in cap
in vol
Long-term decisions of this book.
are outsi de the scope
etc. Such decisions
MAKING PROCESS
STEPS IN DECISION
en:
lowing steps are tak
on making, the fol
io a In a process of decisi
ul so as to
fm roughly analysed
The problem is tho on to study
Identify the purpose
QO. e of the dec isi
<< identity the purpos
of the decision the various aspect
s
x
Oo
ting to
to be gathered rela
Information has keholders
ect e d sta
Gathering the factors and aff
information

ding
a down ail the ideas lea
Introductia na: Short-term
and
Long-term decisions; 7 R Re Brainstorm and Brainstorming to list done on the basis
ad best cho ice . This is
relevant reven ues, d
ann- $e
ificati cision-making, ; Ce ants
t cos d no anatyse different to the
Classi fication of coststei for de . Cos of cause and effe
ct study
cost factors nak ec ision-making;g; Decision-maki db le cos i
tin g; alteratives
_ ae li . markets; ingE and vari
Fixation of selell ;
ing prices; ; E Exploring new
; P 5; nExpSl
ow anes: etn or buy
ortecsal
are evaluated to sele
ct the
s: Plant shut-d
decisi ons wa
; Product mi erential All alternations
vmx decision king and diftor
e, inf orm ed decision:
cost analysis vs It sho uld be
g; Decision ma cost Select the best - best alternative.
or 4 e costin ntial nn
analysis: Adding .
ne further
-
i e; Decisino alternative making.
Key terms; ea
ropping a odduct
pro
pr; lin ae
i ;
Processing g: Problems and solutions ‘ on questions.
plan and
converted into 2
Decision made is wit
int o action by excuting
P tan is converted
a———_ inates.
Introduction Execute the decisi
on
the help of subord
In any
choosy oy business,
tas , the
the basic
basi function
i of management is isi
tose ® course or action oma few possible “tternathws for examale the waneonteat ey in Decision Making
kane Fig. 6.1. Steps
‘ my hae tise i" price of its product, lower it or leave it enchas nd at its pre ant level,
down its openation: fy the et continue to sell its product at a loss i ae ing’ market shot
nt
ra Pe gts For the ume eing. In a company, manufacturing tele ‘sto ets the manage tt Relevant Revenues nt costs and releva
wumvtien Thos the eee J i) make picture Relevant Costs and has to concen trate on onty releva es are tho se
tubes within the eompar orto | yi ‘Nem an eats makes decisions,
it d relevant revenu
selecting the beer nage ent is continuously engaged in evaluati 5 Y. 5 w alternatives ond When management es are yel eva nt. The relevant costs an ent t
alt e ern ati ve cou rses of
revenues. Not ali cos
ts and revenu differ undet differ sly
ese. The decision to be taken will be affected by co:
cost Cond
and athe on
other factors. and expected future Te venues that cted by a decision are obviou
expected future costs d. Cost and revenues chak remain unaffe
sidere
action being con d when making a
decision.
not be considere teristics
Sho 7 -term 201 ie s
irrelevent and need uld have two charac
nt cos ts and relevant revenues sho
Thus releva
T € focus a hisis chapter i
is on short-te T ™ decisions ' such a $0
to future, and
enues must relate action.
(i)(1) Whether
Whether 0 sere Product ini the domestic market or export it {a} The costs and rev ative courses of to
among differen t altern hing cat be done
(ii) Whether to chanee ceatponent part inhouse or buy it from 4 i (b) They must differ be mad e aff ect s only Future. Not 200 1. It can
isi on to purcha sed in
future because dec of plant and machinery
ge the present product mix to make it more ,profitable, etc,
ofitabh supplies The focus is on the can not change the cost fut ure costs that will
differ
.
t. Man age men t eva nt cost s are
mana gers consider a decisi
Most
rt rm ifif ititi involves a
peri change the pas ons . Hen ce, rel in ano ther
by its current decisi
but
but a decision as a shoort-te
nt
cast may be releva
cut off per iod is arb itr
arbi ary
ugh a ticbet ter dis tinction th at to longyearterorm less , This
isi
dececisions change future costs man age men t. In one decision, a mus t dec ide whi ch costs
al capital ny estused tho ati actions of the on, the management
Shan Teq uir e sub sta nti men t wit h imp plicat ion s f er, depending on the For ac hy dec isi
t-term dec* isions are mor easlyi reversed than long-term. For cram severa l fut ure yea rs, Mor eov may not be relevant.
duct can always decision the same cost
be changed according to ore et conditions, export sales can be stopned ve of a pro are relevant.
, lf necessary, g and so on.
6.1

TT
nem
Management Accounting
Cost-Volume-Profit Analysis ce
5.87 5.88
. . BBB
: % 20 per unit
$2. i
The bud geted incom lines of Adarsh Multiproducts Com pany for 2017 “ .
fallews ; e statement by product is as Direct materiais 100 % of direct wages
Variable overheads = 1,20,000
Products ,
Fixed overheads 100 per unit 00}
[Ans. B.E. Point 2 2,40,0
, B c Selling price .

. e ¢ data for the coming year


of 2 factory:
° You are given the following 80,000 units
Sales
2,00,000 9-06,000 3,00,000
55.
Budgeted output
ates Ga units) ¥ 4,00.000
Variable casts:
- Fixed expenses z we
Cost of goods sold
Variable expenses per unit
1,410,060 1,35,006
. 20 2,80,000 gz 20
Selling expenses what will
006 90,000 45,000 Selling price per unit is reduces te ¥ 18 per unit
Fixed costs:
ing the brea k- even point. If the gelling price (LC WA, inter)
overheads Draw a break-even chart show 9,00,000]
5 %
. .
Administrative expenses
30,000
12,006 75,000 45,000 be the new break-even poin
t ?
[ans. BEP € 8,00,000, New BEP
30,000 18,000 for any given venue
before tax 28.000 concluded that its cost
Income . 25,000 it rela tion ship, the Fleming Ltd. to 60% of gale s. The company’s
heome tax @ a0 66, After a study of cost-vol
ume- prof
of fixe d cost s plus variable costs equal
as % 1,00 ,000
of sales could be expressed
10,000 2 00
Net income © 0% 16,800
11,260
15,00 0 34.200 z ero to % 8,00,000 of
sales.
proper sales line to the
graph
range of volume was from volu me etal ship. Also draw a
relartion
All products are manufactured with th iustrate this cost
Prepare a graph ehich will
iliti
break-even point. approximately
Renu are allocated among the products in ovoportion te thelr budgeted sale ete "xed chart. Determine the also has fixed costs of
to form a break-even as that of Fleming Ltd. ce actween
a plant of the same size es 0 f the diff eren
A Competitor operating ,000 sales . What are probable caus (M.,Com.}
{a}‘ to compute the budgeted break-even point
poi of the co i k-even point is Z 3,00
Z 1,00,000 but this brea Fleming Ltd. 2.50.00!
(b} re the budveted ae neat pane to iscetain the eect an hdoeted inca eit al t and that of the fans . BEP %
this break-even poin
. : ht te tule sae in tupeee somain the product A and C in equal rupee amon
:
form the following data
: mon company of the shift suggested in (b) above on the budgeted break-even point of the 57. {a) Drawa break even chart z 5,000
Fixed cost € 20 per unit
Selling price
ns. {a}(a) B REP & eo
: i rot
Fi cast : e000) P/V ratio 32%, (b} Profit 2 83,250 a si 2 10 per unit
' , (c) New BEP % 6,02,150; New P/¥ ratio Variable cost
875% € 20,000.
of safety when sales are Cesaraece arate

53. Show profit and margin fixed cost.
: .
A retail
; a dealer in rv
g armer ts is ITen
curre: ly selli Tg 24,000 shirts annually . He supplies decrease
20% deco
of 20% ae in ee as
t te show the effect
the foitowing
% 8.000}
{b) Draw anot her brea k even char [Ans. (a) BEP © 10,000; (b)

Selling price per shirt


;
Variable cost per shirt
5
Fixed costs:
| °
Staff salaries far the year 1,20,000
General office costs for the year
Ae eutsins casts for the year “O00
$a cost accountant
() Calculate i
the baat of the finn i
you are requized to answer the following each art inde
(i) Assan thee eancevey point and margin of safety in sales revenue and nber of shite ea
Ui) rth dene “ 5 ms were sold in a year. Find out the net profit ons Sale Mt the
c introduce sellin ig commission
is i me ' shits
uv} sevuatine thu year to earn a net income of P1008 shat, Row many shirts would require
of : a ahie ie at they
forWi
the beyearincreased
a 2018 an te additional
a itl staff ‘sala:
salary y ofof =© 33,000 isi anticipated
i i
Shnts and sales ovenue? ry 15%, what should be the break-even point in‘oumbor at
‘ane 1 RE (B.Com. Hons. i
«a Ans. BEP 5 to, a00 or 16,000 skirts, M/S & 3,20,000 or 8,0 ite
note teeving aa (ii) 21,250 shitts, (i) BEP & 5,98.000 or 13 000 shear
54, Bhect ta pk, and determine the break-evan point: me
@ 10 per unit
Management Ac ounting

6.2 d cash flow,


employ ed, discounte
e con sid era tio n of return in capital
involv
Long-term decisions scope of this book.
outside the
atc. Such decisions are
MAKI NG PROCESS
STEPS IN DECISION
ing steps are taken:
on making. the follow
in a process of decisi
CHAPTER

as to
ghly analysed so
The problem 1s thorou the decision to study
Identify the purpose purpase af

DECISION MAKING
identify the
of the decision the various aspect
s

to
be gathered relating
Information nas to kehold ers
ed sta
Gathering the factors and affect
information

ding
down all the ideas lea
Introduction; Short-termmanand

f
Long-term decisions; Brainstorm and Brainstorming to list is don e on the basi s
relevant 7
and a analyse different <————— tg the best choice. Thisdy
cost fe aeons: Cessitication of casts for decision-making, cost stu
alteratives af cause and effect
anne Decision-making and variable costing
Fixation of selfing prices; Make o buy
new markets; Export sales; evaluated to select
the
decisions; Product mix ‘echinnae Plant shut-down decisions; Differential Ail alternations are informed decision
cost analysis vs. variable costin . ti making and differential cost Setect the best <————— *_ best alternative. it sho uld be

analysis; Adding or dropping ae oduct alternative making-


processing; Problems and solutions; Key terms; ' Examination ques further
questions.
and
converted into 4 pian
Hecision made is by exc uting with
int o act ion
|-<————~~ plan is converte! dé
Introduction Execute the decision the heip cf subord
inates.
:soa In cs
o any business,: ' the
. ; i basi
.asic function
: : i : of ane een
Management is ° make seeision’s
iS Making decisions is to
in Decision Making
Fig. 6.1. Steps
lower it or leave it hchanged at it: Pp : ene level
It 7] t 4 ew t pace he preduct,
T oe I oer
: ': : C : . whe
: :He 0 |vy Iinue te sell its Pp produhet * a ess i ia falling Market
i vat orov abut
shut
Relevant Revenues evant costs and rel
evant
wae ave 0 cee Relevant Costs and it has to con centrate Om only rel es are those
e whether to make picture tubes within the eompany
makes decisi ons , nt rev enu
nt costs and releva
orto buy it om an outs ie
When management Tele vatl t. The rel eva
alternati ve cou rse s
s and revenues are differ under different
$ eC lecting the best out o ‘si
of the se. ‘TheThe decision revenues. Not all cost fut ure rev enu es that isi on are obv iou s!y
and expected ffected by a dec
‘ be taken
to We will b eaaf ffected by | cost ond
and oth
other factors.
expected future costs
te
es that remain una
|
d. Cast and rev enu
action being considere n making a decision.
not be considered whe
Sha rt 7 term a th d Long -term D @cl §10 ns
jrrelevent and need characteristics :
es should have two
and relevant revenu
The focus of this chapt er is o qT short -TeT
term decisions, 4 suc 1 as:
Thus relevant costs future, and
enues must relate to ion.
(1} Whethe market or it export
from an
it ; (a) The costs and rev alt ern ative courses of act be done to
(it) Whethe, , se product in the domestic inhouse or buy mus t diff er amo ng dif fer ent
y future. Not hing can
chan : ieee
part
to make it more ee supplier, {b) They
dec isi on to be made affects onl in 200 1. It can
(ui) Whether to ge the present product mix profitable, etc, The focus is on the
future bec aus e
the co st of plant and machinery
pur ch: ased
will differ
nge ure cos ts that
Most manage: ; agement cannot cha relevant costs are fut
gers considera
a ¢decisi - rm ifif ititi involves a
decision as a shortrt-te peti change the past. Man ren t dec isi ons . Hen ce, vant but in ano ther
erbi its cur
riod d isis arbitrary but capita
cut off perio he rele
y thou gh a bette less,
r distinction is tne ongyeartermor decis i This
cisio ns change future costs by men t. In one dec ision, a cost may whi ch costs
ene requite subst ar.ti ai l inee stin impli cati
depending on the act
ions of the manage agement must decide
: ment with plica tions For each decision, the man
‘ -term decisi ‘ ons are mn easly reversed than long-term. For ts for several future years, Moreover, may not be rel eva nt.
be changed according to ore decision the same cost
if n of a product can always
¢ et conditions, export sales can be stopped are relevant.
. if necessary, and so on
6.1
g
Decision Mo Hing Management Accountin
oe
6.4
—_____ 83a profitable smce project
B requires less investment
SP these two projects are rot equally owned by a firm is also-an imputed
Th COSTS FOR DECISION MAKING / profit. Obviously, Similarly, rental value
of building
of decision ha Costs hich aesspecially computed for use by
the man and thus, it should be
preferred.
Ng. These costs may of account for the purpose ,
Sunk
ot be recorded in the books ,
cost .
Costs when the
Opportunity Cost that is last or sacrificed
A sunk ¢ be defined as the potential benefit of action. In other
made now op aq 42 costfuturs,th
in
Gs

Such veady been incurred and that cannot be
at has al changed An opportunity cost
may
kesit necessary to
give UP competing
course
under consideration.
theor enn an alternative
the
for decision-makin anged by any decisip gelection of one co
.
illustrate
:
olved in accep fang considering
nachine Sian cept of such cost, as S$ ate not relevant about the future To words, an opp ort un Ure
ity cos t is the 2sacrifice inv
Othe
h in ban k at 40% p. a. inter est. Now, it is
decides to
that a firm has just paid 7 1,60 000 for a special osi ted 7 1 lak 4. If the com pan y
y has dep yield is 12% p.
ver though ater cost outlay hae heen a pur for example, a com pan the ty cost.
TeUeVE the¢ Words the decisi made, ¢ 1,00,000 invest ment | 4.00,000 ine is a sunk oe amou ne in debent ure s whe
a proposal to invest this have to forego bank interest of ¥ 1,000
re
p. a. which is the opp ort uni
rueve the co ec is found
bay | the machine re machine Is a sunk cost. will uted cost tha t doe s not require cash
the fact that wan af its decision Arture decisio in can cause * no amount of 1 egeet c invest in debentures, it a pure decision-™ aking cost. It is an imp
Fequently na RE costs, whi . tor any
ay Oppartunity cost ig boo ks.
Costs, which to be avoided. Despite ting
. costs, are irrelevant ee nek entered in the accoun
historical com tsed im detail] Defoe justoricale decisions about future courses of actionm aare Aecisions, they ae outlay and it is not
made. . For ex, amale abich ig being
sé cs] b if Ot ana:
asset identical to hak why
Replacement Cast
et Future ta x
1 cted paymentsn which
j
be purchase 4 an
i
will differ depending on the course of actii
future costs will gemgi e nt. Moreover a On .
which there could asse t. When the
alter: an analysis of historica
l cos _ cost of replacing an
Sunk ¢ ost and iffej r under alternative courses of action . ts may provide information about ho Ww This is the cos t at
cost is the cur ren t mar ket
ement cost and
replaced. In simple wor
ds, replacement p in mind its replac
Ctween these+ 4 ineleva nt costs are . © fan asset, it has {3 kee has ed in 1995 at ?
ymous and one should under:
not synonymou t he rep lac eme nt ery pur e
Not all ictelevant cost management considers ier. For example, a machin 009. So the
€Xq
Mole, in: cho "0,
osi ng f but alt sunk costs are ile the difference
To take an it was purcha sed earl
type is purcha sed for © 15,
Same under the om the two altemnativve sunk costs
methods of production, if direct inate not the cost at which nd a new mac hinery of the same
3,a
cost is th @ 10,000 is discarde din 200
it will be & inece emativ: es, it Isis anan isirri elev cost. Bat the op material cost is nota@ awellsunkknowcostn
: : eral
machinery is z 15,000.
auec
t 20 ' and ant
replacement cost of the
inlet on the subject gute st has the same meaning as the past ot rngren, Cost and Implicit Cost)
Ho
(E whereas many
Out-of-pocke Cost xplicit mn (such a wages rent)
Past costs are t ain co sts which require cash payment to be ade costs, also known zs plicit
ex
Diffare 2
. ential (or Increm There ate cert ach as depreciation). Qut-of-pocket lisation of curtent resources.
cash out lay {s e the uti
ental) Costs costs do not require h outla ys of requir , etc. Qut-of-pocke
t
cost m s that involve cas insurance, pewer cost
other hisword 5, diffeber
renc Gar ded as the e difdi ference ini total cost zesulting from a contempl costs, are those cost mat eri al cost ,
direct wages). Dep rec iat ion
if e mplated change. In cket cosis are wages, le (raw macerials and
Course of action “tential cost : is the incr ease of decrease in total cost that results fr ‘om an alternati Examples of out-of-po nag er's salary) or variab is not en out-of-pocke
t cost.
ive eit her fix ed (ma and the ref ore
Ttisa Scertained : cost may be e any cas h out lay an aid in make
altematy ve. The : choice vr 3ubtracting the cost of one ative f oh the cost of another
alternmet ery does not involv frequently used as
vi olume, change al in ternative in hod on plant and machin -of-pocket cost is
ay arise because of change wn as imp lic it cost s. Qut
er decisions .
duc t mi isi of produc
tion, . Inin sales Such costs ate also kno sion and many oth
For
. diffenterenalt al Dro
tem cos t ana
muxis, We
lys
.
makn or buy decisions, take
rem
or
ent
ref
al
use
rev
dec
enu es (th we or buy decision, price
fixation dur ing depres
eed to know the inc
and ine
st {the ch ange i e change in rev enue} made now
"ge In cost) arising from the decision Future Cost t is histo ry and decisions
Marginal Cost
wha t has alre ady happene a, The pas 1g for dec isi on- making are
nge nt cos
Marginat No derision can cha Thus, ¢ ne only releva for
happen in the future. lby provide 3 basis
thing veien cost8 the € adaadditional
as Variable cost of produci
“ys
uni ‘ same
can affect only what will But it is the hist oric a 1 cas ts which genera
giv en due con sid eration
ducing one additionala hechetag Marginal cost is the
: future cost s. cure are alsa
costs to arginal costi
(or variable costing) is predetermined or relationships in the fu
Fixed cost of charging only variable
9 products, Inventory
and written js aesting s. However, changing
computing future cost
:
analytical ft ini Profit
andof decis; valued at variable cost only.
and Loss A ccount of the period. Marginal costing is alsteated
ision- mak
as period co st
0 a very important
while estimating future
costs.
tool in the hand s
-
UY. pricin
ductets .chapteing
Proara
book MNsy 0a sep n : of sales-mix, etc
i
“the tackmteueue oO.i helps 7 in decisi. ons like maksed@ inOI Conversion Cost duc t. This term is used
to denate
imthis
- os
eriat into finished pro
sys
al costing is discus
oF words,
margin con ver tin g a raw mat a product,aeIn cther
It is the total cost of in che production of:
. .
}. cos ts be
y ove rhe ad . this cos t can
tl
ee Costs . our amd factor : : . Appropriate use of
the sum . of direct : lab cos t min us direct material cost
fac ter y
1852 are
VPothetic conversion cost is the
purpose s which are spe ci i
pecially computed outs side the accounting
system f
made in certain manageria
l decisions. ce
interest of dec ision-m, al g. costInt
capit al ; akin t on capital invested i e of imp ute d or the
On usu all y res
18 a common typ . As Direct Material son CosCast
when deci dine al is lud ed in cost , it i ary to tak e re ted cost C onversion
outth Y Hot
interest ¢ ost m9 ayabresult; @ altecnati
inc
, it i s considered necess it into acco Prime cost pireet Labour er
i
of Rs @cnative capital
projects. The failu to consider imputed ted Factory Gverhead
9,000 and proje ct“in an erroneous eiheelavestment
Bt 4g 000. B For example, project A re ire
par t of prime cost
as well as conver
sion cost.
tab our cost is a
. . Both the. projects are expected to yield & 10.000 ,000 =asinvestment
additional It should be noted that
|
Management Accounting
Decision Making
65. a
circumstance
COST AND NON‘COST FACTORS IN DECISION MAKING Selling prices under normal must cover total cost (i.e, varia
ble cast
vost Inhen ‘
decisicn me ing, the management
maki agen h as to consider the long run, unde r norm al circumstances, the sel ling price l for the surv ival of a
i not only cost In le amount of profit. This
1s esse ntia
give a reasonab
Por examples a cctors oF awantiative factors are those which van he as a ence aait Row plus fixed cost) and also
imple, 3 | ingBe taetna
he selling Br price of business. to be fixed below
| production
jon of of which
which i the setting price may have
met adverse market conditions, be temporarily
per unit,20 the management
seeNl wi ad , Say ; ' 20% margin
argir afofaiebt
vargin profitaodand nd dec
deci
decide the selling i pri ch ee In the short run due to In other words , the selling price may
above variable cost. upon demand
the mathe o I Mpany is selling its product at % i apatites a total cost but it should be amount of contribution depends
com tribution basis and the fixation at
25 por unity rl the nee
starts selling g the
the same productct atat F% 95 95 p per unit,uni Now Now thethe compan
company at fixed at marginal cost plus it shau id be note d that
a
will‘incurs
i kes
enes s of comp etit ion, non-cost factors etc. But
“ost considerztione al pre Breauce ‘ and supply, acut
coiapetitor's price because its ow n cost isi € 100 per rani uni Se 4 short term basis.
‘on ares eae,1 wae, the compan cost may be made only on
pany should : stop selling q th the product because seltin vestlte fn i selling price below total
product ty cpoid he te gement has to decide whether to sell at a loss or stop vroduetion ofthe
and depression
Pricing in competition products may have fo be pric
ed below
i In such
wor-tOSt a situati
st Foca e is acut e comp etit ion or in pe riods of depression, able cost tech nique is
cation. on, it‘itit isis difficult
di t 0 makek a decision
is on cost fact i When ther situation. When vari
i. ssary to mee t the special ribution
woking pore nee factors be brought to the attention sf the ee semen total cust, if such a step is nece variable cost so that it make s a cont
should be higher than the
he dent
those cannot
arnieh
s be expressed
@ there is a danger of wrong g decisi
decision being
i made. enNon eecision used for pricing, the price price is just equal to vari
able cost. the amount of
loss. When
possible to guantin in monetary terms with aceur. acy. Fo towards fixed cost and
help redu ce the
situatio ns the selling prices take no
victor te the conn
1 Tei egartons ar
equa l to the amou nt of fixed cost because in suc h
tems the effect of a decline in the morale of er wees or loan 1oss will also be
r
Noncee he ' any closes down the manufacturin conttibution towards fixed cost. etition, if selling price is highe
slans th seer spot ay according to the decision under vonlieation
iliti varticular on ese
al circ umst ance 5 like the trade depression or comp stop ped. This ts
Thus, under speci be
ean
n. For ular a produc
example, the pro duction should not
i ess and it recei
company
varia ble cost, even thou gh it is pelow total cost, duct ion is cont inue d or not, and
Tat a price wh than of whether pro
to be incurre d irrespective
because fixed costs will have
cost.
the cost.
On cost
Howes, ae Ne export order should
factors wld be
be owetighthe
outrightly rejected because it ve 4 ome owe ing the amou nt of loss.
will help in reduc short-term
the company fe ‘atti vmagement should consider the no-cost factors such 0% a “armed i continuing the production
selli ng price belo w total ¢ ost should be made only on a
on, fixat ion of e of short-term
1 g the export house status, earning valuable foreign orchingege ban As a note of cauti com panies to take advantag
from the
the wae
order
ng base d on varia ble cost plus contribution helps long -ter m basi s and thus in
pasis. Prici d toe incur loss on a
and 30 oF For gecern g t hese ber ents. 1 the
A c QI
same time, no firm can affor
pany should consider thcurny 9 a small and tol ela bl ie
opportunities. But at the a Teas on abie amo unt of profit.
and give
price must cover total cost
Variable
the long-run, the selling
Costing and Differentiat Costing a§ Aids DB vecision
ill ion Mak in g

Variabl 2 a arr a. ginal


Example
% 1,06,000 (total)
costini g an d di i
I I ‘ j : J fferential cost anai! y Sl$i dTe the two valuable techn q ues use ftOF
Fixed cost & 7 per unit
Variable cost % 8 per unit
DECISION-MAKING ANB VA Current market price 50,000 units.
The most useful contribution Output
ths of isvariabloyecosting
i isi the assistan
i ce tat
in vital decision-makin
vitae that ititi renders to the manage Should company sell or not?
vei teeta ea ' variable costing is an invaluable aid to
decision-making, § cement Solution
make costing proves its worth in decision-making = © 3,50,000
4 are explained
we explaine @ = 7)
helow: . Variable cost (50,000 units 1,00,000
Fixed cost 4,50,000
1. Fixation of Selling Price Total cost

00 units = = 9
. Although
- factcrspricethas are regulatedd more Cost per unit = Z 450,000 + 50,0
more b market conditions of d
economic by
n by the decisions of management, the mnanegenen wha Rey
and other to cantinue to produce and
not cover the total cost, yet it is wise
ile fixing prices has to Although the selling price does will be meurred if
keep ; in viewow th the level of Pp profit esired,
desi In the long-ru. ; ‘ account of fixed cost) that
he high will reduce the loss (on amount of fixed
gher than the total cost as otherwise the Mitte prices of products or services must sell because such a step d be % 1,00.000 [ the
e is stop ped. If prod ucti on is stopped, the loss woul
Variable costing aids in production as follows:
continued the loss will be
earned But freque atl circumstances
cost), but if productian is
arise for Man agemen
ge t to
consider Spécial condi ons and sell its re g decisions such as : % 4,00,000
Sales (50,000 units @ z 8)
ar Pp reduc
ui ai dq
special price which m may be lower th* at * Fixation of selling prices F 4,50,000
vat price woich cost + Fixed cost)
selling prices is discussed below: an the total cost. Fixation of * Exploring new markets Less: Total cost (Marginal z 50,000
* Make or buy Loss
i % 50.000, Le.
index normal circumstances cost, the loss is reduced by
* Product mix uce and set at below totai
n times of competition
iti and/or
| trade depression Thus, by continuing to prod
* Operate plant or shut down.
from % 1,00,000 to % 50,000.
Decision Making Honagement Accounting
6.7
Special Cireunistarices wh when
ip hie Circin Selling Price is Below Variable Cost
such 0ventwalg fale below variable cost, the loss will be more than the Solution
amount of fixed cost. In ce of 20% Capacity Order
su ” , ent Year Prior to Acceptan
Peer
wil be better tc stop P p producti i : the amount of toss because Budgeted Statement for the Curr
reduction Means loss will be just equal to fined cost Total
Per piece
renee é
the win Jn certain special circumstances tike the followi
produ
' ction may be continued even if
ing price is below the marginal cast
nn ere
L 2 popularise a new product, A new product 14.50 . &.70,000
introduced in the market may be sold at a
Pe price so as to make it popular. very Sales (60,000 pieces) “ 2,52,000
420
. , saminate competitors from the market. Ditect material (2 4 + 5%) 4.05 63,000
po

hee ee of perishable products so as to avoid total loss Direct labour (Re1 + 5%) 3.00 1,80.000
ow

wee re s0 a foreign exchange, Government may allow Variable factory overhead __ 8.25. _ wld 200.
im port quota against foreign
rot at ow aeand profit
profi from import
i quota may be more than the loss on exporting the Variable selling overhead 8.50 3,160,000
i t 6.00 — 3,60,000
. To keep P plant
pl and machini ery ini operatiion as idle machines may be liable to deteri ibution
yaContrtei (Sales - Variable cost)
6.
oration
ti .
To prevent loss a f future orders as temporary closure may = © 2,47,500
Fixed cost % 2,25,000 + 10%
break business connections with
customers that tan he re-established ata heavy expenditure. = Contribution - Fixed cost
% 1,712,500
To helpPinin the sale of a conjoi conji ned product which i i
Profit 3,560,000 - 2,47,500 =
}

ff
8. To maintain production and to xaep omloyets sccapi = 1,60,000
ea ange Profits & 47,500
Planned profit 1,60,000 - 112,500 =

nod
required .
2. Exploring New Markets ‘actease in profit (or contribution)
0 pieces (order
ptokit:Someti
basist mes, a com pany iss not able to fully ly utilise
| Vanaiie cost of additional 20,00 8.50) z 1,790,000
Te we utili plant capacityi when selli 0 « €
for 20% capacity. |.e.. 20,00 . z 47,500
huik otter or sport 2 met me xpiore new markets and find orportunitie contributi on desired
s to receive aditiona Add: Addit ional —_& 247,500
price makes a ‘contribution’ Th eowhichay may Bebe below # total to cost buti above variable
vari cost “so so that
that the th Total sales value
Cost fs ateady 2 tone @ en of contribution form such sales is profi t because fix i + 20,000 units = % 10.875
total : cost is nossite only because
m currentin sales at ; total cost plus profit b: asis. Such additional Selling price per unit = % 2,17,500 Vonroh
t ef
ition sales at Speke b 875 so as to ensure an OVeFar ™
possible minim um price for sale uf additional 20,000 units is = 10.
aceepting bulk orders ice discrimin eiow Thus,
e. In this way spare plant capacity can be utilised to am alttee % 1,60,000.
Rt
profit. “serimination is
Additional Order for Utilising Spare Capacity Export Sales
or below total cost
a foreign market at below normal price
oon when a company has a spaze (or idle) capacity Additional orders may be accepted from sales are at a arice which
which it is5 not able to utili tilise becaus yield additional contritution when such
straint and it receives a bulk order at be how normal
selling price, such an order should be see vel but above marginal cast. Export sales
provided existing sales are not affected ig above varlable cost.
by piice discrimination. Ht will earn the company additional additional factors should
profit, by utitising spare capacity. ing export orders, the following
While determining profitability of accept
be considered. quality checks,
costs like special packing cost, additional
A manufacturer of plastic buckets make . on a setting price of 1. Export sales may result in additional costs shoul d be deducted from
rofi of € 2.50 per piece
$ an average profit if not berne by importer. These
manning and selling 60,000 pi
@ 14.50 by produc freight and insurance charges, etc.,
eces at 60% of potential capacity, His cost of sales is: export order.
contribution to determine profit from government, exemption
may result in certai n cost benefits like expor t subsidy from
2. Export sales from export order, these
Direct materials drawbacks, etc. In determining pro: it
Direct wages
epee or concessions in excise duty or duty on.
aided in contributi
Factory overhead (variable) 100 items should be deducted from cost or
300
Selling overhead {variable} Tllustration 6.2
0.25
025 produces only 2,000
Total fixed cost is F 2,25,000 to produce 5,000 articles but actually
Indo-British Company has a capacity
fixed Dun:
oan current year, he intends costs.
?* articles for home market at the following
i to preduce the same nu saber, of units, . but anticipat
ot ng up by 10%, and (6)-material and tabour t
cos ts will go up by 5% each. pates that (a) z
orice you we € circumstances, . he obtatams an offer 40,000
for a further 204 i i
uld recommend for acceptance to ensure an overall Materials 36,000
syoft oF 1 60,000" » Mba minima
Wages 12,000
Factory Overheads -— Fixed
Decision Making Management Accounting
ee
65 "6.40
ourcing Vs. Outsourcing)
Administration overhead _ mnie 20,000 3, Make or Buy Decisions (ins sion has to be taken by the
management
useful assistance when a deci
Selling and distribution overheads — Fixed 18.000 Marginai cost analysis renders rnally ot purchased from an
outside firm.
should be manufactused inte
- — Variable 16.000 on whether a component part ing 1s the proc ess of purchasing
is prod ucin g the good s by the firm itself whereas outsourc rely on outside
Total Cast "7,52,000. insourcing car manufacture may
de suppliers. For example, 3
the qoods or services from outsi manufacture other parts inter nally .
sue Th | market can consume enly 2,000 articles at a selling price of onent paxts but chooses te w firn:’s own
ddditivnet an % 80 per article An yendors to supply some comp is avail able in the market at a price belo
der for the supply of 3,50G articles is received fr om a foreign country at % 65 article when a comp onen t part
is part icul arly so a deci sion can be
nouldul this order be accepted or nat, if execution of this o ,der entails. an vec This may be misleading. Such
based on total cost analysis
additional packing cost of total cost. This type of decision inal cost. On the face of it,
arin g the outsi de supp iie’s price wit? firm's own marg nt by which
sates arrived at by comp cost. then the amou
(M.Com, Kolkata, Adapted) cost to manu fact ure the component ‘s its marginal Ther efor e, i. Wh be
since the only arises in making.
ier's price is the saving that
marginal cost falls below suppl below firm' s own marg inal cost.
only when supplier's price is
profitable to buy from outside onen t is Z 100 per unit, consisting of =
80 as variahie
Statement of Marginal Cost and Contribution maki ng a comp
— For example, total cost of to supply this comp onen t at $99, if
{of 3,000 articles for export) ose, an outside firm is prepared
cost and 7 20 as fixed cost. Supp y of cost analy sis will show that wach
that it is chea per to buy the component. But a stud fixed cc. °
way appe ar recovery of fixed cost. This
contribution of = 20 towards
Tnirvints (woap ©& 29 per article
Matrviate unit if manufactured makes a the componen t part is only ¢ 80 which
; or buy. The real cost of making
Wages @ € 18 rer article to be incurred whether we make be acce pted beca use if accented, fe
5.0 of % 90 per unit should mot
Vardnable js its variable cost. This offer 20 of fixed cost which caanet
be
ie, F 90 of purchase price plus
2
overhead — Factory @ & 10 per article
sain % 410,
—- Selling and dist. @ % 8 ner artirlo component will really cost
—24000 . -
sles (sano ante 0 Marginal cost
‘ost of sales saved if component is not produced consideration should be-given
to other factors. For
” 1,68,000 final decis ion, due
|
_ However, before arriving at city released by the non-manufacture of
7,95,000 idered as to whether plant capa
example, it should also be cons
some altemative use or not.
eess) Additional packing cast athe component part is put to
memes
“con
. Additional profit Illustration 6.3 the same is
cept d nreoe
Accepta
«oop
that while it costs 6.25 to make component R-518,
A radio manufacturing co. finds
accepte af this export order2 results in additional profit of the
.
to of % 24,000 and th hus the
h order supply. The break-down
;y ee should b e each, with an assurance af continued
available in the market at ¥ 5.75
Note: Fi
ote: Fixed overhead Rave not been taken into acc ount i idi a cost is : z
oveth in decidi
ethead have already been recove zed from sale in theunt home ng
market. the
acceptability of this order because fixed 2.75 each
cost ; Non-C
otoe Materials 1.75 each
ost
t Factors or Qualiti ative
ative Facto
Fact rs. Apart from cost and
oa i i i Labour 0.50 each
route also be ne kept in mind while making an onporting detcision Other variables
fon. these
These includee
e : * 4.25 each
costs
Depreciation and other fixed
6.25
(8) Export house status.
a\fece) :Enh, ancement ini company prestige and goodwill, etc {a) Should you make or buy ? at © 4.85 each ?
if the supplier offered the component
a} Employment opportunities.
a (by) What would be your decision, (B.Com. Hons. Dethi)
Conclusion
Solution.
:
normal
>!. thIn ae
ti . the component is shown below
um
prices should be based on total cost plus profit The variable cost of producing z
plus basis «9 as conditions like trade depression and competition :price may be fixed on margin
al cost . . 2.75
preparers vie, contribution. This is valid only for ashart period. Materials 4.75
.
may be accepted at less ns Pare capacity, bulk orders from home market or fy i Labour 0.50
an but above marginal cost. This adds to th i forei
gn market Other variable costs
company. This is possible onty whe 5.00
n price discrimination is such sales in diferent mae a oe
narxets is possible. Variable or Marginal cost 1.25
:
Fixed cost
ting
Management Accoun

6.14 in two parts :


Decision Making 6.12
product mix may b e discussed
——
the most profita
bte
of selection of
p TM ent at z = 75 ear ! because it j The discussion Key fac tor , and
On the face 0 f i, it a ppears that iti is che a per to buy th, e@ compo 5.75
be saved if
it Is. is no
. {a} When there
tess than ; own cos
an
t
cost of € 6.2 bat it sho uld mak unders
be ing it.
too
Thu
ds that
if thethecomfix ponedent i cannot the re is a key factor.
the ane {b) When
it is decided to buy mt instead of ent .is purchased theen it. ) factor Analysis. When
there
wilt really cost % 7 per component iLe. % 5.75 paid f or purchase ther e i 5 no key (of timiting Cos t Volume Profit iet ed as the
ponent‘
“Diced ae make ined
cost which will (a) When was explained
1 ion is consif
.
continue to be
incur ted
the compon ent, But tor am o unt of contribut s in sales
- int bs saved It is therefore . to b The concept of ke y fac
provi des the hi gh es t due to change
Secision to make or buy ail be
enced by the fact wheth
er the capacity y to to - be released b y the non- t mix that t does not change
js no key factor
, the produc ¥ hen fixed cos
anufacture of the com ponent canb 2 put to some alternati ermative use or not , Tesulting in a savi ng of fixed
; This holds good sales mix
is av i most profitabl e sales mix. cost. then that
cost. If yes, , buying q will
w i be prefi compon :
costs € 6.25 at 5.7-F5, But ch an ges i" fixed relative
. " preferable util sedas, the i
be be utilised, it should ponentnot which
b .25 is available " es m ix ate associ
at ed wi th
le gales mix. in
othe wor ds,
their
if the released capacity cannot sal ab
mix
but made
e
b
by the firm. However, when
chan ge s in the most pr of it
of the b asi s of
(b) Tf the pri
r o t e purchased
. fit is cons idered as profit and not
unit, then the off er can be accept
pro
nee will be a saving
:
hi gh es t is of the is
which provides the s will be evaluated on th a at ed with chang* in fixed cos
is % 4.8 bas t.
. ne price offered
uni 85 per : released cannot be profitably sr oteyed, . This is so because th °
of . 15 paise per unit than i
even i the capanty profitability of
wixe
p rad uet mix 3 associ
price offered is less @ cost of the product . en a change in
contribution wh
. . factory:
Gutsourcing and Id le Capacpaci ity.ty. W When a firm has no spare capacit’ pable of achi
evement in a
iinvolves setting g asiaside oth l alse 2 com ponent
give {ustration 6.5 x are ca
ion tof Sao wai shoa uld to
be 5 mi
production/ sale
also n . due
:
e e loss contribut The following of product C.
consideratio n. In othe r work pro ‘table
to bu
y only whe n the ourc
purcha h se pr od uc t A and 2,000 units : C.
of units of produc
e@
loss of contribu
ti '
ribution of di isplac best fouunnd by y ththe {i} 2,000 anits B and 1, 600
cost plus
o h oes r work . The loss of contribution is ascally fii) 4,000 uni ts 0 f pr od uct B.
A, 2,000 wnits
of pr od uc t
wu unit of son ato . ts of product
se of contribution per (iii) 1,000 uni C
Illustration 6.4 js as follows: B
Cost pet unit A
16
40
Manufacture
t of product
oa A tak ine No. 101. It h i i 20 20
10
@ hours manda
the margi s 7 % 8
Marginal cost of ¥ 11part
vost oFof conga
component OT OY a at made
could bebema
eeal
¥aide supper’ on machine
pie Nar. n0t in
in 4 hours. The Divect material &
C ar e % 2. #4 and
Direct wages ad s per un it 0 fA, Band pec tiv ely .
he pet unit res
yariabl e over 40 and € 100 g from produc
t
ake or buy component Y. Discuss in both ications ah en— ™ € 20,000 and C are q 36, % profits resultin
: Fixed cost is af A, B an d C and the (B .C om )
: - ling prices uni tof A, Band
{2} Machine No . 104 is worki
; is idl e cap aci ty. rking at full'capacity. respectively. Sel
Determine the
ma rg in al co nt ribution per
(b) There and (iii).
mixes (i). Gi)
Solution
‘a a Solution
ia) Contetbution per unit of A = @ 150 - 110 st State ment
Tt vonponent ? 4 machine hour = % 40 + 20 his SP h Marginal Co of products
-
Per unit
the tltotal costo make
cstto mae cnet
in manufi then asas it*it fakes
actured then hours,
takes 4fou
h the k loss
the
s the ibuti
los . of6 contribution is % 8 (i.e, 4 hes. @ %
B -
A gz
z
b se | " z
; hate so
15 andi so
of nd
it i t ta
so it is better
5 {b) IE, however, un of %
upplier’s me price we
contrib a40
be. e tots aof case, a
veo ae te component Pad waly capacity, then cere woul
ee
makingthe product than buy wo be its variable cost, Les @ 8 inh
it would
md
16
contin ical toof make est
Selling price (5)
a decisi " a &5
following
0 non-cost farb ative Factors. Whilei making
i Direct material _ 4 a
(a) Aesura ctors should also be considered ision on make oF Buy com a 68
Direct wages 30
continued supply, if bought from outsid ™ ad
i) Assurance Variable overhe 30 10
32
eee
ic) Aesuranee o aa of the product by the supplies. * 4 TT
ce increase during the e peri of agreement
period Yaniable cast (¥) a
(Ss - V)
Contribution _-
a

4. Product Mix Decisions
. .
i
Sales mix or n in which va when a bts are sold or nproduced .
duct mi denotes the ss
mix profi he pro por tio
m
The ; proble
mo g a
of selectin thus, arises sty
of sales usiness ent:erpprise has a

ot product line s and profitalie ga contribution of of its i


own.helAr
ps erange
:
sales mix also results
ae gin al cost ing
gement in determinin g
e change in
profit 1 ngs
The tec hni:que of mar
sale tion
mix
posi m.
.
the most profitable
.
:,
,
a
Management avve---
Decision Making
6-14

Statatement Showing
i Comparative Profitability golution geted Pr ofit
{i} Statement of Bud
Contribution Fixed cost
Total Profit
~ Fe - contribution
(1) A 2,000 unsts 000 s)
C 2,000 units j 6,600 Budgeted production (unit
7 54.000
14,96,000
° ‘ 20,000
.
Salling price 2
-
(7) B 4.000 un 40,000 *0.008 sales (S) 48,000
.
ss
ts
49,000 20,000 20,800
’ 5,12,000 1,28,000
~ 48,000 .24,000
Raw materials
{1s A 4,000 units 5,200 32,000
48,000
96,000
120,000 | 10,00,000
29.900 Direct wages ___ $4,000
B 2,000 units
Variable overhead $08,000 | _2./2,000
Total variable cost (Vi
a 2 4,96,000
€ 1,600 units
— . 31,200
— ' ‘00 20,000
: 57,200 2,88,000 1,12,000
1,712,200
Conclusion, The sal es Mm. 1x
Contribution (S-¥) 3.24 800
(i 18 th & most pro: fitable as it ylelds the hi g hest amount
and profit,
of COnLT be 10n Less: fixed cost’
_
Profit
(b} When there is a key factor Calculation of Fixed
Cost
= % 57,600
When a ke y factor
Lari is operati
:
profitable sales i i . A= 6,400 unit « zg
70,400
Th :
ing, selection of the most. profi of vontribea on contribution
~ per unit of key facto which makes the highest amount B = 3,200 units x ¢ 22
factor, is the most notte —_————
43,200
ee we Unit of key C= 2,400 units = = 18
2 and its procuction is pushed up. The second
factor aa 1s to be given e © 1,71,200
te product which yields the key
second highest yicoatribution per unit of Total fixed cost a
or and so on andand in th é
whic key factor
yields teast contribution per unit of
uced
end that product should be prod rach is the key factor
extent of availability of the key ced and to the {ii} When raw material B c
. ’ ee A
i Case 2 numb: ero
“TY ke y
Oo 2 kg 1 kq
actors
0 are c perating
r SH ult ar eously ie the basic principle Temalns the same bi t oo a 4 kg
problem : become 5 More mathematical output 2400 = }
Raw material per unit of
i Programming to d 3200 = 2
the optimal product mix ical in nature and one has to tesort te Linear Ing to determine 6400 4
sumed (kg) = 2,400
, Total raw material con = 29,600
= 6,400

6.6 % 112, 000 % 96,000


Mlustration ; = 2,88,000 = “Fan ko. = P00 kg.
rial = 75.600 ko.
A company y¥ m, manufactures
tinder three products. The budgeted quantity, , selling *Contribution per kg of raw mate
selli prices and unit ¢ osts are
== 17.50 =% 40
7 = = = 11.25 I

Po
3
A B ii

a eate(@8i 2p
Ranks as -
raw material is calculated
805 405 20 *Contribution per kg of ed
sum
s (@ © 5 per hour) Total saw materials con
i wage
Vari ble 0 Total contribution = .
overheads
aria
10 30 (raw material is the key factor)
i Suggested sales mix = 2,400 kg
Fixed overheads g 20
2,400 units x 1 kg = 6,400 kg
pan Production {in units) 3,260 Rank 1 — Product £ -
6,400 - 4,200 units « 2 kg = 9,600 kg
sUing price per unit (im 2} 120 2,400 Rank If - Product 8 s x 4 kg (balance)
2,400 unit
Rank Tlf - Product A ~
140
90
ble 18,400 kg
Required ; Total materials availa s
.
s and ¢ - 2,400 unit
TTF

() Present a statement of budgeted profi uct mix is: A - 2,400 units,B 3,20 0 unit
Thus the suggested prod
(it) Set optimal product-; 1
mix and d etermine
. “ Contribution
vine the profit, if the supply of raw materials is restnicte
rest d to 18,400
. k g. Caleulation of Profit = 1,08,000
(B.Com. Hons, Dethi) Product A 2,400 units @ % 45 p.u.
F 1,12,000
Du.
2 ‘3,200 units @ € 35
= 96,000
c 2,400 units @ % 40 p.v
3,16,000
Total contribution 4,271,200
Less: Total fixed cost
Profit z 1,44,800
Decision Making
Management Accounting

5. Plant Shut Down 6.15 6.16


Decisions on
The management under cert 10,000 units
the business, is better
ain cizcumstances might feet Example: .
than Operating at a loss. that plant shut down, 48, ity © of plant\
capacity
is not always so. This However, variable costin closing down Normal
: som :
type of situation usuall g analysis may prove
that this ix
Fixed cost when plant is operating
This type of decision y arises when sufficient sale : ;
May be sith er s cannot be achieved, i
Fixed co: st when plant is shut down
Permanent closing (a) temporary suspensio
down of production, n of Production activiti
es, oy (b) Variable cast per unit
Temporary Closing
Down, Temporary Susp Selling price per unit
is usually to stop ension of activities js a his, pric a - 5,060 units.
operations untit trad short-term measure. imated sales volume at thus. _
when should operations e di epression has passed The object Estim, Marginal Cost Sta tement ~
be suspended? or in othe , The question before Ce
r words, hosy long sho Ma na ge ment js. ee
ite answer to this question is that if uld operations be contin 4 00,006
=
nl
generally Speaking, produc products are making a contri ued? ——
tion shouid not be suspen bution towards fixed cos
t, then ~ Sales (5,000 umits
7 80) OT
nits .@ mis TTY

help minimising loss whic ded. This is 80 because cont “1


h would be incurred if inuing
solve this type of problem Plant ic b+ 9 wen ry us, the inform production wil Less: Marginal cost {5.0
invalves a comparison betw ation heeded tp vom
the loss that would een probab te loss at a give ;
be suffered if production is sus n level of output ang LOntTIOULON
pended temporarily.
Example; A manufacturing Fixed cost if plant is operating
company s upplies you che
following informatien: Loss
Normai capacity of plant
_ s,
shereaas,
0 where an fuseif plant is:
Fixed: cost 10,000 units
on ‘xed cost would be % 86,00
Now if the plant is shut ,
Marginal cost per unit % 1,00,000 ec nt
. al the certa in
operated, the loss would be < 75,000 .t fet ya this Oper Fae : oa ati . ‘3. og
smalt
lant offers
. | “riled
Estimated selling price % 75 k epiee in view me
Estimated sales volume at this z 80
7000 tag
es advan
non-cost e tie
tages ke ing
like keep weeplantan
sng the ae advis
in gear, ning £0ine¢ tinue the praduction even if there
retaiable
selling prize MIL
5,000 units mana gena ial pers onne l. ' ss_ ied cost,
labour and
am n us actors utweiigh the loss.S.
Mar ginal Cost Statement mail
st an d makes no contribution final towas fi
the a i
sellin g pn
or De
ce is below Oe
Ne the marginal cost . deciision
sion in the
Total sales (5,000 units
« = 80) He In acase he derations, the plant should be temp i t down, But a
fess: Marginat Cost-(5,000 4,00,000 then on cost consider iderations, :
orar ily 3a of shut down on plant, fear of
units x # 75) nsideting non-cos t factors
Contribution 3,75,006 regard should be taker after colati‘ onsh ip. witith workers andaq supp liers, etc.
: down of° busi.ness is concerned, suc suctt a
tosing the market,t, effect on Te So mt closing do ect
Fixed cost far
25,000
Permanent Shut Down. So d beas petm+ane : ‘
hen in the long long run, the bus:
iness doas not exp
,
1,00,000 takenonl
y ords, inrun, the long-run
-run,, selling pric ce
decision is a drastic step and shout ‘
Lass ——
the risk invalved. In other able retu
(-} 75,000 to earn a sufficient we a nost but should also give a teason rn on the capital employed.
If plant is shut down
loss would
, the loss due to fixed
cost would be % 1,00,000
=.
must not only cover the to
aly be % 75,000. This . However, if plant is
contribution towards is because selling price operated, the ANALYSIS
fixed cost, is above the marginal DIFFERENTIAL COST
cost and js making a
Role of Committed : -eiqn-making, ement has to compare
spe two . skiag
by management whe
and Dise it has been stated earlier that nes the manag mane gement in decision-making
niqu e to help
n cost analys is
as a
veo spec ial tech of action.
alternatives. Differential alt ernative cour ses
Committed fixed cas wo different under different
discontinued,
ts, which shows how costs and revenues

Tr decisions to cl ose
down temporarily, contribu
Meaning of Differential Cost rnative and another 2 it Lcs ob tal 1 ed by
one alte
to be incurred whe tion should be compared Differential cost is the differenc® a
cost between wher
n plant is shut down, ie. with Fixed cost which is of the other alternative. Fot example,
committed fixed cost. he lev
subtracting the cost of one terns Me
from the cost by
be calculated
Examples of fixed costs diff erential cost wilt
which may be avoided induction ucti on 4
management is considering a cha é O
el of prod
and development, part by cl losing down are: adv l,
of salaries, ete, Longer ertisement Cost, research
at Lower level of p from that of a higher leve
avoidable fixed cost the Period of shut down su bstracting the cost
is likely to be. ~ . , the larger the amount
of
-
PGHUYy cies

Decision Making 6.17, 6.18


638
whereas, differential
Example: Differences fix ed costs from its analysis. se
ting & clud es all able ‘xed costs are tho
Alternatt Alternative
:
1. The technique
of variable cos
or traceable fixed costs. (identifi c alt ern ati ves ).
f we Differential ludes identi fiable iable with spe cifi
if cost analysis inc these are i dentif differential costs
. cost/ revenue bet wee n alternat ives since ac co un ting system while
(units) . 7600 2,500 which cha nge int o the for mal
ic dec isions.
Gutput 5.060 : may he incorporated for making specif
~ 2, Variable casts y for rep ort ing to manag emenit are the mai n yardsticks for
z
are worked out sep
aratel
rat io, key facto 15, etc., on the other
g ° ting, contribution
, p/¥
di fferential cost ana lys is,
Materials 30,000 1 In var iab le cos _de cis ion s. In e for decisian-
20,000 ‘ 3.
mante and making incyem antal teverm
Labour 6,000
2,008
000
s ,200 evaluation of petfor made between differential cost and
. js
hand, comparison
> 000
Variable overhead
4,000
‘ 6,000
or ption costing
as well as variable
i.
Kann .
making pur pos es. case of both abs
1,008 the
ny ver hearl
Fived mad e in
t ana lysis can be
wo 5,000
ie _ 4. Differential cos
al cost 35,000 ~ 81000 16,600 “ costilig.
s oe - oy AL COST ANALYSIS
Sales 50,000 20,000 AND DIFF ERENTI help of
” DECISION MAKING s are ™ ade with the
we pro ble ras © £ alternative choice
Thus, in ode: : - decisions involving
additional 2,500 units, additi onal 16,000 (i.e, 51,000
is = 46.000 is ©should Many manager aj e the following:
In words dite
$5,009}, that if cddition : erential cost for 2,500 units additi
other be noted in thi cos t ana lys is. Such decisio ns includ
‘ 00. It dif fer ent ial
example, does not involve any aids in
differential cost will
be e al output cost to be incurred the Cost Analysis
gual to marginal or variable cost. Thus onal qe of Differential
yari " ed costs remain constant the Optimum Level as :
differenti ial cost is 5 ymonymous wi a I 1, Determinatio
n af the decisions such
the variable costs of producing the n-units.
| pro-
.
that differential cost is rymons with -units. [t is for thi Is reason ng optimum level of
erred to as marginal cost. 4. Determini
: Production duction where
cost is also i sa gs is that teval of pro duction.
The optimum Jevel
Sometimes the term incremental
i @ interchangeabl differential cost. The ive at a decision
| cial order.
readers should m. In ordet to arr 2, Accepting 4 spe
note that i
i cost actuatiy means only an creas with profit is the maximu par ed wit h dro ppi ng a product line.
to another. Tn the shave examples ferential 54,000 to produce 7,500 nits cost from one alternative are com 3, Add ing oT
differential costs or buy alternatives
.
units, the incremental Tae OD = 35,000 to produce af this type, the of output. So
|
4, Evaluating mak e
2,000 ifferential) cost of producing
widitional
,50 ,000, | In
units: is = 16,000. rev enu e at various levels ent ial | joi nt/ by-pro duc ts.
incremental eed s dif fer 5, Further processin
g of
the
eeesame way, ay, different
di ial cost may be referred to a decremental cost, ‘ wW where d ecrease in output is bei ing the inc rem ental Tev' enue exc But as :
lon g as output.
e to incre ase the
i
se
costs, it is profitabl e, it is no more
profitable to increa
: - . r eme ntal rev enu
Increment al Revenue. ue, Di Difference in rev enue of two alternatives i oF exc eed s inc
. «at incremental revenues ential cost equals
Ta differenti al cost analysi decions are taken on the basis of vongarenn a soon as the differ
.
neremental "(or decrem ve on of differential al cost with put.
ue. An alternative i .
considered profital : the volume of out
. # is incremental revenue s
5 more than diff
erential cost. In differential cost is 1 when ina month. The sale
,
revenue is § 20,009. This
means ue above example,
produces additional profit io whereas incremental Iustration 6.7 1,0 0,0 09 umi ts of a certain pF oduct
alternative II (ie, 20 20,000 - uc ing
1b . of % 4,000
0 (i.e, A company has
a capacity of prod sale pri ces is possib
le:
6,000) and is thus, acceptable.
t hat the follow ing sc hedule of Selting price per uni
t =
department reports 0.9 0
n
Diff erential
i Cost Analysis Vs. Variable Costing Volume of productio
aci ty 9,000 units 0.80
As stated earlier, r, diff di
cost. At 60% . cap
erenti oymous with variable theory of 000 units
ari able costing* is only a part a fost may bepresyn que of diff st. in et the The Stet
70, 0.75
vari more com hensive techni diff erential costing. . At 70% capacity
0 units
simi r in som @ Tes : se two 0, 00 0.67
techniweeques are are sim : ila
.
r tain other fields. . The poioints of similarity At 80% capacity
mar ise d jes bec ls but als odi ffe in cer c00 units
og
a.
nd differ enc e are sum go, 0.61
. At go% capacity
4,00,000 units 40,000.
Similarities At 100% capacity ¢ otal fixed cost %
se pet unit and
as

ch
. ctu re is 15 pai of each stag e. At whi
d ifferential cost
.
Variable cost of manufa
et

1. Both the techniques


: are based of
se on the classification
cost and variable cost into fixed and variable. When fixed rev enu e and
costs do not chance
showing incre: mental
A

». Roth witteese ne the differential @ same. Prepare a statemt ant be maximus ?


tafe!

jal costing and vari :


are techniques of cost analysi . a will the profit
* : vale costing volume of productio
2, Both the techniques and presentation,
ele

e y the management in decision-making and : aformulating polic


policies.
ting
Management Accoun
Decision Making

Line a product
Solution opping a Pro duct adding or d ropping
3. Adding or Dr age men t may have to decide on sed and rev erse will
: company, the man costs will a Iso be
increa
Statement. se
of Differential Cost and Incremental Rever: In a multi-product sales and certain agement should
aa ue produc’ t line is add
ed. its on, the man
tapaaly at such a de cisi ition
tne. When 2 new d. In order to arrive the overall profit pos
t Variable red Total | Differential pen when a produc
t line is dro ppe
enu e and study its
eff ec + on
; of cost Sales Inere- hap
cost and incremental
rev
compare the differential
cost cost men
utput @ 0.95
z z ren of the company.
— . : : the product line
60,000 49,000 fiiustration 6.9 dro p one item from
9.000 j 40,000 |
ld
0% 1 500 ; $4,000 a company is thinki
ng whet he r it sh ou
|
70% 0.000 40,000 50,500 The management of bet ow are present cost
and output data:
Percentage
80,000 10.900 6,000 h another.
and replace it wit
Given costs Variable
80% 52,000 ae af sales
“_ 40,000 53.500 1,500 | 60,000 Price
per unit z
90% 90,000 40.000 |
13500 | 40,000
Product z 30%
won | {moO | oon 60,300 40
55 000 60 20%
15,000 61,000 60
; - because at 300
thislevel, i Book shelf 100 50%
At 80% volum e of production
profits
He
maximum. This is 120
i: * 4-000 whereas, differential n revenue Table 200
in additional mrokt of z incremental
ae (Le, % 4,000 Bed € 7,50,000
1,500, resulting
differential cost exceeds incremental revenue thereby per year
300). After 80% level, resulting in a los. 5. Total fixed costs % 25,00,000
adding the line
of
Sales ppi ng the lin e of Tables and put dat a :
2. Acceptance of a Special Order sists dro ing cost and out
consideration con ecasts ihe follow
a The change undet the manufacturet for Percentage
Sometimes
Management has to or refuse an additiitional order for one of its change is mad e, Variable cos ts
to accept
a decision sale Cabinets. If this Price of sales
which is below heap customary price, Such an order Product per unit =
2 price
business isat working
profes below full produ
in in attractive when a et . 50%
costs ion capacity and the price offered results in Incremental re Verne 60
9
10%
ich is more than differential . Book shelf 160
we
40%
120
Rustration 6.8 .
Cabinet
Bed
200
& 7,50,000
there is a bulk off expri for export at 50 paipaise per unitit f for the bala
3
; tin q Hlustration 6.7, 7, if
Con uin t per yet . 26,00,000
Total fix ed cos ¥ (Adapted)
i
r the maxaxi
capacity oveacc ept i mum
num prof p
itr o f i t . volvol ume and the e price quoted willwill not affect the int eral sales, willnce
Sales

you advise ng this bid and why? Co mm en t.
pti al be accepte d?
Should this propos
Solution —
Solution
Statement
Compar ative Profit _
Internat Special Order Proposed situation
(80, 000 ier s)
for export (1.00 foals
ee (20,000 units) 00,000 units) Cabinet
Bed Total

Shelf zt
g g
z Particulars
Variable cos : ‘
zg
12,000
Fixed et © 15 paise per unit 15,000
“ 40,000 pe
oo
Total cast
40,000 3 000 . 15,88.16/
52,000 55,000 1808
““”
97,500 6,24,000
Sales 60,000 | soon sa
15,50,000
70,000
Less:
lost [sie
500,000 3,00,000 7,50,000
1,62,500 416,000 | 10,11,833
Profit one Variable cost 9,50,000 433,333
8,000 2,00,000 5,00,000| 7,£0,000
250,000
:
15,000 Contribution 7,50,000 2,61,8333
It is advisabl @ to accept the b
7 0.50 peran unit for the balance capaci of 20,000 uni ts Less: Fixed cost
(/.e., 1,00,000 - 80,000) for crpott ae it at resuit in increase se ofof prprofit b y =pacity
7,000,

Profit 1,00,000
° .™ 95,00,000 = z
= F 26,00,000 - =
Incremental revenue - z 15,450,000
= = 38,16?
= & 15,88,167 cast
Differential cost enue ~ Differential
Fd Incremental rev
Additional profit 38,167 = % 61,
833
= % 1,00,00 0 -
Management Accounting
Decision Making

products should be sold


Tetal profit has increased
i b; : S
tement of pro fit abi lit y and advise whether
ar e a sta
61,833 from & 2,00,000
to % 2,63,833
‘hovld b @ accepting the propesal. , Thus the ... You are required to prep
propesal to drap the line of wb es and add the line of Cabinets accepted. nt oF sho uld be further processe
d.
at spli t off poi
Working Notes:
solution Sales Incremental
ratte cost is calculated as under: Profit at Incremental revenue
Cost Sales at value
Output split off + cost ?
cok shelf (Present situation), Products Split off z
Kg. allotted z
point psint
ws ~ 25,060,000 « 30% = ¢ 7,50,000 z z
n selling g pri
price of book shelf
“if isis % 60, its variable cost is ¥ 40 Th us: z
1,68,000 53,000
? 2 4,15,000
30,000 66,000
Variable cost * 7,50,000 16 = © 5,00,000 8080 85,000
10,000 4,10,000 20,000
% 60 ~ K
67,c00 90,000 23,000
Sant
Book ook shelf
« (Proposed situation) B 5000 2,000 60,000
§5,000 14,000
Salzs = 25,00,000 « 50% = % 13,00,000 41,000 90,000 10,000
c 3000 18,000
80,000 20,000
4000 60,000
D allotted.
Variable cost = 13,060,000 x x40 =f t off point and cost
betwe en sales at spli
it at split aff poin t is the difference split off poin t.
Similar calculati Notes: 1. Prof ng cast after er
ations are mace for other lines
a of product:
vase,s. is additional processi t an d sales after furth
2. Incremental cost s at split off poin
nue ig the diff erence between gale
4. Evaluatingi Make or Buy Alternatives 3. Incremental reve
or
e than incremental
Make or buy decisi i been di . processing. inc rem ental revenue is mot
y decisions
t, wh when making -om on variablele costing have already
of a based t sho wld be further
proces sed if r increm ent al rev enu e is
However,
{ in this chapter Conclusion. A produc processed because thei
€ should be further cessed further
requires addition investment in ie‘an basedeeos in additional differential cost. In thig
case, pro c act Band
t off point and sho uld nat he pro
in ene situation make or buy lecision should be sold at spli
cost tocostbe andincurred.
ifferent:al
ied incremental ‘even
ue and its effect on profit.profi parison of
Sed on compari more than increment
al cost. Produc t A and
D
revenue.
ental cost is ™ ore than incremental
because thei increm ca
SOLUTIONS —
5, Decision regard J Further Pro sin of Joi y~ roducts
PROBLEMS AND
.-
sometimes m anagement has t further processi ‘ai
afer split off point. For example 0 take a decision regarding
:
joint orit by-products
as suchpoint or ifurthe Tofprocess
upto cream
cost sell
into butt: Price’ for Buik order) unit ona gelling price of
= 14.30 by
ee. In such a caseapportionment
reaatdi
x oajoint may split off
not relevant fo T the purposeer Probiem 6.1 (Selling prafit of % 2.50 per
of 4 ecision-making of j o i point is
by- reran average
mak es
Hg regardin¢ further processing of joint products or
ufa ctu
taken an the ha: sig OF compariso, i sing (a} An umbrella man cent of potential capa
city.
00 units at 60 per
voit ean
incremental ‘eee . i profitable to furthhesor producing and selling 60,0 *
process a product if incremental rownte is morecostthananddifferential cost on further
fa procesi is as follows *
processing. His cost of sales per unit % 3.50
Tlustration 6.10 % 1.25
Direct materials
In a com : . . Direct wages % 6.25 (50% fixed)
. pany engaged ln in process indus try, four products e
only, process of 0.80 (25% variable)
eperation. The total cost of err the period ended 30th September 2000 eet Factory overhead
of output, additiona ost Sales averhead but estimates that his
fixed cost
and sales value of the products ae 20 te detaits to prod uce the sam e number by 8% and 6%
Prochicts plit-off-point” ppended below: year 2017-18, he inte nds
d direct ‘materi alg will
incr ease
. During the coming s of direct wages an
Output
intra procfesspoincost
Addiaftetionr alsplit-of would go up by 10
per cent whil e the rate

4
Kas. e
t Sales
vane respectively. However,
the selling price can
ain
not be changed.

s an offer for a furt


her 20% of his pote
ntial capacity.
ufacturer and
§,000
85,000
60,000 : Under this situa tion. he obt
ceptance 0 F the offe
r to ensure the man
3
57.000 68,000 mmen d for ac om. Hons. Delhi)
5,000 10,000 What minimum p rice
would you reco (B.C
C ‘
3,000
2,000 ad overall profit of 2 4,67
,300 ?
and Profit for the
year 2017-18.
4,000 50000 18,000 the sal 60,000
00 are a Sta tem en t of Marginal Cost
-cost. factors in the
foliowing independent
if °
heen + the products are sold at “sptit-off poi . °
point” without further processi
ng,
$0,0 {b) (I} Prep
owing, exp lain ing the rele vant cost and aon
+
(1) Answer the foll saies will continue
‘ sales value would have that exis ting domestic price of % 9 per
situations, assuming the pot ential capacity at a
A % 145,000 rt arder for a further
3 0% of
y obta ins an expo
(i) The compan
B= F-90000 unit. Should this orde
r be accepted ?
js at & 8.25 per unit
(instead of € 9 per unit)
for a further
-C = % 55,000 (i) What will be your
answer if expo rt orde r

D % 80,000 30% of the potencial


capacity ?
Management Accounting
Decision Making
6.23 6.24
above variable cost and will
{iit} What will be your answer if instead of ex ort :
order, the offer is from a domestic market for a butk sup l upply r at = 10 per anit from doimestic market ig definitely company has to ersure
.
oF 30,000 units at F 10 per unit ? port iti} The orde the domestic market, the
— a profi t. But sinc e this is a case of sale in a diff eren t dome stic market and not
(8.Com Hons. Detaiy resul t in
rimi nati on is possible i.e., the buyer should sell in cted. Thus such an
that price disc e company’s sales will be
adversel y affe
s sales, otherwis as re-sales price maintenance,
price
Solution compete with the company' to certain conditi ons such
be acce pted only subj ect
ardet should
(a) Statement of Marginal Cost and Profit . discrimination; etc.
for the year 2017-18
—ble
Pro —m—6.2 (Export Order} of its product in the home
market at a price of & 60
Par unit 60,0 000as i t Expo rts Lid. is prod ucin g and selling 20,000 units
7 Smar
is as follows :
sales per ynit. The per unit cost g 10 per unit
14.3¢0 Direct materials 2} per unit
‘ 858,000
Direct materials (3.50 + 6%) Direct labou

naltcldiee
3.719 2,22,600 =‘ 12 per unit
Factory expenses -
Direct wages (1.25 + 8%} @ 4 per unit
81,000 Fixed
Variable overhead - Factory 1380
3.125 1,87,500 Variable
« 6 per unit
~ Sales Office and selling expenses Z
0.200 3 per unit
Fixed
Variable cost —.
385 price ofthe70variaBO E Exegution of Austraba n
a - Variable
Australia slac ed an order for 6,000 units ata abov ble cost. Should the Austr
alian
Contribution (Sales - Variable cost) mee An impo rter from of € 10,000 over and e
(B. Com. ifans. Deihi}
354,900 order will resutt in an additional total cost
Fixed cost tomplete working.
2,45,850 order be accepted ? Show
eee
P i
Solution
109-050
“Calculation of fixed overhead “ Marginal Cost Statement
. ne
ee cB

Total = Per urt


Factory overhead — 60,000 unit:
2,87.500
—_-_e-eoeeraee
(20,000 units) z
Sales overhead = — its @ 3.125=F
60,000 units @ Re 0.60 = = 36,000 60
12,009,000
,
Sales

pall
10
Add: 10% increase oan
22,350 Less; Variable costs 2,060,000
3
Direct: materials 1,40,000
Fixed cost 2,45,850 4
Direct labour $0,000
Factory _. Bo
Statatement of Price
i Recommendation (For 20,000 units) Variable overhead = —- _. 60,000
_~ Office and selling 24
4,80,000
Total variable Cost i 20,000
7 3
ane
Margi cast (F 8.385 x 20,000 units) cost) -
; Contribution (Sales - variable 2,40,000
1?
ditional profit required (1,67,300 - 1,09 050) _ 6
eaoen Fixed cost — Factory
__1.20,000
Total sales value 2,25,950
saad _- Office and selting | 18
3,60,000
Seltin g price
ce per unit {2 125,950 ++ 20,000) )
20, = F 11.30
‘ pprox }
( Approx. Total fixed cost ~ 3,60,000
),000 1818
cast)
Profit (Contribution — Fixed
cost (8)
of &(?)B.385
The export
= once ae
should be accepted because the export price of 9i
7 48 more than variable
a

matt) of € 18 ea @ acceptance of this export order will mak


ee it) mare an asciiona\ con tribution (or

Ui) The exe nner ee go


wie ats 825 willi
Ines i orde
the amount ofexport
:
result “ in a loss because it is tess than the vari
gnificant, the export order should not be outrightly ejected but
ut management Cost, Since
should consider i
accepting this .
ordert for non-cost considerations i
j ;
enhancement in the goodwill of the company because of export hous t ‘tue ot of foreign exchange ‘
1 e status, ete.
wwe Fe b L&E
Decision Making
C8;
Pye ne Management Accounting
om 6.25 6.26
Profitability Statement (Export Order
)
Problem 6.4 (Export Order) inf.
Expoct sales (6,009 units @ ¥ 30) < A company currently operating at 80% capacity has the following
particulars:
fess : Varaible cost (@& 24 for 6,000 units) 1,80,000 =
1,44,000 32,090,000
Sales
‘ess > Additional cost uf exparts 10,00,006
36,000 Direct materials
4,00,006
10,000 Direct labour
Meee eee On 2,006,000
:
ont ey - rofit ne
et .
Variable overheads
tt te a
26 0a O80 )
. 1
13,00,000 /
»- me
conclusicn,
tai
tance
————
Fixed overheads
Accep of expor
Pp t order frrom Australiai woul of the factory. The order
—- the export order should he acce d i additional profitMosto¢ An export order has been received that would utilise half the capacity
nuimal camestic
pted, rent ap cannot be split, ie., it has either to be taken ic full and executed at 10% below the
3 (Export Order) . prices or rejected totally.
A j manufacturer has planned i The alternatives available to the management aré +
jis level of (as at present), or
are estimated a3 follows, if 0%
af the pt ant capac
]i
capacity
i plant capacit : ‘ (1) Reject the order and continue with the domestic sales only
ity isisute
utilised: away excess
(2) Accept the order, split capacity between overseas and domestic sales and tuin
d 6.4 pacity of 30,000 units, His expenses

(1) Direc. materials z domestic demand, cr


I
(3) Increase capacity to accep t the export order and maintain the present domestic sales oy :
(tt) Direct wages 8,280
in able and other manufacturing expenses that will increase capacity by 10%. his will result in an increase af
11,260 fa) buying an equipment
-
otal fixed expenses irrespectiv; : thes 3,960 % 1,00,000 in fixed costs, and
The expected sellin g price intthe
the eed wttisation
domestic marke 6,060 (b) work overtime to meet balance of require
d capacity. In that case, labour will be paid at
woes t is % 2 per unit Re
has received a trad
1.45 per unit,
e enqui quiry from an overseas Organisation
interested in purchasin SO one and a half times the normal wage rate.
9 6,000 units atmanufacturer the b est alternative.
As a profes§sional ma age
a price of % Prepare a comparative statement of profitability and suggest
ment accountant, : what wou (B. Com. Hons. Delhi, CA. tater}
of the offer e?y Stp
sy PPA
port youryour suasu gestio ld b @ your suggestion regardin G acc
ti n witi h suitab
i le quantitative inform eptance or rejej ction
(B. Com. , Hons . Dethi, LWA
ation.(B.
Solution 0.WA
. . F, Inter) Solution
; ; _ Profitability Statement
Profitability Statement
Alternatives
"45,000 units
6.000 units1 if
(For
Fe domestic sale) T Ii
{For export) 21.000 ts Present position 50% capacity for 80% capacity for
80% domestic sales domestic sales domestic sales
Sales only and 50% capacity ard 50% capacity
Direct Materials for export for export

Direct wages
z z z
Variable expensas
32,00,000 20,60,000 32,00,000
Total variable cost Sales (domestic)
18,00,000 18,06,000
Contribution 560 9,360 | 1.560! Sales {export} “

Less: Fixed cost Total Sales (A) 32,00,000 38,00,000 50,00,000


ee eee ene

~~ 16,25.006
neh eae

= 6,00
so 0 Direct materials 10,00,000 12,50,000
(-)b60 —_ (-)60
4,068,000 5,006,000 7,006,000
Conciusion. Direct tabour
“ The off er for 3,25 006
There is a nega
“5 tive uci Variable overhead 2,006,000 2,50,000
units. it is thus not protien
rz . : he varireanai .
af % 0.11 per unit resulting in a loss of Fen 356 ai uma. 16,09,000 20,00,000 26,560,000
certain non-cost ¢ " © accept this foreign offer However, . (000 additional Total Variable Cost (B}
actoeag.
may also }be cons‘der any’s
rs like entarging market for com Pany ay : befo taking a final decision,
s products, earnreing fore 16,900,000 18,00,000 23,50,000
(A~B)
ign exchange, ete Contribution
13,00,000 14,00.000
Less: Fixed overheads 13,00,000
\

3,00,000 5,00,000 9,50,000


Profit

of profit.
Conclusion. Alternative II] is the best because it results in the highest amount
Decision Making
Management Accounting

Working Notes:
1. Sales at 30%, capacity
= % 32,900,000 ired: . h of ye four options.
.
at 100% 100 ony A statement of costs and profit under, aseac to Ww not these options the company showi
d
= 32,00,000 x eo 7 = 40,9 on, with
i reaso ns
00,000 (ii) Your recommendati
j
mene (MBA, LCW. A. Inter )
2. Sales (export)
at 50% capacity = 20,00,000 - 10% decide upor.
3. Direct Material = % 18,00,000
cost for alternative I] (100% capac
ity) Solution.
10,600,000
106 Profitability Statement
x 0 = % 12,50,006
4. Diree t material cost
for alternative Til (130% x Ga~ So = my 62-5"
f bho
y
Voge
__.

capacity) Alternatives
10,906,000 x 30
130 —God
2,40
40,000 2,406,090
= © 16,25,000 ay——
2,00,0 00 omer
Sales (units Domestic ,000 eee
Similar calculations
are made for labour cost and varia ble overhead — Exports ——Ty2,00,
00000 200
a, 00008 ie 9 4,030

is
Cost at 130% is ratcu
lated on the assumptiga that overt
at 100% and 130% capacities
, Labour
$40,000 ie —
i me will be required only asl

lacie
additional plant will
taise capacity by 10% which will for 20% capacity because Total . 12,00
34,40,000 u
Thus, labour cost not require overtime working. sates (2) — Domestic ,060 14.40,000 9,40,000
for alternative II,
— Exports
9 40,000 9,460,000
a 70,000
ce
_
At 100% = 4,00,000
100
@ 21,40,000 0
23.8000"000 —"Fra0.000 23,80,000 ee
=
x 80.
Labour cost for alternative
= ¥ 5.06,000,
{A} Total s ales 76,00,000 Sd 80, ———(9,60,000
III
(B) Variable cost =
« 99,000 480.000
la) 540,000 5,80,0
¢
At 130%fe = ( (400,000 x«a—}+!4ao0 2 76.000
‘ 00¢ x* a090 x 25| (C) Contribution (A-3} 2,40,000
2,40,000 assoe00
(D} Fixed cast Oe ys 20,000|
= 5,56,000 + 1,50,000 = % 7,00,000, 360,000 3,25,000 3,24,000 ng
Profit (C-D) a to
Problemim 6.5 Export Ord _ large a
requitAe $s ial
er) # u' alt a byive sh
i snowing 4 P I ¢ F . 00
3,25,0 appear
p s the most profitable, 1i
ut ougn h aitert 6 f capacity.
Pieco Engineering Compan i onclug:en.
fu ure, ' it \ will result in
i under utilis auon
ilisatio
y has receivetin once- off export ozder for its t ent.
iS stm If export t o rders are 7 not | ed
repeat Ww
Tequire the use of half of sole product that would i.
capital :
mnve
the factory's total cap acity,
The condition of the export which is estimated at 4 lakh oe F
2me 24,000
4 whihich is shor
hort byry
Tl
on
3 nly z 1,009.
order is that it has to be units per anntu, alte rnative 1 ? it”alternative Ti offers
} es
a prose
wie
accepted in full, acceptance Ct mM pare a to
not allowed, of part quantity is of capital investment. al d overtime. Thus. alternative is. appears masl
But t it t dees no t cr ea te prev! ems
The factory is curre ntly
operating at 60% level to meet the demand of its domestic acceptable.
against the current Customers. As
Price of 2 6.00 per unit, th © expor Working Notes:
total cost of curren t offer is € 4.70 per unit, which = 2.50
is less than the
t production. Variable cost ger un iti — Material
vabous1 00
1.050
Th ® cost break-down is given
below:
Direct materjat Variable expenses To
% 2.50 per unit
Direct labour Total ——
= 1.00 per unit
Variable expense
% 0.50 per unit i I
Variable cost — Alternative 00
z 17,60,000
Fixed overhead @ 1,00 per unit 4,40,000 units « 7 4 __ 40,000
Total cost =
The company has the following unit F 5.00 per Add: Overtime payment 38.00,000
options: Total
(a) Accept the export order and
cut back domestic sales as neces
(6) Remove the capacity constraint by installing sary.
necessa Variable cost — Alternative TiT z 16,00,000
working overtime to meet both 4,00,000 units «4 50 100008
demestic and expo tt demand.
Met

overheads hy F 15,000 annu This will increase fixed Raw material 40,000 units @ @ 2. __ 80,000
ally and additional cost for overtime work will amou
40,000 for the year, nt to = Add: Overtime payment “17,80,000
{c) Appoint a sub-contractor Total
to manufacture the additional
nd export requirement in fut requirement and meet the dome
by supplying raw materials, payi stic Fixed
i cost Aite iv
iternative H
40,006 G :+
2,49,
15 008 =k 2,55,5,060
2.00 per unit and appointing a ng a conversion charge @ = 2,76 cod
supervis
or at a salary of % 3,000 per mont ‘ Alternative iw 2,40 000 + 36,000 = g
quality of the product and controll h for checking the
(dq) Refuse the order ing operations at manufacturing unit,

a
Management Accounting
Devision Making
6.29 6.30
to profit: This is shown
Problem as it makes a contribution
6.6 (Export order)
The order from overs eas, market should be accepted
(b)
RB. & Com
ompany isj presentlyHy operati : below: z
pe
annually of a patented operating at 50% of practicalal capacityialler prod
Fo nett 50,000 units
at ae component. RB. recently received an Seas market intg a
1,80,000
30,060 components h : * = 6}
. : unit, R.B. af
FOB &.8.’s plant, units output ie ae & sold components Incremental revenue (¥ 30,000 z
market. Bud geted ,production cost for 40,000 and 80,000 . : . .
is as faliows: Less; Incremental cost 45,000
' .
50,000 units Direct material 45,000
80,01 i / 1,50,000
" Direct labour 60,000
pet unil
Durect material / 1,20, Variable overhead @ % 2 30,000

Contribution (or Profit)


Direct lasour ; .
Fectory overhead 7000 profit of = 30,000.
2 20,000 2,60,000— offer wilt result in additional
000 ‘60,
0 : Thus, acceptance of this
Cost per unit of a Special Order)
ise Problem 6.7 (Acceptance and ather contests. Its manufact
uring plant ':45
The sales man ager th i rs
25
manu fact ures medal s for wint ers of athletic events ction and sales level of 7,500
POR Ltd. company has current produ
vec
because he feels the wales me y ‘lll 7" $ | 0 medals each manth. The cost data for the month of
build np
up future
[ture market. The production
master. The manager ae oesSnot the capacity to produce 10,00 meca l is ¥ 150. The
r accepted primaril y because the ord would show production moneee ne not wish
wish tota ree
have the : nt domestic market price
of the
, ’ : a I per unit when computed medals per months. The curre
nade average unit cost. The cost accountan ‘tat mate @ a6 quick computa
quick com putation indicating that accepting the e Octob er is as under :
an actually increase profit. with units produced) ?
Variable costs (that vary
You are Tequired ta: - 2,62 ,500
fe? / Capiai
Saplai
n n +what apparently caused the drop in cost from Direct materials 3,00,000
. 0
¥ 7,00 per unit to % 6.25 per unit when r -
budgeted
g Production i increased f ram 50,000 t i Direct manufacturing labou
g computations. number of batches)
{2) Should tae order he accepted? PANGS ns: Show supportin Variable costs (that vary with
(LOWA. Inter} mater ials handl ing, quali ty control 75,000
Set-ups,
Solution 150 batches » % 500 per batch = - 2,75,000
Fixed manufacturing costs 1,756,009
Statement of Cost
Fixed marketing costs ' g 10,87,500
a T
50,000 units 80,000
* units
nits 2,506 medals at 1Q0 per medal.
special one-time-order for
PaR Ltd. has received a hatch size of 50 medal s.
Total Per uni tt Tot ~ its existing customers in
POR Ltd. makes medals for meda ls}
per batch = 7,500
Direct material | é e : eee (150 batches « 50 medals in 25 hatches of
: to manufacture the medals
Direct labour 75,000 1.50 | 1,20,000 The special order for 2,500 me dals requires POR Ltd.
Factoary overhoad - Variable
. ; sa oo0 1.50 | 1,20,000
20, 2.50
3
100 each.
Fined 00, 2.00 | 1,66,000
rou, 2. Required:
special order? Why? Explain briefly.
on “ne i : (i) Should PaR Ltd. accept the 0 medals each month. The speci
al
__ Total
——
we
3,50,000 7.00 5,00,000 6.25
Supp ose the plant capac ity was 9,000 medals instead of 10,00 PaR Ltd. accept the special order?
(ii) Shoul d
in full or reyec ted totally. (CA. Inter}
order must be taken either
Calculation of fixed overhead ly.
Why? Explain brief
Difference wy in fa ctory overhead betwee
thus n two level = 2,60,000 - 2,0 =
one overhead for 30,000 (additional) units = Solution
60,0 ee aes + Profitability Statement ng =
; nal le overhead per unit = 60,000 + 30,000 Ci}
= % 2 “
Fie
anab le overheai d for 50,000 units - 56,000
0 x 7 2 = % 1,00,00 2,50,000
a cost = 2,00,000 - 1,00,000 = ? 1,080,00
0 on Sales (2,500 medals @ = 100)
Comments ( a) The drop i : Less: Variable Costs: 87,500 }
is : so because at § Pit cost per unit from % 7 to 0 &F 6.25 isi because fi
- of decrease inin fixed
0.000
decr ease invesfixed cast by 7
un: its fixed cost is cake cost per
is © 2 per unit while at 80,000 units tis comes down to O ¢7
unit. t. ThisThi .
1.2 Materials (2,500 medais @ < 35) 1,00,000
total 1.25, This (2,500 medals @ 40}
important to note that f 5 paise has resulted in drop in t wa 7S acen ie, from % 7 to & 6.25, It is Lakour 12,500 2,00,000
: at fixed ; cost per unit: comes down when output ™ (25 batches @ = 500)
nereased. Set ups 50,000

Contribution i
pe n of & 50,000.
it gives a contributio
should be accepted because
Conclusion. The special order
Management Accounting

Decision Making 6.32

r)
(ii) When plant capacity is 9,000 medals. Problem 6.9 (Export Orde % 6,900 each, made
up as under: z
tric kits at a ¢ ost of
A company selling elec
Present contribution on 7500 medals : 3,206
Sales (7500 x 150) . 25,000 Direct materials
400
Less Varitable costs : 1,000
ees Direct labour
200
Materials {7,500 x 35) Variable overhead
200
Labour {7,500 x 40) ~ 0,000 Fixed overhead
Crs) 109
Set up (150 hatches x 7 500) "000
75,000 6,37 Depteciation
e 20)
— Selling overheat -variabl
Contribution Taras
87,500 Royalty
_ 1,006
6,306
Profit
Contribution on 6,500 medals 690
6,900
Central excise duty
Kecountant, would you
Sales (6,500 x 150) 5 % 5,000 each. As a Cost
Less: Variable costs r has offe red ta buy 200 such kits at
sees (a A foreign buye company uoader
the same
offer.

=
Materials {6,500 x 35) a
advise accepting this chased by
for a kit te be pur (C.§. Inter}
Labour = (6,500 x 40} 20-00 pric e shou ld the company quote
(Bry what
ld be at cost.
Set-ups (130 x 500} 65,000
* i000 552,501 management if it shou
Contribution , 4 22,500
238 ;
_Sotution
lows 7
is computed as fol +

Variable cost per kit


x
Acceptance of special order when capacity is 9,000 medals {a) - 3,206
400
Direct material 1,000
Gain
: n ini contribution
ibuti on acceptance of special order 50,000 Direct labour
- 4,22,500} 200 -
oss in contribution {4,87,500
_ 65,000, Variable overhead pro duc tio n}
to be on 4,300
- Net Loss in contribution 15,000 Royalty (assumed
5,009
Variable cost
eign buyer 200
Conclusion,
: When plant Capacity
city is 15 9 " 000 medals, e the special order should not be ac cepted because it Offer price from for
Contribution per kit kit and a total profit
mental profit of < 200 per
ed as it resu lts incr e and (b) selling
Problem 6.8 (Special Order) Offer should be accept re is spare ca pacity,
ts x & 200) . Tt is assumed (a) the
-Ki
menyeneatine
A machineShop
shop inin a 3 factory ory
ry isi workingH g toto itsits full
tul capacity
j and earning
i a contributi of % 40,000 (i.e. 200 r.
evant for export orde ted is computed as
under:
rae eee seadat over wal ae which it wants to execute immediately. Materia wil be antied by the overhead are not rel man age men t, price to be quo
under the sam e z
(b) For the company
ete ea minicum of 10 hours, Wages payable wi hou? and
be 150% of wages. If the customer is prepared ta pay 300 for the oer . 7should the order
val the ae 6,300
be accepted. (LOMA. Inter) ise duty)
Price (excluding exc 1,000
Solution
109 4,100
Less; Profit ———
Caleulation of the cost of special order selling overhead 5,200
Variable
Price to be quoted wherever payzhle.
It may be added,
Direct wages (10 hrs x % 15) 5 : Exci se duty has 9 ot been included.
Note
Variable overhead (150% of % 150) 22
Loss of cantribution (10 hys x % 50) 50 Proble (6.19 (Selting Price Decision) re cookers has drawn
up the following
y Priv ate Limi ted, man ufacturing pressu
Cost of specizl order a Prastige Compan
7-18.
~. Price offered by customer aoe budget for the year 201 26,006,000
6,00,000
Loss =75 Ram materials costs
and other variable
Labour, stores, power
Decisio n, The order shouid not be accepted as it would give @ loss of & 75
Decision Making Management Accounting
. €.33 6.34

Manufacturing overheads sion}


Variable distribution costs 7,00,000 Problem 6.11 (Selling Price Deci presently in the domestic marke
t at 75 per
product which 1s so! Id by it 50% of the capacity
#,00,000 A company produces a single month repre senti ng
General overheads including selling and sale is 40,000 un its per
3,00,000 unit. The present production
of the product are as under :
. Tot
oT
40,00,000
avail able. The cost data

Income fram sales otal Variable casts per unit = 50;


30,00,000 lakhs.
Budgeted profit Fixed costs per manth = 10 an hand as under :
19,00,006 management has three proposals
The General To improve the profitability. the a reduced price of % 60 per
unit.
additional volume Msenager suagests re reduce selling prices by “1 5% and . expects to achi leve a 30,000 units pet month at
orde 1 for
uf. i
intensi " (a) to accept an export supply export packing, duties, etc.
aye
additi . manufacturing capacity. More Btensive
2 5 SUIICeRE t
manufacturining costs of € 5 per un it towards
programme will involve
voty - itional costs of % 50,000 for 100.000 : wat : incurring additional variable chain stores 30,000 units at {55
pe:
necessa Ty to open an additional sales office at a cost of Z Planing. [t will also be (b) to increase the domestic market
a
sates by selling ta domestic
A, er annum. at the existing price.
unit, retaining the exisiting sales department as
The Sales ‘ Mana ger, on the other hand , i sell i sale as advised by the sales
which i the sellin g price for the increased domestic
estimated to reduce sales volume by 16% 'at eam
the same tine,
time, saving in {c) to reduc e
in manufa
mentee
cturinree
g overheads
and general overheads at ¥ 50,000 and z ,00, per annum respectively is expected on this under : Increase in sales expected
reduced volume, Reduce selling price per unit (in units)
by = 10,000
Which of these ; two proposals would your acce pt and why? (2 _ Com. . Hons. . Delhi Delhi. LC. A WA. Inter) 5 - 30,000
8 35,000
Solution advice of
11
of the abov e pro posals and give your comments and
table ta prese nt the tesul ts (LO.WA. Inter)
oo Comparative Statement of Budgeted Prepa re a
Profit
the proposals.
Praposai I Proposal li Solution
Profitability Statement
noes
Sales of Sates ; 71,25,000
: 49,50,000
‘ Existing (b}
position Increase
Raw materials sales
L abour,stores, power and other variable costs 30,006
500000 ‘sana
Manufacturing overheads
+5000 «30-00
Variable distribution cost o4
5000 20-00 zg

ener overheads including selling Selling price 2°
00,000
«00 2,00,000
a0 00 z
7 | Less: Variable cost 14
56,50,000
Contribution per unit =
Profit (Saies - Cost af sales)
1 4,75,000 sco
14,00 on
00,000
Sales {units)
Conelusion .?P [0 posal I gives
i a hi her i B14 Contribution (existing) = lakhs
of 14,75,00¢ and thus should be accepted,
g profit
Working Notes: (on 40,000 units)
Additional Contribution
ales:
Sales: Proposal - € §0,00,000 x =
100 *x =<
100 =F 71,25,000 Z lakhs (on 30,000 units)
Total contribution at
reduce selling prices % lakhs
I % 50,00,000 oan xe 49,50,00
Total Contribution z lakhs
Raw material:
aterial: Proposal -I 2% 20,060,000 + 50% ” = z nao ment, proposal C
labour ete:
Te °
30,00,000
Conclus jon. On the basis of above state
> = 20,00,000 tance of
lakhs and thus is the most profitable.
- 10% =F 18,00 0) n non-cost considerations. Accep
consider accepting export order
However, the company should also ge, entering export market, income
tax
nt roposal | z 6,00,000
' + 50%fo =% ‘oo000
49,00,0 forei gn exch an
company eamung ¥: aluable
II z 6,00,c00 - 10%
! =f 5,740,000
“sooo export order will enable the
etc.
benefits, export status of the company,
Management Accounting
Decision Making __ nee
6.35 6.36
Problem 6.12 (Export Order and Key Factor) Profitability Statement
cost sh og (for Export)
products X and ¥. The
A Ltd.
e operatin. iq at. 75% level of activi produces and sells two to Japan Export to Bangkok
of these’ products are a follows: mr ace. eets Export
Product X Product ¥
, © Per unit ©
Units preduced and soid Per unit
: 000
3,000
’ eon
2,00 26.00
Per unit
10.0000
Per uni 20.00
Direct materials 20.00
rect Materials , 9,00
Direct labour 15.06
" 19.90
Direct Jabour
20 2 Variable factory overhead 16.00
Wied overhead (40% fixed) overhead
5 3 Variable adm. and selling _ 2:59 2:20
m. and selling overhead (60% fixed)
‘0 3 Special packing cost 63,50
61.50
Total cost per unit ead
5 20 {A) Total yariable cost a7.90
Selling price per unit 15 “953595
5 {B) Export price 24.00 16.00
Factory i overheads a re absorb i)
hour rate is 2 10 per hour.
basis of machine hour which is the limiting facter. The mach
. ne Contribution (B - A} 2.5 :
fon the = 10.67
Machine hours per unit 16 = 1.5
Alternative ateceives an offer from Japan for the purchase of product X at a plice 26 + 2.9 = 9.0
pri of F 87.50 hour
i contribution per machine rt offer from
. the company has anathe Me pecking cheno ot Purchase of product ¥ at a mice of in product Y, Thus expo
77-50 per unit. In both the cases.s as spatial paki key facto r (ie, machine hour} is higher surp lus mach ine hours
-50 per unit has te be b Contribution per unit of ¥ tha t can be produced
with
mpany. The company ca P ‘ er of the two export orders by utilising the balance OF 2s a tieWs ld be-a ccep ted. Total num her of units of
capacity. pany ran aces Bangkok shou
Advis ie { he ; Comp any ¥ Wi i thodet. 1 + . js calculated as follows:
i d worki|
e ane Orking es to which
2,333 units of ¥
| prop Osa! should be accepted and prep
tat emer t Show _ 3,500
Surplus machine hours
sta shi th @ overa' r Fit bili ty 0 fF the Cam
ontah ompanry after Incorporat: ig 9 the eXi port proposal suggested b
1.5 -
{B.Com Hons. Dethi, C.A. Inter) Machine hus. per unit
Solution Overall Profit
Statement of or
&
Basic calculations z z
g
1, Machine hours (limiti
are made as follows ‘ ming frtor)
per unit of sutput of product X and ¥ and other related calculati
ons. Home Market
,
er em
{A) Selling price per unit .
20
Variable cost per unit: 10
(A) Factory overhead per unit 20 ad
: Direct. Material
(B} 15 a
Machine hour rate
£10 Direct Labour
4 __ 10
(C) Machine hours per unit (4 + B) Factory overhead 16
“as
{D} Units produced ‘3 Adm, and selling overhead 61 59
3,0000 2,000
00 Total
Total machine hours used (€ x D) 7,500 (B) 4 __ 36.
2.€ i
alculation of surplus machine hours available 2000 (C} Contribution (A ~ B) 3,000 2,000
72,000 2,324,000
¢
Tota machi 1 eh ours
ur used fo r product X and I {D) Number of units 1,62,000
2)
7,500 hours Total contribution (C » BEB
¥
Export Market ~ 37,328
2.000 hours Contribution (2.333 x 16) 2,74, 328
10,500 hours 1,44,000
Machine hours avai ailable at 100% capaci at . = 10,500 Overall contribution
Surplus machin: e hours (14,000 - 10,500) (20.500 = 75%) 14,000 hours Less: Total fixed cost 124,528
3,500 hours
3. Calculatioofn fixed cost Total Profit
Fixed factory overhead :
Decision)
. x: 3,000 units @ = 10 z Problem és (Make or Buy onent. The component
cost
anmu al prod ucti on of 90,0 00 units for a moter comp
¥ : 2,006 units @ % 6 30,000 Aute Parts Ltd. has an 2
Fixed adm. and selling ovéthead A200 42,000 structure is as below :
270 per unit
X: 3,000 units @ F 24
Y : 2,000 units @ % 15 sooe Materials 180 per unit
. _30,000 1,02,000 {25% fixed)
Labour
Total fixed cost 1,44,060
Decision Mating
Management Accounting
6:37 cast , 6.38
~
Expense; :
Variabla Required: released following
90 per unit considerin q that the present facility when
Fixed {1) Should the part be made or bought,
_135_ per unit:
Tatat a busing decision would remain idle? 0, what would be
675 per unit be r ented out to another company for Z 1,50,00
See Coyin case the released capacity can (C.8, Inter}
(@) The purcha
540, Shoul s@ Manager an cffer ftom a su pplierstoppe :
willi to supply the compon
who d?is willing the decision?
d thene campon 2 be has
2nt purchased and production
ent at <

b} Assi e the fesdurce° $


Solution
mow used for t his compe lent’s ma ufacture are to be used to Produce idle, variable costs to make the component
remain
{a} When released capacity will
another new product for whic the seiling price is € 485 % 5 lakhs
| In the atte a ASR. i
“She hid teral Ww wi i lve
Direct material ~ 8 lakhs
price z 200
c Bp per unit. 9 0,000
produc
- ed aa ihe same
me co
co:t basis da 8 above
ve f for ta N¢ ur and
' nits of this product Can be Direct labour _2 6 lakhs
OXpesis e
Hoel. iscuss
aioe Whether it would be advisable ti O Variable overhead
that new bepreduct,
. produced, on the footi € 19 lakhs
¢
produced would, instead of being purchased form the matket the compenent presently being Relevant cost-to-make
@ 7 22 per cent % 22 lakhs
(B.Com. Hons, Dethi: C.A. Enter} Purchase price of the component % 2 lakhs
Saiution
Less: Reduction in fixed cost
¥ 20 lakhs
Relevant cost to buy
Material Tk erefore it is advisable to make the
component is less than cost to buy.
Lab
abour (75% of & 180) The cost to make the
mnable expenses 35 component.
sure
r otal variable é cosc t wh when component released capacity is rented-out
(b) When
oc isi produced 30 to buy will further reduce
ies will remain a! t % 19 lakhs. But cost
In such a situation, the cost to make
ta buy will be -
Exein of purchase [rice over variable 540
by the income of rent. This the cost
cost = 540 - 495 = % 45 = 22 lakhs
Tn @) Fixed expenses
compare vhetlli ake Purchase price
of 2 18e 0 pe:
vompansat
uniti a tote ‘ incurred
i whether 2.00
‘ th ne component is made or Less: Saving in fixed cost
Oe f because if purchased from outside 3.50
‘> Ber unit more and on
90,000 units
it will hve bes Rental income 150
45 if comes to 2 40,50,000. 18.50"
() Cost implicati ions of propo ve 10 Pay to buy
sal to diver t availi able production i facili _ Relevant cost
faciliti
ti es for a new product:
to buy it.
As "tae is less than cost of making, it is advisable
Selli| ng price of per uniti of new
Less Variable costs ~ :
Meter 200 “
|
Problem $715 (Make or Buy Decision}
of ¥ 300 and marginal cost
Labour an machine 99, It has a selling price
Expenses
135 Product X takes 20 hours te process on machin e 99 in 3 hours for a marginal
90 production} could be made
425 of % 6Q. ¥ (a component part used in or buy Y? Discuss in both situations (i)
when
Should one make
Contribution per unit cost of 2 5, The supplier's price‘is € 10,
and (it) when there is idle capacity.
60
Los
ifi present
ass 2 Component is purchesed = 540 machine 99 is working at full capacity (8.Com., Hons, Deihi}
- 495 =
to be kept in mind in such cases?
f company diverts the re scurce
5 tor the p T aduction What are other non-cost considerations
ihe, © 60 - 45} per unis, i oe
of an e We pra 1 juct, ' i will benefit , by 1 15

Gn 90,00C units § itit wilwi save @ Fis, ie, i a: . Solution


Vibine tes inGo theah, manafactcre Fy ¥ plus
facilit of ew@
the new product and the comes it is advisable / to divert the production capacity. In this situation, cost of the component
be bough . LL (i) When machine 99 is working at fall be compared with supplier's price.
ght han his will result in additional pioltet t 4 presentty being manufactured should the period of manufacture should
of % 13,50,000, contribution lost during
= 100
Problem 5.14°(Make of Buy Decision) Setling price of X =
; 60
4¥ Ltd memvfactures auta parts. Less: Marginal cast
: The following costs are incurred for processing 1,00,000 units of 40
component 00, ofa Contribution
Direct mater-al cost 20 hours = = 2
% 45 lakhs Contribution per hour= = ¥ 40 +
yee labour cost lost
= 8 lakhs’ Cost of making ¥ = Marginal cost + contribution
parable factory overhead per hour)
lxed factory overhead
@ 6 lakhs =25+42%6G3 his @ <2
= 5 lakhs =f il
The purchase price of the com Bp ponent is z 2d.4. 5 T he fix ed
Cost of ¥ if bought = © 10.
ove thead Ww ould contltinue
ent is bought fam outsic e altho a to b be Incur
Li} red even
vi
rugh there would b @ Teduction to the extan t of F 2,00,000.
1 advisable to buy it.
Ags the cost of making is more than suppliers price, it is
! '
te wf} ag ouch in,
:

Boy Gs
t
I'
Management Accounting
Decision Making
6.40 r y the
‘ :
ee639 ae
conclusi
on is that it is cheape to. bu
ions. the prima facie nent will really
idle calculat rchased, the compo
ine 99 has idl ; ; _
(ii) When mach
a aaa In this
e capacity. ti
sitinaation, the Conclus
ion. From above
udy will show that if pu
when component ¥ isi Or 0 loss of contribution e pp li er. Bu t a cl ose st be in curred whether
t machine = will be idle during thet petled t
componen from outs
id su
) beca use the fixed cost ha s to
of making will be only ae neginal mace
5. As the supplier's price is © 10 1, iti otadvisabl . The cost 00 of fixe d cast if manufacturing
cost of e to mat,
cost © 65,000 {i.e 45,0
00 +2 20,0 en- shouid be made. But
this component Y,
e
is mmade or pur cha sed . Therefore the compon to some alte rnative use then the
the component of the component are
put
the non-manufacture
Non-cost considerations facilities released by
buy the component. the supplier at the
1, Assurance of contitinued supply, i i com pan y shou ld ccntinued supp.y from
re taki ng a final decision, an ass rance of
‘ Acherence to delivery sche a Hom outside However, befo n.
long-time should be take ows:
- Assurance of reliable quality. game price for a fairly ed on av erage stock whic
h is calculated as foll
cost is calc ulat
4, Assuranc e of no price
i changes during the period of agreement "Working Note : Storage
of E0G
Average stock = Minimum stock + 1/2 s.
3
= 200 + 1/2 of 1.000 = 700 unit
. “ats ,
The Gamma fom an : tag
. ityTranofs purc jobb er a ~eun
hasinng andfromstoraa nearby Jopb con ponent ke or Buy Decision)
g theof feas ibil 4 _ problem 6.17 (Ma
A, B and
necessary eee a
the be‘negligible
inwould erin
it price & 4.50.
-50. portatioion ge costs for the com ponent ing and assemblin
g three ¢ omponents
and, therefore, can be ignored , man ufa ctu res a product ‘'F py mak mac hia es eact of which can make
Stirling Industries Lid. i dantical
4 e shop using three 12,006 machine-
Gamma produce s the compa : raw materi 7
erial. The firm are made in a machin e machines is omy
ponent . from a single
, ¢. The components tota l capacity of the thre g 1s available
Economic Order Quanti
uantit wage isis v0.00 maverials in ponents. How eve r the
dem and. Labour for
ass emb lin
8uding € 1; average
cost oftaxes,
unit rent, annual
i 10, O units of the any of the three com nt to meet the current
component, The yearly storagege cast
percast
(unc The minimum inventory is set
unit.
ret at 200. Investment, etc.), at
‘ hours per month and
is just suff icie
ils are given below
:
present is com puted at 50 paise units, ements. Further deta Market price at
. according te Tequir Vari ab-e cost
Direct labour co ts for the i
& 3 perpe unit. t , Fixed
F i manufactusi is absorbed at Machin e-h our s whic h the component
; of are10,000
compoaent
& 2 ; per unit b ased on. a tt ormat capacity units. In addition t he ake overhead Components per untt
can be purchased,
which the component month o the above costs, the machine on required per-unit
$ are produced is leased at ¥ 100 per if required
Should Gamma mi ake or buy the component? Would you like to qualify your an swer in
i any way? €64
z 48
4 44
(LCW.A. Inter: B.Com. Hons., Delhi) A
45
60
110
Solution B
§
80
C 30 (per unit of q}
-. a ord: @r ta decide ‘ whether
wh @ to ake
mak Q t bu y thi e the ce iporent, the cost of making Ky has to be compared
ip € —-
Assembling at = 300 per unit.
\ t! the pure hase price of v4.50 per unit. For cost of making the camponent the cost of ordering Le Product ‘Z is sold
4 AyING cost is not supplied calculate d by ¥ cost s per mon th amount to € 50,00 0. for 7 tp rise by 25%. AS
the machine
Pp in the question and is usin
i g E0g formuta as, Follows: te Fixed and
the com pan y ©! pects the dem by buyi ng such numbers of A, B
fiom next month onwa
rds, in demand
company wants to meet the increase
capacity is limited, the
Econ o mici Order i
Quantity (EQQ) = —— where A = A nuala con: Ss UTE TF t jon
itable.
ot € which is most prof
§ = Buying (orderin cost
out the following:
5 = Storage cost 9 You are asked to find pany.
profit made by the com bacght fromm the mar
ket to meet
Thus, 1,000 = { 2% 10.000*8 (a) Current demand and y unit s of the same. 5 hould be
how man
0.50 . (b) Which component and LCW.A, later)
?
(1,000)? « a the increase in demand estion in (Bb) is accepted.
the company. if sugg
(c) Profit, made by
0.50 « (1,000)? = 20,000 B
B = 25 Solution 15
49+ 6=
ms per unit of 22-4
Total cast of of making
i the component per year is calculated as follows : (a) Man hours required 12,000 + 15 = 800
made in 12,000 hrs =
No. of units of Z t be 30 = = 218
= Z 4a + 60 + 80+
Ordering cast (% 25 x 10 orders)
0 Variable cost per uit = = 300 - 218 = % Be
= sp - VO
noase cost (700 units x 0,50) 350 Contribution per unit x = 82)
| pat material (40,000 units @ = 1) (800 units 50,000,
000 Tatal contribution
eet labour cost (10,000 units @ % 3} 30.000 Less: Fixed cost ' 75,600
easing {% 100 x 12 months) ~ ; 1,200
“1200 Current Profit

Fixed cost (10,000 units @ & 2) varae cen 20,000


-o.an0
Total cost per annum 61,800
Tote cost ;to buy y the c component
po: { (10,000
| units @ z 4,50 ) 45,000
Decision Making Management Accounting
6.42
{6} Additional cost per hour, if Coatiponent
is parchased from market
i
(Machine hour isi key factar} Solution
Component d Incremental Revenue
ifferential Cost and
Statement of Diffe nm
Market price per unit = A B : ; Differential
64 75 Product Sales after Selling price Sales Incremento
fess; Variable cost of making
48
Number before before . revert cost (Furt-
her proce-
60 further
(a) Extra cost, if purchased rr - ao further see
16 AY i ssing cost)
(6) No. of hours per unit 15 30 processing . racessin
4 processing B z z
Additional cost per hour (a « b} % 5 6 e
z
4 3 5 = 1-4
Component 8 has the least additional cost
age
2) (3) (4422 «3
pey hour. Therefore, . +
it is best to Purchase (5) = -
For next month demand is = 80¢ units + B component -
25% « 1,000 units (1) 2
fe
~
Hours réyuired for 1,000 units of Z OT 0
8 p00 5,000
7.00 28,000 ‘
Component ¢ (1,000 « 6 his} P 36,000 4,00 1,750
6,000 3.50
A (1,000 * 4 hrs) 3 Q 14,900 3,500
12,250 1,750 ,
4,000 : 20,000 2,500
Oo 20,000 _
5 Balance (400 units x 5 hrs)
2,000
8. 10,200 1,200 3,250
: 5 12,000 1,200
Total 9.00 . To.950
The balance of 600 units (1,000 - 400) 12,000 19,000

of B are to be purchased from the market. a
{c) Computation of profit as per ‘b’ above tt oe
cost ecential
- —— —_ her ;
processing is higher at & 10,950 than afferent i ost atat
Conclusion: Incremental revenue i so ‘Thos as a package, the products may ring products further
Sales (1,000 units » € 300) ne ht
we fit 0 .
should
1 the remain
; .
3,00,000z 4
{ % 10,000 resulting in additional i
But if considered individually, only e oduct P should be processed further and * af % 8,000 is more than
Cost of making Component ¢ {1,006 units @ =F 80) —_— 4 se only in cost of P, incremental reve are equal at & 1,750, In
80,600 | be sold at split. of Ee : is is becaw
case of G, incremental revenue ane : ertial cos!
1.200, resulting in a ‘ loss.
° aon
400 units
mite 6 z 20) 24-000 : ere afferent cost of Z 3,250 is higher than incremental 1
Cost of buying B
4,000 4 case Of 5, a
(600 units @ F 75)
Cost of assembling 45,000 :
{1,000 units @ % 30) ‘ 4 ‘ ist
30,060 ix Decision} .
Total variable cost Problem 6.19 (Sales # eal,
227,006 : the following costs and output data
A multi-product company provides Preducts for the last y
Contribution (Sales - variable cast}
hess: Fixed cost 73,000 “3
50,000 4
co
x ¥
Profit 23,000 10% 35%
é
4 2%
— - Sales mix 2 z z
Problem 6,18 (Further Processing Decision)
— 4 Selling price
; 25 30
A food-processing company 10
produces fcur jeint products from a single
raw Material. These four products . 15 8 = 150,000
are obtained simultaneously at the point unit
of separation. The product R does not require Variable cost per uo
before being taken to the market. The ather three products further processing Total fixed cost z 4,00,000
sold. The company follows P, Gand § May be further processed
the net market value method for alloca before being
ting common costs to products. Total sales
The cost of the ra w material .
used for the year just ended was Z 18,000. _ “ The company proposes t ¢ rept di ce uct Z by
Pr oduc
$
P rod uc ts. £ stimated cost and put
outpu data a ar are

% 30,000 for the same period. The initial processing costs were .
The output, sales and further processing costs for the last year were as follows: x
»

4 ¥ s
Product Output 50% 30%
oy

Further processing Sales or


(units) casts (2) j Sales mix
P 4,000 (2) | z g 28
a
5,000 36,000 4 26 a i
3,500 1,750 Selling price
14,000 ‘ Variable cost 10 15 1
R 2,500
~
pet unit Z 1,50,00
. “000
20,000 { Total fixed cost
You are : required
.
to: 200 3,250 12,000
5,00,
. | Total sales
' should take.
(a) Prepare a Statement of differential cost and
Incremental Reverse Analyses the proposed change and suggest what decision the Company
(b) Assess the change in the prcfit/loss (LWA. Inter)
if a Proposal {stated below} made
b y the top management is accep
1

PROPOSAL: To selt all the produ ted. |


cts directly ta other processors '

Processing. The expected price just after separation without any 1


per unit for the products are: further 4
P- <70-% 450, R-% 8, and
5-2 g. I
(Adapted)
g
Management Accountin
Decision Mating
6.44
16
7
12 Hours
Solution Variable overheads Hours Hours
Rate pet hour 8 45
Present Position Direct tabour 5 14 5
Department A 4 3
76 14
Department B 8
4
Products
7 24 ncial yea r, which
x ¥
7 Department © The bud: get for the current fina
Selling price % 25 30 Total for the year are 2 3,00 ;000 .
s:
20 Total fixed pverheads on the fotlowing sale
Less: Variable cost 2 i nary period, is ba sed Seiling price per unit
<
Jo 15 is prepared for a recessio Sates (unit s)
Contribution % “te es Product 210
10. 12 7,500
40% x 220
P/V ratio 50% 40%
0 6,000
mix ¥ 300
Sales
25% , 1 G0%- . 6.000 of the
“ 1.75.635% 500,060 " Z fo r management,
the unit variable cost
is the most
Sales
Contributi
@ 2,230,000
90.000 75,090 ,25,000 wy, 4
you are required to
show in the farm ata statement
io:
for the current yeat
. Which or thes e prod ucts
(€.5. Inter Adapted)
ution (Sales x P/V ratio) %
70,000 50,000
Less: 5S: We, 2,20,000 ; . the total profit expe cted
Fixed cost ? r three products and
1,50,000 3 profitable? Rank the
products.
Profit ¥ a 0.000
B. ; (& 1,50,000 = 44%")
E. Point Solution
* 3.40,909 Profitabitity Statement
“Overall P/V ratio = 220,000 er Products
5,00,009 * 100 = 44% Particulars

Proposed Position
Products Direct materials
x ¥ 5
08 Tetal Variable overhead
Selling price & 20 25 Direct labour:
Less. Variable cost & 16 15 4
Department A
Contribution F
10 10 14 Department 2
P/V ratio re ae
50% Department C
Sales mix 100% 9 Total Variable cost
20%
50.000
2 50,000 50, _
1,50,000 _2-00.000
cant hs P ;
1,25,000 60,000 30,000 Sales Price
.antribution
Fixed cost
(Sales
* PA ratio) 000 2,35,000 Contribution p-u-
7,500
Profit z
_1,50,000
1,50,000
35.000 Units sold
1,65,000
Total Contribution
B.E, :
Point (% 1,50,000 + 47%")
T 319,149 Less: Fixed Cost

"Overall P/V ratio » 225.900 99. 475 Profit


80,000 « 47%
- Ranking of contribution pet
unit,
Conclusion. « | It-may thus, , be concluded that the proposed the highest amount
replPlacement of Product Z by Product S will
change should be acceptabl
eptable because
t Z is the most profitable as it giv es
: Conclusion. Produc
(a)) pnerease
i profiti by % 15,000 (ie, 85,000 - 70,600); Decision)
g down B.E, point from € 3.40,909 to & 3,19,149 Problem 6.21 (Sales Mix duces F, @ and R, it has
an available machine
rin g com pan y pro duc es and sells U tree pro pro duc ts. Pres ently. the company
P Toblem 6.20 (Sales Mix Decision) A manufactu inte rcha ngeable a mong the thre e
price of the three
lakk hour s, sell ing
hour capacity of one The unit’
eac h of @ and R. aforesaid
20,000 units of P and 15,000 cture and the
e stru
Three
; 5 product 3 4, ¥ and Za Te produces and sells P. Qan d & resp ectively. With this pric {pre sent ly z4
made and sold by 4 company.¥ The Tele iV ant
% 32 and & 42 for d char ges
re, exclusive of fixe
informati On i gi Wi en belo Ww :
1s
products are % 25, g loss . The total expenditu ts p, G@ and R is
Products y is incu rrin the produc
sales-mix, the compan amongst
~ unit cast ratio cost and price
13.75 lakhs. The hout changing its
é per unit). ig % rov e its profitability wit lable capacity.
x zF company desi res to imp
sd as to be within its
tota l avai
: r
\
z 4-6: 7. Since the the following three mixes
Standa cons ider ing
. arg costs s :
material structures, it has been
Direct
420 eo
50
°
Decision Making
Management Accounting
6.46
Pr rodugts
f
Mix on of various
Mix IT
about the are a to be aitotted for the cultivati
: the following data and requests you to advise
i -
n units} +
{in units}
.
y contem plates growing apples,
The compan
aM
tin units)5 ation of profit s.
, 5,500 types of fruits, which would result in maximis
20,000 .
36,000
.- ; lemons, oranges and peaches. Oranges Peaches
eas 12,000 Apples Lemans
5,000
0,000 15 15 30 45
18,000
You
15 oae Selling price per box (2) 100 200
are : eq dyed to comput
omputee the qua ntum 560 156
Season's yield in boxes per acre
0 of oss low bein iG incurred
x which could be considered by
and advise the most Profit
prof able
b
z z z z
: he coz mM Pany. Ps
{LOWA. Inter) fost: 90 150
Solution 270 105
Materia\ per acre 150 195
300 225
Vari able cost par uitic is catculated Labour : Growing per acre 3 4.50
as under: per box : 1.50 1.50
pProduct Picking and packing 4.50
Outpata( ani umts Cost caei 3 3 1.50
Equivalent units Jransport per box
5 The total fixed costs in each season would be 7 2,10,000.
15,000
6 90,00
! 00,000 before you:
The following limitations are also placed
voea | on000 ? 95,000 D table for growing only oranges and
1,05,0
out of this 300 acres are sui
(a} The area available is 450 acres, but
190 acres is suitable for qrowing any of the four fruits.
na venadls cost — F 13,75,000 lemons. The balance of
in
area allatted for any fruit should be demarcated
ariable cost per equivalent unit= {b) As the produce may be hypothecated to banks,
ae
13.75,000 + 2,75,000 uni
Variable cost cf P = 4x Saf 20 per ait complete acres and not in fractions of an acre.
“ the compulsory production of all the four types
Fg 8 = © 30 por unit {c) The marketing strategy of the company requires to be 18,060 boxes.
"
of fruits in a season and the minimum quan tity of any ene type
Centributior per urit P = 6-20 : Ps (8.Cam. Hons., Dethi)
ut Calculate the total profit that would accrue if your advice is followed.
Q = 32-302 F2
_ Ro»
Tasata!
ta i
42-35
2% 7 Solution
contribution P = 20,000 units x 5 = 1,00 Comparative Profitability Statement
G = 1£,000 -
units x % 2 = se 00D
R = 16,000 units x % 7 "= Apples Lemans Oranges Peaches
1,05,000
15 15 30 45
Less: Totala fixed
used cost @ % 5 fo. 50,000 (A) Selling price per box (2) 100 200
, unit: $ 500 150
30.000
:
2,506,000 (B} Season's yield in boxes per acre z z
z z
Loss 15,000
7,500 2,250 3,000 9,000
Comparative Statement (C} Sales per acre (A x #)
of Profitability
Variable costs per acre: 180
270 105 90
Hixes Material 195
150
ee
ee
300 225
oe ne eee Growing iabour 300 $00
750 226
"Picking and packing casts 150 900
1,500 450
Contribution rete
F @ % 5 "Transport per acre
1 25,
+5000 1,00,00 2,820 1,005 690 2,445
:
ane, : ’ (B) Variable cost
30,000 24 : 000 16,000
T0000 4,680 1,245 2,310 6,855
.. Contribution (C - B)
_ 70,000 (E} II I
1,26,000 1,05,000 I Iv
atai Ranking
2,25,000 18,000 18,000 48,000 18,009
laste
hon es . 7 100 = Oe
¢ Less: Fj ixed cast = 00 = Tso
2,50,000
50,
2,50,000
nen 2,50,000
pen Minimum acres to be allotted
p88 OO = 36 = 120 = 180 = 490
_ 25.000(-} Nil
h 15,000 36 120 180 90+24= 114
Conclusion. —— Acres allotted out of 450 acres
Mix [1 is the most. profitable as
it gives profit.
Total Contribution
Problem 6.22 (Sales Mix and 168,480 149,400» 4,25,800 7.81,470
Key Factor) {Contribution per acre x Acres allotted)
4 I @ Cost§ A ce cuntant of a Company
a rank Ing ar orc. hard with an adequate supply of labour, Presents
TURRET ALGER |
Decisi
— son Making
-
| 6.47 Management Accounting
Total contribution 6.48
of all products
. = (1,68,48
ie 0 + 149,
49, 400 4 4,15,800 + 7,81,470) | : Profitability Statement
Net Profit . Contribution =
- Fixed cost
, (For the recommended product mix)
© 2518.150
= 15,15,159 ~ 2,16,000
0, = F 13,05,15 A B c v
“Note. Pickin ig and pack. i 46
by the cost per box. (a) Minimum production (Tonnes) 40 40 36
pacsnd cost and a Iso transport cost is calculated by multiplyi 8 a | 2
ng Rumber of 5 {b) Yield per acre (Tonnes) 1 _
P Oxes
3
Toblem 65 (Sates .
Mix and Key Factor {c) Minimum acres allctted (a = b) 4 5 4
) 36 TT iB
{d) Surplus acres allotred*
A . farmer asks your recomme o2
data is given below - i 5 :
adation for optimal mix of production
. 40 5 4
for the coming year. {e) Total acres {c + d)
Th 7,550 9,608
. > *N€ Cartent (f Contribution per acre (*} 5,300 4,900
Particulars 24,500 30,200 489,600 —
(g) Total contribution (2 « § F _2,12,000 __
ltems produced

18
A Total contribution of al! products A+8+C+2
Area occupied (acres)
25 Less: Fixed cost
Yield per acte (tonnes) : ;
10 : Current Profit
Sale Brite per tonne “5
(%)
1,900
in 2
Variable cost per, acre: : ' 50
Material we ‘Working Note:
mm area is 4 and 4 acres for
Lahour oe
we Total acres allotted to A and 8 is 45, i.e, 25 + 20, out of which minimum
allotted to A
Variable overhead
(2}
S00 500 o00
|
A and B. Since contributior per acre per A is more than B, The remaining 36 acces are
2,000
000 30 + 25, out of whic M.nimum area is 4 aiid
Fixed overheads: ca ‘00
200 Similarly total area allettec to C and D is 55 acres, Le.,
2000 ace allotted to
Cultivation and growing z nue none
me
3 acre for C and D respectively. Since D is more profitable chaz C, the remaining 48 acres
i,00,000 “ane
Harvesting and transport dB.
Land revenue 2 40,000
Administration 90,0 Probl (Sales Mix and Key ruven;
i v0.00 for che yee: 2005
a XY¥ZAtd Jywtich produces three products furnishes you the following aes
5,40,000
The tand which is a
c and D. The land
hej ing used for Production
items A and B can b € used
- oo _—
which is being use Zor i
either items b A & C
for items 4 and d for Producing items
. .
i
8. C and p can be used for
either items bate’ Selling price per unit (2) 100 my 5
u

each are to prov na


ide adequate market Profit/Volume ratio 10% 20% 40%
, and B and 36 tonnes serv ice, the farm er must produce each year
each of € and p. at least 40 tonnes Maximum sales potential (units) 40,000 25.000 10,060
OU are required to Raw material content as percentage of variable costs iF 50% 50% 50%
calculate the followin
(i) g :
G the profit for th @ cur The fixed expenses are-estimated at % 6,80,000. The cdupany use$ a single raw material in all the three
rent year; and
(i) the profit for products. Raw material is in short supply and the company has a quota for the supply of raw materials
the producti on mux
Solution which you woutd tecommen of the value of % 18,00,00C for the year for the manufacture of its products to meet its sales demand.
d.
(€.5. Inter)
You are required to i) set a product mix which will give the maximum overall profit keeping the short
supply of raw material in view. (ii) compute that maximum piofit. (8.Com Hons., Bethas
Profitability Statement for the current year
Solution
Sales per acre (2)
(Selling price a B c 2B
Profitability Statement
I 9 price * yield per acre} 10,000 16,000 33.509 7
258: Variable cost 000 13,500 16,200 Products
®
4,700
_ (2) Contribution per acre ) oo 2100 5,950
A 3 c
6,600 {a) Selling price % 100 ny 40
wot ©) Total acres oe
tal contrib
4,900 7,550 9,600 (5) P/¥ ratio 10% 20% 40%
i ution {a =x d) ~ Se °
Total Contribution (A+ B4e4 B) 20 30 25 (¢) Contribution per unit (a x b) & 10 16 ay
_ 132,500 _ 28.000 2,26,500 2,40,00G
Less: Fixed ‘cost’ (d) Variable cost per unit (a - c) z 90 60 30)
: 6 97,000
Current profit (e) Raw material per unit {d = 2) = 45 30 is)
. 4 5, 40.000 (A Contribution perRe 1 cf raw material (c = e) fe 0.22 .50 1.33
Oe 1,57,000 Ranking il II l
. nn
Management Accounting
. 6.49 6.59

ee Statement ef Product Mix and Profit


a Solution
Products Nyva of Statement of Budgeted Profitability
Materialf cost Total material Contribution Total
. om (i Total
. per unt cost
: per ae { ibuti
Contribution Products
B € z
A
. 14,000
ie 15 1,59,000
5d, 20
3,750 7,800 7.800
; oo aes 30 750,000 (units)
15 as 00 Budgeted quantity zg g z
' 45 9,00,000 (Balance) 10 2,00,000 400
270 260
Total (A} Selling price pet unit
Less: Fixed cost ~ 680,000 Variable costs:
64 152 ii?
oo Profit D, Materials $4 222
95,000 160
D. Labour 9 21
16
Variable overhead 235 364
240
PrProblem 6.25 (Sales Mixi and Key Factor) (B) Total variable cost 40
30 25
A . (C) Contribution (A - 8) 7,800 » 2£ 7,800 x 4
__* company produces three preducts. The cost data are as under: 9,750 « 30
Total contribution = 3,42,000 7,99,500
= 2,92,500 = 7,95,00G
480,000
Direct materiats Less: Fixed Cost 3,99,500
: ‘ ; : -
Direct labour - Budgeted Profit
.e m "
Dept Ri ‘ate per. hou- Ars, Hrs. Hrs. Mix
Statement of Profitability with Optimal
(ii) €
:1 5 18 16 A B
; : 5 4 ;
25 40
30
Variable overheads (A) Contribution per unit (3) 4 7
¥ io ; 5
{B) Labour hrs in Dept. 2 6.25 5.71
Fixed overheads
% 4,060,000 ° per annum ° a B) = 6
ae (¢) Contribution per hour {A + I Ill
a Tl
Ranking
11,700 9,750 5,292
.
prices
The budget wa
are - as under S prepared
. Dd.
i
at a time when the market was slu ggish. The budgeted quantities and selling Optional product mix*
. (Balance)
(39,000 hrs) (37,044 hrs}
vo (58,500 hrs)
Product Budgeted Oty. 2,43,750 2,11,680
Selling Price (@)/Unit (®) 3,51,000
Total contribution ‘for optimal mix 2,43,750 + 2,11,680)
‘ ¢ = € 8,06,430 (3,51,000 +
9,250 units Total Contribution of A, B and
7,800 units
270
Less : Fixed Cost 400,000
ee 280 4,06,430
7,800 units 409
Profit

Later the market im roved and the sal it :


each for products © 2 Band P ales quantitit es couid be increa. sed by
20% fo *Note
follows :
at the prices orizinally ede sale manager confirmed that the increased quantities could be ni or Total hrs in Dept. 2 are calculated as Total hours.
the output cannot be int
inc Hours per unit
beyond the budgeted level due te o ire
:
production i manager stated that
limitati : Product
Units
(Cc) (Bx ¢)
Required «
= tutation of direct labour hours in Department 2. teased
(A) (B) 5 48,750
A 9,750
(?) Present a statement of budgeted profitability. 4 31,200
B 7,800
(ii)
ii) SeSet optimal
i product mixj and calculate the optimal profit ? 54,600
(CA. Inter} ¢ 7,800
1,34,550
~ 4a
Total

“Maximum sales quantities:


A = 9,750 units + 20% = 11,700 units
B= 7,800 units + 25% = 9,750 units
C = 7,800 units + 25% = 9,750 units
Management Accounting
Pesision Making 6.54 6.52
ad

rablem 6.26 (Sales Mix and Key Factor) Calculation of total fixed cost
=» ¥ 59,000
A = 10,000 units x @ 5
XY Led. . is mManufa cturing
i three household 43,006
B = 12,000 units x = 4
products 4; B and €, and selling them in a competitive
market. D etails2 .
K of current demand, selling price and cost structure are gi ven below : 49,000
C = 20,000 units « = 2 = 1,38,006
8
Expected demand Total fixed cost te
(units) ~ 10,0 ‘ 12,000 20,000 12,100 kg.
materia 1 is available only
Selling price per unit (2) (a} Product mix, when raw
von 16 1a Tatel
Raw material Contribution
Variable cost per unit (2) - . Product No. of units contribution
required (kg) per unit (*}
Direct Materials (% 10/kg.} 2 é
Direct labour (% 15/hr.} — OT te.003
_ eae

ne
1.50
__ 4,306
Variable overheads 1 C 20,000
8.00 95,000
4,300
Fixed overhead per unit (%) : 4 2 B 12,000
9.00 49,500
5,500 3,300
The com pany
A 1s frequently affected by iy acute scarcity of raw material and high labo nover. a Duri ing A (Balance} Total 2,455,500
the next period,
iod, itit isi expected to have one of the following situations : 138,000
~L.
LAF Be materials available will be only 12,100 ke Less: Fixed cost Profit 2,17 5C0
“{G) Direct labour hours available will be only 5,000 hrs.
(c) it may 5 be possible toi
O increase ; sales of at ny y one product by 25% W without
i any yf additi Lond! l Fixed
xed costs
t: nee onty 5,006
when labour hours available
but by spending g 20,000 on advertisement, There will be no st artage of materials ar labour j (b) Product mix Total
Contribution
Suggest the b eat P iC oduction
i plan i
in each case and the r result af t Pp ro fit 1
that the company would ea ™ No. of Labour hours contribution
4 Product per unit
according to your suggestion. (L.C.WA. Inter} units required -
z
Solution 2,000 “S557 S*~« 0.00
C 20,000 96,000
fT 2,000
9.06
Profitability Statement A 10,080 49,000
8.00
5,000 1,000
Products 4 B (Balance) 240,000
A B Total ' 1.38,000
¢
Less Fixed Cost
Seliing price per unit
: 5
Variable costs: ct and thus
= Cis tae most profitable produ
Materiat
. :
there is no shor tage of material and iabour, ional contr ibution from its
{c} When and sale. Addit
ted for 25% increase in production
Labour this product should be: selec
: 4 sale of extra 5,000 units (25% of 20,000 units) is
Variable overhead
“1 x 2 5.50 = % 27,500
.
Variable cost per unit : 5,000 units
a “3 as
Contribution per unit
tani P/V ratio
c s0 Product mie Contribution Total
45% : 50% 55% Product Ho. of units per umitt cont ibulor
os
9 ¢0,0C0
When raw material is the key factor A ™~ 10,000
= * & 46,000
Rant material required per unit (kg) B 12,000
5.50 137,500
(Material cost + price per kg) c 25,000
_ “
ca “ 3,23,500
| 15
Contribution per kg of raw material % 20 27.50 Total
1,38, 000
Less: Fixed cost 1,548,300
When tabour time is the key factor 20,000
“ . Advertisement
‘about hours yequire: per unit ~
(Labour cost = wages per hour} Profit
: .” . “ ee
Contribution per labour hour %
Ranking
_ Ul
at Ill”
tt fI
Management Accounting
Decision Making
6.54 of
42 to produce 4,500 = 1.5 = 3,000 units
(Salales Mix Decision and K kg. of raw material will be used
Problemem 6.27
srobl 6. ey Factor .- (b) Availability of additional 4,500
X produc ts has alread y been fully met. Additional profit from
a : product Z because the market demand of Y and
A company can produce three
differ usi oe Z is computed as shown below:
facilities. The requisite labour i Mterent products from the same raw
material
production this additional production of product -
8 per hour for ati products. - TheThe onsupply of ra
" plenty at @ 10,400 <
materialadswhich is imported at Pak limited to
. is ad kg. f iablee
budgetion period.is. Th l@ variabl j 7 — ; 37,800
: , for the commiss
g. The selling @ & 12.60)
overhe are % 5.60
the
.60 per hour. The ; fixed overheeads
followin g i are € 50,000.
'S 10% on sales, Additional contribution {3,000 units
(a) Fram t the most suitab le sale
you are required to sugges Less: Additional costs
maximise the company’s information, mix, which wil} :
9,000
ine the profit that will be earned ves
profit. Also, determ b (3,000 4
« %z 5.60 x, 25%) 6,300 15,300
Variable overhead (3,000 x % 5.60 « 25%)
> haa at that level. Labour
roduc Market Sell
ernit Labour hours Raw material 22,500
demand per unit required per
Additional net contribution 20,000
ween ee units z Less: Additional fixed overheads ~ "2,500
8,000 30
unt (ka.)
Additional, Profit
x
‘ 1 07 production of 3,000 units of
2 product from
r 6,000 Conclusion. The company should go in for the additional the profits of the company.
a because it will add % 2,500 to
additional availability of raw material
1s -
800, 50
1.5 :
re
additional : igh
(5) Assume, in above situation. «if if additio 4,500 kg. of raw material is
naleg i :
Problent 6.28 (Nes Mix with Key Facto
r) 1 bility
should the company go i if witt result in udditionat fixed srerkonte pee action,
' * mix: and prepare a statement
of profita
and 25% increa se in the tates Fa ede n 20,0 art - the follow ing particulars. find the most profitable product
abour and variab ton From .
Dethi; CA.
fable overheads. (B.Com. Hons., , of that product-mix + Product 8 Product C
Selution ~ Product A
~ : 3,600 1,200
: and sold 1,800
Statement of Margiginal Cost and ibuti “ Units budgeted to be produced 60 55 50
nee nd Contribution Selling Price per unit ke) 3 / 4
ae 5
unit (kg}
(Per unit of products) Direct material required per 4 3
x 7 i
Direct labour per uniti (hrs,) 13 8
Z , 7
¥ Variable overheads (®) 10 16
i
Direct material 3 7
7 . 10
en kg. 5.60 . 12.00 fixed overheads (%} 4 4 4
Direct wages @ és be thr
. 8.00 1
12.00 Cost of direct material per kg (3) ' 2 2 2
Variable overhead 5.60 0 i labour hour rate (®) 1,500
br Direct 5,200
- Sales commission ¢ «gO er elling price . 8.40 4,000
. ° 3.00
. Maximum possible units of sale using the same type of machines
— 5.00 from the same direct material
All the three products are produced
A) M 22.20
d to 18,600 hours. (B.Com. Hons., Delhi)
(E Selline Print 37.40 key factor is limite
30.50 40.00 and labour, Direct labour which is the ate
{C} Contribution (B - 4) 7.80 5.60 een :
(2) Raw-material per unit (kg.)
’ 0.78
nf
2.60
1.50 solution Profitabiltiy Statement
;
Bet Fo: of wematera
(FE) Contributi
ee Dy a 14.00 00
A B c Total
z
_ Ranking | I z z z
I | 50 —
~ . : og 60
Statem ent of Suggested Sales Mix and Profit Selling PriTice 16
ao
Soa elling 20 12
Uni Raw mater Direct material 4
nits Raw material
aterial Contributi 8 6
er uni: outer i
Direct labour
produced (k
consumed tkg.) z a i _—é.
¥ per unit0.4 {kg.) Variable overheads
5,000 ’ 2,400 2,400 kg. x % 14 35 31 28
, Variable cost 24 22
= ? 33,600 25
a7 et Contribution (2) 2
8,000
5,600 5,600 kg. « T1Lt@ 4 3
; « & 62,384 Labour hes. per unit {b) 6.25 8 11
(Balance) 1,600 Ls Contribution per labour hour {a+b) I 1
, 2,406 2,400 kg. x % 8.40 i
Ranking . Hil
7 “ ‘
= : & 20,160 150 5,000 1,500
buti on (33,600 + 62,384 + 20,160}
‘otal contriTibuti 116.144 Units produced” (c} 33,008 1,56, 750
Less: Bad : 3,750 1,208,000
: Fixed overhead Contribution (total) (a = ¢} 60,000
~—~E0 000
ee Profit z 56 144 Less: Fixed Cost** , 96,750
Profit
™ —=——=
we
> ow ’ * ~ we ~y

a)
Decision Making
Management Accounting
6.56 ne A
*Suggested sales mix = Labour hour being the key Factor
Rank I - Product ¢ - 1,500 units « 2 hours Comments:
H - Product B - 5,000 unitsx 3 hours 3,000 hrs.
its P/¥ catio is higher.
III - Product A - 150 units « 4 hours (Balance) 15000 hys. 4
d — Product A is more profitable because
(2) When sales value is is limite — Product A is more profitable because its contribution per
600 hrs. (ii) When raw material in short supply
m ; Total labour hours = 18,600 hrs. ‘kg. of raw material is higher.
Calculation of fixed cost
product A is more profitable because its contnibutioa
Product A - 1800 units x % 10 (iii) When production capacity js the limiting factor,
Product B - 3000 units x % 10 = 18,000 per hour is higher.
should
= 30,000 more profitabie and thus production of A
Product € - 1200 units x Z 10 (iv) When raw material is in short supply, product A is um
Y productct ion of 4 iipte
= 7 12,000 of taw materia L l per unit. Thus maxim
Totat fixed cost be maximised. It consumes 10 kg. 16,000 kg
= % 60,000 kg.) 10,060 kg of raw materials. Remaining
1,600 units will consume (1000 units @ 10
wy Problent 6.29 (Sales Mix Decision) 8 7, should by utilised to manufa cture 490 units of product 8 (2, 16,000 kg + 25 kg}
m profit.
B is the product mix which yield maximi
(a) The following particulars are taken from the records of a company engaged Thus 1,000 units of A and 405 units of
products, A and 8, from a certain materiat in manufacturing twe z
: « = 1,000 = oon"
700
Contribution from Product A = 1,000 umitsi

= 400 units * q 1,750 = 7,00,


Product A Product 8 Contribution from product B
(per unit) 17,00,6
te oas0a00
(per unit), Total Contribution
z z , _10,00.000
Less: Fixed Cost
Sales ; 7,00.000
2,500 Profil
Material cost (% 50 per kg) ie nt
500 1250
Direct labour (% 30 per hour)
750 1.500
Variable overhead
250 Problem 6.30 (Product Priorities)
Total fixed overheads: = 10,060,000 ‘00
a1 made fron
products x. ¥ and 2. All the three products
A company manufactures and markets three the data given below, indica
Comment on the profitability of each product when : is limited by machin e capacit y. From
the same set of machines. Praduction
(i) Total sates in value is limited, a view to maximising profits :
priosities for products X, ¥ and Z with
(ii) Raw materials is in short supply. Products
~
{iif} Production capacity is the limiting factor. Zz
x ¥
(iv) Total availability of raw materials is 20,000 kg. and maximum
sales potential: of each product is ee -
16.25 mt
21.25
1,000 units, find the product mix to yield maximum profits material cost per umiti (7)
(B.Com. Hons Dethi) Raw ”
i or
Solution Direct labour cost per unit (2) oe
1. . son
Other variable cost per unit (%)
Profitability Statement a :
Selling price per unit (2) cater
umt {in minutes)
- Products Standard machine time required per
A B
{A) Selling price per unit z 2,500 5,600 Sotution _—
Materiat
i cost zg 500 Profitability Statement
Direct labour z 750 1500
Variable overhead zg
Ts Products
(8) 250 ‘s00
Variable cost z ig
1.500 3 3st xe y¢
Contribution
n (A - B) z 1,000
P/V ratio (C/5)} 25: 30 3b
40% Son i rice per umit
Raw material consumed 16.25 71.25
7 500 = 56 z 1250 + 450 14.25
a “ Gow waterst a
2.50 an
2.50
= 10 k = a 0
Contribution per kg of material Direct labour
ite 2
Z 1,000 = 10 ks T1750 + 25 a ee Fe
Other variables
. :
Direct’ labour hours ~ ~ soo we a
~ *
ZT 750 + 30
z I * (B) Variable cost pez unit ,
| & 1,500 : 4 2 ;
Contribution per hour = 25 his . 50 hrs (C) Contribution (A - 8) as Pn
. © 1,000 + 25 hrs os
= 1,750 + 50 hrs (0) St. machine time in minutes i
_
(E} Contribution per minute (C = D)
= = 40 = % 35
Product priorities (Ranking) Te
Decision Making
6.57 ES Management Accounting
§.58
Problem 6.31 (Sales Mix Decision)
requirement to be augmented on rentai basis at an annual rent
The budgeted results of A Ltd. are as under - (b) show the additional machinery
capacity of 36,000 hours per machine;
of Z 1.5 lakhs per machine to provide additional
Product Sales Value {% {c} change in number of machines to be rente dif the annual rental charges reduce to © 1,25,000 per
x P/V Ratio (9 (LEW.A. Inter)
y 2,50, 006 50 () “machine.
; 4,00,000 60
Solution
6,00,000 30
|
Fixed overtiend © for ' Marginal Cost Statement
the peri 02.200. The management isj worriei
required to prepare a stat Per unit of
ement
Pt she,showing the amount of loss, You ar
tecommend a cha chan ge in the if any
; being
ing eure
incurred at present and SIGMA
sales value of each product as well 26aselein PIE
the same sal “mux, which will eliminat in the total sales vaiue maintaining 30
e the Said igss. 20
{C.A Inter) Salling price < 16
Solution 11
tess. Variable cost & 14
9
me Contribution 2
Statement Showing the Present Loss 1
Machine hours 44227?
a Poduct hour % G$+z1=9
° oe "Sales
Sales Vaiue
tes i Value «« P/V
P/V Ratio
Ratio Contribution per machine
Contribut lon I {
;
g, (F Product priorities
= 2,50.000 « 50%
1,25,0
; «
profitable because it gives a higher contribution
% 4,00,000 x 40%
1's0.000 Mai chine time being the key factor, product PIE is more
rotal
= 6,00,000 « 30%
1,80,0
per machine hour. _
leds: Tota! # maximum 0 f 1,00,000 units of PIE should be
‘s: Teta! fixed overheads
Rs 12,50,000 Product Mix — Taking into account market limitation,
“5 OOD
hours available for Sigma = 4 lakhs - 1 lakh = 3,00,000
__ Fatal lass 502 onn sold which will utilise 1,00,000 hours. Balance
hrs.
an 0 = 2 = 2,50,000 units
No. of units of Sigma that may be produced = 3,00,00
For eliminati
inating thisi loss, an additional
iti contribution of % 37,206 is required
Maintaining the same sales mix - =
Means that the (a) Contribution from the best product mix
unchanged, ratio of sales value of XY and Z should remain = % 9,00,000
PIE 1,00,000 units @ 9 per unit
Total contribution 1,50,000 units @ @ 14 per unit = & 21,00,000
4,65,000 SIGMA
Overall P/Y ratio = 30,00,000
#1 ratio = Totalsales ‘12.50.0060 «100
= 37,2,
Tetai Contribution
(b) Additional machines on rental basis.
AdditiMonal sales to earn additi 4,00,000 units.
iti onal contribution of % 37,200 Units of Sigma yet to be produced
2,00,000 hours.
Machine hours required
- Additional contribution per machine 6 +17 machines
37,200 No. of machiness @ 30,000 hours
Combined P/V ratio to provide 1,80,000 hrs giving a contribution
~ 37.29% ~ * 200.000 Of the seven machines, 6 machines can be fully utitised
But the 7th machine will give a contribution
Additional sales of = 1,00,000 af % 2,10,000 per machine against its rent of 7 1,50,006.
should be af 20,000 hours due to market limitation.
¥ % 20,000, ¥F 32,000 and 7 % 48.000 in the old ratio of sales, 25 : 40 : 60 for X, ¥, and Z, ie of only 1,40,000 because it can be utilised only to the extent
Thus this 7th machife will give a contribution of only
This will leave an idle capacity of 10,000 hours.
Pr
‘ oblem
machine rental of & 1,50,000 resulting in a loss of % 10,000. Thus anly 6 machines
6.32 {Sales Mixi and Key Facte t ) = 1,40,000 against
should be taken on rent.
ata, 4 compan y preducin @ products PIE and SIGMA using a single produ the 7th machine could aiso be rented out
ction process has the followin (c) When rent per machine is % 1,25,000 per annum,
t - 1,25,000).
because it will result in a net margin of €'15,000 (i.2., 1,40,000
g cos

it does not change under different alternatives


Fixed cost has not been taken into account because
Selling prics per unit (Z) PIE SIGMA
and thus is net relevant in this case.
rule cost per unit (%)
achin
| e hours requi
quired per uniti of producti
" - Problem 6.33 {Changes in Price and Volume)
Market ‘
limization (units) 2 is expected to show a profit of <
mossenen Os) 1 lakh 2.5 lakhs Reliable Froduct Co. manufactures product MX. The company
Total machine hours MX in.t he year 2005, after charging fixed cost of t
available"= 4 lakhs 14,00,060 fram the production of this product
Fixed cost per annum — = a variable cost of 7 2¢ per unit.
26 lakhs 16,00,000. Product MX is soid for % 50 per unit and has
Consid ur g the mitin factor Market research suggest the following responses to price charges:
Cc s of M1. achin e hours and marke t
market nm
limita nl '
tions
{a} iE 1d icate G u are fr equired d to i
° - th ENE bes combinatio
i i n of products to qiv e optir umi contributi
ticon:
Decision Making
Management Accounting
6.59 6.60
; Alternative a .
Selling price y has.a plan to diversify and make product
Quantity satd Due to fall in demand of the above products, the compan
A reduced by : for C direct materia l and direct labour will be q 2.58
5 increased by C using 40% capacity. It has been estimated that
Other variable cost will be same as applica ble to product A. The seiling price
. 10% and % 3 per unit respectively.
8 be 30,000 units.
7%, 20% of product C is € 14 per unit and production will
ture of A and B, calculate
10% : 25%4 Assuming 60% capacity is used for manufac
Evaluate these 2 alal i (a) present costs and profit,
adopted fo T the forthcomi rernatives and suggest on profitabitity considerations
i alternative should be
oming year 2006, assuming there is no change in th which (by cost and profit after diversification. and (LC WA. inter)
of in é@ cost structure, as to whether to diversify or not.
(c) given your recommendation
Solution (B.Com. Hons, Dethi; CA, Inter)
Solution
of Present Cost and Profit
Total Contributi
ibution = F 4 p (a)
Statement
oe oe nee ne
ae
Contribution pet unit == §14,00,00
- a no ¥6,00,000 = % 24,00,000, ae | Total
Producis
Quantity produced = z 24,00,00 - = % 30 [i i
0 + 230 8 (A + §}
= 80,00
80, 0 unitsunit . A
30,000 20,000
Profitability Statement Production and sales (units)
Total "Per unit Fetal
Per nit zg zt
Alternatives z zg g
42 3,60,000 20 400,000 | — 7.60,000__
c Sales (8)
hese Selling Brice
;. 3zo: z Direct materials . 2.00 60,000 4.00 80,000
1,006,066
1,480,000
1,906,600
3.00 90,000 5.060
ess: Price reduction 50.00 50,00 50.00 Direct labour (1/4 of S.P.)

New ' price 2.50. 3.50


. ‘
5.80
Variable overhead :
1.50- 45,000 2.50 50,000
Factory
1,50 30,200 oe.
fess: Variable cost 47.50 46.50 45.00. Adm. and selling 1,00 30,000
13.00 2,60,000 | 4,85,000
“ New contribtuon 20.00 20.00 20.09 Variable Cost (¥) 7.50 2,25,000
27.50
et 26.50
26.50 00
3825.00 1,35,000 7.00 1,40,000 2,75,000
ales (units) Contribution (5-¥)} 4.50
=
Add: Increase 80,000 80,000 80,000
. . Less: Fixed Cast: '
, ‘
16,000 20,000
(B)B New sales (units) erat Factory 95.000 | 155,000
88,000 96,000 ° Adm and selling 66,000
Total Contribuyti
Less: Fixed weetation (A = B) @ 24,20,000 25,464,000 oso oon Profit
Profit 10,00,000 10,00, 008 10.00.00
14,20,000 15,44,000 15,00,000 Diversification)
Cr . Conclusion.
tit. ai iternative (5) Statement of Cost and Profit (After
B should be adopt:
p 2 d as iti results j
in the largest Praducts Total
amount of contribution and

A B c {A+B+C)
Pp roblem 6.34 (Introducing 30,000
a New Product) 18,000 12,000
Production and sales (units) z r zg
A company or
od
value and ihe procuces i of Product A and =
30,000 units
ts of the two products are 20,000
, units of product
uct B2 per annum, The sales 2,16,000 2,40,000 420,000 _ 8,76,000
|
as follows : Sales (5) 75,000 1,59,000
36,000 48,000
Direct material 60,000 99,000 2,04,000
Sales value e 54,000
Direct labour
Direct material 7,60,000 Variable overhead: 1,902,000
30,000 45,000
27,000
Direct labour 1,40,000 Factory
18,000 «__—«18,000 30,008 66,000
Factory overheads 1,900,660 Adm, and selling 2,640,000 5,317,000
1,35,000 1,56,008
Variable cost (¥}
50% of facts *, e and selling overheads
Administrativ 170,000 _1.80,000 345,000,
: . Ty . overheads
‘ads i
aré~variable and te . Contribution (S$ - ¥} 81,000 84,000
sellin whe din Of A is 712 per unit and B is 20 na administrative and selling overheads are fixed. The
. Less: Fixed cost :
. . 95,000
rect material and prod A is 2 :'3oe and .
the sellingq price j 400%
mdof labour
dj
ratio for Product f : . Factory
____ 60,000
1.95,000.
9 Price is or # is 4: 5. For both the products, Adm and selling
and administrative and ¢ ns direct labour. The factory overheads 1,90,006
ar, : are charged in the ratio of direct lab
of B. - etng overheads © Tecovered at’ a flat-rate of ¥ 2 per unit of A and % 3 Profit
q per A unit
it
ws. Joa | &.59 i You FP? , Fs 1s?
Decision Making
. a c , 2.5 Management Accounting
“6.61 6.62
G9
{c) Recommendation

Before diversification
Oy Less: Fixed overheads 76,000

Profit After diversification Profit T= 1,25,000


1,20,000
1,906,000
(b) If ‘8’ is discontinued, production il increase
of A and C will i by 50%, ie,
i A -~ 4,500 O uni units and €
2.75, 300
P/Y ratio 760.900 » 100 = 36.18% 345,000 - 7,500 units.
876.000 « 100 = 39.38%
Contribution on A = 4,500 x 13 = z 58,506 _
Aiversifieation has resulted
in hi gher profit of = 1,90,000 and
higher P/V ratio at 39.38%, Contribution on € = 7,500 = 18 = @ 4,35,.000
Therefore, diversification plan
should be implemented.
Total contribution enon
Froblem 6.35 (Discontinuance of a : Fano
Product Line) Less: ain
Pre Kay Led. is enga ged in
3 distinct lines of production. Their production cost per unit and selling
Prices are as under: production of A will be oe on of & witt be 3,000 units.
(c)} If ei discontinued,

A Contribution on A (4,500 = 13} '


Production (units) B c
3,009 2,000 Contribution on B (3.000 x 22) z 66,000
5,000
z z ood
Material ons z Total contribution
18 26
Wages 30 ‘ess a overheads . oe,
Yeriable overheads ? 9 10
@ refi
‘ized overheads 3
_ 5 3 : . i
8 9 on
Conctusion. 0 ft he three aiternat 1Vé5, . the highe: st amount of p profit {z ao, 00}
1,25,0 5 earned when 1 A
Tatal cast
32 46
S€ ling price 52
a 49 60
Profit
ee 61
Problem 6.36 (Differential Cost Analysis) | se tend ent
8 14 9
The management wants to discontinue Pioneer Manufacturing Company, makers of a specialized tine or oss received an order for 2,
one line and gives you the assurance B “ shes srpus eapacty an
other Enes shall rise by
50% . They intend to discontinue
that Prcduction in two of toy battle tank from a large mail-order house at a price o
Profita ale. the tine which Produces Article 4 customers at 5 each bu
as it is tess The company sells this type of toy to its other
operations
p for the comin g 1 anth.
{s" No you agree to the Can tak e the special T
order without adver sel y i g
aff ect amg it: s regular
c
scheme in principle? If sa, do you
should be discontinued, think that the line which Produces i
for the preceding month isi an unl der:
‘A’ The income statement of the company
(o) Offer your comments and show the : z
necessary statements to support your
decision. 50,000
Net Sales—10,000 units @ =5 each
Solution (8.Com., Hons.) Costs: 15,000
Direct materials--% 1.50 per unit ° vo000
Total Fixed Overheads:
z Direct labour—% 2 per unit to-a00
A (3,000 units @ % 5)
15,000 Factory overhead io000
B {2,000 tnits @ & 8)
16,000 Selling and administrative overheads a
C (5,000 units @ % 9)
45,000 Total costs ‘ a
Total fixed overhead
Contribution = Selling price - Varia ~76,000 Net Profit
ble cost i ; i order are estimated
d on the special | to be eect
imated Whee of the
Article A = 40 - 27 = % 13 per i li
ial and direct labour costs to be incurre
unit same amount pet uait as for the regular business. Special tools costing % 500 would be req
A= 60 - 38 = F 22 per unit the specifications of the mail-order house.
@ = 61 - 43 = % 18 per unit You_are required te TI
prepare a iff ial
Differential Costs
§ atement
Stat for decidin G about the
( acceptance : of 'thei
; The decision should be taken on relat
Profits from different ive Profitability of vazious alternatives order. B.Co
Production arrangements are shown as ascertainedDelow ;
below :
(@) Tf ‘A’ is given up, sale of 2 and
¢ will increase by 50%, Then the sates
and Article € - 7,506 units. would be, 8 - 3,000 units
.
Contribution on B = 3,000 x 22
zt 66,000
tr

Contribution an C = 7,500 «1a =


=. 135,000
Total contrikution
2,01,006
Management Accounting
Decision Making 6,64
6.63
. these will be
variable in nature and thus at 100% capacity
Solution Labour and material costs are
calculated as under:
Differential Cost Statement
100
8,07,000 x ~ * % 896,667 (approx.}
For 10,000 unit For 12,000 units Differential Cost
z z Analysis.
Statement of Differential Cost
Direct materials 15,000 18,000 . A
5.0% 100% differential
Direct Jab
abour 10,060 12,000 oe
_ 15,000 a 1,500
13,500
Factory overhead 40,000 10,0 un (units)
Production
8,96,667 89,667
Selling & Adm. Ghd. 10,000 vo-008 _ = 807,000
Labour and material cost 1,49,500 4,500
Speciat
. tools — 50 iG 00
5090 = 145,000
Variable expenses 3,00, 400 2,900
seine cost 45,000 50,500 5,500 t 97,500
.
Semi-fixed expenses 3,00.600 100
as 50,000 56,000 6,000 & 300,500
Fixed expenses 97,167
Conclusion. Increase in sal
sales is T 6,000 while increase in cost is 0 Tl ly = 5,500 The special ordey should 13,50,000 14,47.167 anne
r ' .
Total a
be accepted as it will result in increase in profit of 7 500. Tt
Rs. 97, 167
Problem 6.37 (Differential Cost Analysis) Differential cost = F 64.78
; , , Dujerential on.
units 1,500 units
Differential
; A Company at present workin g at 90% capacity
i and producing 13,500 unit Ne Differential cost per walt
which
flexible budgetary control system. The following figures are obtained ftom its budget. NE operates 2 fore, any price above; < §4./8
be no additional profit. There
At a price of € 64.78, th ere will t, assuming ~hat expert witt not affect the
90% 109% should be arceptable for expor
gives at least reasonable profit
internal sales.
Sales : ,
Fixed expenses *eoosoa "sont00 Problem 6.38 (Discontinuance of a Product)
lling 4
a Cost Accountant. After insta
Variable expenses ~~ sis 000 si c00 trial conce rn which had ne costing system appointed produ cts which are
An indus ved that out af the three
Semi-fixed expenses "97,500 00 00 data, the Cost Accountant obser decides to advise
system of collection of cost in produ ct 8. He immed iatel y
other, loss is being incurred
Units manufactured 13,500 am produced independent of each ation:
rted by thz followingBOtabul Product —
to disco ntinu e manu facture of this product suppo dues
Labour and materiai cost per uniti are constant under present conditions. Profit margin is 10% of sales manageme nt
onti nue ttn N o Product C
management to disc Product A Praduct BO.
at 90%
capacity.— z z
e
to 1 {a) You are required to determi ine the differential
i i cost of producing
i 1,500 units by increasing capacity 490,000
1,06,000 65,000
tS
Sales
26,200 1,40.000
{b ) What price
i we ald you recommend for export of these 1,500
' units A ‘takin ] ito ace oun t th.la t 52,000
overseas prices are much lower than indigenous prices. . (8.Com. Hons., Dethi Variable manufacturing cost
Fixed manufacturing overhead 1,05,000
§,500 19,000
Solution ,
(apportioned) 17,000 18,000
18,000
The ! problem does not give
gi i and labour cost
the material whic i i i Variable selling and distribution cost 8800 ae 4,000
cost. It is computed by working backward from sales as rillove is needed for computing differential _ 4,600
costs
Fixed selling and distribution
66,600 267,000
At 90% capacity 81,000
Total cost _ 2,23,000
18,900
Sales (13,500 units) : Net profit _ aa a
Less: Profit (10% of sales} ‘tso000 Net loss ca
aed with ycur views on the basis
of data
Accountant's conclusion? Argue
Cost af goods sold s0eoD Bo you agree with the Cost (LOWA, Inter}
Less: Variable expenses __ 1,45,000 meni
mes expenses 07,500
ixed expenses 3,086,500 543,000
Cost of labour and material (Prime cost) 8.07000
}
Decision
ecision Making
Mes 6.66 Management Accounting
6.65
ce

Solution price for the new market is 10,000 units. The


of Target annual sales volume at the existing selling
estimated variable cost of production is % 12 per unit.
ae Profitability Statement try to establish the new market?
(LCWA. inter)
Should the company
rs
Product A Product B P raduct €P :
> z ° Solution
~ z
Sales Statement of Incremental Cast and Revenue
1,00.000 65,000 4.90,000
Variable cost: Present Proposed Incremental cost
52.000 . ” and revenue
Manufacturing position position
oP 900 26,000 1,40,000
Selling and distribution _.
18,900 17,000 18,000 40,000 50,086 10,800
:
: cee Sales uni ' ts a a
Marginal cost
? 1,006 1.250 250
a.
50.000 63.000 1,58,000 (A) Sales Z “000 —— ——— atl
Fixed
Contribution
cost
066 22,000 3,32,000
Costs: Advertising
100 125 25
" 30
Manufacturing
39,000 1,05,000 Sales salaries 5 5
Selling and distribution on a
4,600 Travelling exp. =
-600 . 4.000 10 10
Total fixed cost 10 10
11,100 23,600 1,09,000 mn _
Profit/Loss _—— —_—
(-}1,606 2,23,000 es 250 310 60
18,900 ——_—- ee
(*) P/Y ratia ~ ~
—_————
Q *
Variable costs of production
30%
; ‘ :
ie 67.8% : 480 00
" he
#) Manufact cost to vartable cast:
or peduring q 4.
™ a — —2
tow 73.1% 75% (B} Fi . ———_—
(5) Setling and distribution , — —— 70
a7. eats . * 270 340
ae Profit (A-B)
(it) Total selling and distribution cost . ;
;
a8 a ratio of sales 22.6% 33.24 incremental revenue of % 2,50,000 resutting
stan
3a 4.3% ; Conclusion. There is incremental cost cf€ 1,80,000 against1,80,000). Therefore the proposal shouid be
di . (ie, = 2,50,000 ~
Conclusion, Product 2 should not be be disconti nued because -
in net additional income of % 70,000
(i) Product B apnea
berause af arbitrary apporti onment of fixed overhead. it is burdened 4 accepted.
with 73.1% fixed neaufacticny ix ti , Teen 3
erhead of its variable cost which is al =* Problem 6.40 (Differential Cast Analysis}
a
.. Product B is no less profitable than product A as P/V ratio nea six times that of productA. . .
70%
of B is 34% whick i rs of 1,060,000 units and presently it is working at
Hi) Alt s more tan ofA. 4 A company has an installed production capacity per unit decreases as follows :
A and €, it is burdened with the s , cost
and Hiceriee uae Sales of B are much less than
ame amount of selling capacity utilization. As production capacity utilization increases
(iv) By di 5. .
; Cost per unit
Iscontinuing product B ibuti 3 Capacity utilization
; the contribu
the toss weuld be equal te th & amount tion of € 22,000 made byti it will not| be availabl
ailable, hence : 70% Zo
of fixed cost of = 23.600 a f
Problem 6.3 , . ' pportioned to it.
3 20% z 92
6.39 le
Frob ential Cost Analysis}
(Differm a"5 90%
z 8 BY
The followin g extracts are taken from sal es budget of a company for a current year: : 100% F 82
t sources as under:
Rupees in ‘000 The company has-received three export orders from differen
Sales : 40,000 units @ F 25 per unit 7 sy Source A - 5,00) its at 755 per unit
1,000
Selling costs:
J" * Source B- 10,000units at 752 per unit
Advertising Source C - 10,000units at ¥ 51 = per unit
106 accepted or not.
Salesmen’s salaries
80 ] Advise the company whether any or all the export orders should be
Travelling expenses (B.Com. Hons., Deiht)
0 + ed
Rent of sales office 2
10
Others .

te 250 "S
i ing a
ent j1s consider “4
The managem
meth :
i : 3
vear. [t : Proposal to establish :
supervisor apposed to crease the advertising expenditure ‘by 25% in fhe eastern region in the next s"4
oh raie appoint an addition al sales
onal
, ty of & 30,000 0 per year to
nd travellin: g expanse shall inerease by 10%. establish a market. This will involve additional travetling
and x

zt
Decision Making
Management Accounting
6.68
Solution
Ltd is rere profitable
Conclusion. It may be concluded form the above analysis that ordez from A
revenue of ¢ 54,000 (2 3,40,000-2,86,000). Order from A Ltd. should,
Statement of Differential Cost (D.C.) because it gives a higher incremental
therefore, be accepted, P/V ratio from A Ltd. is also higher.
;
Capacity Praduc¢tion Cost per unit
; Total BC.C
units : . De. Problem €.42 ential Cost Analysis}
. Cost Total per “unit
z = varying
Le io
zt z A Ltd. Co/has capacity to produce 1,00,000 units of a product every month. Its works cost at
70,000 97 67,96,000 - ievels of prgduction is as under: |
oo 0 80,000. 2 73,60,000 5, 70,000 é/ Level Works cost per unit %
a 99,000 a7 78,30,000 4,70,000 10% 409
i 1
1,00 ,000 82 82,060,000 4,70,000 20% 390
Gain/Loss on Accepting Various Export Orders
30% 380
40% 370
Sour ce = Order size
q i
€apacity BC. Price Sales 50% 360
Profit/loss
PUL Total = PLY, z
A = 60% 350
= :
5,000 75% 57 2 85,000
B 20,000 85%
55 2.75 000 VY ee 70% 340
soonest, " (8.000 80% 330
5 voowanck. 5,20,000 52 5,20,000 Nil 90% 320

ec 10,000
, 9 5% 5,000@47
100% 370
>.
Total 25000 5,000@37 420,000 517 §,10,000 Its fixed administration expenses amount to % 1,50,000 and fixed marketing expenses amount to 2
+80,000
2,50,000 per month respectively. The variable distribution cost amounts to ~ 30 per unit.
Conclusion. Acceptance of orders
only fram source A or Aa nd Tt caa market 100% of its output at 7 500 per unit provided it incurs the following further expenditure :
all the B gives a loss of F 10,000. But
all the three
ordersard ers ave accepted, the company makesa profit of & when
80,000. Thus the company shou (a) it gives gift items costing 7 30 per unit of sale;
ld accept
(b) it has lucky draws every month giving the first prize of € 50,000; 2nd prize of 2 25.000, 3rd prize
Problem 6.41 {Differential Cost
Analysis) of 7 10,000 and three consolation prizes of 5,000 each to customers buying the product.
= vavta
4( hyLtd. has been offered aan n ord er from A Ltd. (c) it spends 7 1,00,000 on refreshments served every month to its customers:
. for 10,0
, 00 units
uni of output @ % , 190
there i anothe og 60 ane will i
involve an outlay of 40,000 for set up, jigs a dies {d} it sponsors a television programme every week at a cost of % 20,00,000 per month.
er of an order from B Ltd, for . At the same the
esti
ne mate
Compdeny at € sec
68 eneach and invol ve and outlay of 8,000 units of output at 2 119
each. Variable costs are It can market 30% of its output at $550 per unit without incurring any of the expenses referred to
% 50,000 for set up, Jigs and
ji dies. Which order should in (a) to (2) above.
Solution Advise the company on its course of action. Show the supporting cast sheets. (CA. Inter)

Statement Solution
of Incremental Revenue and Cost
Cost Sheet far the month of .....
{a} Size of arder {units) ‘ Coe B itd, Capacity level 30% 100%
, . 8,000
(b} Price per unit Cutput units 30,000 1,08,003
e ?
{c) Variable cost per unit 100 110 Per unit Total Per unit Total
(d} Incremental revenue (a) x (5) 10 60 68
{e) Incremental variable cost (a) « (c} € e z z
een 8,80,000
(f) Contribution (¢) - (e} “oe tos 5,446,000 Works cost 380.00 1,14,00,000 310.00 3,10,00,000
igh Outlay for set up, jigs ete. 00,000 3,36,000 Add: Adm. exp. (fixed} 5.00 1,650,000 1,50 1,456,000
(h) Total incremental cost (eh (g) a ‘ 80,000
0 Net incremental revenue (d) - (4) Y 6,60,000 50,000
5,94.000 Cost of Production 385,00 1,15,50,006 311,50 3,11,50,000

) P/V ratio {(f) + (a) « 100 ' “om 2,86,000 Add: Marketing exp. (fixed) 8,33 2,50,000 2.50 2,450,000
e 38% Distribution exp. (variable} 30.08 §,00,000 30,90 $0,00,000

(Contd.}
Management Accounting
Decision
ve Making 6.69

n-making.
Add: Further special exp: . Gut of pocket costs are not relevant for decisio
unit of key factor is the basis of ranking products on profitability.
Gift: items — — 30,00 30,00,000 . Cost per
are relevant for managerial decisions.
Lucky draw price ~ - 1.00 1,00,000 _ Future costs and not the past costs
should not normally be accepted at a price below total cost.
TV Programme — _ 26.00 20,00,000 . Expart orders
. Wages of workmen is an aut of pocket cost.
Refreshments _ _ 1,00 1,060,000 = —
_ Differential cost analysis helps in make or by decisions.
which is even below marginal cost in certain special circumstances.
Total Cast 423.33 1,27,00,000 396.00 3,96,00,090 . Management may sell at a price
Profit 126.67 38,00,000 104.00 1,04,00,000 . Differential cost is also known as incremental cost.
overhead.
_ Conversion cost is the total of direct labour and factory
Sales 550.00 1,65,00.500 500.00 2,00.00,009 when supplier's price is below firm's
. In make or buy decision, it is profitable to buy from outside firm
Advice. It is advised that the company should produce and sell 1,00,900 units at 100% capacity, Profit own total cost.
6. False; 7. Truc; 8. True;
at this level is higher at = 1,04,00.000 in spite of quite a few special further expenditures incurred. This [Ans. 1. False; 2. False: 3. False; 4. False; 5. True;
9. True; 10. True; 11. True; 12. False]
is in comparison to profit of only % 38,00,000
00, at 30% capacity level, ' which is lower by % 66,000, i.
104,600,000 - 38,00,000} , pen eee Questions
Multiple choice

KEY TERMS 1. Which of the following is an out of pocket cost ?


(a) Sunk cost (d) Imputed cost
kin
Decision-making It is the basic functions of mana gement ich
which i
involves selectingi the b (¢)- Labour cost (d) All of these.
selling prices in the short-term are normally fixed on the basis of
course of action from two or more available alternatives. , = _ According to marginal costing,
Relevant Costs These are the future casts which differ among alternative courses of action and (a) Total cost plus profit (b) Variable cost plus contribution
(a) Below marginal cost.
which are used in making decisions. {c) Total cost plus contribution
Irrelevant Costs These are the costs which were incurred in the past and also which do not | Which of the following is an irrelevant cost?

wa
' change between alternatives. These costs are not considered relevant in making decisions. {b) Opportunity cost
(a) Sunk cost
(a) All of these.
Selling Prices Decisions In the long run, under nermal circumstances, the management should (c) Replacement cost
a bulk order which will not affect prices of company
ensure that the selling price must cover total cost and also give a reasonable amount of profit. . A company has an idle plant capacity. It gets its
But sometimes due special market conditions, such as depression, competition, exporting etc., the order may be accepted at a price which is mote than
products in the market: Such a bulk
which may ‘y b ie iW (b) Variable cost
ge it may have to sell its product at a special price lo eT than the
{a}Fixed cost
thist should be done on a short term basis only. total capacity
cost. But
Variable cost plus any opportunity cost of idle
(ce)
Make or Buy Decisions In such decisions, outside supplier's price should be compared with firm’s (d)Total cost.
own variable cost. It is prafitable to buy from outside if supplier's price is below firm's own . An export order is generally accepted at

wn
variable cost of manufacture, considering non-cost factors. (b) Below fixed cost
(a) Below marginal cost
Product Mix Decisions A mix that providesi i
the highest ibution
amount of contributio i i margina l cost {d) Above total cost.
{c} Below total cost but above
the-most profitable mix. mon is considered as not be conside red in a make or buy decisions ?
Which of the following should

an
Selection ofa Suitable Method of Production, the management should select the method which (b) Variable cost of production
(a) Potential use of manufacturing capacity
gives the largest contribution (i.e., the lowest marginal cost), keeping in view the limiting factor. occupie d by production area
{c} Potential rental income from space
Differential cost is the difference in the cost of two alternatives. (d) Unchatged fixed cost
Differential cost analysis is a technique to help in decision-making. Here decisions are based on [Ans. 1 (c), 2 (2), 3 (a), 4 (8), 5 Cc). 6 (09)
comparison of differential cost and incremental revenue.
Incremental Revenue Tt is an increase in revenue due to change in alternative course of action. Essay Type Questions
decision-making. (8.Com. Hons., Deihi}
Explain fully the concept of ‘relevant cost’ in managerial
‘EXAMINATION QUESTIONS | moe
Explain the technique of marginal costing and state
its importan ce in decision making.
(¥.Com., Bangalore)
a reasonable and sound basis for
Gbjective Type Questions "The technique of marginal costing is more used to provide
(C.8. Inter)
kab

managerial decisions than to arrive at product cost” Explain.


True or Fatse? costing can be a valuable aid to management”. Discuss.
“The technique of marginal (B.Com. Hons., Delhi}
>

1. Depreciation ef plant is an example of imputed costs


the quantitative as well as qualitative
2. in differential cost analysis, managerial decisions are based on P/V ratio and contribution What do you mean by make or buy decisions ? State
{B.Com. Hons. Dethi)
wn

considerations influencing a ‘make or buy’ decision.


Hie
|
Secision Making 4
6. Management Accounting
1 6.72
6. Explain bri .
- Explain briefty the circumstances under which selling below mazginal cost may be justified 3 —
2. Lovely Snow Company manufactures and sells direct to consumers 10,000
. Jjazs of Lovely y $ SHOW per
. . (B.Com, Hons. , . :
7. cost ? if . Det) month at % 5 per jar. The company’s normal production capacity is 20,000 jars of snow per month.
Do you thinks a producer can sell his output even below variable
circumstances, * Af SQ, mention thase. An analysis of cost of 10,000 jars shows:
z
(8.€om. Hon : . . 4,000
: general recessionoo in the country which is considered te de of a temporary nature 4 vehi Direct materials
8. There isa
_ Direct labour , . 4,900
ode in Which you are employed as a cost accountant does not want to retrench ersornes the
Fowet 560
an
What at ‘oar for a product at a selling price which evidently is below the cost of product 1,720
9. Ment vice will you give to the management regarding this and why ? (B.Com. Hons. Denn Miscellaneous Expenses
- Mention five areas of business activity that frequently require managerial decision as to olig yy dats 2,400
cost analysis is helpful in decision-making ia inl. «.048? polscy. How Fixed 3h321,820
(B.Com, Caleuttsy ixed ExpeExpenses
9 +48: 50.400
19, ' Indi ot
“a circumstances
marginal any five under which you will allow to fix a price which is tess than i
nal cost. . é ore vane of |. seat
additionat
. . : . (2.5. Enter) The company has received an offer far ihe export under a different brand name of 1,20.000
11, Discuss the role o epg
of selling prices * contribution Tne costing and ns televance to decisions relating to fixation jars of snow at 10,000 jars per month at @ 3 per jar. . .
: yas
. a
ces which
justi ixati :
might justify the fixation of selling prices equal Write a note showing your calculations, on the advisability or otherwise of accepting the affer.
to or below marginal cost. (M.Com., Madras)
12. State the costing data required for (i . , (LE.W.A. Inter)
[Ans. Offer for export should be accepted as the offer price is above variable cost}
or buy decisions, quired for (1) determining the priority of various Pnaa and (i) make
13. What are the specific cost and non-cost factors that are considered before a ‘ k _ Hons Pethi) 3, A manofacture of a certain product has been selling exclusively in tie Indian market up to now. He
is taken? . make or fecision has just received his first export enquiry wyand wants to quote as competitively as the circumstances
4. Inte,
will allow. . latest tein cost sheet is :
: . .
a” ai
im Whe is differential costing ? Explain its importance in decision making.
- : : . . . .
(B.Com. Hons Det
. . - er uni
. : * . .
and diffe ey Differential Cost Analysis? Explain the essential features and points af similarit Dive aton ° 13 “
16. Wha a fence between Differential Cost Analysis and Marginal Costing. (Com, Calcutta} ved Pind seat “ bane : j
of differential - ervices On Your oisame haw)
O i © you understand by Differential Cost Analysis? Explain (a) the essential features(B.Com. Andh Ta Ciwed (7 Works overhead 7
osting,
cogs :
and (b) its practical
.
applications.
. ie a f + .
—1?. Distinguish between -marginal costing and differential costing. (8.Com. Hons et Ofice overhead 3
Profit earned in India 6
Practical Questions 68
4 Indian selling price
ca ;
Management is thinking of quoting a selling price somewhere between % 62 and € 68 per unit for
y
, mpany producing 24,000 units provides you the following information: on the princigles
z this export order. One of the directors suggests quoting an even lower price based
. . 1,20,000 of marginal costing. As the firm’s accountant, you are requested to compete the lowest price the
Direct material make on
86.000 4 management could quote on these principies. State clealy any assumptions that you may
Direct wages s the above facts and also on any other costs or facts. (CA. Inter)
Variable overheads : 48.000
28,000 {Ans. Variable cost @ 5% per unit. A price ebove this is acceptable}
Semi-variable overheads
80.000 : (Assumptions. Works overhead and office overhead have been assumed to be fixed. Services are
Fixed overheads assumed to be € 2 fixed and % 4 variable. There is sufficient spare capacizy.)
Total cost 3,60,000 |
4, A toy manufacturing company presents the following information fer the year 2006.
The product is sold at % 20 per unit.
; e
to incr ease the i ; : .
The : management proposes
. d
It is estimated that the semi-yariable overheads vill ‘nctoase by 1000. 8 it he eae ae
y % 1,000.capi But the product will be 4:
Material cost
Labour cost . 2,40,000
aie oon
sold at 14 per unit in the foraiteign no additional i will:
on.
ee
: , as However,
market.
i neurred. The management seeks your advice a cost ' accountant. What pital expenditure
will you advise rn f
ae: Fi
. Fined verhet’ ;
: ariable overhea ,
_ 4 Selling price per unit 50
BC
pen) " Units produced — 12,000
market should-be scot ted b ony
[Ans. Proposal for sale in the foreign 4 The available capacity is a production of 20,000 units per year. The firm has an offer for the purchase
- . - give additional Noh aie it wil
this offer thete
Profit of & 9,500] 4 of 5,000 additional unit at a price 7 40 per unit. It is expected that by accepting
will
:
‘ will be a saving of 7 1 per unit in material cost on all units produced, the fixed overhead
% 35,000 and the overall efficiency will drop by 2% on all production. Shouid this offer
increase by
(LOWA, Inter)
be accepted?
Decision Making
6.73 Management Accounting
~ —— 6.74
———
jAns. Offer should be accepted, Profit will increase from % 60,000 to & 60 200} Is it worthwhile
the fixed overheads will increase by 10%.
g
5. AY¥Z Cc, manufactures lathes of Captain brand and sells them at 2 36,500 each made up as follo : of the product (over initial 10,000 units) (€.8. Inter}
to try to capture the foreign market?
WS;
7
d because it will add % 5,400 to profit.
fAns. Order should be accepte
Direct materiais 16,000 cost 7 1,600]
Direct labour 2.000 Fotal variable cost % 64,000; Additional fixed
you the following data:
Factory variable overhead 5,000 gp. X Ltd, markets a single product and provides
Fixed overhead 3.000 Per unit ¢
Depreciation 2.000 13
Selling variable overheads ‘500 Materials
12
Royalty 1,000 Conversion casts (variable) 4
Profit 2,000 Dealer's margin
40
Tutat Selling price
Central excise duty "2000 Fixed cost : % 5 lakhs
Sales tax 3,000 Present sales : 90,000 units
Selling price 36.500
Capacity utilisation : 60 percent for
Thete is enough idle capacity. to se' Ul. Suggestions have been made
efforts are necessary
There is acute competition. Extra
(a) ache
A firm inee,
Africa has offered to b uy y 1010 lathes
lat at & 28,500 each. Should the company be interested
i increasing sales:
(a2) By reducing sales price by 5 percent.
(6) It bas
: also decided to seli five such | athes to an engineering
iner company und per cent over the existing: rate.
at sare cost? What price should be charged. ° em mane mC LWA.
CWA. Inter
(b) By increasing dealer's margin by 25 desires to maintain the
would recommend, if the company
anter
Which of these two suggestions you (C8 Inter)
[Ans. {a) Offer should be accepted presuming sales tax and excise duty
present profit? Given reasons. % 2,20,000;
San vocise are exempted. (b) € 29,500, i.e, marginal cost + excise duty + sales tax] (b) % 7. Presen t profit
{Ans, Contribution per unit (a) 6.20 57 units]
. An ent nysiastie marketing manager suggests to his managing director that if he is permitted to Suggestion (b) is recommended, sale 1,602,8

cidbes
g price of a product by 20%, he would be able to achi i i New selling price
is 10% of selling price. Alternative (a)
volume. The, z managing director, . findin g that the sales volume increases
increases exes fsinin pereentage
exceeds percen the (Hint : [t is assumed that dealer's margin
extent of requested reduction in price, gives the clearance. You are given the following infortions Z 38. Dealer's margin % 3.80).
its primary product. It
nents (materials) in manufacturing
Present selling price per unit = 7.50 . AB Company uses three different compo ane {desig nated as Compo nent 1) from outside
and purchases
Present volume of sates 2,600,000 units manufactures two of the components plan. Sales are highly seasonal.
developing the annual profit
supplies. The company is currently 3 can be ouichased. The three
Total variable costs % 10,50,000 rs: however, Component
Component 2 cannot be acquired from outside for the following
The annual profit plan provided daca
Total fixed costs = 3,60,000 Components have critical specifications.
Assuming nc changes in the costs pattern in the coming period computations: Component 3 unit cost
(a) Examine the
bxamine the consequences
or of th fe mana i
anaging i
director's. decision
iso assuming
i that 30% increase
i ini (At 12,000 units)
z
° . : 1.40
(b) AAt fat
what volume of sales can th e present quantum of profits
; be sustained, after erecta the price
- . A. Inter} Material (direct) 2.20
Labour (direct) 0.40
{Ans. (a) Additional loss of € 2,55,000, (2) At 6,00,000 units Fixed overhead (apportioned)
the present profit of € 90,000 can b ined} machine rental
Annual 0,50
(Hint. Use cifferential cost analysis). see used onty for component 3)
(Special machine 1.60
- 5pameon ® fe. annuelly
z j Variable factory overhead :
manufactures 10,000 units of a product at a cost of ¢ 4 per unit. There is 0.40
re re market for consusting the entire volume of production at the sate price of % 4 25 per unit Average storage cost per year {fixed}
5.30
a 1! year, there is a fall in the' demand for home market whi ch can consume , 70,000 uni Total
only at a sale price of F 3.72 per unit. The analysis of the cast per 10,000 units is ; _
Average inventory level is 500 units.
sign an year contract
Material % 15,000 e suppliers and found one that would
The purchase manager investigated outsid Serious consideration
during the year at ¢ 4.20 per unit.”
Fixed overheads 8,000 to deliver “12,000 top quality units as needed n the
tive. Should the Company make or buy Component 3? Explai
Wages ™, 11,000 is being given to this altema (1.C.W.A. Inter)
Variable overheads 6,000 retevant factors influencing your decision. [Ans. It is advisable to make the Component.
The aforeign market is ¥ 5.20]
Variable cost is % 5.10 whereas supplier's price
’ is e xplored and it
it 1s
i found that,i thisis ]market { c can CONSiUA e 20,0 a i
units of hi
the
p a cz if offered at a sale of % 3.55 per unit. It is also discovered that for additional 10,000 units
Decision Making Qi, 1a Management Accounting
6.75
. | 6.76
sals and
to evaluate these three propo
you, as & Cost Accountant, him take
- EY2 Co - produce: 5 product in
P in i
its worksho Pp ‘A’. The overhead recove y rate is 100% ie of direct' Wai ges The Managing Director asks in each ease, in order to help
volume that would be required
based on the following budgeted figures: :
calculate the additional sales (LCWA. Inter: CA. inter)
zt a decision. units earn a profit of %
Direct wages to break-even (ii) Additional 10,000
1,60,000 [Ans. {i} Additional 10,000 units units eam a profiz of % 4,000}
Variable overhead 5,000 (iif) Additional 40,000
64,000 following data:
Fixed overhead ” the sales mix from the
96,000
12. Catculate the effect of change in
The production plan
: for the same budget Pp eriod q
envis ages an ¢ utput of 18,000 units
i of product , Pr : Products

C D Total
A B
% per unit 20,000 18,000 1,20.000
Setting price 40,000 50,000 78,000
‘2 Sales (2) 34,000 16,000 4,000
Direct materiats 12 Variable cost (2) 24,000 _ 29,400
_ _
Direct wages _ 1,20,000
: Fixed cost {%} 40,000 6,000
44,000
Total overheads
Sales mix changed to 30,000 (.C.WA. Inter)
The company
n Proposes to use the balance capacity of workshop p ‘A’
‘A’ after com i i by @ 4,320)
production plan for the manufacture of component ‘G’ whose cast data are render sbove said {Ans. The new sales mix will reduce prof
® per unit ion, you are renu ired to :
Direct materials a Ah Given the following informat
uct cost and contributicn pet unit.
prod
Direct wages
16 (a} Calculate the marginal end to managemert ard why.
sales mixes you would recomm
Total overheads 16 (b) State which af the alternative Per unit
The component
production Q i $ used by yf the company
department, ini the manufacture of some other product in another Per unit
¥
x
7 20
zThe30 coeacgany receives
i an export order from abroad for the purchase of 2,000 units of product P - @25
Selling price z= 6
the company a ee accepted by diverting the capacity from component Q. in that event zt 8B
Direct materials 16 hours at 25 paise
i oa y the component which is available from an outside supplier at a price of & 24 hours at 25 pais?
Direct wages per hour
per hour.
You are requized
\ to ate |> the’
evaluate alt i courses of action a nd
the alternative i
Spare capacity should be utilised for the manufactuze -or: State With reasons whether the Fixed overhead - = 750
wages
{a) The component @, or Variable overhead ~ 150% of direct
(8) ) 2,0 0 uni
00

You might also like